Gynae

ARIMGSAS

Index Rubella exposure in pregnancy (1st trimester) ...... 1 Varicella exposure in pregnancy (1st trimester) ...... 4 Advanced age pregnancy ...... 7 Epilepsy in pregnancy ...... 11 Use of combined OCPs ...... 14 Secondary - ...... 17 Acute Urinary Retention d/t Fibroids ...... 20 Acute Urinary Retention d/t HSV ...... 24 Recurrent HSV in Pregnancy...... 26 Ovarian tumor + Urinary retention ...... 28 Subfertility ...... 29 Preterm Labor ...... 34 PPROM ...... 37 Ectopic Pregnancy...... 39 Pre-eclamptic Toxemia ...... 42 Lichen sclerosis ...... 45 Uterovaginal prolapse ...... 48 Osteoporosis ...... 51 Gestational Hypertension ...... 55 Gestational Diabetes...... 57 Atrophic / ...... 59 Placenta Previa ...... 62 Mild revealed Abruptio Placenta ...... 65 Severe concealed Abruptio Placenta ...... 68 Lactational ...... 71 LSIL ...... 73 Hydatidiform mole ...... 77 Threatened Miscarriage ...... 80 Hyperemesis Gravidarum ...... 84 Secondary Post-partum haemorrhage ...... 86 Post CS ...... 89 Primary Post-partum hemorrhage d/t retained placenta ...... 92 ...... 94

© 2020 ARIMGSAS

Contraception options ...... 96 Contraception in Mentally-disabled child ...... 99 Elective induction of labor ...... 101 ...... 103 Bartholin's Abscess ...... 106 Trauma during pregnancy...... 109 Postpartum Pyrexia - Mastitis ...... 111 Dysfunctional Uterine Bleed ...... 114 Pubertal Menorrhagia ...... 117 Breakthrough Bleeding/ Emergency Contraception ...... 119 SLE Pre-pregnancy Counselling ...... 121 Mitral stenosis in pregnancy ...... 123 Polycystic Ovarian Syndrome ...... 126 Recurrent Miscarriages ...... 129 Mobile head at term ...... 132 Vaginal Birth after Caesarean Section ...... 134 Home Birth Advice ...... 137 C-section Counselling ...... 139 Primary ...... 142 Secondary amenorrhea - Exercise-induced ...... 146 Recurrent Candidiasis ...... 149 UTI in Pregnancy/Pyelonephritis ...... 152 HRT Counselling ...... 155 HRT Counselling II ...... 158 Thalassemia Minor ...... 161 Twin Pregnancy ...... 163 Transverse Lie...... 166 Permanent Sterilization ...... 169 /Trichomoniasis ...... 171 Reduced Foetal Movement ...... 174 Iron Deficiency Anemia ...... 177 Post-dated Pregnancy ...... 180 Bipolar Disorder in Pregnancy ...... 183 Urinary Incontinence ...... 186

© 2020 ARIMGSAS

OCP-induced Hypertension ...... 190 Fibroadenoma ...... 192 OCP Counselling in a 14-year-old ...... 196 Stage 1 Breast Cancer ...... 200 Stage 2 Breast Cancer ...... 204 Cyclical and Non-cyclical Mastalgia ...... 207 Lymphedema ...... 209 Speculum Exam ...... 211 Contraceptive advice ...... 212 Rape of a 20-year old Woman ...... 215 Pre-pregnancy advise for DVT ...... 218 Pre-pregnancy advise to a 24-year-old woman with Type 1 Diabetes Mellitus ...... 220 Anencephaly ...... 223 Primary amenorrhoea ...... 225 Investigation for male factor ...... 227 Anemia in a 28 year old pregnant woman ...... 231 Breech in labour ...... 235 in a 23-year-old woman ...... 237 Cessation of periods in OCP use ...... 241 A positive culture for GBS ...... 243 Vaginal bleeding ...... 245 Eclampsia in a 22 year old primigravida ...... 247 Abnormal GTT ...... 250 Abdominal pain and vaginal bleeding ...... 253 Fundus greater than dates ...... 257 Severe postpartum hemorrhage ...... 259 Sterilization in intellectually disabled child ...... 262 Blood transfusion consent ...... 264 Meconium staining ...... 267 Urinary incontinence in a 50-year-old woman ...... 269 Fundus less than dates ...... 273 Nausea and vomiting in the first trimester ...... 275

© 2020 ARIMGSAS

Rubella exposure in pregnancy (1st trimester)

Karen, a 25-year-old kindergarten teacher comes to your GP clinic because she is worried because she had contact with a child in her class who has rubella. She wants your advice regarding further management.

TASKS 1. Relevant focused history 2. Explain investigations and management to the patient

APPROACH ▪ History o History of present illness Doctor: Hi Karen, I'm Dr.______your GP for today, what seems to be the problem? Karen: I am a bit worried because I had contact with a child with rubella in my class. D: When did the contact happen? K: It happened about 2 days ago? D: Was rubella infection confirmed in the child? K: Yes. The mother came to school and told me that her child indeed has rubella infection. D: Have you had a previous rubella infection, or have you been vaccinated for rubella? K: No, I haven't had a previous infection nor have been vaccinated for rubella. D: Have you been feeling unwell or feverish? Have you noticed a rash, or lumps and bumps behind your ears and the neck, or do you have a sore throat, runny nose? K: No doctor, I haven't got any of those. o Period history D: When was your last menstrual period? K: It happened about 6 weeks ago. D: Is it regular? What is the usual length of your cycles? K: Yes, regular, about 28 days. D: Do you have any problems with your periods like pain or clots? K: No, no problems with my periods. o Sexual history D: Are you currently sexually active? Do you have a current stable partner? K: Yes, I've been with my partner for 2 years now. D: Are you using any contraceptives? K: I've been using combined OCPs before, but I stopped taking them 6 months ago. D: Have you done a home pregnancy test? K: No doctor, not yet. D: Have you had any pregnancies or miscarriages in the past? Any history of sexually transmitted infections? K: No doctor. 1

D: When was your last pap smear? What is the result of that? Page

© 2020 ARIMGSAS

K: I had it done 1 year ago, and the results were normal. D: Any previously diagnosed medical or surgical illnesses? K: None. o SADMA D: Do you smoke, drink alcohol or take any recreational drugs? K: No I don't smoke nor take any recreational drugs, but I drink alcohol occasionally. D: Okay, if indeed you are pregnant now, you must remember to cease alcohol drinking during pregnancy as it will not be good for the baby if you continue drinking alcohol. K: Yes doctor. ▪ Diagnosis and Management D: Okay, I'll have you do a urine pregnancy test right now for us to confirm if you are pregnant. Excuse me, I'll just talk to the examiner for a while. (to examiner) What is the result of the urine pregnancy test? Examiner: PT is positive D: Thank you for that examiner. (to the patient) D: So PT results are positive which means you are indeed pregnant right now. I would like to arrange some blood tests for you: FBE, blood group and Rh factor, BSL, urine dipstick and urine microscopy culture and sensitivity, UEC, vitamin D estimation, rubella, varicella serology and STI screening. I will also start you on folic acid for the first three months of your pregnancy. Was this a planned pregnancy? K: Yes, we've been wanting to have a baby. D: Congratulations on your baby. As I've said, I'd like to order rubella serology test for you for us to know if you have got a rubella infection due to your exposure. Do you know anything about rubella? K: Not so much. D: Rubella is a mild infectious disease caused by a virus which is usually transmitted through air droplets. For you, rubella is a mild infection. However, for the baby, if you contact a rubella infection at this time of the pregnancy, there is a high chance that the baby gets infected and it results in congenital rubella syndrome. Birth defects associated include deafness, blindness, heart defects, limb defects, and mental disability. I need to check for certain factors called antibodies in your blood which tells us if your body has the power to fight the infection or not. There are two antibodies that we have to check for, and there are 3 possible scenarios. (write on paper) IgG IgM

+ - If IgG is positive and IgM is negative, it means you do not currently have rubella infection and it is safe for you to continue the pregnancy. I would arrange for your antenatal blood tests, start you on folic acid until 3 months of your pregnancy, then we'll proceed as in a normal pregnancy, ultrasound at 8 weeks, at 18 weeks, sweet drink at 26 weeks, bug test at 36 weeks and regular antenatal checks where your BP and weight will be

recorded along with other assessment to note any alterations.

2 Page

© 2020 ARIMGSAS

- + If IgG is negative and IgM is positive, unfortunately, it means you have a current rubella infection. I will have to refer you to the high-risk pregnancy clinic where you will be seen by the obstetrician and infectious disease specialist, and you have two options at this point. You can terminate this pregnancy or you can continue with this pregnancy, then we'll monitor the baby through repeated ultrasound scans. Whatever your decision is, you will be supported through that. Also, rubella being a reportable disease, it has to be reported to the Department of Health Services.

- - If both IgG and IgM are positive, then we have to repeat the test in 3 weeks just to confirm the results. During this time, do not come in contact with any other rubella cases. If both antibodies remain negative, it will be safe for you to continue the pregnancy. After delivery, you will again be checked for your rubella status, and if still negative, you will be given the MMR vaccine. After you've been given the vaccine, you should not try to be pregnant within 28 days or better 3 months.

D: Other than those, for a healthy pregnancy I advise you to take a healthy balanced diet rich in fruits, vegetable, cereals and bread, drink ample amount of fluid about 2 litres per day, and adopt a sensible, regular and non-contact exercise. Take your folic acid regularly. I'll arrange a review with you once your blood test results are in. If you experience any unusual pain or bleeding, call for an ambulance immediately. I will also give you reading materials for your further information. Do you have any other questions? K: None doctor. D: Okay then, thank you very much!

3 Page

© 2020 ARIMGSAS

Varicella exposure in pregnancy (1st trimester)

Karen, a 25-year-old kindergarten female comes to your GP clinic because she is worried because she had contact with a child in her class who has varicella.

TASKS 1. Focused history 2. Explain investigations and management

APPROACH ▪ History o History of Present Illness Doctor: Hi Karen, I'm Dr._____ your GP for today, what seems to be the problem? Karen: I am a bit worried because I had contact with a child with varicella in my class. D: When did the contact happen? K: It happened about 2 days ago? D: Was rubella infection confirmed in the child? K: Yes. The mother came to school and told me that her child indeed has rubella infection. D: Have you had a previous rubella infection, or have you been vaccinated for rubella? K: No, I haven't had a previous infection nor have been vaccinated for rubella. D: Have you been feeling sick or feverish? Have you noticed a rash, blisters or lumps and bumps around your body? Do you have a sore throat or a headache? K: No doctor, I haven't got any of those. o Period history D: When was your last menstrual period? K: It happened about 6 weeks ago. D: Is it regular? What is the usual length of your cycles? K: Yes, regular, about 28 days. D: Do you have any problems with your periods? K: No, no problems with my periods. o Sexual history D: Are you currently sexually active? Do you have a current stable partner? K: Yes, I've been with my partner for 2 years now. D: Are you using any contraceptives? K: I've been using combined OCPs before, but I stopped taking them 6 months ago. D: Have you done a home pregnancy test? K: No doctor, not yet. D: Have you had any pregnancies or miscarriages in the past? Any history of sexually transmitted infections? K: No doctor. D: When was your last pap smear? What is the result of that? 4

K: I had it done 1 year ago, and the results were normal. Page

© 2020 ARIMGSAS

D: Any previously diagnosed medical or surgical illnesses? K: None. o SADMA D: Do you smoke, drink alcohol or take any recreational drugs? K: No I don't smoke nor take any recreational drugs, but I drink alcohol occasionally. D: Okay, if indeed you are pregnant now, you must remember to cease alcohol drinking during pregnancy as it will not be good for the baby if you continue drinking alcohol. K: Yes doctor. ▪ Diagnosis and Management D: Okay, I'll have you do a urine pregnancy test right now for us to confirm if you are pregnant. Excuse me, I'll just talk to the examiner for a while. (to examiner) What is the result of the urine pregnancy test? Examiner: PT is positive D: Thank you for that examiner. (to the patient) D: So PT results are positive which means you are indeed pregnant right now. I would like to arrange some blood tests for you: FBE, blood group and Rh factor, BSL, urine dipstick and urine microscopy culture and sensitivity, UEC, vitamin D estimation, rubella, varicella serology and STI screening. I will also start you on folic acid for the first three months of your pregnancy. Was this a planned pregnancy? K: Yes, we've been wanting to have a baby. D: Congratulations on your baby. However, I would also like to order a varicella serology test for you for us to know if you have got a rubella infection due to your exposure. Do you know anything about rubella? K: Not so much. D: Chickenpox or varicella is an infectious disease caused by a virus which is usually transmitted through air droplets and direct contact. For you, rubella is a mild infection. However for the baby, if you contact a varicella infection at this time of the pregnancy, there is a high chance that the baby gets infected and it results in congenital varicella syndrome. Birth defects associated include microcephaly or a small head, deafness, blindness, heart defects, limb defects, intrauterine growth retardation or your baby doesn't grow well and mental disability. I need to check for certain factors called antibodies in your blood which tells us if your body has the power to fight the infection or not. There are two antibodies that we have to check for, and there are 3 possible scenarios. (write on paper) IgG IgM

+ - If IgG is positive and IgM is negative, it means you do not currently have varicella infection and it is safe for you to continue the pregnancy. I would arrange for your antenatal blood tests, start you on folic acid until 3 months of your pregnancy, then we'll proceed as in a normal pregnancy, ultrasound at 8 weeks, at 18 weeks, sweet drink at 26 weeks, bug test at 36 weeks and regular antenatal checks where your BP and weight will be

recorded along with other assessment to note any alterations.

5 Page

© 2020 ARIMGSAS

- + If IgG is negative and IgM is positive, unfortunately, it means you have a current varicella infection. I will have to refer you to the high-risk pregnancy clinic where you will be seen by the obstetrician and infectious disease specialist, and you have two options at this point. You can terminate this pregnancy or you can continue with this pregnancy, then we'll monitor the baby through repeated ultrasound scans. Whatever your decision is, you will be supported through that. (If symptomatic) If you decide to terminate the pregnancy, acyclovir which is an antiviral medication (800mg 5x/day x 7 days) will be given to you. Also, varicella being a reportable disease, it has to be reported to the Department of Health Services.

- - If both IgG and IgM are positive, since your exposure only happened 2 days ago, I could give you the varicella-zoster immunoglobulin which gives you immediate protection against the varicella virus. However, because immunoglobulins are not 100% protective, you should still be alert for symptoms of chickenpox such as fever, fatigue, sore throat, rashes and vesicles, and report back when you experience such. (If more than 96 hours since exposure say: we have to repeat the test in 3 weeks just to confirm the results.) During this time, do not come in contact with any other rubella cases. If both antibodies remain negative, it will be safe for you to continue the pregnancy. After delivery, you will again be checked for your rubella status, and if still negative, you will be given the MMR vaccine. After you've been given the vaccine, you should not try to be pregnant within 28 days or better 3 months.

D: Other than those, for a healthy pregnancy I advise you to take a healthy balanced diet rich in fruits, vegetable, cereals and bread, drink ample amount of fluid about 2 litres per day, and adopt a sensible, regular and non-contact exercise. Take your folic acid regularly. I'll arrange a review with you once your blood test results are in. If you experience any unusual pain or bleeding, call for an ambulance immediately. I will also give you reading materials for your further information. Do you have any other questions? K: None doctor. D: Okay then, thank you very much!

6 Page

© 2020 ARIMGSAS

Advanced age pregnancy

Jane, a 40-year-old, single woman, came into your GP clinic because she had a positive home pregnancy test. This is her first pregnancy.

TASKS 1. Counsel her on the risks of pregnancy for her and her baby

APPROACH ▪ History Doctor: Hi Jane, I'm Dr. ______, I'm your GP for today. What brings you here today? Jane: I'm here because I took a home pregnancy test and I'm a bit worried about the risks of this pregnancy for me and my baby. D: Okay, I understand you have some doubts about this pregnancy. I am here to assist you with your doubts. I just have a few questions for you, is that alright? J: Sure doctor. o Period history D: When was your last menstrual period? J: I had it 8 weeks ago. D: Is your cycle regular? Do you have any problems with your periods? J: Yes, my cycles are regular. No problems with my period. o Social history D: Do you have a stable partner right now? J: No, I broke up with my partner 6 months ago, I thought we were going steady, but things didn't really work out. D: Okay, but you are willing to continue with this pregnancy? J: Yes doctor, as long as I get the assurance from you that it is safe for me to carry on with this pregnancy. D: Do you have someone to support you during this pregnancy? J: Yes, my mother and sister live nearby. I could rely on them to help me through this pregnancy. (If no support, D: I will refer you to a social worker who could help you find support for your during your pregnancy) D: How are your finances? Will you be able to support yourself for this pregnancy? J: Yes, I have a stable job, and it provides adequately for my needs. (If no finances, D: I will refer you to Centrelink who can assist you with money matters that could help you during your pregnancy) o Sexual history D: Do you have any history sexually transmitted infections? J: No doctor. D: When was your last pap smear and what was the result?

J: I had it last year, the results were normal.

D: Were you using any contraceptives before? 7 Page

© 2020 ARIMGSAS

J: I was using combined OCPs before, but I stopped it a year ago because I thought I wouldn't get pregnant anymore o General wellbeing D: How are your diet and exercise? J: I eat moderately, but I'm mostly just sedentary, doing my office job. D: Sensible, regular, non-contact exercise is also important, let's arrange for another session to talk about lifestyle modifications. Do you smoke, drink alcohol, or take recreational drugs, take any medications or over the counter medications? J: No doctor, none of those. o SADMA D: Do you smoke, drink alcohol, take any recreational drugs, medications or over the counter medications or any allergies? J: No I don't smoke or take drugs or medications, but I drink occasionally. D: Alright, remember that drinking alcohol is not safe during pregnancy, so no alcohol while you're pregnant okay? Thank you for the information, Jane, I'll just talk to my examiner. D: Any history of medical or surgical illness? J: No doctor, I've been pretty healthy. D: Do you have any family history of birth defects? J: No doctor.

▪ Physical Examination: D: (to examiner): What is the BP, PR, RR, temp and O2 sat of my patient? What is her BMI? E: BP is 110/80, PR 80, RR, 15, temp 36.8, sats 99% room air. BMI is 24.

▪ Diagnosis and Management: D: (to the patient) Okay Jane, I would just like to do an office pregnancy test for you just to confirm your pregnancy. D: (to examiner): I would like to know the results of the pregnancy test. E: Pregnancy test result is positive. D: (to the patient): Okay, now that we've confirmed your pregnancy, I would like to arrange some blood tests for you: FBE, blood group and Rh factor, BSL, urine dipstick and urine microscopy culture and sensitivity, UEC, vitamin D estimation, rubella, varicella serology and STI screening. because of your advanced age, I will need to refer you to the high-risk pregnancy clinic where you will be seen by the obstetrician and you will have regular antenatal checks every 8 weeks during the 1st trimester, every 4 weeks up to 28 weeks, every 2 weeks up to 36 weeks, and every week up to your delivery. During each visit, weight and BP will be recorded so any alterations will be noted. I will also start you on folic acid 0.5mg for the next three months. You don't have to worry so much because many women at your age go through a normal pregnancy and delivery. It is not automatic that you or the baby will have complications. But since you are concerned, I could tell you more about that. For you, in your early stages of pregnancy, you can have a miscarriage or an ectopic pregnancy, a condition where the fertilized egg is lodged elsewhere in your reproductive

tract other than the where it should normally be.

8 Page

© 2020 ARIMGSAS

However, an ultrasound at 8 weeks called a dating scan will be done to confirm your dates and also rule out an ectopic pregnancy. You can also have pre-eclampsia, a condition where there is a sharp rise in blood pressure, weight gain and kidney stress which shows up as protein in the urine; gestational diabetes mellitus, or placenta previa. The placenta is the part that connects you to your baby. It is normally at the upper pole, but when it attaches to the lower part, it could lead to bleeding.

Towards the end of your pregnancy, you can have preterm labour, prolonged labour, or an increased chance of induction or C-sections. For the baby, he/she can have a risk for heart defects, kidney defects, neural tube defects which are defects in the spinal cord and the brain. However, the most important complication is Down syndrome. At age 40, the risk for Down syndrome is 1 in 106. (At age 42, 1 in 70; 45, 1 in 30; 35, 1 in 385; 30, 1 in 900). There are 2 types of tests for Down syndrome, 1 is screening and the other is a diagnostic test. The screening test does not tell you with certainty if the baby has Down's or not, but diagnostic tests are confirmatory tests. Screening can be done in the 1st and 2nd trimester. In the 1st trimester what we usually do is a combined test, combine blood testing with an ultrasound. A blood test is done between 9 - 13 weeks, and we look for 2 factors: B-HcG and PAPPA

(pregnancy-associated plasma protein-A). B-HcG is increased and PAPPA decreased in Down

syndrome. The ultrasound test is called the nuchal translucency scan where we test the 9

skinfold thickness on the back of the baby's neck. This is done between 11 - 13 weeks. Page

© 2020 ARIMGSAS

Detection rate for the combined test is 87%. These do not pose any risks to the baby. Another test is the NIPT or non-invasive prenatal test, where we take a blood sample from you at around 10 weeks of pregnancy to look at the genetic material of the baby. Detection rate for NIPT is 99%. In the 2nd trimester, we usually do a triple or quadruple test to look for 3/4 factors in maternal blood: AFP and are decreased, and B-HCG and Inhibin A are increased in Down's. This is usually done in 15-17 weeks, and detection rate is 71% for the triple test, and 81% for the quadruple test. There are two diagnostic tests: chorionic villus sampling or CVS and amniocentesis. CVS is done between 11-14 weeks. A needle guided by ultrasound will be passed from down below and a portion of the placenta will be taken and analyzed for chromosomal/genetic defects. The risk for miscarriage is associated with this procedure, about 1 in 100. Amniocentesis is done between 15-18 weeks. A needle guided by ultrasound is passed into your tummy to your womb and a portion of the fluid in the bag surrounding the baby will be taken and analyzed for genetic defects. The risk for miscarriage is 1 in 200. Other than those, for a healthy pregnancy, I advise you to take a healthy balanced diet rich in fruits, vegetable, cereals and bread, drink ample amount of fluid about 2 litres per day, and adopt a sensible, regular and non-contact exercise. Take your folic acid regularly. I'll arrange a review with you once your blood test results are in. If you experience any unusual pain or bleeding, call for an ambulance immediately. I will also give you reading materials for your further information. Do you have any other questions? J: Doctor what is the chance of me getting a baby with Down syndrome in a next pregnancy? D: The risk is that of the risk for your age plus 1%. For example at 40 years old, the risk is 1%, so the risk for a Down syndrome baby in your next pregnancy is 2%. J: Thank you, doctor. D: Thank you, Jane. I'll see you on our next review.

10 Page

© 2020 ARIMGSAS

Epilepsy in pregnancy

26-year-old Laura who is a known epileptic presents to your GP clinic for advice regarding her chances and preparation to become pregnant. She is on sodium valproate and had been seizure-free for the last 2 years.

TASKS 1. Counsel the patient appropriately

APPROACH ▪ History Doctor: Hi Laura! I'm Dr. _____, one of the GPs in this clinic, what can I help you with today? Laura: Doctor I've been epileptic for the past 5 years, I would like to ask for some advice regarding my chances and preparation for pregnancy. D: I see Laura, I'm happy to help you with that, but I'll have you answer some questions beforehand, is that alright? L: Sure doctor. D: You said you've been diagnosed with epilepsy for 5 years, do you know what type? L: My doctor said it's focal epilepsy. D: Okay, when did you last see your neurologist? Do you go for regular checks? What was his last advice to you? L: Yes doctor, I see him regularly. He started me on this medication, Na Valproate, and it's been doing well for me. I've been seizure-free for 2 years now. D: Alright. Has this been your medication or usual dose ever since? Do you experience any side effects with this drug? L: Yeah. It's been this since the start, no side effects. o Period history D: Okay, when was your last menstrual period? L: It was 3 weeks ago. D: Is it regular? The normal length of cycles? L: Yes. D: Is it a mild, moderate, or severe bleeding during periods? Do you experience pain or clots during your periods? L: Just a moderate bleed, no problems. o Sexual history D: Are you currently sexually active? Do you have a stable partner? L: Yes, my boyfriend and I have been together for 3 years. D: Do you use any form of contraception? L: Yes, I used combined OCPs in high dose because of my medication. D: Alright. Do you have any previous miscarriages or pregnancies?

L: No. D: Any history of sexually transmitted infections? When was your last pap smear and what was the result? 11

L: No. It was 1 year ago, the result is normal. Page

© 2020 ARIMGSAS

o SADMA D: Do you smoke, drink alcohol or use any recreational drugs? L: No. D: Alright. Have you been diagnosed with any medical or surgical illness in the past other than your epilepsy? Do you take any other medications or over the counter medications? L: No.

▪ Diagnosis and Management D: Alright, thank you for that information, Laura. As for your doubts regarding your chances and preparation for pregnancy considering your condition, it would be reassuring for you to know that 80 - 90% of people with epilepsy go through a normal pregnancy and delivery. The chance of you having a seizure during pregnancy is only around 10 - 20% and risk during labour are only 1 - 2%. Aside from that, you've mentioned that you have been seizure-free for 2 years which is, in fact, one of the criteria we consider for planning for pregnancy in epilepsy. There could be certain complications for you and the baby due to the antiepileptic medications that you are taking or just in case you experience a seizure attack. You may experience vaginal bleeding, especially in the 3rd trimester as antiepileptic medications decrease vitamin K in your blood which is necessary for blood clotting. You may also experience a condition we call abruption placenta. Normally placenta separates from the womb after the baby's delivery, but just in case it happens during pregnancy, this is abruption placenta. There could be pain and bleeding and the baby could also become unwell. Another complication is preterm labour or labour before 37 weeks or premature rupture of the bag of water, and increased risk of induction and C-section. Complications in the baby are mainly neural tube defects or defects in the brain and spinal cord, heart and skeletal malformations, intrauterine growth retardation, and if you go into premature labour, prematurity of the baby. Once the baby is born, the baby can have a condition called hemorrhagic disease of the newborn or bleeding tendencies because of the decrease in vitamin K. D: When are you planning to get pregnant Laura? L: Probably 7 months from now, doctor. D: Alright, that's a fair amount of time to prepare. So before you get pregnant, I will refer you to the neurologist who will do one of three things, either 1, a supervised safe withdrawal of medication over 3 to 6 months; or 2, if stopping the medication is not possible, a change in medications from sodium valproate to any new generation anti- epileptic like lamotrigine which carries the risk of fewer birth defects in the baby compared to sodium valproate; or 3, if the medication cannot be changed from sodium valproate, the neurologist will keep the medication below 1g/day, because above 1 g/day, it carries a high risk of birth defects to the baby. I will also start you on high dose folic acid 5g/day 3 months before you get pregnant and 3 months after you get pregnant. I will also do your antenatal blood checks now like FBE, blood group and Rh factor, BSL, urine dipstick and urine microscopy culture and sensitivity, UEC, vitamin D estimation, rubella, varicella serology and STI screening.

Once you become pregnant, I will refer you to high-risk pregnancy clinic where you will be seen by the neurologist, obstetrician and me as your GP. You should continue taking your antiepileptic medication as well if it is indicated by the neurologist. Your baby will be 12

monitored for neural tube defects or defects in the spinal cord and brain by looking at the Page

© 2020 ARIMGSAS

AFP in your blood at around 16 - 20 weeks, and an ultrasound at 18 and 34 weeks to look for acrania or absence of skull bones. The detection rate of these tests is around 95%. You will also take the sweet drink test at 26 weeks, bug test at 36 weeks, and blood levels of the medication will be monitored once in each trimester. I will start you on vitamin K, 20mg/day orally during the last month of your pregnancy. If everything goes well for you and the baby, you could attempt for a normal vaginal delivery but it should be in a tertiary hospital under specialist guidance. You and the baby will be continuously monitored during the delivery. Your baby will then be checked by the pediatrician after delivery and will be given vitamin K injection 1mg to prevent bleeding. You are encouraged to breastfeed your baby since only small amounts of the medication are secreted in breast milk. The risk of your baby having epilepsy is slightly higher in the general population which is 3- 3.5%. I will give you reading materials for further information and will arrange a review with you after your neurologist consult. Do you have any other questions at this point, Laura? L: None doctor, thank you very much. D: Thank you, Laura, I'll see you next time.

13 Page

© 2020 ARIMGSAS

Use of combined OCPs

Your next patient at your GP is a 19-year-old, university student, Jessica, asking you for a prescription of oral pills, as she is now planning to become sexually active.

TASKS 1. Focused relevant history. 2. Examination findings from the examiner. 3. Counsel the patient accordingly.

APPROACH ▪ History Doctor: Hi Jessica, I'm Dr. ______one of the GPs of the clinic, what might I help you with today? J: Well doctor, I'm going to Uni soon, and I'm considering starting on oral contraceptive pills. o Sexual history D: Alright I understand that. Have you been sexually active before? Do you have a stable partner? J: No, I haven't. But my boyfriend and I are going to Uni together. (If yes, do a urine PT to confirm if the patient is not pregnant at the moment) D: Have you received the Gardasil vaccine? J: Yes, I received 3 shots of that already. o Period history D: When was your last menstrual period? J: 3 weeks ago. D: Are your cycles regular? Average cycles? J: Yes regular, 28-day cycles. D: Is your bleeding mild, moderate and severe? Any pain or clots during periods? Any bleeding in between periods? J: Just moderate, and no problems with periods or bleeding in between periods. o Contraindications to OCPs D: Any previous surgeries in the past 3 months or recent trauma? J: No. D: Any previous travel, especially more than 12 hours? J: No. D: Any weight loss, loss of appetite, lumps and bumps around the body? J: No. D: Any calf pain or prominent veins in the legs? J: No. D: Any family history of coagulation disorders or history of deep vein thrombosis?

J: No. D: Any previous history of migraine, heart disorders, liver disorders, stroke, high blood pressure, diabetes, breast malignancy or severe depression? 14

J: No. Page

© 2020 ARIMGSAS

o SADMA D: Do you smoke, drink alcohol or use recreational drugs? J: No. D: Do you take any prescription or over the counter medications? Any allergies? J: No. D: Thank you for that information, Jessica. I will just talk to my examiner and I will get back to you shortly. ▪ Physical Exam D: (to examiner) What is the BMI of my patient? E: 20.5 D: Any pallor, icterus, cyanosis, lymph node enlargement, edema, poor skin turgor, dry mucous membranes, delayed capillary refill time? E: None. D: What is the BP, PR, RR, Temp and Sats of my patient? E: 110/70, 85, 15, 36.8, 99% room air D: How are the heart sounds? Is the rhythm regular? Any murmurs? E: Normal S1 and S2, regular, no murmurs D: Is air entry equal? Any adventitious breath sounds? E: Air entry equal, no abnormal breath sounds D: How are the motor and sensory exam of the upper and lower limbs? (stroke symptoms) E: All normal D: Is there any visible distention or mass of the abdomen? Is there any hepatosplenomegaly, any mass or tenderness? E: None. D: Any edema or tenderness of the lower leg? E: None. D: Any palpable breast lumps, tenderness or visible distortion or dimpling? E: None. D: Are there any visible lesions in the and ? Any discharge or bleeding? E: None. D: I'd like to do a urine dipstick test and a blood sugar level. E: Urine dipstick is normal, blood sugar is 5. D: Thank you for that information. I'd speak with my patient again. ▪ Diagnosis and Management D: (to the patient) Okay, it seems like it would be possible for you to start on combined OCPs, so I would like to give you information about it. Combined OCPs contains 2 hormones, and progesterone which is normally present in your body which regulates your periods. What it does is it inhibits ovulation, the release of an egg from the ovary. To a lesser extent, it increases the thickness of your cervical secretions so the sperm will find it difficult to get through. And just in case fertilization happens, it changes the lining of your womb so that implantation does not happen, because only after implantation does a full bloom pregnancy happen. (TAKE PILL PACK) In a pill pack, there are 28 pills, 21 are hormonal, 7 are sugar-coated or dummy pills. Starting taking the hormonal pill from the 1st day of your next period, 1 pill a

day, at the same time every day. Continue the hormonal pills for 21 days and then on starting the sugar pills, you get your periods. However, if you want to start taking the pill right away without waiting for your next period, you may, but use alternate methods like 15

condoms for 7 days. Page

© 2020 ARIMGSAS

While taking the pill, you may experience side effects such as nausea and vomiting, abdominal bloating and breast tenderness. Breakthrough bleeding or bleeding in between periods will usually settle in 3-4 months. Major side effects such as DVT, stroke and MI could happen but are rare with low dose pills, such as what you will be taking. Advantages of the pill include periods become more regular, lighter and shorter. There is less dysmenorrhea. There is a decreased incidence of benign breast lumps and pelvic inflammatory diseases, decreased incidence of endometrial and ovarian cancer, and thyroid disorders. However, you must remember that OCPs do not protect against sexually transmitted infections, so you must use condoms along with it just in case you're concerned about STIs. If you experience diarrhea and vomiting within 2 hours of taking the pill, take a pill again and keep going with the rest. You need to use condoms as long as diarrhea and vomiting last. If you go to a doctor or pharmacist, make sure you tell them that you're on pills as there are medications that decrease the efficacy of pills like vitamin C, some antibiotics or antifungals, and antiepileptics. If you miss a pill for more than 24 hours, take the recently missed pill and just keep going with the rest even if it means taking 2 pills on the same day. If you keep going with the rest, and the dummy pill period falls within 7 days of missing the pill, skip taking the dummy pills and start the hormonal pills from the next pack. This will mean that you will miss having your periods. Alternate methods of contraception like condoms should be used for 7 days after missing the pill. I will give you reading materials for your further information, and if you experience any problems with the pill, report back to me. Remember to do a pap smear every 2 years. I will review you in 3 months, and then yearly after that. Do you have any questions at this point? J: No doctor. Thank you. D: Thank you, Jessica, I will see you next time.

16 Page

© 2020 ARIMGSAS

Secondary dysmenorrhea - Endometriosis

Your next patient at your GP is 28 years old Samantha, complaining of severe abdominal pain during menstruation since the last 6 months.

TASKS 1. Focused and relevant history 2. Examination findings from the examiner 3. Diagnosis 4. Relevant investigations and management

Differential Diagnosis: ▪ Endometriosis ▪ Fibroids ▪ Pelvic Inflammatory Disease ▪ Intrauterine Contraceptive Device

APPROACH ▪ History o History of Present Illness Doctor: Hi Samantha, I'm Dr. ______your GP for today. I read from the notes that you've been having abdominal pains during your periods. Do you have your period now, are you currently in pain? Samantha: No doctor, I'm not in pain right now. D: is Okay. Can you point where exactly is the pain? What does the pain feel like? S: It's around here in my lower tummy, and it's a sort of a crampy pain. D: Does the pain go anywhere else? S: Sometimes it goes right through my back. D: Any associated symptoms like nausea, vomiting? S: Yeah sometimes I feel a bit sick as well. D: How long have you been experiencing this pain? S: I've been having this pain during my periods for the last 6 months. D: Concerning your periods, when does it start and when is it relieved? S: It usually starts 2 days before I get my period, then it gets worse as my period starts. D: Do you get pain in between periods? S: No. D: On a scale of 1 to 10, 10 being the most pain, how bad is the pain usually? S: It's around 8-9 on bad days doctor. o Period history D: When was your last menstrual period?

S: Around 3 weeks ago D: Regular, average cycles? S: Yes. 17

D: Is your bleeding mild, moderate or severe? Page

© 2020 ARIMGSAS

S: Mostly heavy bleeding doctor. D: Any clots? Bleeding through your urine or tarry stools at the time of your periods? S: None doctor. o Sexual history D: Are you currently sexually active? Do you have a stable partner? S: Yes, I have a boyfriend of 4 years. D: Do you experience pain during coitus? S: Yes oh my god doctor, it hurts so bad during coitus D: Okay I see. Any previous pregnancies or miscarriages in the past? S: None D: Do you use any contraception? (IUCD) S: Yes, I'm currently on the pill. D: When was your last pap smear and what was the result? S: It was just last year, everything's normal. D: Do you feel any mass in your tummy? (fibroids) S: No. D: Any fever, abnormal or offensive ? (PID) S: None. D: Okay, thank you for those information Samantha, I'd just speak to my examiner for a while, I'd get back to your shortly. ▪ Physical Exam: D: (to examiner): Is there pallor, lymph node enlargement? E: None. D: What is her BMI? E: 23 D: What is her BP, and temp? E: 120/80, 37C D: Is the abdomen soft, non-tender? Any visible distension, palpable mass? E: It is soft, non-tender, no visible distention, no palpable mass D: On inspection, are there any abnormal/offensive discharge or bleeding from the vulva and vagina? E: None D: On speculum exam, any discharge or bleeding from the ? E: None D: On per vaginal exam, is there cervical motion tenderness? E: None. D: What is the position and mobility of the uterus? Any tenderness? E: It is fixed and retroverted, no tenderness D: Any mass or tenderness in the adnexa? E: None. D: On per rectal exam, any nodularity or tenderness in the pouch of Douglas and the uterosacral ligament? E: Yes, there is nodularity and tenderness in the pouch of Douglas and uterosacral ligament. D: I would like to do an office urine pregnancy test, urine dipstick test and blood sugar level

E: PT is negative, UDT is normal, BSL is 5. D: Okay thank you for the information examiner, I would like to speak to my patient.

▪ Diagnosis and Management 18 Page

© 2020 ARIMGSAS

D: (to patient) Okay Samantha, based on your history, physical exam and office tests, your most likely diagnosis is endometriosis. Do you know anything about this? S: No doctor. D: Okay. The tissue lining your womb is called . Each menstrual cycle, part of grows and becomes filled with blood and then is shed as a period. Endometriosis is a condition in which fragments of the endometrium grow in other places, such as the wall of the uterus, the ovaries and ligaments inside the , the fallopian tubes and on other pelvic organs.

Each cycle, the blood from these fragments cannot escape because it is embedded in tissue in the pelvis. Small blood blisters develop and irritate the tissues. The exact cause is not fully known, but one theory is the backward flow of blood through the tubes or retrograde menstruation. Another theory is it can also spread through blood and lymphatics. It untreated, it can lead to complications such as dysmenorrhea, menorrhagia (excessive bleeding), and infertility. Endometriosis is a common problem, especially in its mild form, about 1 in 100 women will be affected by it. The symptoms are the same as yours, painful and heavy periods, pain in the back, pelvis and abdomen during periods, pain during intercourse, nausea and tiredness. I would order blood exams for you such as an FBE, UEC, CRP, ESR, coagulation profile, and imaging tests such as a transvaginal ultrasound and laparoscopy for us to visualize your pelvic structures. I would give you painkillers that you could use at the time of your periods, and refer you to an OB- G specialist. There are two types of treatment, medical and surgical treatment. For medical treatment, you could take a combined oral contraceptive pill continuously for 6 months. You will not have your periods and the deposits will not bleed as well and start regressing. Another option will be progestogens like Depo-Provera and Mirena. GnRH analogues can also be given via nasal spray daily, or injections monthly or 3 monthly for 6 months which would induce medical . However, it should not be used for more than 6 months because you might develop severe menopausal symptoms like hot flushes, bone pain and osteoporosis. Or lastly, Danazol can be given for 3-6 months which would also induce medical menopause, but should not be used for more than 6 months because it could cause you to develop male characteristics like voice changes, hair growth because it is an androgen. If medical management fails or if you present with infertility or severe symptoms, our next option would be surgical management via laparoscopic excision which is cutting away of endometrial deposits through keyhole surgery or laparoscopic ablation techniques using laser or electrocautery. I will give you reading materials and arrange for a review with you once your blood test results are in. Do you have any questions at this point? S: No doctor, thank you. D: Thank you, Samantha, I will see you next time. 19

Page

© 2020 ARIMGSAS

Acute Urinary Retention d/t Fibroids

You are an HMO at the ED of a major hospital, seeing a 34-year-old lady, Vanessa, complaining of severe abdominal pain. She had not passed urine since yesterday night.

TASKS 1. Focused history 2. Examination findings from the examiner 3. Relevant investigations 4. Management

APPROACH ▪ History o History of Present Illness Doctor: Hi Vanessa, I'm Dr. _____, your GP for today. How can I help you? Vanessa: I'm very much in pain doctor, please help me. D: Where is your pain? How bad is it? V: It's here in my tummy. I am very much in pain, please help me. D: Okay Vanessa, I'm here to help you, let me just talk to my examiner. D: Is the bladder palpable per abdomen, and are there rash/vesicles around the ? E: Bladder is the palpable per abdomen, no rash/vesicles around the urethra. D: I would like to proceed with a Foley catheterization in my patient with the consent of my patient under all sterile precautions. I would like to know how much urine has been drained and would like to send a sample for microscopic examination. I would like to keep the catheter there until she is seen and evaluated by a specialist. D: How are feeling right now, Vanessa? V: I'm much better now, Doctor. D: Great, may I proceed with some questions for you so we could find out what is causing you to retain urine? V: Sure doctor. D: When did your pain start? Did it start after you failed to pass urine? V: Yes doctor. I wasn't able to pass urine since yesterday night, then I experienced severe pain in my tummy. D: Is this the first time you experienced this? V: Yes, this is the first episode. D: Do you feel any mass, or experience any heaviness or fullness in your tummy? V: No. D: Is there any burning sensations when you urinate, or is your urine dark-coloured and smelly? Are you running a fever?

V: No. D: Do you notice any bulge from down below? V: No. 20

D: Do you move your bowels regularly? Page

© 2020 ARIMGSAS

V: I move my bowels every day. o Period history D: When was your last menstrual period? V: It was 2 weeks ago. D: Are your periods regular? Do you bleed heavily? Do you experience pain or clots during your periods? Do you bleed in between periods? V: Yes my periods are regular. And I noticed that I bleed a lot during my periods and it's prolonged, and I usually get pain during my periods and sometimes see spotting in between periods. o Sexual history D: Are you sexually active? Do you have a stable partner? V: Yes, and yes. D: Do you experience pain during intercourse? V: At times doctor. D: Do you have any history of sexually transmitted infections, especially HSV? V: No. D: Any previous pregnancies or miscarriages? V: No. D: Do you use any form of contraception? V: Yes, I am on the pill. D: When was your last pap smear and what was the result? V: It was last year, normal results. o SADMA D: Do you smoke, drink alcohol or take any recreational drugs? V: No. D: Do you take any prescription or over the counter medications? V: No. D: Any previous medical and surgical illnesses? V: No. D: Thank you for that information, Vanessa, I would like to speak to my examiner, I'd get back to you shortly.

▪ Physical Exam D (to examiner): I would like to know the BMI of my patient. E: BMI is 34. D: Is there pallor, lymph node enlargements, poor skin turgor, dry mucous membranes, delayed capillary time in the patient? E: None. D: What is the BP sitting and standing, and temperature of the patient? E: BP sitting and standing is 110/70, the temperature is 36.8 D: Is there visible distention or palpable mass and tenderness in the abdomen? E: Yes, there is a mass in the mid-lower abdomen. D: Can you get to the lower border of the mass? E: No.

D: What is the consistency, mobility and tenderness of the mass? E: It is firm, mobile and non-tender. D: On inspection, are there any rash or vesicles around the vulva and vagina? Any discharge 21

or bleeding? Page

© 2020 ARIMGSAS

E: No. D: On speculum exam, is there any discharge or bleeding from the cervix? E: No. D: Per vaginal exam, is there cervical motion tenderness? E: No. D: What is the size, mobility and tenderness of the uterus? E: Size is enlarged to 14 weeks size, mobile, non-tender. D: Is there any mass or tenderness in the adnexa? E: None. D: I would like to do a urine dipstick test, urine pregnancy test, and a blood sugar level. E: UDT is negative, UPT is negative, BSL is 5. D: I would like to arrange for a full blood exam, urea, creatinine and electrolytes, blood group and Rh factor, coagulation profile, follow-up on the urine microscopy culture and sensitivity, and arrange for a transvaginal ultrasound. D: Thank you for the information examiner, I would like to go back to my patient. ▪ Diagnosis and Management D (to the patient): Vanessa, based on your history, physical exam and result of the office tests, most likely what you have is fibroids. Do you know anything about fibroids? V: No doctor, what is that? D: Fibroids is a non-cancerous growth from the muscle layer of the womb. They can vary in size from a pea to grapefruit and also vary in the rate of growth. These growths are dependent on the hormone estrogen so it can regress at the time of menopause and increase in size during pregnancy. Fibroids can develop anywhere in the uterus from deep within the muscular wall of the uterus to on the outside or inside the wall.

It is most common in women between 35-45 years. Most have no symptoms however if you do experience symptoms, it is usually due to a large fibroid. Common symptoms are similar to what you are experiencing, heavy and lengthy periods, spotting between periods, painful periods and painful intercourse, bladder problems, and a hard lump or swelling in the lower abdomen. You need to be admitted, start you on IV line and I will refer you to an OB-G specialist. The specialist will decide surgical management for you. It could be a myomectomy either by laparoscopy, hysteroscopy or open surgery, wherein only the fibroids are removed leaving the uterus. Possible complications include severe bleeding that might necessitate a hysterectomy,

recurrence of the fibroids or if you go for a pregnancy, scar rupture. The second procedure is uterine artery embolization. It is done by an interventional radiologist wherein a small catheter or

tube will be passed to one of your leg arteries under ultrasound guidance. It will proceed to the 22 Page

© 2020 ARIMGSAS

artery that supplies the fibroid. An inert material like colloid or jelly will be injected which will block the artery making the fibroid shrink. I will give you reading materials for further information. If you experience any heavy bleeding, pain, bladder or bowel problems, call for assistance in the ward. Do you have any questions at this point? V: No doctor, thank you. D: Thank you, Vanessa, I will see you next time.

23 Page

© 2020 ARIMGSAS

Acute Urinary Retention d/t HSV

You are an HMO at the ED of a major hospital, seeing a 34-year-old lady, Vanessa, complaining of severe abdominal pain. She had not passed urine since yesterday night.

TASKS 1. Focused history 2. Examination findings from the examiner 3. Relevant investigations 4. Management

APPROACH ▪ Pain questions. Offer painkiller. ▪ Can you feel the bladder per abdomen? Yes. ▪ Any vesicles or ulcers around the urethra? None. (If there is rash IN THE URETHRA, do a suprapubic catheterization NOT a Foley's catheterization) (If there is rash, vesicles, ulcers in the vulva and vagina, you CAN still do a Foley’s) ▪ Examiner, I would like to do a Foley catheterization with the consent of my patient and under all sterile precautions. And I will send a sample of urine for microscopic culture and sensitivity. I would also like to know how much urine has been drained.

▪ History o History of Present Illness • Is it the first episode of urinary retention? • Do you have any burning sensation or stinging while passing urine before this happened? • Any colour changes in the urine? • Any smelly urine? (rule out UTI as the cause of AUR) • Do you move your bowels regularly? (fecal impaction can lead to urinary retention) • Have you had any rash or vesicles around your private parts? Yes. (explore HSV) • Is this the first time? (to know if recent infection or recurrent) Yes, this is the first episode. • Any other abnormal vaginal discharge or bleed? (to rule out any other STIs) Do you have any fever? Lumps and bumps around your body especially in the groin? o Period history • When was your last menstrual period? (to see if pregnant) around 2 weeks ago • Is your period regular? Several days of the cycle? IS the bleeding mild, moderate, or severe? Several days of bleed? Any pain or clots during periods? No problem. o Sexual history

• Are you sexually active? Do you have a stable partner? Do you use any contraception? 24

Does not use condoms. Any history of STIs? Pap smear is up to date? Page

© 2020 ARIMGSAS

• Are you using any prescription and over the counter medications? (antipsychotics can cause AUR) o Any medical or surgical illness in the past? (Stroke, diabetic nephropathy, multiple sclerosis can cause AUR)

▪ Physical Exam o General appearance: Pallor, lymph node enlargements? BMI? Skin turgor, dry mucous membranes, delayed capillary refill, JVP? o Vitals: temperature, blood pressure o Abdomen: Any visible distention, visible mass? Any palpable mass or tenderness? The abdomen is soft and non-tender. o Pelvic exam with the consent of my patient and the presence of a chaperone is needed. • Inspection of the vulva and vagina, any abnormal discharge or bleed you can see? • What is the distribution of the rash and vesicles? • *Do not do a speculum exam or a per vaginal exam because it is painful, and you will be carrying the infection inside. o Office tests: I would like to do a blood sugar level.

▪ Management Most likely what you have is a herpes virus infection. It is a sexually transmitted infection. It produces urinary retention because of extreme pain. (It can also infect the nerves supplying the bladder causing paralysis of the bladder called sacral myeloradiculopathy. It can also produce a severe bladder infection causing acute urinary retention.) After the primary infection, this virus can remain dormant in your body and could get reactivated resulting in a less severe but similar infection, when your body immunity goes low. To confirm the infection, I need to take a swab and send it for PCR. Inquire about the partner, and if the partner is infected, he needs to be treated also. Refrain from sexual intercourse until the lesions heal. I need to screen you as well for other STIs with your consent. Advice about safe sex practices: use condoms along with the pills to prevent sexually transmitted infection. I will start you on acyclovir 800mg for 5 to 7 days, and painkillers like Panadeine or any strong painkillers. Use loose cotton underclothing. You can also use ice packs over the lesions to bring the pain down and dry the rash afterwards. Do not rupture the vesicles. Just in case you experience any difficulty in passing urine again, you can try to pass urine under a warm salt bath. (salt bath will decrease the pain, and the warmth will stimulate the bladder) I will give you reading materials regarding HSV for further insight and I will arrange a review with you in 2 days.

25 Page

© 2020 ARIMGSAS

Recurrent HSV in Pregnancy

The 20-week pregnant lady presenting to your GP with complains of painless ulcer in her vagina. This is her fourth episode.

TASKS 1. Further history 2. Examination findings from the examiner 3. Explain management with the patient

Differentials: ▪ Syphilis - syphilitic chancre does not recur. This differentiates the case because it is her 4th episode. Also, the syphilitic chancre is always solitary. ▪ HSV

▪ History o History of Presenting Complaint • What is the duration of the ulcer? Is it a single ulcer or multiple ulcers? Other vesicles or rash. I can other rash around. • I have read from the notes that this is the fourth episode, when did the first episode happen? Is this the first time you had the ulcers during pregnancy? Yes, this is the first episode during pregnancy. (pregnancy is a low immune state, that is why ulcers are recurring) Any treatment you have taken for the previous episodes? • Have you had any burning or stinging sensation before the onset of the ulcers? Yes. • Any other vaginal discharge or bleed? • Any fever, lumps or bumps around the body? • Are you passing urine normally? Any or pain at the time of passing urine? • Are you moving your bowels regularly? o Pregnancy • Have you had your regular antenatal checks? Blood checks, blood group? Have you taken your folic acid? Had your Down syndrome screen? Ultrasound at 18 or 20 weeks? (Arrange for an ultrasound for her now if she hasn’t had her UTZ at 18 or 20 weeks) • Have you felt your baby kick? Any tummy pains? • Are you sexually active? Have a stable partner? Any pregnancies/miscarriages before? Any history of STIs? Do you use contraception? When was your last pap smear, what was the results? • Do you some, drink alcohol or take any recreational drugs? • Any medical or surgical illness in the past?

• Take any prescription or over the counter medications? ▪ Physical Exam o General appearance: Is there pallor, lymph node enlargements, edema? 26

o Vital signs: What are the blood pressure and temperature? Page

© 2020 ARIMGSAS

o CVS/Respiratory/CNS o Abdomen • What is fundal height? 20cm. • Is the fetal heart rate appreciated? What is the rate? 140 bpm. • What are the lie and presentation? Longitudinal and cephalic. o Pelvic Exam • On inspection of the vulva and vagina, is there any abnormal discharge or bleed? • Where is the ulcer, and what is the size? Any discharge from the surface of the ulcer? How is the surrounding area of the ulcer? Any tenderness in the ulcer? What is the consistency? • Any other rash or vesicles that you can see around the ulcer? • *Don’t go for speculum and per vaginal exam o Office test: I would like to request for urine dipstick test and blood sugar level. ▪ Management I'm thinking of a condition, recurrent HSV infection. Do you know about HSV? Yes, I've heard of it before. HSV infection is sexually transmitted infections. After a primary infection, this virus can remain dormant in your system and can get activated, once your immunes system becomes low and as you know pregnancy is one of the conditions where your body immunity goes down, so that is why you're having your recurrent HSV infection now. Recurrent HSV infections as such will not cause any birth defects in the baby as the infection is less severe and your body already has got antibodies against this infection which might pass through the placenta and protect the baby. But we'll still monitor the baby for any birth defects as well. I will refer you to a high-risk pregnancy clinic where you will be seen by the obstetrician and infectious disease specialist. To confirm the infection, I need to take a swab and send it for PCR. Inquire about the partner, and if the partner is infected, he needs to be treated also. Refrain from sexual intercourse until the lesions heal. An STI screen needs to be done only if you are not with the same partner, and also if you never had an STI screen during your pregnancy. Do you know about safe sex practices? Advise if she doesn't know. I will arrange an ultrasound for you now, and if necessary, repeat ultrasounds will be done. I will start you on Acyclovir (can be given from 20 weeks of pregnancy), which you need to take for 5-7 days. I will also give you prophylactic acyclovir from 36 weeks until delivery if it is a secondary or recurrent infection to prevent recurrent episodes of HSV as it could be dangerous for the baby and could affect your delivery during the last weeks of pregnancy. (not given if primary infection because antibodies in the body are pretty high. There is no chance that she can get a recurrent infection during pregnancy. If secondary or recurrent, antibodies are low, the baby can get infected) If there are no active lesions at the time of your delivery, you can opt for normal vaginal delivery. But just in case you have active lesions, then a C-section needs to be done. I will give you reading materials about recurrent HSV during pregnancy. I will arrange a review with you in around 5 days.

(Just in case the patient delivers vaginally with active lesions, as soon as the baby is born, give IV

acyclovir to the baby. HSV in a neonate- viral sepsis, skin, eye and mouth lesions, pneumonia, herpes encephalitis. 27

Page

© 2020 ARIMGSAS

Ovarian tumor + Urinary retention

▪ History o Relieve the urinary retention. o Same history for fibroids and urinary retention. Nothing positive in history. o Rule out differentials as in fibroids.

▪ Physical Exam o General appearance: Pallor, lymph node enlargements, dehydration, BMI. o Vital signs: Temperature, blood pressure. o Abdomen: No mass per abdomen, soft and non-tender. o Pelvic exam: • Inspection of vulva and vagina, discharge, bleed, rash, vesicles • Speculum exam look at the cervix. Any rash, bleed, discharge. Nothing positive. • Per vaginal exam: is there CMT? None. (CMT positive in PID and ectopic) ▪ What is the size of the uterus? Normal size. ▪ What are the position and mobility? Tenderness? Non-tender ▪ or tenderness? You can feel an adnexal mass on the right side, no tenderness. (ovarian mass vs PID, tenderness will be there if PID) OR You can feel a mass, orange shaped mass pushing the uterus from behind (ovarian tumour) OR you cannot feel the adnexa properly. (Do a per rectal exam) o On a per rectal exam, is there any mass in the pouch of Douglas. Yes, there is a mass in the pouch of Douglas (95% it is an ovarian tumour; 5% abnormal growth of pouch of Douglas) o Office tests: I will order a BSL, UDT. ▪ Investigations o I will arrange for a full blood exam, CRP ESR, U&C, CA-125 and transvaginal ultrasound. Ultrasound confirms the ovarian tumour. ▪ Management The cause of the urinary retention is the mass from the ovary as it can push the uterus from behind which will obstruct the urinary tract. I will admit you and you need to be seen by the specialist, remained on catheter until the specialist comes and has a look. I will start you on IV, take blood for all investigations, and arrange for an ultrasound. The specialist will decide on the surgical management to remove the ovarian tumour. Further management will differ if it is a cancerous growth or not. Just in case it is a cancerous growth, then further investigations need to be done for staging and grading the tumour, and you will be dealt with by a multidisciplinary team which also has got the oncologist. Further chemo and radiotherapy will be arranged.

28 Page

© 2020 ARIMGSAS

Subfertility

27-year-old Janet is your next patient at your GP. She tells you that she had been trying to fall pregnant for more than 1 year now, but she could not, and she is quite worried about this.

TASKS 1. Appropriate history 2. Examination findings from the examiner 3. Relevant investigations 4. Management

Sub fertile - if less than 35 y/o, unprotected sexual intercourse x 12 months; if >35 y/o unprotected sexual intercourse x 6 months or more

Differential diagnosis: • Unknown • Infrequent sexual intercourse and ignorance about the fertile period • PCOS • Endometriosis • PID • Fibroids • Thyroid disorders - hyper/hypothyroid • Hyperprolactinemia • Eating disorders • Medical- DM, SLE, renal disorders • Surgical- Asherman’s syndrome • Medications- antipsychotics, spironolactone, chemotherapeutic agents • SAD - marijuana • Stress, exercise • Family history of subfertility

APPROACH ▪ History o History of Present Illness D: Hi Janet, I'm Dr. ___ your GP today. Be assured that everything will be private and confidential, and I may touch on personal and sensitive matters. So, I have read from the notes that you have been trying to get pregnant for 1 year now, how frequently do you undergo intercourse? Do you live with your partner always? J: About once a week or so cause we're not living together always cause he's working

in the state and he only comes down once a week or so. D: Do you know about your fertile period?

J: What is that doctor? 29 Page

© 2020 ARIMGSAS

D: Fertile period is the time during which you undergo intercourse, there's a high chance of pregnancy. D: When was your last menstrual period? D: Regular? The number of days of the cycle? Mild, moderate or severe? Several days of bleed? Any pain or clots during periods? J: Yes, regular. No problems with periods. D: Has your partner evaluated subfertility so far? J: No, he hasn't. D: Does your partner have any medical or surgical conditions? Any medications that he is taking? J: None. D: Sorry to ask you this, but does your partner have any children from previous relationships? J: No, he has none. D: How long have you been in a stable relationship? J: We've been together for 2 years now. o Differential Diagnosis D: Any pregnancies and miscarriages in the past? Any history of STIs? (tubal obstruction) J: None D: Use of contraception before trying for pregnancy? D: Any recent weight gain, acne, excessive hair growth? (PCOS) J: None D:Any pain during intercourse? (Endometriosis) J: None D: Any mass or lumps that you can feel in your tummy? (Fibroids) J: None D: Any abnormal, offensive vaginal discharge? Any fever, back pain? (PID) J: None D: Do you have any weather preference? How are your bowel habits? (Thyroid disorders) J: None D: Any headache, blurring of vision, milky discharge from nipples (Hyperprolactinemia) J: None D: Do you think that you are excessively overweight? Do you try to reduce your weight through crash dieting or excessive exercise? How often do you exercise? (Eating disorders) J: No. D: What is your occupation? Do you have any stress at home or work? (Stress) J: I do office work. None. D: Have you had any surgical procedures especially those done down below? (Asherman's) J: No o SADMA

D: Do you smoke, drink alcohol, or take any recreational drugs? J: None D: Do you take any prescription or over the counter medications? 30

J: No Page

© 2020 ARIMGSAS

D: Do you have a family history of subfertility? J: None D: Thank you for that information, Janet. I'd just talk to my examiner and will get back to you shortly. ▪ Physical Exam D (to examiner): What is the BMI? E: 23 D: Acne, excessive hair growth? Pallor, lymph node enlargements, edema? E: None D: What are the BP and temperature? E: 110/70, 36.8 D: Any visible distention? Any mass? Palpable mass or tenderness? E: No visible distention, no mass or tenderness D: Inspection of vulva and vagina. Any bleed, discharge? E: No bleeding or discharge D: Speculum exam, is cervix healthy? Discharge and bleed? E: No discharge or bleed from the cervix D: Per vaginal exam, is there CMT? E: None D: What is the size, position and mobility of uterus? Tenderness? E: Uterus size is normal, anteverted, mobile, no tenderness D: Adnexal mass or tenderness? E: No adnexal mass or tenderness D: I would like to do a blood sugar level E: BSL is 5. D: Thank you for that information, examiner, I would like to go back to my patient. ▪ Diagnosis and Management D: Hi again Janet, I would also like to arrange for some investigations such as FBE, U&E, TFTs, serum prolactin, mid gluteal hormone assessment (21st-day serum progesterone; if >3nanograms/mL it means the lady is ovulating), FSH, LH, estrogen, progesterone, transvaginal ultrasound. If everything turns out to be normal, we need to arrange for a hysterosalpingogram. It is done to check the patency of the tubes. A dye will be injected through the cervix, and we will look if the dye is passing through the tubes or not. After the initial work-up, I need to refer you to a specialist for further evaluation. It is always better to treat both partners together. I'd like to see your partner when you come for your next appointment. He also needs to be done in certain investigations. As far as the details you have given me and after examining you, I could not find anything wrong, except for your infrequent sexual intercourse and your ignorance regarding the fertile period. So, I'll advise you further on this. If you are planning for a pregnancy, better to undergo intercourse 2-3 times/week. To find out about your fertile period or whether you're ovulating or not, you can get ovulation kits from the pharmacy which are urine kits and you can start testing from the eleventh day of your periods. Other methods by which you can detect your ovulation is by basal body temperature method. For this you have to get a basal body thermometer from the

pharmacy, a chart will also be given, and you need to record the temperature from your armpit, every morning before you get out of bed, and record it on the chart. A rise of 0.2C for 3 days over the previous 6 days temperature will tell you that you're ovulating. Next is 31

the cervical mucus method or Billing's ovulation method. You need to observe the cervical Page

© 2020 ARIMGSAS

mucus. A cervical mucus at the time of ovulation will be thin, increased in amount and lubricated. Once ovulation is over, it changes into thick mucus. If you undergo intercourse during this time, there is a high chance that you will be pregnant. I will arrange a review with you once the results are out and we'll take it from there. I will also give you reading materials about subfertility. If you are not leaving with your partner, it may be better if you stay together.

*** If it is due to PCOS, you need to do lifestyle modifications. I will refer you to a dietician and you need a structured exercise program. You need to continue this for 6 months. If it is not working, I will refer you to a specialist who will start you on ovulation-inducing agents like clomiphene citrate and give you metformin along with that. If that still doesn't work, you can undergo a surgery we call laparoscopic ovarian drilling. We put multiple holes in the ovary through keyhole surgery and that itself can induce ovulation as it drastically brings down the testosterone or the male sex hormone production.

*** If it is due to fibroids, we need to do surgical management such as myomectomy or uterine artery embolization.

*** If it is due to endometriosis, we need to do a laparoscopic excision of endometrial deposits or we can burn away the deposits with electrocautery or laser.

*** If it is due to thyroid disorders, treat the disorder with medications.

*** If it is due to hyperprolactinemia, look for the cause, either micro or macroadenoma (<1cm micro, >1cm macro), use of dopamine antagonists like antipsychotics, or hypothyroidism. Refer to a specialist, Do serum prolactin, TFTs, high-resolution MRI. Pituitary microadenoma - treat by giving bromocriptine continued for 1 year, review every 6 months Pituitary macroadenoma - do a transsphenoidal approach to remove the macroadenoma

*** If due to Asherman's (post-surgical unexplained adhesions within the uterus, like D&C especially after septic abortion) - hysteroscopic removal of adhesions under antibiotic cover, then prevent adhesions by inserting an IUCD or Foley's catheter to keep the uterine cavity apart until healing takes place. To regrow the endometrium, we need to give you estrogen.

*** If both tubes completely blocked, your option is to do in-vitro fertilization. IVF is when fertilization occurs outside the female body. It is done by a specialist. You will be put in ovulation-inducing drugs such as clomiphene, and once the eggs are mature, it is taken out and combined with the sperm from your partner in the lab. Once embryo happens, a healthy embryo is chosen and is implanted into the uterus. If only 1 tube is completely 32

blocked, 1 tube patent, we could do a GIFT procedure. It is a gamete intrafallopian tube Page

© 2020 ARIMGSAS

transfer. Fertilization takes place inside the female body. Introduce a healthy egg and a sperm into the patent tube and allow fertilization to take place in the tube. Once the embryo is formed, it will be implanted in the uterus. We could also do a ZIFT procedure. IT is a zygote intrafallopian transfer. Here fertilization happens outside the female body. And once the embryo is formed, it is introduced into the patent tube. Then it will travel down the tube and get implanted in the uterus.

33 Page

© 2020 ARIMGSAS

Preterm Labor

A 28-year-old primigravida presents to you at 30wks of gestation with sudden onset of abdominal pain. You are a GP in a rural practice about 300km away from a hospital with O&G facility.

TASKS 1. A relevant and focused history 2. Examination findings from the examiner 3. Discuss management with patient

APPROACH ▪ History o History of Present Illness *Check for hemodynamic stability. D: Is my patient hemodynamically stable? What is the BP, pulse, RR, temp and sats of my patient? E: You may proceed with the examination. D: Okay, thank you. D (to the patient): Hi Lisa, I'm Dr. ___ your GP today. I understand that you suddenly experienced abdominal pain. How bad is your pain? L: It is 8/9 D: I would like to give a painkiller if you don't have allergies. L: None D: How long have you been having the pain? D: Any trauma to the tummy? D: Where is the site of pain? All over the tummy. D: Does the pain go anywhere? It goes to my legs, thighs, back D: What is the character of the pain? Continuous or on/off the pain? It is an on/off the pain (Abruptio is continuous pain) D: Is the pain coming at regular intervals? Do you think that the intervals are coming shorter? Is pain duration coming longer? Coming regularly, shorter intervals, pain coming longer. D: Is anything making it better or worse? Nothing. o Pregnancy D: Any associated symptoms like bleeding or discharge from down below? D: Is the baby kicking well? D: Any burning or stinging while passing urine? D: Are you running a temperature? Any recent infections? Any rash? D: Regular antenatal checks? Blood checks, blood group? Folic acid? Sweet drink test? D: Down screen, UTZ at 18/20 weeks? Is it a single baby? Position of the placenta? Sweet drink test at 28 weeks?

D: Any associated headache, blurring of vision? Any swelling or edema? (pre- 34 eclampsia)

D: When was your last pap smear? Page

© 2020 ARIMGSAS

o SADMA D: Do you smoke, drink alcohol, or take recreational drugs? o Social history D: Do you have good support? ▪ Physical Exam D: Is there pallor, edema, lymph node enlargements? D: What is the BP, temperature? D: What is the fundal height? 30cm D: FHR? 150bpm D: Lie, presentation? Longitudinal, cephalic D: Can you feel uterine contractions per abdomen? How frequent are the contractions coming? How long do the contractions last? Contractions happening 3-5minutes, lasting 30seconds to 1 minute. D: Is there uterine tenderness? None D: Is the baby's head engaged? Not yet D: Inspection of the vulva and vagina, any discharge or bleed, rash or vesicles? D: Sterile speculum exam, any bleed or discharge from the cervix? Cervical os dilatation? Dilated 3cm **Don't do a per vaginal exam. You are at a GP clinic, you may rupture the BOW. **If you are at a hospital, do a per vaginal exam under specialist supervision. Do Bishop's scoring. D: I would like to take a vaginal swab and give it for fibronectin test. Fibronectin is the test to look for preterm labour. If it is positive, she might deliver in the next 7-10 days. If it is negative, she will not deliver in the next 7-10 days. ▪ Diagnosis and Management D: What you may be having is preterm labour. Normally labor happens in and around 40 weeks, but if it happens before 37 weeks, that is called preterm labor. It can be unknown such as with you, but anything that over distends the uterus such as excessive fluid in the bag surrounding the baby, multiple pregnancy, cervical incompetence or when the cervix is weak and cannot hold the baby inside, or maternal infections, other maternal conditions such as diabetes and preeclampsia which is a sharp rise in blood pressure with leakage of proteins to urine. can also predispose to preterm labor. She needs a referral to a tertiary hospital with a neonatal intensive care unit. I'll arrange an ambulance for you, I'll ring up the hospital and make them aware of your condition so that all arrangements will be made once you reach the hospital. I will start you on an IV line, take blood for investigations like FBE, ESR, CRP, UCE, blood group and Rh factor. I will give you your 1st dose of steroid (betamethasone), to bring about lung maturity in the baby, (if less than 34 weeks) (2 doses at 12-24 hours interval), and 1st dose of tocolytic. Tocolytic is the medication given to prevent further uterine contractions. (Nifedipine, salbutamol, ritodrine, atosiban) Once you reach the hospital, you will be admitted, seen by the specialist, ultrasound and a CTG will be done to look for the wellbeing of you and the baby. The specialist will try to prolong your pregnancy as much as possible. They will give you further doses of tocolytics, and further dose of steroid. In spite, if your labor progresses and you deliver, the baby will

be taken care of by the team at the neonatal intensive care unit.

What are the complications can the baby have if I deliver at this week of pregnancy? 35

i. RDS or breathing difficulties in the baby Page

© 2020 ARIMGSAS

ii. Neonatal sepsis or infections iii. Bleeding into the brain or intraventricular hemorrhage iv. Difficulty in maintaining body temperature and sugar levels v. Feeding difficulties

Do you need me to call your partner to be with you during this time?

Criteria for identifying preterm labor • Contractions happening every 5 - 10 minutes lasting for 30 seconds to 1 minute • Cervix > 2.5cm dilated • Fibronectin test is positive • Labor is happening before 37 completed weeks

36 Page

© 2020 ARIMGSAS

PPROM

25-year-old Mary, who is 32 weeks pregnant, presents to you at your GP clinic with complaints of passing fluid from the vagina since the past 1 hour.

TASKS 1. Relevant history 2. Examination findings from the examiner 3. Management

APPROACH ▪ History o What is the colour of the fluid? (meconium-stained - fetal distress) It's just like water o Is it smelly? How much fluid is leaking? Several pads soaked? o Any abnormal vaginal discharge before this happened? o Baby kicking well or not? o Regular antenatal checks? Blood group? Folic acid? Down syndrome screening? Ultrasound at 18 weeks? Sweet drink test at 28 weeks? ▪ Physical Exam o Abdomen: • Fundal height? 32 weeks (not a massive fluid has been lost. Because is a lot of fluid, smaller fundal height) • FHR? Lie? Presentation? 145, Longitudinal, cephalic • Are there uterine contractions per abdomen? • Is the head engaged or not? o Pelvic exam: • Inspection of the vulva and vagina: fluid leaking out of the vagina? Trickling of the fluid or gush of fluid that you can see? Color of the fluid? Smelly or not? Any other abnormal bleed or discharge that you can see? Any rash or vesicles? • Sterile speculum exam - can you see the watery discharge on the cervix? (if there is discharge from cervix, it invariably tells you that it is coming from the uterus) ▪ Is the OS open or closed? ▪ Can you see fluid collecting in the posterior fornix? (If yes, it is amniotic fluid) Yes, you can see fluid in the posterior fornix ▪ I'd like to take a sample of this fluid and send it for amnisure test or nitrazine test (to confirm if amniotic fluid) ▪ I'd like to take a vaginal swab and a cervical swab and give it for microscopic culture and sensitivity. I'd like to take a vaginal swab and an anorectal swab for GBS. • Do not do a per vaginal exam.

o Office tests: I'd like to do a UDT and BSL. ▪ Management Normally when labor sets in, around 40 weeks, it is pain that happens first and it is then followed 37

by the rupture of the membranes. But if the membranes rupture, before pain sets in, it is called Page

© 2020 ARIMGSAS

premature rupture of membranes or PROM, and if PROM happens before 37 weeks, it is called PPROM or preterm premature rupture of membranes. So, from the details that you have given me, I could not find a cause of why you ended up with a PROM. But anything that over distends the uterus can lead to PROM like polyhydramnios, multiple pregnancy, cervical incompetence, maternal infections, maternal conditions like gestational diabetes and preeclampsia. One of the complications that happen in PROM is infection. WE need to be careful about this. You need to be referred to a tertiary hospital with a neonatal intensive care unit. I'll arrange for an ambulance and call the hospital and liaise with the ED. I need to start you on an IV line with a slow IV drip, and take blood for certain investigations like FBE, ESR, CRP, UEC, Blood group, blood sugar level, coagulation profile. I will also send urine for microscopic culture and sensitivity. I will give you your 1st dose of steroid, to bring about the lung maturity of the baby just in case you progress into labor. I will also give you your first dose of antibiotic, erythromycin (also has tocolytic effect), and you need to be continued on it for 10 days. (if patient is transferred to a far hospital, give tocolytic. If not, don’t give tocolytic.) Once you reach the hospital, you will be admitted, seen by the specialist, who will do an ultrasound and CTG. Ultrasound will be repeated weekly and CTG every 2nd day. Bloods will be done every 2-3 days. If no signs of infection, labor will be induced at 36 completed weeks. Just in case she has any signs of infection like fever, blood counts going high, or baby becomes unwell, delivery will be planned immediately either by induction or C-section.

Do you have support? Anybody, to phone in to be with you? I can give you reading materials that you can read on the way. I'll arrange a review with you once you are out of the hospital.

*** 36 weeks pregnant, cervical suture in place, presenting with PROM. *usually remove cervical suture at 37-38 weeks But in this case, in the GP do all blood and swabs, don't give antibiotics. In the hospital consider the removal of cervical suture (risk for uterine rupture if cervix remains tight) and sent for microscopic culture and sensitivity. If labor does not set in, induce labor by 36 completed weeks if no infection. Once the suture is taken out and sent for microscopic C&S, give her antibiotics.

38 Page

© 2020 ARIMGSAS

Ectopic Pregnancy

You are an HMO at the ED of a major hospital, when a 28-year-old female presents to you with severe right-sided abdominal pain since the past 1 hour.

TASKS 1. Relevant history 2. Examination findings from the examiner 3. Investigations with the examiner 4. Diagnosis with patient 5. Management with patient

Never miss an ectopic pregnancy for a reproductive age female who is presenting with one-sided abdominal pain.

Differential Diagnosis ▪ Ectopic ▪ PID ▪ Mittelschmerz ▪ Ovarian cyst torsion/rupture ▪ UTI- pyelonephritis, renal calculi ▪ Abdominal emergencies - acute appendicitis, acute bowel

APPROACH ▪ History o Ask the name of the patient. o In a scale of 1 to 10, 10 is maximum, where is your pain? It's like 7/8 I would like to arrange a painkiller for you if you have no allergies. o Pain questions: • Where exactly are you feeling the pain? Right lower part of the tummy • What were you doing when the pain started? • What type of pain? Continuous or on/off pain? Cramping sort of pain • Any radiation of pain? Going to legs, to the back, going to the back passage or rectum (levatory sign) • Anything that makes the pain better or worse? (Mittleschmerz - follicles rupture, some blood and fluids leak in the peritoneal cavity which irritates the peritoneum. Movement relieves the symptoms because it is not dependent on the peritoneum anymore) • Any nausea, vomiting, fever? Any dysuria, chills, rigor? Any bowel problems? • Any abnormal vaginal bleed or discharge? (Prune juice bleed - ectopic pregnancy; in 10-

15% vaginal bleed is absent) o When is your last menstrual period? 6 weeks ago. o Is it regular? Yes. Any problems with your periods otherwise? No issues with my periods so 39

far. Page

© 2020 ARIMGSAS

o Are you sexually active? Stable relationship? Were you looking forward to a pregnancy? We've been planning for a pregnancy for the past 6 months. Any pregnancy tests that you have done? No, I haven't done any so far. Do you experience any early signs of pregnancy such as breast tenderness, morning sickness (in ectopic, absent or very minimal in ectopic pregnancy)? o Do you use contraception? How long have you been off your contraception? Pap smear and result? Any history of STIs? Past pregnancies and miscarriages? o Other medical and surgical illness? o Prescription and over the counter drugs? o Do you smoke, drink alcohol or take recreational drugs? ▪ Physical Exam o General appearance: What is the BMI? Is there pallor, lymph node enlargements, edema, dehydration? o Vital signs: What are the BP and temperature? All stable (if ectopic, it has not ruptured since vitals are stable) o CVS/Respiratory/CNS o Abdomen: Any visible distention, any mass? Palpable mass or tenderness? There is definite tenderness in the right iliac fossa • Any guarding or rigidity? Guarding on the right lower area • Any McBurney's point tenderness? (Appendicitis) • Any Rovsing's sign? • Any renal angle tenderness? (Pyelonephritis) • Auscultate for bowel sounds (rule out bowel obstruction) o Pelvic exam: • Inspection of the vulva and vagina - any abnormal offensive vaginal discharge? Any bleeding? Rash, vesicles? speculum - is the cervix healthy? Any discharge/bleed? • Per vaginal examination - is CMT positive? Yes, it is positive. ▪ What is the size of the uterus? Uterine size is normal. Is there tenderness? ▪ Adnexal mass or tenderness? Definite tenderness in the right adnexa +/- mass o Office test: I'd like to do an UPT - could be negative in the minority of patients, or be weakly positive (because B-Hcg is very low to be detected, BSL, UDT ▪ Investigations FBE, ESR, CRP, UEC, B-HCG, blood group and Rh typing, coagulation profile, and transvaginal ultrasound - pick up an ectopic sac and reactive fluid in the pouch of Douglas ▪ Management I'm sorry to tell you, the news I have to tell you is not very good. I know you and your partner are looking forward to this pregnancy. Do you want your partner to be with you while I discuss with you this? The pregnancy that you have is called an ectopic pregnancy. Normally the egg from you and the sperm from your partner fuses in the tube and once the embryo is formed, it travels down the tube and gets implanted in the womb of the uterus. And then it goes to full-blown pregnancy. But an ectopic pregnancy, due to certain reasons, that impair the mobility of the tube, the embryo can get implanted within the tube, and that is called an ectopic pregnancy. Here the tube cannot distend beyond a certain extent, and as the embryo

grows, the tube can give way or rupture. This is a life-threatening condition and we need to prevent this from happening, You need to be admitted and will be seen by the specialist. I will start you on an IV line, take blood 40

for the investigations, start an IV fluid, and arrange for a transvaginal ultrasound immediately. Page

© 2020 ARIMGSAS

Once confirmed, there are two methods of management for ectopic pregnancy: conservative management and surgical management. For conservative management, certain criteria need to be fulfilled, B-Hcg < 5000, and no fetal cardiac activity. Methotrexate can be given by single intramuscular or multiple intramuscular injections or it can be given directly into the sac hysteroscopically under ultrasound guidance. Methotrexate is a cytotoxic medication which can destroy the cells and brings about resorption of the embryo. Surgical management is 3 types: laparoscopy + salpingostomy where a linear cut is made in the tube over the ectopic and the ectopic is taken out and the tube stitched back; laparoscopy + segmental resection where the portion of the tube having the ectopic is removed and the cut ends are joined together; laparoscopy + salpingectomy where the tube having the ectopic is removed altogether. If the tube is beyond repair, then we go for salpingectomy.

What is my chance of getting ectopic in my next pregnancy? Around 20%. Follow-up after treatment of ectopic? If giving methotrexate, is by doing serial B-Hcg estimate weekly; if it declines it is working, if it remains the same, there are still ectopic activity going on, give repeat Methotrexate injection, if it is going high, treatment is not effective, she has to be operated on.

Risk factors for Ectopic Pregnancy ▪ Previous ectopic pregnancy ▪ PID ▪ Any abdominal surgeries ▪ Artificial methods of conception esp. GIFT and ZIFT

Right iliac fossa tenderness, +/- vaginal bleed, CMT positive, uterus empty, definite adnexal tenderness, +/- mass = findings that tell you it is an ectopic pregnancy If PID, same abdominal findings but with fever, with abnormal offensive vaginal discharge

RUPTURED ECTOPIC Generalized, excruciating abdominal pain due to peritonitis +/- shoulder pain d/t irritation of the diaphragm Hemodynamically unstable - you'd like to go for a DR ABCDE protocol, shift the patient to the treatment room, start her on oxygen, put in 2 large bore IV cannulas, and start infusing IV fluids, call for help A board like the rigidity of the abdomen Inspection of the vulva and vagina, don't go any further because speculum and per vaginal can aggravate the shock - the majority of the patients there will be bleed Shift her to the operating theatre immediately, arrange for an immediate ultrasound, blood group, cross-matching, B-Hcg, and all other basic blood Needs to be seen by the specialist immediately, do laparotomy and salpingectomy

41 Page

© 2020 ARIMGSAS

Pre-eclamptic Toxemia

32-year-old Maria, who is 32 weeks pregnant, presents to your GP, with a headache since the last 2 days. She had regularly done her antenatal checks with you and a week before when you saw her, she had mild swelling of her legs. At that time all relevant investigations were done, and they were all normal.

TASKS 1. Relevant history. 2. Examination findings from the examiner. 3. Explain the diagnosis to the patient. 4. Management.

APPROACH ▪ History: o Headache history: • How long have you been having a headache? Sudden or gradual? • How bad is your pain? Offer painkiller • Where is the site of the pain? Pain all over the forehead • What were you doing when the pain occurred? • Pain around the neck or the shoulders? (Tension headache) • What type of pain? (band-like: tension, pulsating: migraine) dull pain over the forehead going to head • Anything that makes it better or worse? Getting worse with exposure to light or noise? • Any associated symptoms like fever, rash, neck stiffness? Any previous infections in- ear, teeth or sinuses? History of head injury? • Any medications that she has taken? • How is the swelling now? It has worsened, now has come up to the knees • Signs of imminent eclampsia: the blurring of vision, vomiting, dizziness, confusion? o Pregnancy • Is the baby kicking? • Any discharge or bleeding from down below? • Waterworks? Are you going to the loo as often as before? No, I'm not going to the loo as often as before • How are your bowels? o Any past history of migraine? o Any high blood pressure before your pregnancy? o Any family history of migraine or high blood pressure? ▪ Physical Exam

o General appearance: pallor, edema up to the level of the knees with facial puffiness • Any dehydration? Cyanosis, jaundice?

o Vitals: BP 180/100 42 Page

© 2020 ARIMGSAS

• I'd like to shift her to the resuscitation room, put in a large-bore IV cannula just in case she starts fitting and I can't access the vein at that time, and then proceed to the rest of my examination. o CVS: S1, S2, murmurs o R/S: air entry, adventitious sounds o CNS: tone, reflexes there is hyperreflexia and clonus o Fundoscopy to look for papilledema mild papilledema o Abdomen: • Fundal height 32cm • FHR? 150bpm • Lie, presentation? Longitudinal, cephalic • Hepatic tenderness? • Uterine tenderness (abruptio placentae) o Pelvic exam • Inspection of vulva and vagina, any discharge or bleed, rash or vesicles • Speculum exam: any bleed or discharge? Is the OS closed or open? o Office tests: UDT look for urinary proteins 3+ (in GH or CH, there are no urinary proteins), BSL ▪ Management Most likely you are having pre-eclamptic toxemia. It is a condition where there is a sharp rise in your blood pressure with massive swelling and leakage of proteins in the urine. The exact cause is unknown but anything that decreases the blood supply to the placenta can cause the placenta to secrete certain chemicals which could damage the lining of the blood vessels of all major organs. Pre-eclampsia that is not controlled can result in eclampsia which could be life-threatening to the baby. You need an immediate referral to the tertiary hospital. I would arrange an ambulance for you, and I will ring up the hospital and make them aware of your condition so they can set up everything for when you arrive. But before you go, I need to take blood for investigations such as FBE, ESR, CRP, UEC, LFTs, coagulation profile. I will give you the first dose of IV antihypertensive (IV labetalol) and then you can be transferred to the hospital. Once you reach the hospital, you will be admitted and seen by the specialist. UTZ and CTG will be done to monitor her and the baby. You have to take absolute bed rest. Continuous monitoring of the vitals, BP recording every 2 hourly, urine protein twice daily, fluid input output. Antihypertensive medications: IV hydralazine and IV magnesium sulfate to prevent eclampsia. 4g initially over 10-15 minutes, and then 1g/hour as continuous infusion. If everything is well-controlled with you and the baby is also doing well, you will be induced labor by 37 weeks. But you need to be in the hospital for the rest of your pregnancy. Just in case the baby becomes unwell or your condition becomes uncontrolled, an immediate delivery will be planned either by C-section or induction.

37 weeks pregnant: same management; but plan her delivery immediately. Because delivery is the treatment of choice for pre-eclampsia and eclampsia

While taking the examination findings, the patient will start fitting. The patient has started fitting. I would like to shift to the DR ABCDE protocol now. Call for help. Put her in a side-lying position. Start her on oxygen by mask, 6-8 liters per minute. Give her IV diazepam and IV labetalol. (but if in

a hospital, give her IV MgSo4 + IV hydralazine) Now I need to re-assess all the systems, and I need to refer her immediately to a tertiary hospital, seen by the specialist, UTZ and CTG done, all the blood investigations, IV MgSO4 and IV 43

hydralazine to control her condition. And once eclampsia has happened, the baby has to be Page

© 2020 ARIMGSAS

delivered no matter the AOG is. Never give ergometrine in the 3rd stage of labor in eclampsia because you will induce vasoconstriction and increase BP.

MILD MODERATE SEVERE

Diastolic BP 90-99 100-109 >= 110

Systolic BP 140-149 150-159 >=160

Proteinuria 1+ 2+ 3+/more

Edema minimal up to the level of the calf Massive

Management Can be managed at Requires admission to the DRSABCDE. home hospital. Secure airway, Oxygen by Bed rest She needs to be seen by the mask, I/V line and blood for Salt and protein- specialist. investigations. restricted diet. Bed rest with toilet privileges MgSO4- 4 gm bolus, then Review by GP every Blood pressure recorded 4th 1-2gm infusion/hour at 2nd day. hourly, urine protein twice least for 24 hours after last Red flags. daily, fluid input and output seizure. If seizure recurs, If not controlled, chart, UTZ, CTG give 2 gm bolus. referral to hospital. Oral antihypertensives: first If MgSo4 not available, I/V line is labetalol: diazepam 2mg/mL, max 200mg(o)/hour, max 3 doses, 10mg. then 100-400 mg bd. Hydralazine I/V. Methyldopa, nifedipine Catheterize, Fluid intake and output chart. Once her condition is under Immediate referral to a control, and the baby is doing tertiary hospital. well, you can discharge her CTG, U/S. but refer her to a high-risk pregnancy clinic

If already at 37 weeks, plan delivery by induction or C- section. No need to wait any longer

44 Page

© 2020 ARIMGSAS

Lichen sclerosis

Michelle, a 54-year-old lady is you next patient at your GP. She complain of severe itching in her vulva for the past 2 months.

TASKS 1. Further relevant history 2. Examination findings from the examiner 3. Investigations 4. Management

Differential Diagnoses ▪ Atrophic vaginitis ▪ Infections o Candidiasis o Lichen sclerosis ▪ Skin conditions o Dermatitis o Eczema o Psoriasis o Skin allergies due to cosmetics or undergarments ▪ Diabetes ▪ Steroids

Key investigation: multiple punch biopsy to rule out a pre-cancerous condition

Lichen sclerosis will never infect the vagina It is a white shiny wrinkled plaque

APPROACH ▪ History of the present complaint o Duration? (past 2 months) o Continuous or on and off itching? (continuous, worsening) o Started interfering with lifestyle? o Sleep and itching at the time of sleep? o Do you have any vaginal discharge? (candidiasis has cheese-like discharge) o Any associated bleed? o Any rash, ulcers, or vesicles around the area? ▪ Differential questions o Atrophic vaginitis: do you still have your periods? Or have you gone through menopause?

Age of menopause? Hot flashes and other menopausal symptoms? Do you feel that your vagina is dry and thin? Any burning sensation in your vagina? o Sexual activity questions: are you sexually active? Do you have stable partner? any painful 45

intercourse? Post-coital bleed? Have you done your pap smear and mammogram? What Page

© 2020 ARIMGSAS

were the results? Any history of sexually transmitted infections? How many children have you had? o Skin allergies: Any other skin conditions that you have, like dermatitis or psoriasis? Any allergies? Have you changed your cosmetics lately? Any use of douches or pessaries? o Any history of medical or surgical illness, especially diabetes? o Are you using any medications, especially steroids? o How are your waterworks and bowel movements? Is there any stinging or burning sensation while passing urine? ▪ Physical exam o BMI, pallor, lymph node enlargements, generalized rash, eczematic or psoriatic patch? o Vital signs: temperature, BP with postural drop, HR with rhythm, RR with saturation o Systemic examination • CVS • R/S • CNS • Abdomen ▪ Visible distention, any mass? ▪ Palpate for any mass, tenderness (soft and non-tender) ▪ Pelvic exam (with the consent of the patient and presence of a chaperone) • Inspection of the vulva and vagina: visible discharge, any bleed? Does the vagina appear thin or atrophic? Any other skin lesions? (white, shiny, wrinkled plaques in the vulva and perivaginal areas in a laced-like pattern) • Any discharge in the plaque? • Inspection of the anal area: is there any lesions, excoriation marks, plaques? • Speculum exam: is the cervix healthy or not? Any discharge or bleed? • Per vaginal exam: uterus size and tenderness (normal, no tenderness), adnexal mass and tenderness o Office tests: UDT, BSL (rule out diabetes) ▪ Investigations o FBE, UEC, key investigation: multiple punch biopsy, TFT (rule out autoimmune thyroiditis) ▪ Management From the history and examination, most likely you have a condition called lichen sclerosis. It is a chronic inflammatory skin condition. The exact cause is unknown but thought to be linked to the autoimmune system. The immune system of your body usually protects the body against infections, but in autoimmune conditions, the system can get confused, and it starts attacking your own body cells. And that is one of the postulated theories for lichen sclerosis. This usually presents with severe itching and causes white, wrinkled plaques in your genital area. It can result in scar formation and it can join up with the surrounding genital skin leading to adhesions. And one of the most common complication because of the adhesions is labial fusion. 4% of the lichen sclerosis can turn nasty or be pre-cancerous. Because this is an autoimmune condition, there is no permanent cure for lichen sclerosis. But of course, we can keep the condition under control. Management is with steroid creams which you need to apply twice daily for the first one month

then once daily for the second month and then depending upon your response, you can decrease the strength and number of applications. But the key point: she needs to be put on a maintenance

therapy (even if the lesions gets controlled), of lifelong 1-2 applications per week. If not 46 Page

© 2020 ARIMGSAS

responding to steroids, she will be given retinoids or ultraviolet therapy. She needs to be seen by the specialist because the multiple punch biopsy needs to be done. When do you go for surgery? If there is scar formation or adhesions, and also if there is any malignant change. Meanwhile, good genital hygiene should be followed, and she needs to be on lifelong surveillance because of the malignant change that we are anticipating, to begin with, 6 monthly intervals, and then annually. Make sure that she takes her mammogram at the correct time. Red flags: in case you experience any bleeding, abnormal discharge, or if the itching is becoming worse, please report back. I will arrange a review with you in around 1-week time and see how you are doing. I will give you reading materials for lichen sclerosis. Is it due to menopause? Lichen sclerosis has nothing to do with menopause. It is an autoimmune condition, not associated with any hormones. It can happen at any age, not only in menopause, but it can also happen in young children.

For the exam, the role-player will tell you that she's been having this for one year. She has seen some doctors who have prescribed her with creams which are not helping. (estrogen cream is for atrophic vaginitis) Go and delve on the estrogen creams they are using.

Note the white, shiny plaque

47 Page

© 2020 ARIMGSAS

Uterovaginal prolapse

56-year-old Maya comes to your GP complaining of a feeling of something bulging from down below. She appears really concerned about this.

TASKS 1. Focused relevant history 2. Examination findings from the examiner 3. Management

APPROACH • History o How long have you been having this? (last 1 year) o Is it there always, or just at times? (assess the degree of prolapse. If it is always there, it is the 3rd degree) (it is just there at times) o When do you have the feeling of the bulge? (when she stands for a long time) o What relieves you of the feeling? (when she lies down, she can feel the bulge going inside) (probable 2nd-degree prolapse) o Associated symptoms: bleeding, discharge, itching, rash? (prolapse can get infected) o Causes of prolapse: • Chronic cough: any chronic cough? Do you smoke? How long have you been smoking? How many sticks per day? Alcohol drinking? • Constipation: ask about waterworks as well (can be associated with a or , can have urinary retention) any problems with passing urine? Any incontinence of urine? Is there any burning or stinging during ? Any history of constipation? Do you open your bowels regularly? • Pregnancy history: how many pregnancies have you had? (3 pregnancies and 3 deliveries), any complications during the pregnancy? Were all your deliveries vaginal? Any C-sections? What is the reason for the C-section? Any history of big babies? Any obstructed labour? Any prolonged labour? Any tears or lacerations that you had at the time of delivery? Any instrumental deliveries? (contributing factors to pelvic floor muscle weakness) • Menopause: period history: have you had your menopause? At what age did you have your menopause? Any menopausal symptoms like hot flashes, heavy sweating, mood changes? Rule out an estrogen deficiency: are you on any hormones or HRT? Any bleed after menopause? • Sexual history: are you sexually active? Are you in a stable relationship? Any problems with intercourse? Any bleed after menopause especially after intercourse? When was your last pap smear and mammogram? What was the result?

• Any other medical or surgical illness? • Physical Examination o General appearance: BMI (35, obese - can lead to weakness of the muscles), pallor 48

o Vital signs Page

© 2020 ARIMGSAS

o Systemic exam • Respiratory system (chronic cough is a cause): air entry, adventitious sounds • CVS: normal S1 and S2, murmurs • Abdomen ▪ Visible mass, distention? ▪ Palpate for mass and tenderness (abdomen is soft and non-tender) • Pelvic exam ▪ Inspection of the vulva and vagina: any discharge/bleed? Any prolapse? Any thin, dry atrophic vagina? ▪ Speculum exam: is it healthy? Any discharge or bleed? • Key point: ask the patient to cough or bear down • Ask if there is a prolapse (cervix is coming up to the level of the introitus) • Identify the degree of prolapse • First degree: remains inside the vagina • Second degree: cervix comes at or near the introitus • Third degree: most of the uterus and cervix lie outside the vagina • Ask for any incontinence that you can see (pelvic floor muscle weakness can result in prolapse or incontinence) *always rule out incontinence in prolapse, always rule out prolapse in incontinence ▪ Bimanual exam • Uterine size (size is normal), tenderness • Adnexal mass or tenderness o Office test: UDT, BSL • Management The condition you are having is a uterovaginal prolapse. It is a common condition where the cervix or the uterus or both bulge into the vagina. For you, it is a second-degree prolapse as it is not coming outside the vagina. The cause of a uterovaginal prolapse is due to the weakness of the pelvic floor muscles and ligaments, that support the pelvic structures like the uterus and the cervix. There could be a lot of reasons for pelvic muscle floor weakness, like chronic cough, constipation, but as far as your case goes, I can see that you are having a bad obstetric history. So, at the time of pregnancies, due to the extra weight gain, and the hormonal changes, these muscles can become weak, and at the time of labour, when you push or strain, these muscles become weaker. One of the deliveries that you had is a big baby, that is why you had prolonged labour due to that, and that might have contributed to you prolapse.

Why didn't I have it then? Why am I presenting with the prolapse now? The contributing factors for prolapse are menopause and the extra weight gain. At the time of menopause, as the ovaries shut down, estrogen is formed in very low levels. And this estrogen is necessary, to maintain the strength and the stability of the pelvic floor muscles.

You need to be referred to a specialist. Start with lifestyle modifications (key point). Your BMI is a bit concerning, so I would like to refer you to a dietician who could give proper advice regarding your diet. You need to have regular exercise and maintain the BMI within the ideal range.

You need to also do pelvic muscle exercises. I can refer you to a physiotherapist who could advise you about this. 49

How do you do the exercise? Page

© 2020 ARIMGSAS

You can do the exercise in a sitting, standing or lying down position. Contract your bottom muscles, for a count of 8, relax it for the same amount of time that you have contracted the muscles, or count of 8 also, and do it 8 times at a go, three times a day.

How long does the pelvic exercise take to work? Usually, it takes around 3-6 months.

Another option is a vaginal pessary. It will help to keep the prolapse in place and prevent it from getting worse. But it is not a definitive treatment of prolapse. You need to change it every 3-4 months. Surgical management/option is considered if the conservative management fails and if the symptoms become worse. One is the repair procedure and it is of two types. One is called colporrhaphy where you reinforce the pelvic floor muscles by using stitches. Another type is you put in a synthetic graft thereby strengthening the pelvic floor muscles. The next procedure is a sacrocolpopexy. This is where you re-suspend the vagina, cervix and other pelvic organs and secure it to the backbone or the sacrum. The last option is a total hysterectomy.

***Another case is the patient has undergone hysterectomy and has prolapse: vaginal vault prolapse. Causes of vaginal vault prolapse: The weakening of the suspension: due to extra weight gain, chronic cough, constipation Management: Referral to the specialist, lifestyle modifications (usually BMI will be high), Pelvic floor exercises, vaginal pessary (but not a definitive treatment), Surgery: sacrocolpopexy - resuspend the vagina and secure that to the sacrum

50 Page

© 2020 ARIMGSAS

Osteoporosis

Angela, 64 years presents to your GP saying that she thinks that her height is becoming less. She is a known hypertensive well controlled on Coversyl (perindopril). She lives alone.

TASKS 1. Further history 2. Examination findings from the examiner 3. Management

Differential Diagnosis ▪ Trauma to the spine (can compress the vertebrae and cause a reduction in the height) ▪ Secondary deficits in the spine ▪ Multiple myeloma ▪ Osteoporosis

APPROACH ▪ History of present complaint o How long have you been noticing that your height has become less? (1 year) o Apart from you, has anybody else noticed that your height is becoming less? (her children have also told her the same) ▪ Differential Diagnosis o Any history of trauma? Any injury to your back? (no history of trauma) o Secondary deficits/multiple myeloma: Any recent weight loss? Any lumps or bumps around the body? o Any pain or limitation of movements of the spine? Any weakness or numbness that you are feeling, especially in the lower limbs? o How are your bladder and bowel habits? o Osteoporosis: • Causes: ▪ Diet: do you take dairy products? (poor in taking dairy products) ▪ Exercise: do you have a regular exercise? (does not exercise much) ▪ Smoking, alcohol, recreational drugs: greater than 2 standard drinks/day, coffee greater than 4 cups per day can cause osteoporosis (takes around 8 cups of coffee per day) ▪ Sunlight: do you expose yourself to sunlight quite often? (I have no idea about that) ▪ Medical and surgical illness especially diabetes, thyroid disorders (thyrotoxicosis, hyperthyroidism), celiac disease (is she on any special diet?), kidney disorders ▪ Medications such as steroids, anticonvulsants ▪ Menopause • Period history: when did you have your menopause? Any symptoms of

menopause like hot flashes, sweating, mood changes (if she is 51 Page

© 2020 ARIMGSAS

menopause, that would tell you that her estrogen is deficient), does she use HRT? (no, she did not take because she's worried about side effects) • Obstetric history: number of pregnancies that she had? ▪ Family history: any history of fractures in her mom or her sister? ▪ Physical examination o General appearance: BMI (a low BMI is associated with osteoporosis. Less than 19 can contribute to osteoporosis) (her BMI is 19) • Ask whether the height is less in comparison to age and weight? (yes) • Pallor, lymph node enlargements o Vital signs o Systemic exam • CVS • R/S • CNS (because if their vertebral collapse, it can affect the nerves leading to numbness, weakness and bladder and bowel symptoms) • Abdomen (normal routine) • Musculoskeletal system - spine ▪ Look for any deformities (there's a bit of kyphosis) ▪ Any localized tenderness and range of motion of the spine ▪ Management This could be due to a condition called osteoporosis. It is a condition wherein your bones become thin and cause your bones to become porous, and there is an interruption in the mobilization of calcium from the bones to the blood, so the bones become soft and becomes more prone to fractures. The main factor that results to osteoporosis is menopause. At the time of menopause, the estrogen levels go low, and this estrogen is necessary to maintain the strength of your bones. As far as you go, the other contributing factors could be, a lack of calcium in your diet, and also lack of exercise. First what we need to do is to confirm that you are having osteoporosis by doing some blood investigations like a FBE, UEC, BSL (to rule out DM), TFT (rule out hyperthyroidism), LFT and RFT (just in case she needs to be put on bisphosphonates and other supplements), 25-hydroxy vitamin D (key investigation), an x-ray of the spine (to look for fractures), DEXA scan/bone density scan (key investigation) (can be done in the femur or spine). [2 scores in DEXA: T-score and Z-score. T- scores less than or equal to -2.5 = osteoporosis; between -1 to -2.5 = osteopenia; greater than or equal to -1 = normal]

Key management: a. Lifestyle modification. i. Advise a healthy balanced diet. Include more of low-fat dairy products, small fish with bones like anchovies, almonds. ii. Adopt a regular structured exercise at least 30 minutes, 5 days a week, moderate exercise against gravity like jogging and brisk walking iii. Address smoking, alcohol and recreational drugs. iv. Limit coffee intake to less than 4 cups per day. v. Exposure to sunlight for around 5-15 minutes on warm days and 25-50 minutes on cold

days. Avoid the danger times (where the ultraviolet rays are very high) usually between 10 AM to 3 PM. b. Attention to fall prevention (she's living alone and has osteoporosis). Ask her if she has 52

support. Ask her to avoid falls as much as possible as it can easily lead to fractures. Page

© 2020 ARIMGSAS

c. Medications: start first with calcium and vitamin D supplementation. Definitive management is with bisphosphonates (alendronate 70mg/week, orally single dose) taken once a day. Side effects: esophagitis, jaw bone necrosis. Take this medication in the early morning, empty stomach, with a full glass of water and then sit upright for 30 minutes (to prevent reflux esophagitis) Newer medications: denosumab (Prolia) - a monoclonal antibody, available as injections, usually given subcutaneous every 6 months.

Osteoporosis alone is not an indication for HRT. If the patient has other menopausal symptoms, then you can put her on HRT.

[other medications: raloxifene - a selective estrogen receptor modulator; strontium granulate; calcitriol - particularly effective in steroid-induced osteoporosis (if you give calcitriol, DO NOT give your patient calcium supplements]

Give reading materials regarding osteoporosis. I will arrange a review with you once the results of your DEXA scan come in.

------Case 2: You are a GP, your next patient is 67-year-old Mary, who has come to you for back pain that she got 2 days ago while she was picking up a heavy shopping bag. The pain is at the lower end of her back and she took Panadol for 2 days and the pain has not settled yet. Her son has advised her to see a doctor as the pain is getting worse. She had menopause 18 years back and at that time she suffered hot flashes, mood swings, and dry vagina but she did not receive HRT because she was afraid of the side effects. Apart from that, her overall general health is fine, and she does not have any significant past medical or surgical history. You had ordered an x-ray of the spine.

X-ray shows a wedge compression fracture at the L4 vertebrae

53

Page

© 2020 ARIMGSAS

TASKS 1. Explain the x-ray to the patient 2. Tell her how that happened 3. Explain the implications of this condition on her life, with reasons 4. Tell her what you are going to do for further management

APPROACH Explain the x-ray This is an x-ray of your lower back. This is a side view that you can see. The spine is made of lots of bones called vertebrae, placed one on top of the other, with space in between. And normally, all these vertebrae should be of equal height but as you can see here, one of the vertebrae has a reduced height and this vertebra, has undergone a wedge compression. The bones also are having a glassy appearance, which means that the bone density has become low. Tell her how that happened The most common causes of a wedge compression fracture of the vertebrae are osteoporosis. It is a condition where your bones become thinned out, soft and porous, as the calcium from the bones is mobilized towards the blood. One of the most common contributing factors to osteoporosis is menopause. At the time of menopause, you were having symptoms of estrogen deficiency and it is the estrogen deficiency that contributes towards osteoporosis. There could be other causative factors as well, like a diet that is deficient in calcium, lack of exercise, inadequate exposure to sunlight, smoking, alcohol and excessive coffee intake. Explain the implication of this condition on her life Once osteoporosis occurs, a major complication that you can get fractures with trivial trauma. As in your case, you were just lifting a shopping bag from the ground when you got this fracture. If you sustain a fracture at this age, it might take a long time to heal. Tell her what you are going to do for further management Confirm that she is having osteoporosis Order blood tests and DEXA scan The first priority is to treat the wedge compression fracture Refer to the orthopaedic surgeon to have an assessment of the fracture Put her on adequate painkillers like Panadeine or NSAIDs A specialist might decide to give you a brace (thoracic-lumbar-sacral orthosis) and might require a rehabilitation therapy after that. If your nerves are getting affected, then surgery would be the option. Treat the osteoporosis Lifestyle modifications Supplementation of calcium and vitamin D Put on bisphosphonates or denosumab Attention to fall prevention - refer her to a fall prevention clinic where you will be educated about fall prevention, and there will be physical therapists

54 Page

© 2020 ARIMGSAS

Gestational Hypertension

You are at a GP clinic, a 30-year-old Jane who is 34 weeks pregnant with regular antenatal check-up comes in for a a consult. During the last antenatal check 2 weeks ago, her BP was 110/80. Today, BP was 150/100. The rest of the VS are stable.

TASKS 1. Relevant history from the patient 2. PE from the examiner 3. Discuss diagnosis and management with the patient

Differential Diagnosis ▪ Pre-eclampsia ▪ Gestational hypertension ▪ Chronic hypertension ▪ Stress and other lifestyle

Positive points: mild swelling around her ankles, 34 cm fundal height, longitudinal lie, cephalic presentation, cervical OS is closed, no proteins on UDT

APPROACH ▪ History o How is your pregnancy going so far? Any issues in your pregnancy? Any tummy pains? Any bleeding or discharge from down below? Is the baby kicking well? o Symptoms of high BP: any dizziness, tiredness, chest pain, racing of the heart? o Any time during the pregnancy and high blood pressure has been recorded? o Did you have any high blood pressure before pregnancy? o Symptoms of pre-eclampsia: any headaches, trouble with vision, vomiting? Any swelling or oedema? o Did you have regular antenatal check-ups? Blood checks and blood group done? Did you have a Down syndrome screening? Did you take your folic acid? Did you do the ultrasound at 18 weeks, was it normal? Sweet drink test at 26/28 weeks, what was the result? Any repeat ultrasound at 32 weeks, if so, what was the result? o Any smoking, alcohol, or recreational drugs? o Any stress at home or work? o Any other medical or surgical illness? o Any medications? Any allergies? o Do you have a family history of high blood pressure in your mother or sister when they were pregnant? (risk factor)

▪ Physical Examination o General appearance: any oedema? PICCLED BMI o Vital signs 55

o Systemic examination Page

© 2020 ARIMGSAS

• CVS - normal s1/s2? Any murmurs? • CNS - is there papilledema? Is there hyperreflexia or clonus? Any problems with the other cranial nerves? • R/S - good air entry? Visible and symmetrical chest rise and fall? Adventitious breath sounds? • Abdomen ▪ Fundal height ▪ Foetal heart rate ▪ Foetal lie and presentation ▪ Uterine tenderness? • Pelvic examination ▪ Inspection of vulva and vagina: any bleed, discharge, rash, vesicle ▪ Speculum - look at cervical OS • Office tests: UDT - look for proteins (key point), BSL

▪ Management From the history and examination, most likely you are having a condition called gestational hypertension. This is a new-onset of high blood pressure at or after 20 weeks of pregnancy. This is not associated with leakage of proteins to the urine or any end-organ damage. The cause is usually due to hormonal changes during pregnancy and it usually resolves within 3 months of delivery. One of the complications that this can lead to is pre-eclampsia where along with a sharp rise in your blood pressure, there will be massive swelling, and leakage of proteins to the urine and this could lead to seizures or eclampsia as well. Gestational hypertension as such, do not lead to any complications in the mom or the baby. But of course, we need to monitor you from the complications. In order to confirm that you have gestational hypertension, we need to repeat the blood pressure recording after 4 hours. And if that reading is found to be high, you indeed have gestational hypertension. We also need to do some blood tests, such as FBE, UEC, LFT, urine protein creatinine. ratio. I will refer you to the high-risk pregnancy clinic where you will be seen by the obstetrician and the internist, who will start you on blood pressure lowering medications like labetalol or methyldopa. Urine protein needs to be done once weekly from now on. Ultrasound will be done now and also every 4 weeks along with a CTG. You need to do some lifestyle modifications like no added salt in your diet, and rest on the left lateral position so as to take off the weight of the baby from the major blood vessels. You need to watch out for dizziness, headache, vomiting, trouble in vision or blurring of vision, tummy pain, swelling, or the baby not kicking well. And if any of these happens, please report to the emergency department as soon as possible. Our goal is to control your blood pressure, to recognize pre-eclampsia early of it develops, and thereby preventing eclampsia, and also optimize birth for both you and your baby. I will give you reading materials on gestational hypertension for further insight. I will arrange a review with you once the blood test results are available. Do you have

56 Page

© 2020 ARIMGSAS

Gestational Diabetes

Your next patient at your GP is 29-year-old Maya, who is 28 weeks pregnant. She returns to you for the results of GCT done 2 days back. The value of plasma glucose level in GCT is 9.1 mmol/L after 1 hour.

TASKS 1. Further relevant history 2. Explain result to the patient 3. Further management

APPROACH ▪ History o How is your pregnancy going so far? Any issues in your pregnancy? Any tummy pains? Any bleeding or discharge from down below? Is the baby kicking well? o Diabetes questions: Any symptoms of frequent urination? Do you feel more thirsty nowadays? Any recurrent skin or vaginal infections? Any numbness or tingling sensation in your extremities? o Pre-eclampsia questions: any headaches, blurring of vision, oedema or swelling? o Routine antenatal history: Any blood test and blood group done? Down syndrome screening? Folic acid? Ultrasound at 18 weeks? Any complications in the position of the placenta? o How is your diet? Activity and exercise? o Any smoking, alcohol? o Do you have good support? o Any other medical and surgical illness? Any medications? o Any family history of diabetes?

▪ Management The results of the sweet drink test, the value is a bit high which could be due to a condition called gestational diabetes. That is when your blood sugar level goes high during pregnancy. But the test that we did for you is just a screening test, so we need to do a confirmatory or diagnostic test now called the glucose tolerance test, which will tell with certainty if you have gestational diabetes or not. You need to come for this test after fasting, and a blood sample will be taken. You will then be given a sweet drink and then blood samples will again be collected at 1 hour and 2-hour intervals. Any one of these values come out abnormal, the GTT is considered to be positive. During pregnancy, the placenta secretes certain hormones like human placental lactogen, beta HCG, and cortisol, all of which has got anti-insulin properties. And insulin is the hormone that keeps your blood sugar level under control. Usually during pregnancy, your production of insulin is heightened up, however in your case, it isn't the case.

If your blood sugar doesn’t get controlled, complications can happen in your or the baby. Complications for you include polyhydramnios. This is a condition where the fluid in the bag surrounding the baby becomes high. Due to this, you could also go into preterm labour or 57

premature rupture of membranes, or membranes rupture before labour pains sets in. There is Page

© 2020 ARIMGSAS

also increased chance of induction or C-section. Another complication is pre-eclampsia, which is a condition where there is a sharp rise in blood pressure with leakage of proteins in the urine. Next is placental abruption, which is a condition where the placenta separates from the womb resulting in bleeding. In your baby, complications include macrosomia or a big baby. He can also have birth defects like neural tube or nervous system defects, heart defects, and vertebral defects. And if the blood sugar level gets uncontrolled for a long time, the baby can even go for intrauterine growth retardation. There can be complications after birth as well, such as low blood sugar levels in the baby. He can also develop respiratory distress syndrome or difficulty in breathing, and a high incidence of jaundice or yellowing of the skin. It is still possible to avoid these complications as long as we keep your blood sugar level under control. I will refer you to the high pregnancy clinic where you will be seen by the obstetrician, the diabetic physician, the dietician and the diabetic educator. The first thing to do is a strict diet control for 2 weeks. You will be given a diet chart by the dietician and you need to monitor your blood sugar level 3-4 times a day, and record that in a diary. Your aim is to maintain the blood sugar level between 4-6 mmol/litre before meals. If the diet control is not working, the diabetic physician will decide to start you on insulin. Again, you have to monitor your blood sugar level 3-4 times/day. You will monitor your blood sugar using a glucometer. This and the proper administration of insulin will be taught to you by the diabetic educator. You will be monitored for diabetic control by doing an HbA1c. You may also be seen by the ophthalmologist and the nephrologist if necessary. You need to have weekly antenatal checks from 30 weeks, ultrasound at 32 weeks and then 4 weekly, CTG weekly from 32 weeks. You can go in for normal vaginal delivery if everything goes on well with her antenatal checks, but it should be at term at the latest. During delivery, your blood sugars will be checked intermittently, and insulin injections will be given as needed. The baby will be monitored by continuous CTG. After delivery, there is a high chance that your blood sugar level will come back to normal because the hormones present during pregnancy will now be gone. You can already stop insulin injections when that happens. Your baby, however, will be checked by the paediatrician and monitored for low blood sugar levels. Once you have gestational diabetes, there is a 30-60% chance, that you could develop diabetes later in life. We will repeat an OGTT, 6 weeks after delivery, and then a blood sugar level 3 yearly.

***GCT is a screening test for diabetes. You will be given 50 grams of oral glucose. The blood sugar level is taken after 1 hour. If it is greater than or equal to 8 mmol/litre, the GCT is set to be positive. The next test is an oral glucose tolerance test. You should be fasting. Then we will give you 75 grams of glucose and then you record the blood sugar level at 1 hour and 2-hour intervals. If the fasting blood sugar level is greater than or equal to 5.5 mmol/litre, 1-hour blood sugar greater than or equal to 10 mmol/litre, and 2 hours greater than or equal to 8 mmol/litre. Even 1 value abnormal, this will be positive.

***Do a GTT at the first antenatal visit if the patient has a previous history of gestational diabetes or a family history of diabetes. Otherwise, do a BSL at the first visit, and GTT at 28 weeks.

***Another case: Random blood sugar level is 14 mmol/L. If the patient has NO symptoms, do a GTT to confirm if she has gestational diabetes. If she has symptoms, don't do a GTT. 58

Page

© 2020 ARIMGSAS

Atrophic Vaginitis/Endometrial hyperplasia

60-year-old Mary presents to your GP, with complaints of spotting since the last 2 months. She does not have any other medical or surgical illness and is not on any medication.

TASKS 1. Focused, relevant history 2. Examination findings from the examiner 3. Order relevant investigations 4. Further management of this patient.

Atrophic vaginitis: Positive points in history: bleeding is becoming worse now, compared to the last 2 months; 3 pads used since yesterday Positive points in PE: abdomen is soft and non-tender, the vagina is thin, dry and atrophic with diffuse erythema with same findings in the lateral and posterior walls; uterine size is normal, no tenderness

Endometrial hyperplasia: Positive points in history: everything normal except for postmenopausal bleeding Positive points in PE: bleeding from the cervix on speculum exam, the uterus is normal in size Investigations: endometrial thickness is 7mm.

Differential Diagnosis ▪ Atrophic vaginitis ▪ Endometrial hyperplasia ▪ Endometrial polyps ▪ Endometrial cancer

APPROACH ▪ History o Is the patient hemodynamically stable? o How long has your bleeding been going on? Is this the first time that you had bleeding after menopause? Has your bleeding becomes worse, or is it the same for the last 2 months? What is the color of the bleed? Are there any clots? Is it smelly? How many pads have you used since yesterday? Do you have any abdominal pain, fever, nausea and vomiting? o When did you have your menopause? Did you experience any hot flashes, mood swings, heavy sweating at that time? (means deficient in estrogen, can predispose to atrophic vaginitis) Are you on any hormone replacement therapy? (can predispose to endometrial hyperplasia) o Atrophic vaginitis: any dryness from down below? Any burning or itching sensation down

below? Any abnormal discharge? (can sometimes lead to a moderate non-offensive yellowish discharge)

o Polyps: any bulging from down below? 59 Page

© 2020 ARIMGSAS

o Endometrial cancer: any weight loss? Any lumps or bumps around the body? Have you done your pap smear? What was the result? o Sexual history: are you sexually active? Are you in a stable relationship? Do you experience painful intercourse? (atrophic vaginitis) Any postcoital bleed? (cervical cancer) o Do you experience burning or tingling when you pass urine? Have you experienced any incontinence? Do you open your bowels regularly?

▪ Physical Examination o General appearance: BMI, pallor, lymph node enlargements, dehydration, oedema o Vitals: BP with a postural drop, temperature o CVS, R/S, CNS o Abdomen: Visible distention, mass, tenderness o Pelvic exam • Inspection of the vulva and vagina: bleeding, gush or trickling of blood, can you see the same findings in the lateral and posterior walls of the vagina? • Speculum: does the cervix appear healthy? Any bleeding from the cervix? • Per vaginal exam: size and tenderness of the uterus, adnexal mass and tenderness o Office tests: UDT, BSL

▪ Investigation o FBE, UEC o Transvaginal ultrasound • The thickness of the endometrium (especially in endometrial hyperplasia; in postmenopausal women, the thickness should be less than 5mm, in premenopausal, can go up to 11mm) • Polyps in the endometrium or cervix • Any other abnormal growths

▪ Management From the history and examination, most likely you are having atrophic vaginitis. It is a condition where the lining of the vagina becomes thin and dry, and breaks down at times, leading to a bleed. This happens due to menopause because, at the time of menopause, the ovaries shut down completely so that only very low levels of estrogen are formed. The lining of the vagina is dependent on estrogen for strength and integrity, so when the estrogen levels go low, the vagina becomes thin and atrophic. You can use local estrogen therapy in the form of vaginal estrogen cream, or vaginal estrogen tablets (Vagifen), which you have to use for some time (up to 8 weeks) before results are seen. You can also use vaginal moisturizers, especially during intercourse. Just in case the bleeding becomes severe or you start passing clots, you develop fever, please report immediately. I will give you reading materials regarding atrophic vaginitis for further insight. I will review you in 2 day’s time. Should I be put on hormones doctor? For atrophic vaginitis alone, HRT is not indicated. However, if you have other symptoms of menopause along with atrophic vaginitis, you can be put on HRT.

From the history and examination, most likely you are having endometrial hyperplasia. The endometrium is the normal inner lining of your womb. After menopause, the endometrial thickness should be less than 5mm. But in your case, the thickness has gone up to 7mm. When 60

there is an increase in the endometrial thickness, it is called endometrial hyperplasia. It is due to Page

© 2020 ARIMGSAS

an unopposed stimulation of the endometrium by estrogen in the absence of progesterone. After menopause, the ovaries shut down completely, so that only very low levels of estrogen are formed, with no progesterone at all. However, estrogen is still being produced in the body through the peripheral conversion of fat tissue and this estrogen causes the stimulation of the endometrium. In contrast, there is no alternate source of progesterone. In endometrial hyperplasia, the endometrium becomes quite unstable and breaks down at times leading to bleeding. The other complication of endometrial hyperplasia is endometrial cancer which could happen in around 7% of the patients. This depends on the type of endometrial hyperplasia (typical vs atypical; atypical is more prone to develop cancer) I will refer you to the specialist who will do a hysteroscopy and biopsy (key point). You will be given progestogens either orally or an intrauterine contraceptive device like Mirena will be placed. This will ensure that you have a continuous supply of progestogens. If the biopsy shows atypical changes, the specialist may decide to do a hysterectomy or removal of the uterus. I will give you reading materials regarding endometrial hyperplasia for further insight. I will arrange a review with you after the specialist consult.

Postmenopausal bleeding: bleeding after 1 year of menopause Cancers only form 10% of postmenopausal bleed

61 Page

© 2020 ARIMGSAS

Placenta Previa

You are an HMO at the ED of a tertiary hospital when 28-year-old Jenny, who is in her 34 weeks of gestation presents to you with vaginal bleeding.

TASKS 1. Further history 2. Examination findings from the examiner 3. Diagnosis and management

Differential Diagnosis ▪ Placenta previa ▪ Abruption placenta ▪ Trauma ▪ Bleeding disorders ▪ Blood thinning medications

Positive points from history: Was just getting dressed for the office, when she noticed there was blood trickling from her thighs; the first pad is not fully soaked Positive points from PE: 34 cm fundal height, normal FHR, longitudinal lie with the cephalic presentation, cervical OS closed Investigations: grade IV placenta previa on ultrasound

APPROACH ▪ History o Is the patient hemodynamically stable? o What is the duration of the bleed? What were you doing when you got the bleed? Any hit to your tummy? o What is the color of the bleed? Is it smelly? What is the severity of the bleed? How many pads have you used so far? Is the first pad fully soaked or not? Any clots? Any pain? (differentiates abruptio and previa) is this the first episode of a bleed during your pregnancy? o Is the baby kicking well? Did you do your antenatal checks, blood tests and blood group? Have you done your pap smear? What was the result? Have you done your down syndrome screening? Did you take your folic acid? Ultrasound at 18 weeks? What is the position of the placenta? Repeat ultrasound at 32 weeks? What was the result? (placenta can migrate up to 32 weeks) sweet drink test at 28 weeks? o Symptoms of pre-eclampsia: any headache, blurring of vision, dizziness, oedema? o Any bleeding disorders? Were any medications taken? Any past history of medical or

surgical illness?

o Any smoking, alcohol or recreational drugs? 62 o Do you have good support?

Page

© 2020 ARIMGSAS

▪ Physical Examination o General appearance: pallor, oedema, dehydration o Vital signs: BP with postural drop o Systemic examination • Abdomen: fundal height, foetal heart rate, foetal lie, foetal presentation ▪ KEY POINT: UTERINE TENDERNESS (differentiate abruptio from previa) ▪ Is the head of the baby higher than the lower uterine pole? (higher in previa) • Pelvic examination ▪ Inspection of vulva and vagina: can you see the bleed? Is it a gush or trickling? Any clots? Any rash or vesicles? ▪ Speculum exam: can you see the bleeding from the cervix? (invariably tell you that something is bleeding from the uterus) is the cervical OS open or closed? ▪ NEVER EVER DO A PER VAGINAL EXAM IN PREVIA (also in PROM, active Herpes Simplex infection, Virginal) • Office tests: UDT, BSL

▪ Management From the history and examination, most likely you have a condition called placenta previa. The placenta is the part that connects you to your baby, and which carries oxygen and food from you to your baby. This placenta is usually situated on the upper part of your womb. But if it attaches to the lower pole of your womb, it is called placenta previa. There are certain risk factors for placenta previa such as multiparity, advanced maternal age, chronic hypertension, smoking, alcohol and drugs, previous C-section. One of the major complications of this is bleeding, and you can go to a shock or coagulation failure or you can go into preterm labour. The baby can have hypoxia or decreased oxygen supply, and intrauterine growth retardation. The bleed is mainly because of the head of the baby pressing against the placenta leading to bleed from the placenta. You need to be admitted and seen by the specialist. I will start you on IV fluids, and take blood for examinations which include FBE, UEC, blood grouping, cross-matching and hold. Ultrasound will also be done to confirm the diagnosis of placenta previa. CTG will also be done to monitor the baby. If the bleeding stops and everything is well with the baby, you will be admitted in the hospital for the rest of your pregnancy until 37 weeks when the specialist will do a planned C-section. (there is a high chance that she can bleed especially after 34 weeks because after 34 weeks, there will be the descent of the baby's head). If the bleeding continues, and you become unstable, or if the baby becomes unwell, an immediate C-section will be done. I will give you reading materials about placenta previa for further insight.

[Grade 1: low lying placenta previa, the placental edge is nowhere near the OS Grade 2: marginal placenta previa, the placental edge comes up to the level of the OS Grade 3: partial placenta previa, partially covers the internal OS Grade 4: total placental previa, completely covers the external OS]

**** Another case: 34 weeks with grade 4 placenta previa. She is completely asymptomatic. 63

Management: Page

© 2020 ARIMGSAS

There are 2 options for her. One, you can admit her and will stay in the hospital until 37 weeks, where a planned C-section will be done. Second is she can be managed at home. However, 3 criteria should be met: 1. She lives nearby to the hospital 2. She should have a constant companion 3. With the informed consent of the patient Tell her that there is a high chance of bleed after 34 weeks, she can go in for a shock or coagulation failure and premature labour, and the baby can have hypoxia and IUGR.

***any grade of placenta previa, any gestational age, if the patient has heavy bleeding, becomes hemodynamically unstable, or if there is foetal distress, stabilize the patient and go in for an immediate C-section.

***In minor grades (grade 1 and 2) of placenta previa, usually C-section is done in 38 weeks. If major grades (grade 3 and 4), C-section is done at 37 weeks.

***If the patient insists on a vaginal delivery, tell that it is always a C-section that is preferred in placenta previa (any grade). But a trial of vaginal delivery can be considered if the placental edge is greater than 2cm from the OS and if the baby's head is below the placental edge.

64 Page

© 2020 ARIMGSAS

Mild revealed Abruptio Placenta

You are an HMO in the ED of a major hospital when 28-year-old Tracy at 34 weeks’ gestation comes to see you with complaints of vaginal bleeding since the past 1 hour.

TASKS 1. Relevant history 2. Examination findings from the examiner 3. Diagnosis and management

Differential Diagnosis ▪ Placenta previa ▪ Abruption placenta ▪ Trauma ▪ Bleeding disorders ▪ Blood thinning medications

Positive points in the history: bleeding for the past 1 hour, was just preparing lunch in the kitchen when the bleed started, just used 1 pad, mild tummy pain all over her tummy, no radiation, continuous Positive points in the PE: Vital signs normal, FHR 152 bpm, definite mild tenderness, no rigidity and guarding, the cervix is closed

APPROACH ▪ History o Is the patient hemodynamically stable? o What is the duration of the bleed? What were you doing when you got the bleed? Any hit to your tummy? o What is the color of the bleed? Is it smelly? What is the severity of the bleed? How many pads have you used so far? Is the first pad fully soaked or not? Any clots? Any pain? (differentiates abruptio and previa) is this the first episode of a bleed during your pregnancy? o Details of the pain: where exactly do you feel the pain? Is it going somewhere else or not? Is it a continuous of on and off the pain? Is it coming at regular intervals? Are the intervals getting shorter and pain getting longer? How severe is your pain from 1 to 10? Do you need a painkiller? Do you have any allergies? o Is the baby kicking well? Did you do your antenatal checks, blood tests and blood group? Have you done your pap smear? What was the result? Have you done your down syndrome screening? Did you take your folic acid? Ultrasound at 18 weeks? What

is the position of the placenta? Repeat ultrasound at 32 weeks? What was the result? (placenta can migrate up to 32 weeks) sweet drink test at 28 weeks?

o Symptoms of pre-eclampsia: any headache, blurring of vision, dizziness, oedema? 65 Page

© 2020 ARIMGSAS

o Any bleeding disorders? Were any medications taken? Any past history of medical or surgical illness? o Any smoking, alcohol or recreational drugs? o Do you have good support?

▪ Physical Examination o General appearance: pallor, oedema, dehydration o Vital signs: BP with postural drop o Systemic exam: • Abdomen: fundal height 34cm, FHR: 152bpm, longitudinal lie, cephalic ▪ Uterine tenderness, rigidity, guarding ▪ Uterine contractions per abdomen • Pelvic examination ▪ Inspection of the vulva and vagina: bleed, color, severity, rash and vesicles ▪ Speculum: bleeding from the cervix, cervix open or closed ▪ DON'T DO A PER VAGINAL EXAM because you don't know if it is a previa • Office test: UDT, BSL

▪ Management Most likely you are having a mild revealed abruptio placentae. The placenta is the part that connects you to your baby, and which carries oxygen and food from you to your baby. This placenta usually separates from the uterus after the delivery of the baby. But if the placenta separates from the womb, during pregnancy, it is called abruptio placentae, which could be revealed or concealed depending on whether the bleeding is coming out or not. In your case, it is the revealed kind, because the blood is coming out. There could be several causes for this like multiparity, high blood pressure, diabetes, injuries or trauma, smoking and recreational drugs (especially marijuana) or sometimes, unknown. So far, from the details that you have given me, the cause could not be determined. If the bleeding continues, then you can go into shock. You can also develop bleeding tendencies (DIC), and the baby can become unwell as the oxygen supply to the baby can be affected. It is good that you have gone to the hospital and we detected it early so we can prevent these complications from happening. I will now admit you, and you will be seen by the specialist. Meanwhile, I will start you on an IV line and IV fluids, take blood for investigations such as an FBE, UEC, blood group, cross-matching and hold, and coagulation profile. I will also arrange for an ultrasound to see if it is indeed abruptio, if any blood is collecting inside and also check the condition of the baby. CTG will also be arranged to look at the well-being of the baby. It is advisable to take complete bed rest at this time, until the bleeding stops. Once the bleeding stops, you will be asked to ambulate, and we will look if the bleeding is recurring or not. If the bleeding stops, and everything is well with you and the baby, you will be discharged, and the rest of your pregnancy will be monitored in the high-risk pregnancy clinic. You need to go for more frequent antenatal checks. Once you are at home, any time you experience any bleed or spotting, any pain, baby is not kicking well, or if your water breaks, please report immediately. Just in case you continue to bleed, or if the baby becomes unwell, immediate delivery will be

planned, either by induction or C-section. Do you have someone with you? Do you need me to call for your partner? 66

***If mild abruptio happens after 37 weeks, you can already do an induction or C-section. Page

© 2020 ARIMGSAS

***moderate revealed abruptio placentae Usually, blood loss will be very heavy. The patient loses around 1 - 1.5 litres of blood. There will be severe abdominal pain. The patient is in shock, and the baby is in distress. Management is stabilizing the patient first, then immediate C-section.

***severe revealed abruptio placentae There will be heavy bleeding, greater than 1.5 litres. The patient is invariably in shock, and the baby is usually dead. Management is stabilizing the patient first, then induction can be done since the baby is already dead.

For any reason the baby becomes dead, there are 3 things that you can do: 1. Wait for normal vaginal delivery. The baby will deliver in 2 weeks. Monitor the mom for DIC because the antibodies of the baby will go to the blood of the mother. 2. Do an induction of labour. If the patient requests, you can go for a C-section.

67 Page

© 2020 ARIMGSAS

Severe concealed Abruptio Placenta

You are an HMO at the ED of a major hospital, when 25-year-old Maria who is 32 weeks pregnant, presents to you with severe abdominal pain since the last 2 hours.

TASKS 1. Relevant history 2. PE from examiner 3. Diagnosis and management

Differential Diagnosis ▪ Preterm labour ▪ Abruptio placentae ▪ Trauma ▪ Torsion of ovarian cyst ▪ Medical causes: UTI, appendicitis, cholecystitis, pancreatitis

Positive points from the history: Pain is 8/10, pain all over her tummy, the pain has been there since last night but progressing for the past 2 hours. It is a continuous pain. No bleeding from down below. Doesn’t feel the baby kicking since this morning. The patient is feeling dizzy. Positive points from the PE: Pale, anxious and sweaty, Vital signs unstable (80/60, 94bpm, 26bpm, normal temp, O2 92%); fundal height 36cm (higher than the AOG), FHR is not heard, hard to feel lie and presentation, the uterus is extremely tender with rigidity and guarding

APPROACH ▪ History o Is the patient hemodynamically stable? o I read from the notes that you are having pain for the past 2 hours. Can you score the pain from 1 to 10, 10 being the worst pain? I will give you some painkillers, do you have any allergies? o Where is the site of pain? Is it the sudden or gradual onset of pain? Is the pain going somewhere else? o Rule out preterm labour: do you think that the pain is coming intermittently or is it a continuous pain? Has the pain been coming at regular intervals or not? o Rule out trauma: any hit or falls before the abdominal pain? o Any bleeding from down below? o Does anything make it better or worse? o Rule out UTI/appendicitis/cholecystitis/pancreatitis: how are your waterworks? Any burning or stinging during urination? Any smelly urine? How are your bowels? Do you open your bowels regularly? Any fever or vomiting?

o Do you feel your baby kicking? How long have you noticed that your baby is not kicking? o Did you do your antenatal checks, blood tests and blood group? Have you done your

pap smear? What was the result? Have you done your down syndrome screening? Did 68 Page

© 2020 ARIMGSAS

you take your folic acid? Ultrasound at 18 weeks? What is the position of the placenta? Sweet drink test at 28 weeks? o Symptoms of pre-eclampsia: any headache, blurring of vision, dizziness, oedema? • The patient is feeling dizzy, ask the patient to lie down and talk with your examiner. Ask for the GA and vitals of the patient. o Any bleeding disorders? Were any medications taken? Any past history of medical or surgical illness? o Any smoking, alcohol or recreational drugs? o Do you have a good support?

▪ Physical Exam o GA o Vitals o Proceed with the DRABCDE protocol • I would like to call for help. • Put in 2 large-bore IV cannulas, and take blood for examinations (FBE, UEC, Blood group, cross-matching, hold, coagulation profile, BSL) • Give a bolus of Crystalloids, then start on IV infusion • Put on O2 by mask 6-8L/min o I would like to re-assess the patient checking the cardiovascular, respiratory and the central nervous system and once she is stable, I would like to proceed with an abdominal examination o Abdomen • Fundal height: 36 cm (blood is collecting inside) • FHR: not heard • Lie and presentation: hard to feel the lie and presentation • Look for uterine tenderness, rigidity and guarding • Feel for uterine contractions o Pelvic examination • Inspection of the vulva and vagina: bleeding, discharge, rash, vesicles • Speculum: bleeding from the cervix, if the cervix is closed or not • DON'T DO PER VAGINAL EXAM, because the patient is in shock o Office tests: UDT, BSL

▪ Management From the history and examination, most likely you are having a condition called a severe, concealed abruptio placentae. The placenta is the part that connects you to your baby, and which carries oxygen and food from you to your baby. This placenta usually separates from the uterus after the delivery of the baby. But if the placenta separates from the womb, during pregnancy, it is called abruptio placentae, which could be revealed or concealed depending on whether the bleeding is coming out or not. In your case, it is the concealed kind, the blood is collecting inside. Because you have been bleeding heavily inside, you have become unstable, and have gone into shock. But your condition has been taken good care of.

Breaking bad news: Unfortunately, the next news that I have for you is not very good. Do you want somebody to be with you while I discuss this with you? I can call your partner so that he can be with you 69

during this time. As you said, you have not been feeling the baby kick since this morning. And Page

© 2020 ARIMGSAS

during my examination as well, I could not hear the baby's heartbeat. There is a high probability that something nasty might have happened to the baby. But of course, we need to confirm this by doing an ultrasound to look at the condition. You will be admitted and seen by the specialist and the specialist might decide to do a C-section, as the blood that has clotted inside needs to be evacuated or removed. Along with the placenta and the baby, the leaking blood vessels need to be seen. Once the emergency management is over, we can offer you and your partner counselling services as well. I know this is a difficult time for you as you are hoping for this baby, but we are here to help you.

70 Page

© 2020 ARIMGSAS

Lactational atrophic vaginitis

23-year-old Amy has come to your GP clinic for a 6-week post-partum check-up. She had a normal vaginal delivery and she delivered a 3.3 kg healthy baby boy.

TASKS 1. Relevant history 2. PE from examiner 3. Counsel Amy accordingly

Positive points from the history: last pap smear was 2 years ago, uses contraceptive pills before, Positive points from the PE: dry thin atrophic vagina

APPROACH ▪ History o Congratulate her on her delivery. o Is the baby doing well? Any concern that you have regarding your baby? Was he given the first dose of hepatitis vaccine? o Are you currently your baby? Any sore or cracked nipples? Any lumps or pain in your breast? Do you know the proper breastfeeding technique? (if the baby is not latching well, it can lead to sore and cracked nipples, leading to mastitis and breast abscess) o Any problems with your waterworks? Any burning or stinging while urinating? Any incontinence? Are you opening your bowels regularly? o Any conditions that you had during pregnancy or was it uneventful? (follow-up certain conditions if present) Are you immune against rubella and varicella? o Any complications during your delivery? Any cuts made down below or any tears that you had at that time? o Have you had excessive bleeding right after delivery? (primary postpartum haemorrhage) have you stopped bleeding now? (secondary postpartum haemorrhage) o Do you have any tummy pain? Any fever? Any abnormal, offensive vaginal discharge? Any calf swelling or pain? Are you enjoying your motherhood? Any changes in your mood? o Any smoking, alcohol, recreational drugs? o Do you have good support in looking after your baby? o When was your last pap smear done? What was the result? o Any other concerns that you would want to discuss? • What contraception was you on before you planned for your pregnancy? Have you resumed your sexual intercourse? Any problems that you are having?

▪ Physical Exam o General appearance: BMI, pallor, calf for swelling/tenderness

o Vital signs: temperature o Systemic exam:

• Abdomen: visible mass, tenderness 71 Page

© 2020 ARIMGSAS

▪ Palpable mass (usually the uterus goes back to the pelvis in 2 weeks’ time) has the uterus involuted or not? ▪ Tenderness • Pelvic examination ▪ Inspection of the vulva and vagina: any bleed or discharge, thin, dry atrophic vagina ▪ Speculum: arrange for a pap = ▪ Bimanual examination: the size of uterus, tenderness ▪ Adnexal mass and tenderness • Office tests: UDT, BSL

▪ Management It is not advisable to take a combined pill if you are breastfeeding as the estrogen content in the pill can suppress the breastmilk production. But there are other methods that you can use, such as only pills, Depo-Provera, Implanon, or intrauterine contraceptive device. When I examined you, I could see that your vagina is dry and atrophic. Have you felt any dryness, itching and burning down below? What you are having is lactational atrophic vaginitis. This condition happens when the prolactin or the milk secreting hormone acts against the estrogen and the loss of the placental estrogen. Normally, it resolves spontaneously in 2 weeks once the hormones come back into place, but you can use vaginal moisturizers during this time. If it becomes severe, then we can use vaginal estrogen creams or tablets. I will give you reading materials regarding contraceptive options and pelvic muscle exercise. 4 B's to check in post-partum: 1. Baby 2. Breast 3. Bladder 4. Birth canal

Usually, do a post-partum check-in 6 weeks. But in C-section, do it in 2 weeks.

72 Page

© 2020 ARIMGSAS

LSIL

Maria, 34 years, presents to your surgery to discuss the results of the pap smear taken 3 weeks back. Results show changes consistent with LSIL. Her last pap smear done 2 years back was normal. She has no history of any other medical or surgical illness.

TASKS 1. Further relevant history 2. Explain pap smear result to the patient and discuss further management

APPROACH ▪ History o Period history: at what age did you start your periods? (early menarche at risk) Do you get it every month? How many days do you usually bleed? Do you have mild/moderate/severe bleed? Any clots present? Do you feel any pain during your periods? Key point: any bleeding in between periods? Any abnormal blood-stained discharge or any other vaginal discharge? o Sexual history: are you in a stable relationship? (multiple sexual partners can predispose to cancer of the cervix) history of STIs? o Have you been pregnant before? (multiparity at risk) Do you use any contraception? o When did you do your last pap smear? What was the result? Anytime that you had an abnormal pap smear? Have you taken the Gardasil vaccine? o Any weight loss? Any lumps or bumps around your body?

▪ Management Do you know about pap smear and why it is being done? Pap smear is a screening test for detecting any pre-cancerous lesions or pre-cancerous conditions in your cells. In LSIL, there are mild abnormal changes in the cells of your cervix, and it is considered to be a pre-cancerous condition. There is nothing to be scared of at the moment as it is not cancer yet. And the majority of these lesions, will get corrected and come back to normal. Certain infections can also lead to these changes in your cervix, like the infection with human papilloma virus. This virus is usually sexually transmitted, and it can remain asymptomatic within your cervix. Normally the body clears the HPV infection in 8-14 month’s time. We will repeat the pap smear in 1-year time. o Pap smear has come back to normal --> repeat pap in 1-year time --> normal result --> go back to routine pap every 2 years o Pap smear remains in LSIL level --> refer for colposcopy and biopsy --> biopsy-proven LSIL --> wait and repeat pap smear in 1-year time --> LSIL --> repeat pap smear in 1-year time --> LSIL --> some specialists will decide to do destructive procedures like cryosurgery (freeze),

laser (burn), cold coagulation technique (hot probe) • A colposcopy is a binocular-like instrument with a microscope and a light. It never enters

your body. Before the biopsy, your cervix will be coloured with acetic acid. The areas 73

that turn white, the biopsy will be taken from there. Page

© 2020 ARIMGSAS

• According to the recent guidelines, a biopsy-proven LSIL does not require immediate treatment. However, we will need to repeat the pap smear every 1 year for monitoring. o Pap smear becomes HSIL --> immediate referral for colposcopy and biopsy --> biopsy proven HSIL --> immediate treatment (Has my partner cheated on me? There are other modes of transmitting this virus, other than sexually.)

In HSIL, the cells show changes to a moderate/severe grade. This is a pre-cancerous condition and requires immediate confirmation. So, I need to refer you immediately to the specialist for colposcopy and biopsy. Once it is confirmed that it is HSIL, the specialist will give you immediate treatment. The treatment could be destructive procedures like cryotherapy or freezing the affected area, laser or burning away the affected area, cold coagulation or LEEP procedure (loop electrosurgical excision procedure) OR Large loop excision of the transformation zone, where a wire loop through which electric current is passed to cut off the abnormal area.; or surgical procedures like cone excision, where the abnormal area is taken out in the form of a cone with clear margins. If you go for pregnancy after this procedure, you can have cervical incompetence, where the cervix becomes weak and cannot hold the baby inside, or cervical stenosis due to the scar. Last option is hysterectomy. We will do a repeat colposcopy and pap smear after 6 months. Pap smear and HPV 12 months after treatment. Pap smear and HPV again 24 months after treatment. If the last 2 pap smears are normal, then you can go in for a pap smear every 2 years. (TREATMENT ----6 months--- colposcopy + pap smear ----6 months---- pap smear and HPV ---12 months ---pap smear and HPV) (Moderate dysplasia - destructive procedures. Severe or in-situ - surgical)

74 Page

© 2020 ARIMGSAS

Criteria CIN Modified Australian Bethesda System

Normal Normal Within normal limits (WNL) Repeat every 2 years until 70-year-old After 70 years, continue if: ▪ any previous 2 pap smear result has been abnormal ▪ patient requests the pap smear to be continued ▪ The patient never had a pap smear before Atypical Atypical Atypical squamous cells of Repeat pap smear in a 1- undetermined significance year time (ASCUS) The cells are showing mild changes, midway between normal and abnormal. Majority of the ASCUS will regress and come back to normal

HPV HPV LSIL

Mild dysplasia CIN I LSIL

Moderate CIN II HSIL dysplasia

Severe CIN III HSIL dysplasia

Carcinoma in- CIS HSIL situ

Invasive Invasive carcinoma Invasive carcinoma carcinoma

***Abnormal pap smear + pregnancy Can do pap smear up to 20 weeks of pregnancy LSIL + pregnancy: can continue the pregnancy, and 6 weeks after delivery, repeat pap smear HSIL + pregnancy: refer for colposcopy but NO BIOPSY, confirm that this is HSIL; can continue the

pregnancy, but requires a colposcopy in each trimester to make sure that it has not gone to the 75

invasive stage, and 6 weeks after delivery, repeat colposcopy and biopsy Page

© 2020 ARIMGSAS

Invasive cancer + pregnancy: 1st trimester: can terminate the pregnancy; 2nd trimester onwards, if she is willing, continue pregnancy but early termination of pregnancy around 30 weeks by C- section and hysterectomy

76 Page

© 2020 ARIMGSAS

Hydatidiform mole

28-year-old Jess, who is 10 weeks pregnant, comes to your GP, with complaints of abdominal pain and vaginal bleeding since the past 2 hours.

TASKS 1. Further relevant history 2. PE from examiner 3. Relevant investigations 4. Explain diagnosis and management to the patient

Differential Diagnoses: ▪ Miscarriage ▪ Ectopic pregnancy ▪ H mole ▪ Trauma ▪ Bleeding disorders ▪ Blood thinning medications

Positive points from the history: pain has been there for the past 2 hours, pain happened while making lunch, pain right from the morning but progressing the last 2 hours, continuous pain, nothing makes it better or worse, pain in lower tummy radiating down the thigh and to the back passage, cramping pain, vomiting like anything, the pain started first then she started bleeding. Bright red bleeding, have used 2 pads not fully soaked, bleeding looks like bubbles Had a positive pregnancy test, no antenatal checks yet Positive points from the PE: visible distention in the abdomen, palpable mass in the abdomen, can get to the lower border of the mass, it is the uterus palpable in the abdomen Bleed with vesicles and grapes, OS is closed Uterus enlarged to 14 weeks size

APPROACH ▪ History o Is the patient hemodynamically stable? o Which started first, was it the pain, or the bleeding? o Pain questions: • Where is the site of pain? Any radiation of pain? o Bleedings questions: • How long have you been bleeding? What is the colour of the bleed? How many pads have you used so far? Is it fully soaked? Are there any clots or tissues? Any vesicles or grapes? Any dizziness or tiredness? Do you know your blood group (key point)?

o How did you confirm your pregnancy? o Have you done any antenatal checks so far? o Period history: 77

• When was your LMP? Is it regular? Page

© 2020 ARIMGSAS

o Exaggerated symptoms of pregnancy: (key questions) • Do you have any excessive nausea/vomiting? o Do you have any history of the hit to the tummy or abdomen? o Any previous miscarriages? Was it an H-mole? Any history of STIs? Do you use any form of contraception? When was your last pap smear and what was the result? o Do you smoke, drink alcohol or take recreational drugs? o Any previous medical or surgical illnesses especially bleeding disorders? Are you on any medications? ▪ Physical Exam o General appearance: pallor, dehydration o Vital signs: BP with postural drop o Systemic exam: • Abdomen: visible distention, any mass; any palpable mass? Can you get to the lower border of the mass or not? (if yes, abdominal mass; if no, pelvic mass), is it the uterus that you are palpating in the abdomen? Any uterine tenderness? • Pelvic exam: ▪ Inspection of the vulva and vagina: is there any bleed with vesicles or grapes? Is the bleeding just trickling or is it a gush of blood? ▪ Speculum exam: can you see bleeding from the cervix? Is the OS open or closed? ▪ Bimanual exam: CMT, size and position of the uterus, tenderness; adnexal mass and tenderness o Office test: BSL, UDT ▪ Investigations o FBE, UEC, serum Beta HcG (will be very high), blood group, cross-matching and hold, coagulation profile o Ultrasound (will show snow storm appearance)

▪ Management Look Jess, I know that you are looking forward to this pregnancy and delivery of a normal baby, but I am sorry to say that unfortunately, you do not have a normal viable pregnancy. I know that this is hard for you to take, but we will be here for you, I can even call your partner inside so that he can be with you during this time. What you have right now is an H-mole or a vesicular mole. It is due to an abnormal development of the placenta, where the baby and the placenta are replaced by grape-like vesicles/structures, and no baby tissue is usually found. The exact cause why this happens is unknown but supposed to be a genetic disorder where the sperm fertilizes an egg which is empty or which has no genetic material inside, or it can also be due to two sperms fertilizing a single egg at the same time. The placental part starts producing fluid-filled cysts which multiply rapidly replacing the placenta and the baby tissue. Occasionally these moles can become invasive when they penetrate deep into your womb or it can enter into the bloodstream reaching distant organs like the lungs, the brain, and liver, then they are called invasive moles. Rarely, these moles can become nasty and undergoes a cancerous change, but it is not correlated with cancer. You need to be transferred to the hospital immediately and be seen by the specialist. I will arrange

an ambulance for you, and I will liaise with the hospital and inform them of your condition. In the hospital, you will be put under general anesthesia, and the vesicles in your uterus will be

evacuated by suction, following which, a gentle scraping will be done (curettage), to ensure 78 Page

© 2020 ARIMGSAS

complete removal of the moles. An oxytocin drip will be given during the procedure to prevent bleeding after the procedure. Ultrasound will be done after 2 weeks. You will be followed-up further with serial blood beta Hcg estimations weekly until it becomes zero (it might take 8-12 weeks to come back to zero), then after than beta Hcg every month for the next 1 year. If ever the beta Hcg goes high, you need a specialist referral who will do further investigations and might put you on Methotrexate. Avoid pregnancy for 1 year after the beta Hcg becomes zero. (if she becomes pregnant, the beta Hcg will go high, and you will not know if the elevated beta Hcg is due to pregnancy or due to invasive mole or choriocarcinoma) You can use any type of contraception during this time to avoid pregnancy. Counselling services to you and your partner can be given to you after the emergency management is over.

***If you get an Rh-negative case, ask for the partner's blood group. If the partner's blood group is positive or unknown, give anti-D injections to the patient. Before giving the first dose anti-D injections, do an indirect Coomb's test. If the indirect Coomb's test is negative, give anti-D injections. If it is positive, it means that the antibodies are there, and there is nothing you can do now. 3 anti-D injections you normally give if the patient is Rh-negative to prevent isoimmunization: 28 weeks, 34 weeks, and within 72 hours of delivery of the baby if the baby is Rh-positive. If the baby is Rh-negative, no need to give the 3rd dose.

79 Page

© 2020 ARIMGSAS

Threatened Miscarriage

You are at your GP when 27-year-old Jane presents with bleeding from the vagina since the past 2 hours. She has done a home pregnancy test which has turned out to be positive.

TASKS 1. Take a further history 2. Get the examination findings from the examiner 3. Discuss your diagnosis and further management with the patient

Differential Diagnosis: ▪ Miscarriage ▪ Ectopic pregnancy ▪ H-mole ▪ Trauma ▪ Infections ▪ Bleeding disorders ▪ Blood-thinning agents ▪ Cervical polyps ▪ Normal bleed during pregnancy

Positive points in the history: bleeding has been present for the last 2 hours, 2nd pad used but 1st pad not fully soaked, no tissues, clots or grapes, bright red in color, mild tummy pain, LMP 7 weeks ago, regular periods, planned pregnancy Positive points in the PE: soft, non-tender abdomen, no masses, bright red bleed with no clots or tissues, the cervix is closed, uterine size is 7 weeks, no tenderness, no adnexal mass and tenderness

APPROACH ▪ History: o Is the patient hemodynamically stable? o Bleeding questions: • How long have you been bleeding? What is the colour of the bleed? How many pads have you used so far? Is it fully soaked? Are there any clots or tissues? Any vesicles or grapes? Any dizziness or tiredness? Do you know your blood group (key point)? o Associated symptoms: • Any pain? ▪ Pain questions: Where is the site of pain? Is radiation present? How severe is the pain? Anything making it better or worse? • Any fever? Any dizziness of tiredness? o Pregnancy questions:

• Period questions: When was your last menstrual period? Is it regular? • I have read from the notes that you have done a pregnancy test and it has turned out to be positive 80

• Early signs of pregnancy: do you have any vomiting, breast tenderness? Page

© 2020 ARIMGSAS

• Exaggerated symptoms of pregnancy: do you have excessive vomiting? • Do you have good support? • Any antenatal checks you'd done so far? Have you taken your folic acid? • Any previous history of miscarriages? Any pregnancy in the past? • Any hits or trauma to your tummy? o Risk factors for miscarriages: • Diet: any intake of raw meat? (predisposed to toxoplasma), how many coffees do you take in a day? • Do you smoke, drink alcohol or take recreational drugs? • Any pets at home? (toxoplasma in cat litter) o Any other medical or surgical illness? o Any prescription or over the counter medications? Any allergies? ▪ Physical Exam o General appearance: o Vital signs o Systemic exam • Abdomen • Pelvic exam ▪ Inspection: colour of the bleed? Trickling or gush? Tissues, clots? ▪ Speculum: look if bleeding is coming from cervix, cervix closed or open? ▪ Per vaginal: CMT, uterine size, position, tenderness adnexal mass and tenderness o Office test: UDT, BSL ▪ Management Most likely what you're having is a threatened miscarriage. It is bleeding from the vagina with tummy pain, before 20 weeks of pregnancy. The exact cause is unknown but there are certain risk factors associated with this like smoking, alcohol and recreational drugs, excessive coffee intake, infections, trauma, could be due to problems in the placenta and thereby feeding of the growing baby, or could be due to genetic abnormalities in the baby. From the details that you have given me, I have not found any risk factors in you, so there is nothing that you have done that has caused this miscarriage, and there is no way by which we can predict a miscarriage. More than half of the women stop bleeding and they continue to have a normal and healthy pregnancy. You need to be referred to the hospital now and seen by the specialist. Blood investigations need to be done such as FBE, UEC, ESR/CRP, blood group and Rh typing, coagulation profile, vitamin D, antibodies against rubella and varicella, STI screen, and also a TORCH screen. Urine needs to be given for microscopy and sensitivity. Ultrasound will also be done to check if the pregnancy is viable or not and also to rule out other causes of bleed. As you are not bleeding heavily, and the opening of your birth canal is closed, if the ultrasound shows a normal viable pregnancy, then the specialist might advise you to return home. Once you are at home, you need to avoid overexerting yourself. No activities like no sports, lifting heavy weights. Rest is not usually advisable, because rest will not prevent the miscarriage from progressing. Do not insert tampons into the vagina for the bleed, but you should use pads for the bleed. No sexual intercourse until the symptoms have gone completely for 1 week. Seek urgent

medical advice in the emergency department if the bleeding becomes heavy, any passage of tissues to the bleed, and if the cramping worsens, or if you develop a fever. A repeat ultrasound needs to be done after 1 week. As the pregnancy progresses, you need to 81

come for regular antenatal checks and do a Down syndrome screening. I will start you on folic acid Page

© 2020 ARIMGSAS

which you need to take for the first 3 months of pregnancy. Another ultrasound will be done at 18 weeks, sweet drink test at 28 weeks, a repeat ultrasound at 32 weeks if needs, and a bug test at 36 weeks. Maintain a healthy diet, do not eat any raw meat, no smoking, alcohol or recreational drugs and limit coffee to 2 cups per day. I will arrange a review with you once you are out of the hospital.

***if a woman is Rh negative, a threatened miscarriage up to 12 weeks, there is no need to give anti-D, unless the patient goes in for a complete miscarriage. But if it is any other kind of miscarriage, you need to give the anti-D.

TYPES OF MISCARRIAGES MISSED The patient does not know if she has gone in for an abortion. No bleeding, no pain. No increase in the uterine size. Has amenorrhea, but no corresponding uterine size increase. Do ultrasound and find out that this is a non-viable pregnancy. The uterine size will be smaller than the gestational age.

THREATENED Presents with a mild to moderate bleed, with no issues. The patient will also have mild abdominal pain. When you examine the patient, you can see bleeding from the vagina with no tissues, the cervix is closed. The uterine size will correspond to the gestational age.

INEVITABLE The products of conception have separated from the uterus and started collecting in the lower pole of the uterus, ready to be discharged but have not started discharging yet. The patient will present with a severe bleed with no tissues and severe abdominal pain. When you examine the patient, you can see severe bleed from the vagina with no tissues, the cervix will be open. The uterine size will be smaller than the gestational age.

INCOMPLETE The products of conception have started discharging out. The patient will present with severe bleed with tissues, along with severe pain. When you examine the patient, you can see bleeding with products of conception or tissue, the cervix is open. Can see products of conception coming out from the cervical os. Uterine size is much less than the gestational age.

Book case: Have done the DRABCDE protocol but still, the patient is unstable. You have to clear the products of conception from the cervix as much as possible. Because as long as the products of conception are there, she will bleed like anything.

As an HMO in the ED, do it, don't wait for the specialist to come. 82 Page

© 2020 ARIMGSAS

COMPLETE History of passing blood with tissues and having tummy pain before she came to you. And now what she's having is mild bleeding. Cervix is closed. Uterine size is normal.

83 Page

© 2020 ARIMGSAS

Hyperemesis Gravidarum

We consider it HG if the patient has severe nausea, vomiting, and no weight gain during pregnancy.

Causes: ▪ H-mole (due to beta Hcg) ▪ Multiple pregnancies (due to beta Hcg) ▪ UTI ▪ Normal pregnancy (10%) (due to other pregnancy hormones and beta hcg)

Always ask in the history of the patient can keep something down when eating.

Criteria for admission to the hospital ▪ Signs of dehydration ▪ Electrolyte imbalances ▪ Ketones in the urine ▪ When the patient tells you that she is throwing everything out, not even a sip of water

APPROACH ▪ History: ▪ Vomiting questions: ▪ Content, frequency, duration, blood-stained vomitus (Mallory-Weiss syndrome) ▪ Pregnancy questions: ▪ any bleeding, discharge from down below ▪ Antenatal checks ▪ Water works: ▪ any burning or stinging while passing urine? Any fever?

▪ Physical Exam: ▪ General appearance: signs of dehydration ▪ Vital signs: BP with a postural drop, temperature ▪ Systemic Exam ▪ Abdomen: if the uterus is palpable per abdomen or not (if yes, might be H-mole or multiple pregnancies) ▪ Pelvic exam: uterine size ▪ Office test: ▪ UDT: ketones ▪ BSL: r/o hypoglycaemia ▪ Investigations: ▪ FBE, UEC, serum beta-Hcg, LFT (in some cases of hyperemesis, it can present with derangement in liver function)

▪ Ultrasound ▪ Management

▪ Admission if criteria are met 84

▪ Order her bed rest for the time being Page

© 2020 ARIMGSAS

▪ NPO ▪ Start her on IV fluids ▪ Give her metoclopramide 10mg IV initially, then shift to oral 10mg twice a day; you can also give ondansetron ▪ Give vitamin B6/pyridoxine (anti-nauseating agent or it brings nausea to a great extent) ▪ Further management will depend upon the cause. ▪ UTI: give cefalexin ▪ H-mole: suction curettage

85 Page

© 2020 ARIMGSAS

Secondary Post-partum haemorrhage

You are at your GP clinic, 25-year-old Jane presents to you with bleeding per vagina since today. She had a vaginal delivery, 10 days back.

TASKS 1. Relevant history 2. Get the examination findings from the examiner 3. Discuss your diagnosis and reason for diagnosis with the patient 4. Management

Secondary postpartum haemorrhage: bleeding greater than 500 mL after 24 hours up to 6 weeks after delivery

Causes: ▪ Retained product of conception: placenta or membranes ▪ Laceration, tears, episiotomy wounds ▪ Hematomas in the birth canal ▪ Endometritis

Positive points in the history: worse this morning, used 3-4 pads - heavy bleed, no tissues, not smelly, mild tummy pain since yesterday, feeling slightly flushed with fever since yesterday, continuous fever, a cut was made down below at the time of delivery (episiotomy) Positive points in the PE: temp: 38, BP normal, slight abdominal distention, palpable mass, can palpate uterus in the abdomen, the uterus is tender, OS is slightly open, CMT is negative, the uterus is enlarged to 14 weeks size, slightly tender

APPROACH ▪ History o Is the patient hemodynamically stable? o Congratulate her on her delivery. Ask about how the baby is doing. o Bleeding questions: • How long have you been bleeding? What is the color? How many pads have you used? Are the pads fully soaked? Any clots or tissues that you can see? Is it smelly or not? o Associated symptoms: • Any pain? ▪ Where is the site of pain? How long have you been having the pain?

• Fever

86 Page

© 2020 ARIMGSAS

▪ How long have you been running the temperature? Is it continuous or on and off? Have you recorded the temperature? Were any medications taken? Any rash? • At any time, have you had any offensive vaginal discharge? • Are you feeling dizzy or tired? • Do you know your blood group? o Sources of fever: • Breast ▪ Are you breastfeeding your baby? Any lumps in your breast? • Bladder ▪ Any dysuria, burning, or stinging while passing urine? • DVT ▪ Any calf swelling, or pain? • Respiratory ▪ Any cough, colds, or runny nose? o Pregnancy and delivery • Any complications during pregnancy? Any illnesses during the pregnancy? • I have read from the notes that you had a vaginal delivery, was it a term delivery? • Any complications during delivery like prolonged or obstructed labour? • Any cuts made down below? Is the cut healing well? Have you noticed any discharge or any excessive pain in the wound? (bleeding from the episiotomy wound could be a cause of secondary postpartum haemorrhage) • Any tears or lacerations at the time of delivery? • Any excessive bleeding that you had immediately after delivery? o Is your partner supporting you in looking after your baby? o Are you coping with the baby well? Are you enjoying your motherhood? (rule out post- partum blues) o Do you smoke, drink alcohol, take recreational drugs? o Do you have a good diet? o Have you done your pap smear? What was the result? o Any previous medical and surgical illness?

▪ Physical exam o General appearance: pallor, dehydration, rash, bruises, skin petechia, lymph node enlargements, oedema, cyanosis o Vital signs: temperature, blood pressure o Systemic examination • Abdomen: visible distention, visible mass, palpable mass, is it the uterus that I can feel in the abdomen? Is the uterus tender? Auscultate for bowel sounds • Pelvic exam ▪ Inspection: bleeding, color of the bleed, tissues or clots in the bleed, episiotomy wound is well healed or not, any other lacerations or hematomas, any other offensive discharge that you can see? ▪ Speculum: is the os open or closed? Is there bleeding from the cervix?

▪ Per vaginal exam: CMT, uterine size, tenderness; palpate adnexa for mass, tenderness o Office test: BSL, UDT 87 ▪

Management Page

© 2020 ARIMGSAS

What I am suspecting is a condition called endometritis. The endometrium is the normal inner lining of the uterus. The infection of this endometrium is called endometritis. This is a common infection that can happen after a normal vaginal delivery leading to severe bleeding, or post- partum haemorrhage with fever and abdominal pain. There are certain normal bacteria in the vagina and during delivery this can ascend high into the uterus, causing infection in the lining of the uterus, especially at the site where the placenta has been attached. Another possibility could be a retained product of conception like a part of the placenta or the membranes, remaining inside the uterus and which has got infected. You need to be immediately referred to the hospital. I will arrange an ambulance for you, and I will liaise with the ED of the hospital and I will make them aware of your condition. Meanwhile, I will start you on an IV line, and start you on IV fluids, take blood for all investigations such as FBE, UEC, ESR/CRP, blood group, cross-matching and hold and Rh typing, coagulation profile, and blood culture. A urine sample needs to be sent for microscopic culture and sensitivity. Once you are in the hospital, you will be admitted, seen by the specialist and an ultrasound will be done to look for any retained product of conception. If there is no retained product of conception, you will be started on IV antibiotics (triple regimen: co-amoxiclav + gentamicin + metronidazole) and when you start responding to the IV antibiotics, you will be shifted to oral antibiotics like Augmentin which needs to be continued for 10 days. You will also be given IV oxytocin +/- IV or IM ergometrine (if she is not responding to oxytocin, give ergometrine), and Panadol for fever. If there is a retained product of conception, an exploration under anesthesia will be done and a gentle blunt curettage will be done to remove the products of conception. This will be done under antibiotic cover to prevent infection, and oxytocin and ergometrine will be given. **If the episiotomy is infected, a swab will be done, and the wound will be re-stitched.

88 Page

© 2020 ARIMGSAS

Post CS Endometritis

You are an HMO in the maternity ward. You are asked to see Mary 30 years old who had undergone an elective lower segment C-section 3 days ago for breech at 38 weeks. This is her first baby. The birth weight of the baby is 3.5kg and the baby is doing well. She wants to get discharged today. She's taking paracetamol for pain. She was given 1 dose of ceftriaxone intraoperatively and enoxaparin 2 doses given post-surgery. The patient's vitals chart is given outside the door: BP range 120/70 - 130/80, PR normal, RR normal, O2 sat normal, the temperature on day 1 is normal, but on day 2 it is 38.0C and day 3 it is 37.6C.

TASKS 1. Take a further history 2. PE from examiner 3. Talk to the patient about her findings 4. If allowed, discharge giving reasons.

Causes: ▪ Endometritis ▪ Wound infection ▪ Mastitis ▪ Breast abscess ▪ UTI ▪ DVT

Positive points in the history: minimal bleed, no discharge, day 1 she cannot breastfeed properly but day 2 latching very well Positive points in the PE: pad over the wound is minimally soaked with blood, mild erythema towards the end of the wound, the uterus is extremely tender, cervix slightly open (normal because it takes 5-6 days for it to close), uterus enlarged 16 weeks size, definite uterine tenderness

APPROACH ▪ History o Congratulate her on her delivery. Is she coping with her baby well? Is she enjoying her motherhood? o Look, Jane, I have a bit of concern regarding your situation. Before I decide in letting you go home, let me just ask a few questions. o I have seen in the chart that your temperature has been high for the past few days. Have you been feeling a bit flushed? o Endometritis: • Are you having any tummy pain at the moment? Have you noticed that the tummy

pain has increased, or is it coming down? (by the third day, the tummy pain should be coming down) • How about your bleeding? Are you bleeding from down below? Is it a heavy bleed or a 89

minimal bleed? Page

© 2020 ARIMGSAS

• Any other offensive vaginal discharge? o Pregnancy: • Do you have any other conditions like pregnancy or diabetes? Did the pregnancy go on well? o Wound: • Have you noticed any smelly discharge from your wound? Any excessive pain you have felt around the wound? o Differential Diagnosis: • Atelectasis: are you feeling any shortness of breath? Chest pain? Cough? • Breast: are you breastfeeding your baby? Any problems with the breastfeeding? Any lumps or redness in the breast? • UTI: how are your water works? Any burning or stinging while passing urine? Any chills or rigor that you are feeling? • DVT: (find our why enoxaparin was given) during pregnancy, did you have any calf pain or swelling? Any condition that causes increased blood clot? o Any other medical or surgical illness? ▪ Physical Exam o General appearance: PICCLED BMI o Systemic exam: • ENT: r/o simple URTI • Respiratory: air entry equal, adventitious sounds • CVS: murmurs, added sounds • Calf: tenderness, swelling, • Abdomen: ▪ Wound: discharge from the wound? I would like to remove the pad and have a look at the wound, looking for abnormal discharge, smelly discharge, erythema in the wound margins or along the wound, wound dehiscence? ▪ Uterus: uterine tenderness? • Pelvic exam: ▪ Inspection: bleed, abnormal vaginal discharge ▪ Speculum: cervical os open or closed, bleeding from the os? ▪ Per vaginal: size and tenderness of uterus o Office test: UDT, BSL

▪ Management I am a bit concerned as your temperature has been high for the past 2 days. It is better that you do not go home just yet because I would like to do some investigations to look for the cause of the high fever. There could be several causes for fever after you undergo a C-section. First is endometritis, where there is an infection of the inner lining of the uterus. Endometritis is common after a C-section. In endometritis, you can have a pain in your tummy along with the fever, which you are having right now. The second cause is wound infection. But I am not suspecting a wound infection because your wound appears to be normal and the pain from your wound is coming and down and there is no smelly or abnormal discharge. Another cause is atelectasis. This is a partial lung collapse that can happen after surgery. In this condition,

you can have shortness of breath, chest pain and cough. However, you do not have these symptoms. Another cause is mastitis or infection in the breast, but there are no problems as far as your breastfeeding is going. UTI is another possibility but not in your case because you do not 90

have burning or stinging while passing urine and your UDT has come up negative. DVT or Page

© 2020 ARIMGSAS

increased clotting tendency in your veins could be another cause but you are not having any calf pain and swelling, and you were given 2 doses of blood thinning medications. So, the favourable diagnosis is endometritis. But this needs further investigations such as FBE, blood culture, urine microscopy culture and sensitivity, ultrasound to see if anything is happening in the uterus or not. But all these will be done after you are seen by the specialist. Endometritis can be managed by giving IV antibiotics. Once you start responding to IV antibiotics, you will be shifted to oral antibiotics. So, it would be better for you to stay in the hospital while we find the cause of your fever. Also, I will need to liaise with the specialist as to why the blood- thinning medications were given to you.

91 Page

© 2020 ARIMGSAS

Primary Post-partum hemorrhage d/t retained placenta

You are an HMO in the obstetric unit of a major hospital, and 29-year-old Jessie has just delivered her baby 15 minutes back. The baby is alive and healthy, but after delivery, the umbilical cord snapped, and the patient had heavy bleeding.

TASKS 1. Ask PE from the examiner 2. Explain diagnosis to the patient 3. Discuss immediate management with the patient

Primary post-partum hemorrhage: Greater than 500mL of a bleed within the first 24 hours.

Positive points in the PE: pale, anxious, sweating, unstable vital signs, the uterus is relaxed, the cord is seen 3cm outside the uterus

Causes: ▪ Uterine atony ▪ Retained products of conception ▪ Lacerations, tears ▪ Coagulopathy ▪ Uterine inversion

APPROACH ▪ Physical exam o General appearance: pale, anxious and sweating o Vital signs: BP is 80/60, PR 110-120, normal temp, RR 24, 93% o I would like to proceed with the DRABC protocol • Call for help • Put in 2 large bore IV cannulas • Take blood for investigations: FBE, UEC, BSL, blood group, cross-matching and hold, coagulation profile • Start infusing IV fluids • Start O2 by mask, 8-12L/minute I would like to re-assess all the systems, starting with: ▪ Cardiovascular system, looking for S1 S2, murmurs ▪ Respiratory system: bilateral air entry, adventitious sounds ▪ CNS: cranial nerve examination, neurological exam of the upper and lower limbs ▪ Abdomen:

o Visible distention o Palpation: can you feel the uterus in the abdomen? Is it contracted or relaxed? Uterus is relaxed 92

o Uterine tenderness (if there is scar rupture, uterus will be extremely tender) Page

© 2020 ARIMGSAS

▪ Pelvic o Vulva and vagina: • active bleeding, can you see the cord inside the vagina? The cord is seen 3cm outside of the uterus • Lacerations, episiotomy wounds o Speculum: active bleed from the cervix, can see cord from the cervix, lacerations o Is the cord clamped or not? If not clamped, I would like to clamp the cord close to the introitus o Palpate for the bladder, is it full or not? If full, put a catheter and drain the urine out

▪ Diagnosis Primary post-partum hemorrhage, which is a severe bleed during the first 24 hours of delivery. It could be due to several causes, but the cause we found in you is a retained placenta because the cord has snapped while trying to deliver the placenta, and the placenta was trapped inside. Normally after delivery, the placenta should deliver at around 10 - 20 minutes time. Because of the bleeding, you went into shock, but do not worry, we have done measures to revive you from the shock. ▪ Management Massage the uterine fundus to make the uterus contract. To control the bleed, we will give you 10 units of oxytocin IM/IV. (Never give ergometrine because it causes a tonic contraction of the uterus, and it will be very hard to deliver the placenta) I would like to do gentle controlled cord traction once the uterus starts contracting to deliver the placenta. If the placenta is still not coming out, manual removal of the placenta will be done under anesthesia. After delivering the placenta, bimanual compression massage of the uterus will be done to prevent further bleed.

Just in case she continues to bleed, you can give an intramyometrial prostaglandin (PGF2 alpha) (inject through the tummy wall into the uterus), then try uterine tamponade (Bakri balloon), B- Lynch procedure (put continuous sutures in the uterine wall), uterine artery ligation, internal iliac artery ligation, hysterectomy.

93

Page

© 2020 ARIMGSAS

Mittelschmerz

25-year-old Isa is your next patient at your GP with complaints of abdominal pain since the past 1 hour. She has no history of any medical or surgical illness and has had no allergies so far.

TASKS 1. Relevant history 2. Examination findings from examiner 3. Diagnosis 4. Explain management to patient

Positive points in the history: pain present since morning, but has become worse in the past 1 hour, the right lower part of the tummy pain radiating to the middle of the tummy, cramping pain, getting worse with time, relieves when leaning forward and supporting the tummy with the hand, has spotting LMP: 2 weeks ago, the second episode of this kind of pain, took Panadol during the first time which relieved the pain Positive points in the PE: a little bit of bleed from the vagina and cervical OS, red ring around the cervical OS (cervical ectropion), slight tenderness in the right adnexa

Causes: ▪ Appendicitis ▪ Bowel obstruction ▪ Right sided pyelonephritis ▪ Ectopic pregnancy ▪ PID ▪ Ovarian cyst

APPROACH ▪ History o I have read from the case notes that you've been having abdominal pain for the past 1 hour. How bad is your pain on a scale of 1 to 10? Offer painkiller. Since how long have you been having the pain? Where is the site of pain? Any radiation of pain? What type of pain are you having? Anything that makes is better or worse? o Differentials: • Appendicitis: Any fever, nausea and vomiting? • Pyelonephritis: Any dysuria, burning or stinging while passing urine? • Bowel obstruction: How are your bowel movements? • Any bleeding from down below? Do you need to use any pads? • PID: any other offensive vaginal discharge o Period history: when was your LMP? Are your cycles regular? Any problems with your

periods? At the time of periods, do you experience excessive pain or bleed? o Sexual history: are you sexually active? Are you in a stable relationship? Any history of STIs? Do you use any contraception? What type? Have you done your pap smear recently? What 94

was the result? Page

© 2020 ARIMGSAS

o Is this the first episode of pain? Any interventions done during the first episode? o Past history and Family history o SADMA history

▪ Physical Exam o General appearance: BMI, pallor o Vital signs: temperature o Systemic examination • Abdomen: visible distention, visible mass, palpate for any mass or tenderness, McBurney's point tenderness, Rovsing's sign, renal angle tenderness, auscultate for the bowel sounds • Pelvic exam: ▪ Inspection of the vulva and vagina: any bleed or discharge ▪ Speculum: is the cervix healthy or not? Bleed or discharge ▪ Per vaginal exam: cervical motion tenderness, palpate the uterus for size, tenderness, adnexal mass and tenderness o Office test ▪ Diagnosis and Management The cause of your pain is Mittelschmerz or an ovulation pain or a mid-cycle pain. Normally, during a cycle one of the follicles in the ovary enlarges and becomes mature, and then it breaks open to release the egg or the ovum. During this time, a small amount of follicular fluid and blood is discharged into the tummy cavity which irritates the lining of the tummy causing pain. There is also associated spasm of the tubes, that is why you are experiencing this pain. There is nothing much to worry, no need for a further consult as this is just a normal pain of ovulation and it will go away eventually. You can take simple painkillers like Panadol or Panadeine. You can also put hot packs over the tummy to relieve the pain. Just in case the pain worsens, or you develop any vomiting or fever, please report to the ED, then we'll do a further ultrasound to find out what is happening. If it is interfering with her lifestyle, you can put her on oral OCPs to inhibit ovulation, but NOT if she has cervical ectropion.

The cervix or the neck of your womb has an outer surface called the ectocervix, covered by flat cells (squamous cells), and the inner part called endocervix, covered by tall cells (columnar cells), and these two meets at the squamocolumnar junction or the opening of the cervix. A cervical ectropion happens when the rim of the cervix, rolls outwards exposing the endocervix into the vagina, and this is what you see as a red ring. And this endocervix has the appearance of granulation tissue or a reddish appearance. There is nothing to worry about cervical ectropion. This is more common during pregnancy, teenagers and in women who use oral contraceptives. We can just leave it behind if it does not cause symptoms, but make sure that you are up to date with your pap smear. If you experience symptoms such as excessive mucosal discharge or post-coital bleed, then we need to refer you to the specialist to do ablation techniques using cryotherapy or electrocautery.

95 Page

© 2020 ARIMGSAS

Contraception options

25-year-old Jenny had given birth to a healthy baby boy 2 weeks back. She comes to you to know her options for contraception. She is otherwise healthy and has no issues post-partum.

TASKS 1. Explain the method of contraception available to her.

Key point: Know if the patient is breastfeeding or not o If the patient is breastfeeding, the combined OCP is contraindicated o Mini pills/progesterone-only pills o IM injections - Depo Provera o Implanon o IUCD o Barrier methods Positive points: breastfeeding, on combined OCP before, comfortable with it. ▪ How was your delivery? How is your baby? Are you breastfeeding? Any problems with breastfeeding? Any burning or stinging while passing urine? Have you stopped bleeding? Any abnormal offensive discharge? ▪ I would like to ask you some sensitive questions now. ▪ What sort of contraception was you on before you planned for the pregnancy? ▪ Do you have any contraception in mind other than the oral pills?

**If she is not breastfeeding the baby, she can have combined OCP, which can be started in 6 weeks postpartum

The combined oral contraceptives are not indicated as it contains estrogen and estrogen can reduce the breastmilk production. As long as you are exclusively breastfeeding your baby, combined OCP is not an option for you. As long as you exclusively breastfeed your baby, and as long as your periods do not return, breastfeeding itself acts as a method of contraception, but it is not a reliable or safe method. Options available for you are: ▪ Barrier methods like condoms if your partner is willing, and for your vaginal rings or diaphragms. ▪ Progesterone only pills o Content: progesterone alone, thus it does not interfere with your breastmilk supply o Contraindications: unexplained vaginal bleeding (before she became pregnant, did she have bleeding), breast cancer, liver disorder, stroke, previous history of ectopic pregnancy, or on anti-epileptic medications/enzyme-inducing agents o Action: it acts by thickening the cervical secretions making it hard for the sperm to get through. To a lesser extent, it also changes the lining of the womb so that just in case fertilization happens, implantation is prevented. o Administration: a pack contains 28 pills, all are hormonal pills. You can start taking the pill 3 weeks or 21 days post-partum. Take 1 pill a day, at the same time every day. Protection will

be achieved after taking 3 pills. 96

o Side effects: irregular bleeding, breast tenderness, weight gain Page

© 2020 ARIMGSAS

o It does not protect against sexually-transmitted infections o If you miss a pill for more than 3 hours, you have to take the recently missed pill and just keep going with the rest, but you need to use alternate method of contraception for the next 48 hours. ▪ Depo-Provera o Content: injectable contraceptive which contains progestogen, thus it does not interfere with your breastmilk supply. Medroxyprogesterone acetate o Contraindications: bleeding disorders, breast cancer, undiagnosed vaginal bleed, severe medical illnesses like cardiac illness, if she wants to become pregnant within 1 year (can cause delay in the return of ovulation) o Action: inhibits the secretion of gonadotropins which, in turn, prevents follicular maturation and ovulation and results in endometrial thinning, lesser extent cervical mucus thickening o Administration: IM every 12 weeks, contraception lasts for 12 weeks. Take it 6 weeks postpartum (do not give immediately after delivery even if it can work because depo can go to breastmilk, the baby's liver is not mature enough to handle the depo) o Side effects: amenorrhea, weight gain, breast tenderness, delay in the return of fertility (6 months to 1 year to return after you stop Depo), bone thinning, osteoporosis (especially if given for more than 2 years) o Advantages: high efficacy, compliance, doesn't interfere with oral medications o Disadvantage: does not protect against STIs o Advise lifestyle modifications such as take a diet rich in calcium to prevent osteoporosis ▪ Implanon o Content: progestogen, does not interfere with breastmilk supply. Etonogestrel o Contraindications: bleeding disorders, severe liver disease, breast cancer, enzyme-inducing agents o Action: inhibiting ovulation, cervical mucus thickening and change in the lining o Administration: 4cm rod containing progestogen that will be inserted just below the skin on the inner aspect of the upper part of the non-dominant arm, under local anesthesia; can give Implanon 3 weeks postpartum; contraception acts for 3 years o Side effects: bleeding, amenorrhea, weight gain, breast tenderness, headaches, acne, mood swings o Advantages: high efficacy, compliance, doesn't interfere with oral medications, rapid return of fertility (a major advantage over Depo) o Disadvantages: does not protect against sexually-transmitted infections ▪ IUCD/IUS o There are two types: inert type which contains copper called Multiload, and progestogen/hormonal IUCD called Mirena o Contraindications: previous history of ectopic, active PID, undiagnosed vaginal bleeds, valvular heart disease, increased risk of PID (multiple sexual partners) o Multiload - unknown; Mirena: releases the progesterone into the system which inhibits ovulation and also makes the cervical mucus thick. But all these IUCD, because it is in the uterus, it can prevent the sperm from reaching the egg, and also prevents the implantation of the fertilized egg just in case fertilization happens o Administration: after 6 weeks if NVD, 12 weeks if C-section; Mirena: 5 years, Multiload: 10

years; it is a T-shaped rod with a string attached to its bottom end that will be inserted into your uterine cavity by using an applicator. The string will be hanging down into the vagina which is cut short. It will not interfere with sexual intercourse and will not produce any pain 97

as well. Page

© 2020 ARIMGSAS

o Side effects: tummy pain, low back pain, which usually settles in a few weeks, breakthrough bleeding for 2-3months, which settles by its own. o Complications: risk of PID, extrusion (be expelled by the body) which usually happens in the first month of insertion --Teach self-palpation of strings; needs to check it once a month to ensure that it is in place. Translocation - it can go somewhere else, perforation - can perforate the uterus and go to the abdominal cavity

I will review you in 6 weeks, will give you reading materials about contraception.

98 Page

© 2020 ARIMGSAS

Contraception in Mentally-disabled child

You are visited by the mother of 13-year-old Jessica at your GP clinic. The mother informs you, that Jessica is intellectually disabled and an epileptic as well. She is on carbamazepine for epilepsy. She is worried about the risk of pregnancy in Jessica and wants your advice regarding the methods of contraception that is available for Jessica.

TASKS 1. Relevant history 2. Address the mother's concerns

Positive points in the history: Jessica is mentally-disabled and going to a co-ed school, no periods yet, with breast development and hair growth; severe Down syndrome, partial assistance in her daily activities, the mother is the primary carer; appendectomy when she was 7 years old

APPROACH ▪ History o Why is she concerned about giving Jessica contraception? o I can imagine your concerns, but I need to ask a few questions about Jessica o Period history: • Has she had her periods already? As long as the periods don’t happen yet, she does not need to worry about contraception o Secondary sex characteristics • Breast development • Hair growth in the axilla and pubic area o Sexual history • Is she sexually active? o What type of mental disability is Jessica having? Does she require partial or complete assistance with her activities of daily living? Who is the primary carer? Is she going to a special school? Any problems that she has at school? o Do you have enough support to take care of Jessica? Is your partner supportive? Any other children that you have? o What type of epilepsy does Jessica have? For how long has she been an epileptic? When was the last time that she had epilepsy? Does she have regular consultations with her specialist? Any side effects of the medications that she is taking? Any other medical and surgical conditions? Has she been given Gardasil vaccine?

▪ Management As long as Jessica does not yet have her periods, you need not worry about contraception. As she is having secondary sexual development, I think that she will start her periods soon. I can discuss

with you the options. People with intellectual disability should have the same choices about contraception as other people. But they often need clear information and support to make their own choices. Once Jessica has her periods, I would like to see her, and discuss the contraceptive 99

methods with her, after taking help from her teachers as well. But I am happy to discuss the Page

© 2020 ARIMGSAS

contraceptive options that are available. Combined oral contraceptive, which has to be given in a high dose as she is taking carbamazepine and it needs to be given 1 pill a day preferably at the same time every day. If you think that it will not be a problem for you, I could discuss with you further regarding this. The next option is Depo-Provera which is an injectable contraceptive which needs to be given every 3 months but can cause thinning of the bones or osteoporosis. Sometimes she might stop her periods altogether or she can go in for irregular bleeding as well. What about permanent sterilization? It is not allowed for girls under 18 years of age and being mentally retarded should not deprive your daughter to be treated like other people. I can imagine your concerns, and for this sort of procedure, you require approval from the family code or the Guardianship board who only have the legal authority to allow this kind of procedure. I will give you reading materials and arrange a review with you once Jessica gets her periods.

100 Page

© 2020 ARIMGSAS

Elective induction of labor

24-year-old Rachel presents to your GP clinic. She is 35 weeks pregnant and requests you to induce labor in her as her husband is leaving overseas on a business trip.

TASKS 1. Relevant history 2. Examination findings from the examiner 3. Respond to patient request

APPROACH ▪ History o When is your husband leaving? o Do you have enough support? o Antenatal history o Any headache, swelling, blurring of vision o Any medical or surgical issues? o Have you done your pap smear? What was the result? o Is the baby kicking well? ▪ Physical exam o General appearance: pallor, edema o Vital signs: BP, temperature o CVS/Respiratory/CNS o Abdomen: fundal height (35cm), fetal heart rate (normal), fetal lie, fetal presentation (cephalic) o Pelvic exam: inspection of the vulva and vagina: any lesions and vesicles, any bleeding or discharge o Office tests ▪ Management I can clearly imagine the concerns of your husband; the time of delivery is very important as he is one of the best persons who can offer you psychological support at that time. Have you had a discussion with your husband if he could delay his trip for 8 weeks? With your consent can I have a discussion with your husband? (the patient says that it is not possible for the husband to delay his trip) Induction of labor needs to be considered when the risk-benefit analysis indicates that delivering the baby is a safer option for the baby, mom or both, rather than continuing the pregnancy and when there are no indications for a C-section or contraindications for a vaginal delivery. Induction of labor is not an easy or quick procedure. It has several complicated steps. The conditions where induction of labor is done is:

In the mom: o If it is a post-date pregnancy o Has premature rupture of membranes (do IOL after 36 completed weeks) o Maternal conditions like uncontrolled high blood pressure, diabetes 101

In the baby: Page

© 2020 ARIMGSAS

o If the baby is not growing well or has IUGR o Has infections like chorioamnionitis where the membranes around the baby get infected There are also contraindications for inductions of labor is: o A big baby o Abnormal presentation of the baby (oblique or transverse) o Previous 2 C-sections o A classical or vertical C-section The risks associated with IOL are: o Induction of labor can fail, necessitating an emergency C-section o When you do the artificial rupturing of the membranes, the cord can sometimes relapse, or the cord can become compressed between the birth canal and the baby's head, thereby decreasing the oxygen supply to the baby o The medications that we use to induce labor can cause a non-rhythmic contraction of the uterus which can lead to uterine damage to the muscles that support the pelvis, uterus, and vagina predisposing to incontinence and prolapse. o After IOL, there is a chance of post-partum hemorrhage or excessive bleed as the uterus might not contract properly The most important complication is in the baby: o Prematurity in the baby is another complication of early IOL o The baby can develop breathing difficulties, feeding difficulties, difficulty in maintaining the blood sugar levels of the baby and the body temperature The more that the baby is inside the womb, the better the outcomes of delivery. So, induction of labor is not most appropriate for you and the baby. Inducing labor involves making your baby and your body do something that it is not yet ready to do. So, having said that, induction of labor is not indicated for social reasons. I can give you reading materials regarding induction of labor and the complications associated with that, and you can even give a medical certificate to the husband with her consent.

One of her friends had an IOL at 35 weeks, and she and her baby are well: there may be indications in her that necessitated IOL, that is why it was done. But we never do an IOL for social reasons.

102 Page

© 2020 ARIMGSAS

Ovarian Cyst

You are an HMO in the ED of a major hospital when 25-year-old Natasha presents to you with severe right-sided abdominal pain.

TASKS 1. A relevant, focused history 2. Examination findings from the examiner 3. One relevant investigation 4. management

Differential Diagnosis: • Appendicitis • Cholecystitis • pyelonephritis • Ectopic pregnancy • PID • Ovarian cyst • Mittelschmerz

Positive points in the history: pain is 6/10, pain in the right lower part of the tummy, been there for the past 2 hours, sudden onset of pain, it's a dull kind of pain, nothing is making it better, and getting worse as the time is going by, feeling sick but do not have nausea and vomiting, LMP is 8 weeks ago, planning for a pregnancy, hasn’t done a home pregnancy test nor done antenatal checks, off the contraception for 6 months, has early signs of pregnancy, takes alcohol occasionally Positive points in the PE/investigations: definite tenderness in the right iliac fossa, uterine size is 8 weeks, no uterine tenderness, adnexal mass and tenderness in the right adnexa, 8-week viable uterine pregnancy, and present right ovarian cyst

▪ History o Is my patient hemodynamically stable? o How severe is the pain in my patient? Offer painkiller. o Pain questions: • Where is the site of pain? • What is the duration of the pain? • Is it a sudden or gradual onset? • What is the type of pain? • Is the pain going somewhere else or not? • Anything making it worse or better? • Associated symptoms: are you having nausea and vomiting?

o Waterworks: any burning or stinging while passing urine? Any frequency?

o Bowel: have you opened your bowel today? o Differential questions:

• Ectopic: any vaginal bleed? Any abnormal, offensive vaginal discharge? 103

o Period history: Page

© 2020 ARIMGSAS

• When was your LMP? Is it regular? Any issues with your periods like heavy bleed or pain? o Sexual history: • Are you sexually active? Do you have a stable partner? Were you planning for pregnancy? Have you confirmed your pregnancy yet? o Early symptoms of pregnancy: • Any morning sickness? Breast tenderness? How long have you been off your contraception? o Is this your first pregnancy or not? Have you had miscarriages in the past? Do you have any history of STIs? Have you done your pap smear, and what was the result? o Do you smoke, drink alcohol or take recreational drugs? • If you are found to be pregnant, you have to stop drinking alcohol altogether o Any medical or surgical illness in the past? Do you take any medications? ▪ Physical Exam o General appearance: pallor, dehydration, lymph nodes o Vital signs: BP, temperature o Systemic exam • CVS/Respiratory/CNS • Abdomen: any visible distention, any mass, palpate for any mass or tenderness, any rigidity or guarding, rebound tenderness? McBurney's point tenderness, Rovsing's sign, renal angle tenderness, auscultate for bowel sounds • Pelvic exam ▪ Inspection of the vulva and vagina: any bleed or discharge ▪ Speculum: cervix healthy or not, discharge or bleeding from the cervix ▪ Per vaginal/bimanual exam: CMT, uterine size, tenderness, adnexal mass and tenderness ▪ Investigation o Transvaginal ultrasound • I am looking for an intrauterine viable pregnancy and a possible right-sided ovarian cyst • What is the size of the cyst? What is the type of cyst? (simple or complex) o Other relevant investigations: FBE, B-HCG, BSL, antenatal check blood tests (blood group and Rh, coagulation profile, vitamin D, antibodies against rubella, STI screen) tumor marker: Ca-125 ▪ Management The management of the cyst will depend on the size and type of cyst. Simple cyst < or = to 5cm: The majority will be functional cysts. These cysts usually regress by itself. We could wait and order a repeat ultrasound in 6 week’s time. When you repeat the ultrasound in 6 week’s time, there could be 2 scenarios: The cyst has subsided. So, there is nothing for you to do further. The cyst is increasing in size. If less than 10cm, do a transvaginal aspiration of the cyst under ultrasound

guidance. If above 10cm, do a laparoscopic removal of the cyst. Simple cyst > 5cm or complex cyst of any size: If not so big, repeat ultrasound in 4 week’s time. 104

The cyst has subsided. Page

© 2020 ARIMGSAS

The cyst is increasing in size and a simple cyst. If less than 10cm, do a transvaginal aspiration of the cyst under ultrasound guidance. If above 10cm, do a laparoscopic removal of the cyst The cyst is increasing in size and a complex cyst (has multiple fluid-filled cavities) Do a laparoscopic removal of the cyst. ***If the woman is pregnant, do the procedures after 14 weeks of AOG. If done in the first trimester, it can carry a risk for miscarriage.

Arrange for a specialist consultation. She needs to be given adequate painkillers. Once the pain settles, she can be sent home, and usually, ovarian cysts that occur during pregnancy settles by 12-14 weeks of gestation. Because usually the cyst that is formed during pregnancy is a , and this usually regresses by 14 weeks. However, we need to repeat the ultrasound in 6 week’s time to check the condition of the cyst. Just in case you develop severe abdominal pain, any bleeding or discharge from down below, any excessive vomiting, please report to the ED immediately. When we report the ultrasound in 6 weeks time, the majority of the cyst might have gone, but just in case the cyst is increasing in size, and it has gone up to 10cm, it needs to be treated with a transvaginal aspiration under ultrasound guidance, and if above 10cm, then it needs to be removed via keyhole surgery, and both of this can be done after 14 weeks of pregnancy. We will wait after 14 weeks of pregnancy because one it can regress after that, and two because it carries a risk for miscarriage before 14 weeks. I will arrange for your antenatal blood checks and start you on folic acid. Take a healthy balanced diet, avoid smoking, drinking alcohol, and do go for regular antenatal check-ups.

105 Page

© 2020 ARIMGSAS

Bartholin's Abscess

Your next patient at your GP is 25-year-old Melinda, complaining of a lump in her vulva since the past 2 days. She is feeling severe discomfort to even sit and walk and wants you to help her.

TASKS 1. Relevant focused history 2. Examination findings from the examiner 3. Diagnosis 4. Management

Differential Diagnosis: ▪ Bartholin's cyst: painless lump, no fever, no lymph node enlargement, no discharge from the lump, normal skin colored lump, non-tender, cystic in consistency, with fluctuations ▪ Bartholinitis: no discharge, has a fever, some lumps in the groin, lymphadenopathy present, the lump is reddish-yellow, the local rise of temperature, tender lump, consistency is firm, no fluctuations ▪ Bartholin's abscess: extremely painful lump, purulent discharge is present from the lump, has lumps in her groin, has fever, lymphadenopathy present, reddish local appearance, local rise of temperature, cystic and tense in consistency, there could be fluctuations except if it is tense

Bartholin's glands are glands that are situated at the 4 o'clock and 8 o'clock positions of the minor, and like any other glands in the body, the Bartholin's glands have ducts that open into the vagina. The function of the Bartholin's glands is to produce secretions to lubricate the vagina. If the ducts of the Bartholin's glands get blocked, the secretions collect inside, and this results in a Bartholin's cyst. Once the Bartholin's cyst form, infections can set in, most commonly a Staph aureus. This is Bartholinitis. If this remains untreated, pus starts to form inside and leads to Bartholin's abscess.

Positive points in the history: reddish lump, LMP is 2 weeks ago, periods are usually regular, no problems with periods Positive points in the PE: left-sided inguinal lymphadenopathy which is tender, temp: 38, a reddish- yellow lump at the 4 o'clock position on the left side or 8 o'clock in the right side, tense cystic lump,

APPROACH ▪ History o Are you having pain now? Give painkillers. o Since how long has the lump been there? Was it painful to start with? Is it increasing in size or not? What is the color of the lump? Any discharge from the lump? Where is the exact site of the lump? Any other lumps? Is it a single or multiple lump? Any fever? Any other lumps

and bumps that you have noticed in your groin area? o Rule out sexually transmitted infections: any rash or vesicles? Any other abnormal vaginal discharge? Any bleeding from down below? o Know if pregnant or not: 106

• Period history: when was your LMP? Is it regular? Any problems with your periods? Page

© 2020 ARIMGSAS

o Sexual history: are you sexually active? Do you have a stable partner? Do you use any contraception? Any history of STIs? Any previous pregnancies or miscarriages? Have you done your pap smear? What was the result? o Do you smoke, drink alcohol or take recreational drugs? o Any medical or surgical illness like Diabetes? o Bladder: any problems in waterworks? Do you have burning or stinging while passing urine? o Bowel: do you open your bowels regularly? ▪ Physical Exam o General appearance: any pallor, dehydration? Lymph node enlargement? Is it tender? o Vital signs: temperature? o CVS/Respiratory/CNS o Abdomen: visible distention, mass, palpate for mass and tenderness o Pelvic exam: • Inspection of the vulva and vagina: is a picture of the lump available? Where is the site of the lump? What is the size of the lump? What is shape? What is the color? Any discharge? • Palpation: is there any local rise in temperature? Consistency? Any fluctuations? • Any other abnormal vaginal discharge? Any rash or vesicles? • DON’T GO IN FOR SPECULUM OR VAGINAL EXAM because it is very painful o Office test: BSL, UDT ▪ Diagnosis and Management Most likely what you're having is a Bartholin's abscess. Bartholin's glands are glands that are situated at the 4 o'clock and 8 o'clock positions of the labia minor, and like any other glands in the body, the bartholins glands have ducts that open into the vagina. The function of the Bartholin's glands is to produce secretions to lubricate the vagina. If the ducts of the Bartholin's glands get blocked, the secretions collect inside, and this results in a Bartholin's cyst. Once the Bartholin's cyst form, infections can set in, most commonly a Staph aureus. This is Bartholinitis. If this remains untreated, pus starts to form inside and leads to Bartholin's abscess. I will refer you to the hospital, give you painkillers, and once in the hospital, you will be seen by the specialist. If it is a small abscess, an incision and drainage will be done where a cut will be made on the surface of the gland and the pus will be evacuated. If it is a fairly large abscess, then what will be done is a procedure called marsupialization. This is done under anesthesia, where a cut will be made over the gland, the pus will be drained, and the pus will be sent for microscopic culture and sensitivity, and the cut edges of the gland will be everted and stitched to the skin folds leaving the gland open. This will prevent further collections, and also will promote the healing of the gland. It also prevents recurrences from happening. You may be given antibiotics as well. You can do a hot sitz bath (saline baths) to promote healing and bring down the pain as well. Also, do good genital hygiene.

Could it be STI? STIs can also produce Bartholin's abscess especially gonorrhea and chlamydia. If she thinks that it could be an STI, always make sure that you offer her an STI screen, and also advise her regarding practising safe sex.

I will arrange a review with you once you are out of the hospital. Do you have somebody to take you to the hospital now?

Bartholin's cyst management: 107

Use sitz bath, and the cyst will come down with that. Page

© 2020 ARIMGSAS

Red flags: just in case the lump becomes painful, you start running a temperature, please report to the ED. If the Bartholin's cyst is interfering with sitting and walking, do an aspiration of the cyst.

Bartholinitis: Give analgesics like Panadeine and NSAIDS, put her on antibiotics like cephalexin, or dicloxacillin for 7 days. Arrange a review in a week's time. Red flags: just in case the lump is increasing in size, becoming more painful, any purulent discharge from the lump, you start running high spikes of temperature, go to the ED immediately.

108 Page

© 2020 ARIMGSAS

Trauma during pregnancy

38-year-old Jane is brought to the ED of the hospital, where you are an HMO, with a history of a motor vehicle accident. She's 32 weeks pregnant and is complaining of abdominal pain.

TASKS 1. Focused history 2. PE from examiner 3. Diagnosis and Management

Positive points in the history: pain in the upper part of the tummy (rule out abruptio and preterm labour), it is a dull pain, the pain remains the same, bruise along the line of the seatbelt, the baby is kicking well Positive points in the PE: bruise along the line of the seatbelt in the abdomen, fundal height: 32cm, FHR: within normal limits, longitudinal lie with cephalic presentation

APPROACH ▪ History o How are you feeling at the moment? o Ask for consent. If it is okay with you, could you please tell me how that accident happened? o Were you driving the vehicle? o Were you driving within speed limits? o Were you wearing a seatbelt? o When did this happen? o Were you able to walk out the vehicle alone? (will tell you that she doesn't have any major injuries) o Did you hit your head or tummy anywhere? o At any time, did you lose consciousness? o Are you having any headache now? Any blurred vision? Any nausea or vomiting? o Any neck pains? Any limitations in moving your neck? o Any shortness of breath? Any chest pains? Any pain or limitation of movements of your extremities? o I have read from the case notes that you are having some pain. Where is the site of pain? How severe is your pain? It is a continuous pain or an on and off pain? Is the pain coming at regular intervals? What is the type of pain that you are having? Is the pain going somewhere else or not? Is the pain worsening? o Any vaginal bleed or watery discharge?

o Have you got any bruises over your tummy? o Is the baby kicking well or not? (most important question) o Did you have your regular antenatal checks? What is your blood group? Down syndrome screening? Have you taken your folic acid? Ultrasound at 18 weeks? Sweet drink test at 26 109

weeks? Page

© 2020 ARIMGSAS

o Other medical or surgical conditions? Any medications that you are taking? Any known allergies? o Do you smoke, drink alcohol or take recreational drugs? o Do you have a stable partner? Do you have good support?

▪ Physical Exam o General appearance: pallor, dehydration, oedema, bruises o Vital signs o Head: hematoma, swelling, depressed fracture o ENT: bleed, discharge o Neck: midline tenderness, limitation of movements of the neck, o Chest: accessory muscle breathing, the position of the trachea, air entry, abnormal sounds; S1 S2 normally heard, added sounds, murmurs o Extremities: a range of motion o CNS: cranial nerves, neurological exam of the upper limb and lower limb o Abdomen: fundal height: 32cm, FHR: within normal limits, longitudinal lie with cephalic presentation • Uterine tenderness, uterine contractions: none o Pelvic examination: • inspection of the vulva and vagina: bleed or discharge • Speculum: bleed or discharge from the cervix, OS open or closed? o Office test: UDT, BSL

▪ Diagnosis and Management From the history and examination, there is no major issue that I can find, except for the pain that you are having over your tummy. This pain may be due to the bruise over your abdomen. The baby is doing well, as you can feel the baby kicking and I can also appreciate the foetal heart rate very well. However, this is an accident during pregnancy, you need to be admitted, seen by the specialist, to do an ultrasound to look for complications inside the uterus, and a CTG every 4 hours for the first 24 hours to check on the condition of the baby. Your vitals also need to be monitored, and you need to be observed in the ED for 24 hours as complications like premature labour, and abruptio placentae can sometimes happen, within the first 24 hours. Once you are discharged, if your pain becomes more severe or regular, if you break your waters, or if you have any bleeding from down below, or if the baby is not kicking well, report immediately to the ED. Do you have someone to be with you now?

110 Page

© 2020 ARIMGSAS

Postpartum Pyrexia - Mastitis

28-year-old Emily, mom of a 5-week-old baby, comes to your GP with complaints of tiredness and fever since the past 2 days.

TASKS 1. Focused history 2. PE from examiner 3. Diagnosis and Management

Differential Diagnosis: ▪ Breast: Mastitis/Breast abscess ▪ Birth canal: endometritis, episiotomy wounds, laceration that has become infected ▪ Bladder: UTI ▪ URTI ▪ DVT

Positive points in the history: 2 days history of symptoms, continuous fever, no rash, can feel a lump in the right breast, the lump has been there for the past 2 days, slowly increasing in size, painful when she touches the lump, it is warm, the nipple feels a bit sore, Positive points in the PE: positive lymph nodes in the right axilla, tender, temp 38, abdomen soft and non-tender, a lump at the upper inner quadrant of the right breast, skin over the lump is reddish, the local rise of temperature over the area, regular borders, consistency is firm, mobile lump, normal uterine size

APPROACH ▪ History o Congratulations on the pregnancy. How is your baby doing? o Since how long are you having the fever? Is it a continuous fever or an on and off fever? Have you recorded the temperature? Any rash? Any runny nose, cough or colds? o Are you breastfeeding your baby? Any problems with breastfeeding? Any lumps that you have in your breast? How long have you been feeling the lump? Is the lump increasing in size? Is it warm and painful to touch? Any other lumps that you can feel in the same breast or the opposite breast? Do you have a sore or cracked nipple on that side? Any blood- stained or purulent discharge from the nipple? Is the baby being positioned to the breast correctly? Has somebody taught you the correct positioning of the baby during breastfeeding? o Did you have any conditions during your pregnancy or was your pregnancy uneventful?

What type of pregnancy did you have? Any cuts made down below? Any tears that you had at the time of delivery? Any abnormal foul-smelling discharge from down below? Have you stopped bleeding? Are you having any tummy pain? o Any burning or stinging while passing urine? Any chills or rigor? Any constipation that you 111

are having? Do you open your bowels regularly? Page

© 2020 ARIMGSAS

o Any calf pain or swelling? o Rule out depression: Do you have good support from your partner to look after the baby? Are you enjoying your motherhood? o Do you eat a healthy diet? o Do you smoke, drink alcohol or take recreational drugs? o Have you done your pap smear? What was the result? o Any medical or surgical conditions in the past?

▪ Physical Exam o General appearance: pallor, oedema, lymphadenopathy, is it tender? dehydration o Vital signs: temperature o CVS, Respiratory, CNS o Breast: compare right with the left breast, look for any visible lumps in the right breast, size, shape of the lump, color of the skin over the lump, inspect the nipples for any cracked nipple, palpate for the local rise in temperature, confirm the site, palpate the borders if well-defined or not, palpate the consistency, mobility of the lump, any fluctuations, examine the same breast for any other lumps, and the opposite breast as well o Abdomen: any abnormal distention, any mass, on palpation, do you still feel the uterus in the tummy or has it involuted already? Any other mass or tenderness in the tummy? Auscultate for the bowel sounds o Pelvic exam • Inspection of the vulva and vagina: abnormal discharge, bleed, episiotomy wounds or lacerations • Speculum: cervix healthy or not, OS open or closed, discharge or bleeding from the cervix • Per vaginal exam: uterine size, CMT, tenderness, adnexal mass and tenderness o Office test: UDT, BSL

▪ Diagnosis and Management Most likely you've got a condition called Mastitis of the right breast. This is the cause of your fever and tiredness. Mastitis is an infection of the interlobular connective tissue of the breast caused by bacteria, and the bacteria that usually causes this is staph and e. coli. Where did these bugs come from? It usually enters into the breast if there are any cracks in the nipple, from the baby's mouth or the surrounding area of your skin and this condition will worsen if there is poor milk drainage from the affected breast. The usual cause of the cracked nipple is due to poor positioning of the baby to the breast. You need to continue breastfeeding from the affected side, the milk is not affected by the bugs, and if any bugs come in contact with the milk, the bugs will be neutralized by the baby's tummy. Put the baby on the affected side first so the breast will be drained completely. Before breastfeeding, you can put some hot washers on the breast, so that the milk ducts will dilate, and during breastfeeding, you can massage the lumps towards the nipple. And after breastfeeding, you can put some cold washers on the breast. Take plenty of fluids and take adequate rest. I need to put you on antibiotics such as cephalexin, or flucloxacillin 500mg QID, for 7-10 days, and

analgesics such as Panadol. I will give you reading materials regarding proper breastfeeding techniques. Just in case that there is no improvement in your symptoms, the lump is increasing in size, becomes more painful, you have high spikes of temperature, report to ED immediately. I will 112

arrange a review with you in 1 week’s time. Page

© 2020 ARIMGSAS

Breast abscess management: Do an ultrasound if you have doubts about whether it is mastitis or breast abscess. If it is breast abscess, she required a referral to the hospital. If it is a small breast abscess: do an aspiration of the abscess If it is reasonably big: do surgical drainage under anesthesia Curve-like incision over the breast to drain the abscess. The discharge will be sent for microscopic culture and sensitivity. Put in a drain for 2 days. Antibiotics and painkillers. Go in for temporary weaning from breastfeeding. Make sure the breast is empty by using a breast pump.

113 Page

© 2020 ARIMGSAS

Dysfunctional Uterine Bleed

Your next patient at your GP is 38-year-old Susan who is complaining of heavy menstrual bleed for the past 4 months. She is very bothered about this and wants your advice.

TASKS 1. Further focused history 2. PE from examiner 3. Investigations 4. Diagnosis and Management

Differentials: ▪ Fibroids ▪ Endometritis ▪ PID ▪ Ovarian tumours/ovarian cyst ▪ Thyroid disorders (menorrhagia for hypo usually, but sometimes hyper also) ▪ Bleeding disorders ▪ Blood-thinning medications

Positive points in the history: LMP is a couple of weeks ago, periods come every month, uses 8 pads per day, bleeds 7 days per cycle, no pain, there are clots, worried to go out when she has her periods because she is staining her dresses, feels a bit tired, had 2 pregnancies and deliveries, had a tubal ligation done after her last delivery, Positive points in the PE/Investigations: pale, no postural drop, CMT is negative, uterine size is normal, mobile, no tenderness Hgb is low, iron deficiency, endometrial sampling normal

APPROACH ▪ History o Are you having your periods now or are you bleeding now? No. o Period history: • When was your LMP? • How long have you been having this bleed? • Were your periods normal prior to these past 4 months? • How often do you get your periods? • How many pads are you using per day? Is it fully soaked? • How many days do you usually bleed?

• Do you have pain with your periods? Do you have clots in your periods? • Does it interfere with your lifestyle already?

▪ Reassure her that you will do everything to manage the problem. (key point) 114 • Are you feeling tired or dizzy because of the bleed?

• What is your blood group? Page

© 2020 ARIMGSAS

o Sexual history: • Are you sexually active? Are you in a stable relationship? Have you ever been pregnant? Do you use any contraception? Any history of sexually-transmitted diseases? Have you done your pap smear and what was the result? Any history of post-coital bleed, any pain during sexual intercourse? (keypoints) o Differential questions: • Any lumps in the tummy that you can feel? (fibroids) • Any other abnormal discharge from down below? Any low-grade fever that you are running? Any back pain? (PID) • Any weather preferences? Any recent weight gain or weight loss? Any problems with your bowel habits? (Thyroid) • Any lumps or bumps around your body? (ovarian tumour/ovarian cysts) o Any other medical or surgical illness? Any bleeding disorders or easy bruising of your skin? o Are you taking any medications especially blood-thinning agents?

▪ Physical Exam o General appearance: pallor, BMI o Vital signs: BP with postural drop o CVS/CNS/Respiratory o Abdomen: visible distention, visible mass, palpate for any mass/tenderness o Pelvic: • Inspection of the vulva and vagina: any bleeding, abnormal vaginal discharge? Any rash, vesicles? • Speculum: the cervix is healthy or not? Any bleed or discharge from the cervix? • Bimanual exam: CMT, uterine size, position, mobility, tenderness; adnexal mass and tenderness o Office tests: BSL, UDT

▪ Investigations o FBE especially Hgb and platelet count, if Hgb is low do iron studies, UEC, TFT, blood group and Rh, coagulation profile, LFT, pelvic ultrasound (look fibroids, PID, ovarian cysts or tumours, endometritis), (if you suspect that she is pregnant, do a B-Hcg, but not in this case), endometrial sampling, (if patient is overweight, has DM or HTN, age is greater than 35 refer to specialist to have endometrial sampling done because there is high risk for malignancy in these conditions)

▪ Diagnosis and Management What you are having is a dysfunctional uterine bleed. It is an abnormal uterine bleed that occurs in the absence of any recognizable pelvic disease, general medical conditions or pregnancy. It is due to a disruption in the normal cyclic pattern of your menstrual cycle due to some hormonal disturbance. No organic cause can be found for this condition and the bleeding could be heavy, prolonged, frequent, or random. I will refer you to the specialist who will decide on either of two types of management:

o Conservative • Give this for 6 months. • Non-hormonal management: ▪ Anti-fibrinolytic agents: Tranexamic acid 1-gram 4x/day for 1-4 days of the 115

periods Page

© 2020 ARIMGSAS

▪ Anti-prostaglandin agents: Mefenamic acid 500mg 3x/day 4 days before the start of the periods until the end of the periods • Hormonal management: ▪ Combined oral contraceptives: high dose; period becomes regular, shorter and lighter ▪ Mirena ▪ IUCD ▪ GnRH analogues, Danazol (last resorts) --but DO NOT use for more than 6 months because you will induce a medical menopause Give iron tablets. (key point) o Surgical - consider surgical management if the hormonal conservative management is not working for 6 months • Endometrial ablation: remove the inner lining of the endometrium by using laser or electrocautery • Hysterectomy: if she has already completed her family Give reading materials regarding DUB (ovulatory). I will review you regularly.

**DUB ovulatory: reproductive age; DUB anovulatory: menarche and menopause

116 Page

© 2020 ARIMGSAS

Pubertal Menorrhagia

Your next patient at you GP is 13-year-old Maria, brought in by her mom Jenny. Jenny tells you that Maria had just had her first period and she is having a heavy bleed since the past 5 days.

TASKS 1. Take a further history from the mom 2. PE from the examiner 3. Diagnosis and Management with the mom

Differential Diagnosis: ▪ Pubertal menorrhagia ▪ Pregnancy ▪ Thyroid disorders ▪ Bleeding disorders ▪ Blood-thinning medications

Positive points in the history: first period, use 5 pads/day, complains of mild pain Positive points in the PE/Investigations: the patient is actively bleeding

APPROACH ▪ History o Is Maria still bleeding? Yes. o Is she hemodynamically stable? She is pale, anxious and sweating. BP is 80/60, PR 90 bpm regular, RR 24/min, Temp 36.7, O2 93% o I would like to shift the patient to the treatment room and proceed with the DRABC protocol • Call for help. • Put in 2 large bore IV cannulas, take blood for investigations such as FBE, ESR/CRP, UEC, B-Hcg, thyroid function test, blood group cross matching and hold coagulation profile • Start IV bolus, then as an IV infusion • Start O2 by mask • Once the patient is stabilized, I would like to proceed with the rest of the history from the mom o Since Maria was a little unstable a while ago, we have done measures to stabilize her. She is stable now. I would like to ask further questions about her condition. o Is this her first period? How many days has she been bleeding? What is the severity of the bleed? How many pads has she used per day? Is it fully soaked? Any clots? What is the color

of the bleed? Has she complained of any pain? o Was she feeling tired and dizzy earlier as well? Was she also feeling short of breath? Any chest pain or funny racing of the heart? o I assume that she is not sexually active yet. Has she had her Gardasil vaccine? 117 o Differential questions: Page

© 2020 ARIMGSAS

• Any weather preferences that she has? Any changes in her weight recently? How about her bowel habits? (A thyroid disorder) • Any history of bleeding disorders? Does she bruise easily? (Bleeding disorders) • Any medications that she is taking? o Do you have any concerns about her growth and development? o When you had her periods for the first time, did you have a similar situation? Any elder sisters that she has? Any history of heavy bleeds in her sister? (key point)

▪ Physical Exam o I hope that the vitals of the patient is now stable, so I would like to re-assess all the systems o Respiratory - equal air entry, abnormal breath sounds o CVS - normal S1 and S2, abnormal heart sounds o CNS o Abdomen - visible mass, distention, palpate for mass and tenderness o Thyroid o Skin - abnormal bruising or petechiae o Pelvic • inspection of the vulva and the vagina - any bleed? What is the color? Any clots? Rash and vesicles? • DON’T GO FOR SPECULUM AND PER VAGINAL EXAM o Office test: UDT, BSL

▪ Diagnosis and Management What she is having is pubertal menorrhagia. It is a condition where she gets heavy bleeding during the first periods. Usually, there are many hormones that regulate the menstrual cycle, and a disruption in the hormonal balance can lead to a heavy bleed. It will take quite some time for the hormones to come back to normal. But because she is bleeding heavily and has gone into shock, she requires a referral to the hospital and needs to be admitted and be seen by the specialist. Bleeding has to be stopped immediately by giving her IV Premarin 25mg (high dose of conjugated estrogen). Even with a single dose, the bleeding can be stopped. But if not, she can be given up to 4 doses. This will inhibit the access in the brain that controls the menstrual cycle. But she has to be consequently given oral Progestogen for the next 14 days to compensate for the high levels of estrogen (otherwise she will go in for a withdrawal bleed). Once the active bleeding stops, we need to consider medications or treatment for the next 2 -3 cycles, as it will take around 2-3 months or even up to 6 months for the hormone levels to stabilize. For this, we can put her on non-hormonal conservative management with Tranexamic acid or Mefenamic acid, or hormonal management with combined oral contraceptives, which regularizes her periods and makes it lighter and shorter as well. These are the options that I can put her on for the next 3 months. Please provide her with adequate psychological support during this time and give her adequate nutrition especially iron-rich foods.

If iron is low, start her on iron pills. ***Anovulatory DUB

118 Page

© 2020 ARIMGSAS

Breakthrough Bleeding/ Emergency Contraception

23-year-old Lisa presents to you at your GP clinic to discuss with your certain issues she has regarding the combined oral contraceptives that she is taking.

TASKS 1. Take a focused history 2. Manage the patient accordingly (can include investigations)

Positive points in the history: have been on the pills for 3 months, taking Microgynon 20 (low dose pill), but she has stopped it 1 month ago, having bleeding in between periods, had unprotected intercourse not with stable partner but with her consent, does not take pill at the same time every day, LMP is 2 weeks ago, Positive points in the PE/Investigations

APPROACH ▪ History o What are your concerns regarding the pills? o How long have you been on the pills? o What type of pill are you taking, low-dose or high-dose? o Was your intercourse with a stable partner? Was it with your consent? o What contraception were you on before you were started on the pills? Why were you switched to the combined pills? o Period history: • When was your last menstrual period? • Any issues with your periods after stopping the pills? o Combined pill history: • Do you take it at the same time every day? • Do you take other medications like anticonvulsants? o Do you smoke, drink alcohol, take recreational drugs? o Any medical or surgical illness in the past? o Any history of STIs? o Any pregnancies or miscarriages in the past? When was your last pap smear? What was the result?

▪ Diagnosis and Management Lisa, what we can give for you is emergency contraception. It's an effective and safe way of preventing unplanned pregnancies. It is around 95% effective when it is taken within the first 24

hours of unprotected intercourse. Example of this is Postinor or otherwise called morning-after pill. It contains a high dose of progesterone and is available in 2 strengths 750mcg and 1.5mg. It acts by preventing ovulation if ovulation hasn’t happened yet, or by preventing fertilization if ovulation has already happened, or by preventing implantation if fertilization has already 119

happened. Postinor 1 is 1.5mg of Levonorgestrel which you can take 1-tab STAT, and Postinor 2 is Page

© 2020 ARIMGSAS

750mcg which you can take 1 tab now and after 12 hours take the 2nd dose. If the patient has a history of gastric irritation or any peptic ulcer, you can consider giving Postinor 2 over Postinor 1, otherwise, Postinor 1 is the first choice. Along with this, we will give an antiemetic like Maxolon. After 4 weeks, we will review you to check if you have your periods back. If no bleeding within that time, we will do a pregnancy test. And because you had unprotected intercourse with an unstable partner, we could also do an STI screen with your consent. Advise also about safe sex practices. Other alternatives: YUZPE method: high dose combined OCP which contains 50mcg of ethinyl and 250mcg of Levonorgestrel. Give 2 pills initially, and after 12 hours, give another 2 pills. However, this is not as effective as Postinor and is only around 60-75% effective in protecting against pregnancy. Danazol: 200mg tab, 2 tabs STAT, and after 12 hours, another 2 tabs. All these 3 medications can be given within 72 hours after unprotected intercourse. If a patient comes in after 72 hours, up to 5 days, you can give copper-containing IUCD and Ulipristal. It is a selective progestogen released later. It is given as 30mg single tab dose. After 5 days, there is nothing you can do.

Breakthrough bleeding: Once you are started on combined pills, around 20% of the women can have a bleed during the early months of taking the pills. It could be due to not taking the pills preferably at the same time every day. Cycles are extremely sensitive and with combined pills, we are adding hormones from outside, so it is preferable and better that we add these hormones at the same time every day so the cycles will get used to this. The other causes of this sort of a bleed could be a low dose of estrogen that is when the estrogen is not enough to maintain the stability of the endometrium of the inner lining of the womb so that it breaks down causing an irregular bleed. Another factor is the type of progestogen in the pill as well. And breakthrough bleed is more common with first- generation progestogens rather than 2nd or 3rd generation progestogens. Other contributing factors would be smoking and certain medications that increase the efficacy of the drugs. It could also be due to STIs. As far as you're concerned, what we can do is, after you have your next periods, if you're happy, you can continue with the pills. The breakthrough bleeding usually settles in 3-6 months. Make sure you take the pill at the same time every day. And if the bleeding is continuing for more than 6 months, then we can put you on a high dose combined pill which has a different progestogen as well. Just in case you are not happy with the pills at all, I am happy to discuss with your other methods of contraception that is available for you.

120 Page

© 2020 ARIMGSAS

SLE Pre-pregnancy Counselling

You are a GP and your next patient is 25-year-old Mary, who was diagnosed with SLE 5 years back. She wants to start a family and is now seeking your advice regarding her future pregnancies.

TASKS 1. Counsel the patient 2. Answer her queries

Positive points in the history: skin rash, joint pain during a bout of SLE, was on prednisolone, off it for 3 months, has regular follow-ups, the last consult was 3 months ago

APPROACH ▪ History o How long have you been diagnosed with SLE? o What symptoms did you have when you had your SLE? o Do you experience any symptoms of a kidney problem? (key point) o Any symptoms of SLE now? How long have you been symptom-free? • Criteria for pregnancy: Symptom-free for 6 months • (If epilepsy 2 years to 6 months, if DM 3 years) o What medications were you put on? How long have you been off the medication? (key point) o Do you go for regular appointments with your specialist? When was the last consult? o Have you ever been hospitalized? o When are you planning for your pregnancy? o Know if she has good reproductive health: • Period history: when was your LMP? Is it regular? Any problems with your periods? • Have you ever had miscarriages or pregnancies in the past? (key point) • Any history of STIs? (can result in tubal blockage and cause ectopic pregnancy) o Do you smoke, drink alcohol or take recreational drugs? o How is your diet? o Any other medications you are taking? Any other medical illness?

▪ Diagnosis and Management As you are already aware, SLE is an autoimmune disorder in which certain factors called antibodies to develop against your own cells leading to inflammation of the body tissues especially the skin, the joints, and the kidneys. Can I become pregnant like any other females?

SLE is not known to interfere with pregnancy, so you can go for a normal pregnancy and have a healthy baby even if you have SLE. But just in case you have been put on a certain type of medications, called cytotoxic medications as part of the treatment regimen, your fertility can become low. 121

What are the effects of pregnancy on SLE? Page

© 2020 ARIMGSAS

In most women, in which SLE has remained stable without any symptoms, especially during the last 6 months, as in your case, flare-ups of SLE are unlikely. There is only a 30-40% chance of a flare-up during pregnancy. And even if you have a flare-up, it will be mild to moderate only. What are the effects of SLE on pregnancy? Reassure her that many women with SLE go for a normal pregnancy and delivery. But sometimes, it can lead to complications in you and the baby. The complications in the mom include: • Miscarriages: incidence is 20% • Preeclampsia: incidence is 20% • Premature labour The complications in the baby include: • Intrauterine growth retardation: this happens because blood clots can form in the placenta and that can interfere with the nutrition of the baby • Birth defects: especially if the mom is on medications • Prematurity: if the mom goes in for premature labour • Neonatal Lupus syndrome: after birth; it is not SLE in the baby. The baby can present with skin rash and some unusual blood counts and it usually settles in 3-6 month’s time. But if the baby has neonatal lupus syndrome, then there is a high chance that the baby can develop SLE in the future. These are the effects of SLE on pregnancy.

Counselling: We have to do some blood tests such as routine antenatal blood checks, and along with that, we have to screen for antiphospholipid antibodies and also thrombophilia screen to make sure that you are fit for pregnancy. We can also do a urine microscopy culture and sensitivity. 6 months being symptom-free is one of the criteria for considering pregnancy. Start on folic acid 3 months before you consider pregnancy, and for 3 months after you become pregnant. Once you are pregnant, you will be referred to as the high-risk pregnancy clinic. A MDT which includes the obstetrician and rheumatologist will look after you. You have to go for more frequent antenatal checks, ultrasounds, and blood screening. If ever you get a flare-up of SLE, it will be managed by the specialist using steroids and sometimes you may be started on blood-thinning medications like aspirin or LMWH as well. Usually, the steroids will be at the lowest dose that is necessary to control your symptoms, and it usually does not cause any complications in the baby. However, in you, it may cause a high rise in blood pressure and blood sugar levels. You can have a normal vaginal delivery if everything goes well with you and the baby. But usually, after delivery, there could be a flare-up of SLE. Are you a member of the SLE Association of Australia? If not, refer her to that. A healthy diet, exercise. Give reading material about SLE.

122 Page

© 2020 ARIMGSAS

Mitral stenosis in pregnancy

You are at your GP, but 32-year-old Jane presents to you at 20 weeks of your pregnancy, with complaints of shortness of breath. This is her first pregnancy.

TASKS 1. Take a further history 2. PE from examiner 3. Discuss your diagnosis and management with the patient

Differential Diagnosis: ▪ Anemia (a most common cause of SOB in pregnancy) ▪ Uraemia ▪ Asthma ▪ Cardiac causes ▪ Stress-related

Positive points in the history: gets short of breath when she walks, goes away when she rests, occasionally has palpitations, no chest pain, feels tired sometimes, had rheumatic fever at the age of 7 years, took medications for 3-4 months only, no ultrasound at 18 weeks Positive points in the PE/Investigations: loud S2, low-pitched rumbling diastolic murmur best heard in the apex at the left lateral position with little radiation

APPROACH ▪ History o Are you feeling short of breath now? No. o How long have you been feeling short of breath? Is it continuous or on and off? When do you usually get short of breath? What relieves it? o Do you become short of breath while you sleep? (PND) Do you need to prop your head up with pillows during the night? o Any funny racing of the heart? Any chest pains? o Are you feeling tired? o Any cough, fever, any wheeze? Do you have a history of asthma? o Any calf swelling or pain? Any history of travel? o Anemia: • Does your diet include meats, green leafy vegetables? • Are you bleeding from anywhere? Any black tarry stools? Any history of heavy bleeding during periods before you became pregnant?

• Is this your first pregnancy? (if not first pregnancy, ask the gap between two pregnancies, because a short gap may predispose to anemia) o Any history of heart problems, high blood pressure, diabetes, other illness? o Were you on any long-term treatment? 123 o Do you smoke, drink alcohol, or take recreational drugs? Page

© 2020 ARIMGSAS

o Any stress at work or home? o Pregnancy: • Did you do your regular antenatal checks? Blood group? Down syndrome screening? Folic acid? Ultrasound at 18 weeks? ▪ No ultrasound at 18 weeks -- address that. Tell her that she needs an ultrasound now. • Have you felt the baby kicking? Any tummy pains? Any bleeding or discharge from down below? • Do you have good support? • When was your last pap smear and what was the result?

▪ Physical Exam o General appearance: pallor, cyanosis, lymphadenopathy o Vital signs: HR, RR with saturations, BP with postural drop o Respiratory: normal air entry, any accessory work of breathing (nasal flaring, tracheal tug, overactivity of the pectoral muscles and SCM, indrawing of the intercostal muscles) adventitious breath sounds, basal crepitations (present in some cardiac conditions) o CVS: JVP raised or not, any visible pulsations, dilated veins, any thrill or parasternal heave, any carotid bruit, normal S1 and S2, any added sounds? o Abdomen: tender splenomegaly, fundal height, foetal heart rate, foetal lie, presentation o Calf: swelling, tenderness o Office test: UDT, BSL, ECG

▪ Diagnosis and Management What I’m suspecting is mitral stenosis in pregnancy. Draw the chambers of the heart. The heart has 4 chambers, two lower pumping chambers which we call the ventricles, and two upper receiving chambers which we call the atrium. Normally, blood flows from the atria to the ventricles, and this blood flow is regulated by valves. The valve on the left side is called the mitral valve. In mitral stenosis, the opening of this valve becomes narrow so that there is a restriction of blood flow from the left upper to the left lower chamber. This causes a sort of backpressure in the lungs, leading to shortness of breath. This mitral stenosis could be a complication of the rheumatic fever that you had in childhood. You have become symptomatic now because during pregnancy, there is an increase in the blood volume and the heart needs to pump more blood. I will refer you to a cardiologist, and he will arrange further tests like echocardiography which is a scan of the heart and its blood flow, for the confirmation of the diagnosis and assessing the severity, and the functioning of the heart. Further blood tests also need to be done such as an FBE, ESR/CRP, UEC, TFT, LFT, and ultrasound. I will also refer you to the high-risk pregnancy clinic where you will be looked after by the obstetrician, cardiologist and anesthetist. Mitral stenosis can carry certain complications in pregnancy as well. It can lead to heart failure and also preterm labor. In the baby, low birth weight and prematurity if the mom goes into preterm labor. As these complications can happen, you need to go for more frequent antenatal visits. Just in case heart failure happens, you need to be admitted to the hospital and will be managed with medications. Ultrasound scanning will also be done frequently with CTG.

You can have a normal vaginal delivery up to term, but it should be in a tertiary hospital under the guidance of the specialist, with continuous monitoring of your vitals and continuous CTG for the baby as well. You will be given oxygen during labor and after delivery. You will be offered excellent pain relief in consultation with the anesthetist, like epidurals. Usually, the second stage of labor is 124

cut short by using instruments such as forceps or vacuum. Close monitoring will be done for at Page

© 2020 ARIMGSAS

least 48 hours after delivery. I will give you reading materials regarding mitral stenosis in pregnancy, and I will arrange a review with you after you consult with the specialist

125 Page

© 2020 ARIMGSAS

Polycystic Ovarian Syndrome

You are at your GP clinic when 20-year-old Amanda presents to you with a history of irregular periods over the past 1 year after no periods at all for the past 3 months.

TASKS 1. Take a further history 2. PE from examiner 3. Investigations with examiner 4. Advise the patient regarding further management

Differential Diagnosis: ▪ Pregnancy ▪ PCOS ▪ Premature ovarian failure ▪ Post-pill amenorrhea ▪ Asherman's syndrome ▪ Thyroid disorders ▪ Hyperprolactinemia ▪ Anorexia or Bulimia ▪ Stress-induced amenorrhea, exercise-induced amenorrhea ▪ Medications - anti-psychotics

Amenorrhea: no periods for 6 months if she is having regular periods, and no periods for 3 months if she is having irregular periods

Positive points in the history: at the start it was irregular, but it became regular, but it became irregular in the past 1 year, in the beginning, the bleed was moderate then it became mild then it stopped, not sexually active and has never been sexually active, gaining weight, sister had a similar condition, has excessive hair growth, acne Positive points in the PE/Investigations: BMI is 27, 130/85, abdomen is soft and non-tender

APPROACH ▪ History o When did you have your first period? Were your periods regular until the past 1 year? How often do you get irregular periods? Since the past 1 year, have you noticed any change in the amount of bleed? Any pain or clots? o Are you sexually active? Have you ever been sexually active or not? Have you taken the

Gardasil vaccine? o Differential questions:

• POF: do you have any hot flushes, heavy sweating, mood changes 126

• Thyroid: any weather preference? Any changes in bowel habits? Page

© 2020 ARIMGSAS

• Hyperprolactinemia: any headache, blurring of vision? Any milky discharge from the nipple? • Eating disorders: do you think that you are overweight? Do you try to lose weight through crash dieting or excessive exercise? • Exercise-induced amenorrhea: do you exercise regularly and how many hours do you exercise? • Stress-induced amenorrhea: Any stress at home? Any stress with your family? o PCOS: • For what duration have you gained the weight? • Any excessive hair growth anywhere? • Any acne? • Any frequent urination, frequent thirsts? (DM is a complication of PCOS) o Do you smoke? (smoking predisposes to PCOS) o Any family history of a similar condition especially in your mom or sister? o Any previous medical or surgical conditions? Any medications that you are on?

▪ Physical Exam o General appearance: hirsutism, acne, BMI, pallor, LN o Vital signs: BP o CVS, Respiratory, CNS o Thyroid exam o Abdomen: any visible mass, distention o Pelvic: • Inspection of the vulva and vagina • Speculum and Per vaginal exam CAN be done in PCOS, but NOT in this case because she is virginal. o Office test: BSL, UDT

▪ Investigations o FBE, UEC, ESR, CRP, LFT, serum lipid profile o TFT o FSH/LH ratio (Normal 2:1) - to confirm PCOS o Testosterone levels o Pelvic ultrasound - greater than 10 follicles less than 10mm in size, this is PCOS o ROTTERDAM CRITERIA

▪ Diagnosis and Management Normally during each menstrual cycle, even though several follicles develop in each ovary, one of the follicles becomes mature and then will break open to release the egg. PCOS is complex condition in which, the ovaries are larger in size, and they develop many small immature follicles or cysts, none of which grows to maturity and breaks open to release the egg. In other words, ovulation does not happen. That is why your periods have become scanty and finally stopped. There are also other hormonal disturbances in PCOS, like the body becomes resistant towards

insulin, which is a hormone that keeps your blood sugar level in check. This causes a rise in the blood sugar level, leading to Diabetes. The ovaries also secrete a small amount of male sex hormones called testosterone which gets out of balance leading to weight gain, hirsutism and

acne. The exact cause of PCOS is unknown but it has been found to run in families. This can also 127 Page

© 2020 ARIMGSAS

lead to complications like Diabetes, high blood pressure, high cholesterol levels, infertility and even depression.

I will refer you to the specialist. You have to employ lifestyle modifications. (key point) I will refer you to a dietician for a proper diet chart. You need to follow the advice of the dietitian as even a 10 per cent reduction in the body weight can normalize your cycles. And once the cycles are normalized, all the other hormones can also be put back to place. The body becomes more responsive towards insulin, and the testosterone production also stops. We will also start you on low dose combined OCPs to bring back your menstrual regularity, decrease hirsutism and acne and to decrease the insulin resistance and lipids to a certain extent. We will also start you on Metformin. In PCOS, the LH is remaining high all throughout the cycle that is why the follicles are not maturing. Metformin can bring down the LH levels, it restores the menstrual regularity and can decrease the free testosterone levels. GnRH analogues. If the patient is not responding to any of the above medications, this can be used. This will suppress the HPO axis and bring about the regularity of the periods. If medical management does not work, we think about surgical management. What we do is laparoscopic ovarian drilling. Under general anesthesia, using keyhole surgery, laser or electrocautery is used to break through the thick outer layer of the ovary, and then destroys small portions of the ovary. When the small portions of the ovary are destroyed, the testosterone production comes down drastically. If you break any part of the circle, all the rest will fall back into place.

Meanwhile, because these medications take some time to act, you can go in for cosmetic therapies, like laser or electrolysis. She can be put on combined oral contraceptive which contains the 3rd-generation progesterone (has anti-androgenic property) cyproterone acetate (Dyne 35) I will arrange a review with you in 1 month’s time.

------Infertility + PCOS Management: Lifestyle modifications first for 6 months. If not working, put on clomiphene citrate + Metformin. If not working, laparoscopic ovarian drilling.

128 Page

© 2020 ARIMGSAS

Recurrent Miscarriages

37-year-old Lisa presents to your GP clinic. She thinks that she's pregnant now as her home pregnancy test has turned out to be positive. She gives a history of having 3 miscarriages before.

TASKS 1. Take relevant history 2. Counsel the patient

Positive points in the history: LMP is 6 weeks ago, regular periods, no problems with periods, feels a bit sick in the morning, all miscarriages were around 8-10 weeks, during the 2nd miscarriage a D&C was done but, in the rest, it just miscarried

Recurrent miscarriage: greater than or equal to 3 miscarriages which is consecutive. Causes: ▪ Unknown ▪ Chromosomal abnormalities in the mother/father or baby (parental or embryonic) - most common cause of first-trimester miscarriages ▪ Immune-mediated (APAS, SLE, thrombophilia) ▪ Infections (TORCH, STIs, Hepatitis B and C) ▪ Uterine abnormalities (cervical incompetence, gynecological surgeries, septate or bicornuate uterus) ▪ Endocrine causes (DM, Thyroid disorders) ▪ Epidemiological factors (advanced maternal age, number of previous miscarriages - after 3 consecutive miscarriages, the chance of a miscarriage is 40%) ▪ SAD

APPROACH ▪ History o I have read from the case notes that you had a positive home pregnancy test, how are you feeling about it? o Reassure her that you will look into the matter, and you will be there to support and help her. o When was your last menstrual period? Is it regular? Any problems with periods? o Were you planning for a pregnancy now? Do you have a good support? o Any early signs of pregnancy like nausea and vomiting, breast tenderness, mood changes? o Condition of the pregnancy now: Are you having any tummy pain, bleeding or discharge from down below?

o Causes questions: • Any history of STIs? Have you done your pap smear? What was the result? What is your blood group? • Any weather preferences? Any changes in your bowels? (thyroid) 129

• Any history of diabetes of high blood pressure? Page

© 2020 ARIMGSAS

• Have you ever been tested for immunity against Rubella? Any pets at home? (Toxoplasma - mainly the cat litter that contains the Toxoplasma) Any raw meat in your diet? (Toxoplasma) • Do you smoke, take alcohol or take recreational drugs? • Have you had any history of skin rashes and joint pains? (SLE) • Any increased clotting tendency in your leg veins? (APAS, thrombophilia) o Any other medical conditions that you have? Any medications that you are taking? o Have you started taking your folic acid? o Miscarriages • I'm sorry to hear about your 3 miscarriages before. At what weeks of pregnancy did you have your previous miscarriages? • At what age did you have your miscarriages? • When was the last miscarriage? • Any successful pregnancy for you so far? • Any trauma or infections prior to these miscarriages? • Were you with the same partner with all these miscarriages? • What interventions were done at the time of miscarriages? • Any analysis being done on the foetal parts at that time? • Have you and your partner received counselling after the miscarriages?

▪ Counselling o Confirm the pregnancy by doing the UPT o Explain the causes one by one o One of the most common cause of recurrent miscarriages during the first trimester is genetic abnormalities, although we have to look at other causes as well, that is why we need to arrange some investigations such as: • Routine: FBE, UEC, ESR/CRP, LFT, BSL, UDT, Urine microscopy culture and sensitivity • Antenatal: blood group and Rh, antibodies to rubella, varicella, • Causes: TORCH screen, TFT, thrombophilia screen, antiphospholipid antibody screen, STI screen including hepatitis B and C ▪ I will refer you and your partner for karyotyping, and we will also do a pelvic ultrasound o I will refer you to a high-risk pregnancy clinic. You need to go for regular antenatal checks, and it is always advisable to do a Down syndrome screening as well (key point because she is already 37 years old). Make sure she is put on folic acid (key point) o Just in case any of these investigations turn out to be positive, you will be managed accordingly o Lifestyle modifications - healthy diet, regular exercise, avoidance of alcohol altogether o Reading materials regarding recurrent miscarriages. Arrange a review once the blood test results are out. I will also refer you to the ’s clinic.

▪ What if there is a defect in me or my partner? o You will be referred to a clinical geneticist, and he will be the best person to give advice for

you. ▪ Do I need to have a cervical stitch? o Cervical stitch is usually given for cervical incompetence and given 14 weeks before pregnancy. Cervical incompetence is one of the most common causes of miscarriages during 130

the 2nd trimester. But since you are having all your miscarriages in the 1st trimester, it is Page

© 2020 ARIMGSAS

unlikely that you have cervical incompetence. But during the ultrasound, we will look for this condition. And if you have it, then a cervical stitch will be done for you.

***3 recurrent first trimester miscarriages, come in for counselling. You will be given a card of investigations. All will be there EXCEPT karyotyping. ALWAYS look for ALL the investigations, so ORDER a karyotyping.

131 Page

© 2020 ARIMGSAS

Mobile head at term

Jane who is 28 years, who is 40 weeks pregnant is referred to you at your GP by the midwife as the midwife is concerned that the baby's head is not getting engaged.

TASKS: 1. Further history 2. Examination findings from the examiner 3. Management with the patient

Causes of mobile head: ▪ Occipitoposterior position ▪ Cephalopelvic disproportion o Big baby o Birth defects in the baby o Pelvic bone deformities (osteomalacia, Rickets) o Uterine abnormalities o Fibroids ▪ Polyhydramnios ▪ Placenta previa

Positive points in history: all normal Positive points in the PE/Investigations: fundal height is 40cm, FHR normal, longitudinal lie with the cephalic presentation, baby's head is 2-3 cm above the pelvic brim,

APPROACH ▪ History o How is your pregnancy so far? Is this your first pregnancy? (key point) any tummy pain, any bleeding or discharge from down below? Is the baby kicking well? o Antenatal history: • Do you go for regular antenatal checks? • Blood group and Rh? Have you had your blood checks done? • Ultrasound at 18 weeks? Repeat ultrasound at 32 weeks? Is it a single baby? Any birth defects in the baby? Have they commented on the position of the placenta? Any excess fluid in the bag surrounding the baby? Any benign growths or fibroids in the uterus? • Sweet drink test at 26/28 weeks? Bug test at 36 weeks? o Any headache, blurring of vision, any swelling? (preeclampsia symptoms)

o Any other medical or surgical illness? Any medications taken? o Do you have a good support? o Do you smoke, drink alcohol or take recreational drugs? 132 ▪ Physical Exam Page

© 2020 ARIMGSAS

o General appearance: pallor, LN o Vital signs: BP o CNS, CVS, Respiratory o Abdomen: fundal height, FHR, lie and presentation, is the baby's head engaged or not? (key point) o Pelvic exam: • Inspection of the vulva and vagina: bleed, discharge, rash, vesicles • Speculum: os is open or closed? Discharge or bleed from the cervix? • Don't go in for a per vaginal exam as a GP o Office test: UDT (rule out proteins), BSL

▪ Diagnosis and Management From the history and examination, I can see that the baby's head is still not engaged, or it has not gone into the pelvis. Usually, we expect the baby's head to get engaged by 36-38 weeks, if it is a first pregnancy thereby preparing the baby to come out during labour. At the moment, many pregnant women whose baby's head has not yet engaged, when they go into labour, because of the uterine contractions, the baby's head will get engaged eventually and will have a normal vaginal delivery. But there could be certain conditions which can prevent the baby's head from getting engaged and one of the most common causes is an occipitoposterior position. Normally, the position of the baby is occipitoanterior when the back of your baby's head will be towards your front. However, in occipitoposterior, it is the opposite, the back of the baby's head is towards your back as well. Because of this, the baby usually has a de-flexed head, or the baby cannot tuck his chin towards his chest. The de-flexed head presents into the pelvis, with a wider diameter, making it more challenging to descend. Even if the baby is in an occipitoposterior position, you can still have a normal vaginal delivery, but the delivery will be a bit prolonged. You will be given adequate and proper hydration and excellent pain relief during that time. But just in case the labour does not progress well, or if the baby becomes unwell, then a C-section needs to be done. Another possibility is a cephalopelvic disproportion, which could be due to a big baby or a narrow birth canal. Also, polyhydramnios could be a possibility where there is a lot of fluid surrounding the baby, but unlikely in your case because your fundal height corresponds to your gestational age. It can also be due to a placenta previa, when the placenta gets attached to the lower pole of the uterus preventing the baby's head from coming down, but unlikely also in your case as this should have been detected during your repeat ultrasound, and usually, the delivery should have been done by around 37/38 weeks by C-section. Another possibility is fibroids, but this should have been detected as well by the ultrasound. What I think is that the head might go down within the next few days. I would like to refer you to the specialist. Have an ultrasound done, to find the cause if any. And also, if the pregnancy is prolonging beyond 40 weeks, then a Doppler ultrasound needs to be done as well to check the placental perfusion because after 40 weeks, the blood supply in the placenta can interfere or there can be placental calcification. CTG should be done now, and also 2 times per week if the pregnancy is progressing. You will be checked for cephalopelvic disproportion by the specialist. If it is positive, then delivery will be done by C-section. Meanwhile, maintain baby kick charts. And just in case you develop any tummy pain, any bleeding or discharge from down below or if you think that the baby is not

kicking well, please report to the ED immediately. I will give you reading materials regarding mobile head at term. I will arrange a review with you after you are seen by the specialist.

133

Page

© 2020 ARIMGSAS

Vaginal Birth after Caesarean Section

Case: your next patient at your GP is Jenny, a 28-year-old lady, who had previous delivery by CS 2 years back. She's 8 weeks pregnant now and wants to discuss possibilities of having a vaginal birth this time.

TASKS 1. Take relevant history 2. Ask examiner for previous medical and surgical notes of the C-section 3. Discuss the possibility of a vaginal birth this time with the patient

C section: maximum of 3 C sections ▪ Can't advise a vaginal birth after 2CS procedures ▪ Type of C-section is important ▪ Must ask for the baby's weight: may have CPD ==> narrow pelvis ==> cannot go for a VBAC ▪ History of other uterine/pelvic surgeries

Advantages of vaginal birth ▪ Can have any number of deliveries ▪ Shorter stay in the hospital ▪ Pain duration will be less

History: current

APPROACH ▪ History o I read from my notes that you are 8 weeks pregnant, is this a planned pregnancy for you? Congratulate her o How did you confirm your pregnancy? Home pregnancy test, no antenatal checks so far => confirm the pregnancy! • I will do a confirmatory office PT as well o When did you have your LMP? 8 weeks ago o Are your periods regular? o Do you have any breast tenderness? Morning sickness? Yes o How long have you been off your contraception? o Do you have any tummy pain, or bleeding, or discharge from down below? o How's your diet? Do you regularly exercise? o SADMA o I have read that you had a previous C section done. When was it done? 2 years ago • Was it an elective or emergency C section? Emergency C-section was done

• Why was it done? Sort of obstruction during the labor ▪ Think of CPD! Do you know the weight of your baby at birth? 4.2kg • What type of C-section was done on you? [draw a photo]

▪ The scar is along the lower border: bikini cut ==> likely a transverse C section 134 Page

© 2020 ARIMGSAS

• Did you have any complications during your previous pregnancy? Like high blood pressure or diabetes that you had? • Did you have any complications after the surgery like any excessive bleeding? Infections? Or any complications? Clotting in your veins? • Did you have any other surgeries done on your womb apart from the c section? None o Do you have enough support for this pregnancy o Past Medical History o Family History

• Examiner o What is the reason for the c section: the obstructed second stage of labor o What is the cause of obstructed labor? Was there any cephalopelvic disproportion [KEYPOINT]: the baby was big, but the pelvis was adequate o At what gestational age was the C-section done? o What is the type of C-section done? [key point]: low uterine segment • Vertical ==> high risk of rupture o Any complications during or after surgery? o How long until the patient was discharged from the hospital (normally should be 3 days) o Any previous uterine/pelvic surgeries done to her? [key point] o How was the condition of the baby after birth?

• Counselling o Vaginal birth after a C- section is an option for all women who had a previous c section provided that the indication of the previous C- section does not recur and in many women, successful vaginal birth could be achieved safely for both mom and the baby. The success rate of vaginal birth after c section is 55-85%. o In your case, the previous c section was done as the baby was a little big and your labor was not progressing smoothly. But this is not a recurring condition and your baby might not be that big this time. And from the notes, your pelvis is not narrowed but quite roomy as well. At present, you do not have any contraindications for the vaginal birth, and other points in favor for it is the type of C-section, which is a lower segment C section, and also you don't have any previous uterine surgeries. o You are in the early weeks of pregnancy now, and as the pregnancy progresses, if any complications develop in you like uncontrollable high blood pressure, diabetes, or bleeding during pregnancy, placenta previa, then a C-section needs to be considered again. Also, certain complications in the baby like the big weight of the baby, or any abnormal presentation or lie of the baby in the womb can lead to a C-section. o There are certain advantages of vaginal birth over the C-section. It avoids the risk of C- section like complications of anesthesia, excessive bleeding, infection of the womb, and also injury to other organs. The pain during the delivery will be short, and also you will have a shorter duration of stay in the hospital. o [Key point] If you have one successful vaginal birth after a C-section, you can go in for any number of vaginal births afterwards.

o VBAC carries risks as well. These include failure of the vaginal birth which will necessitate an emergency C-section, and there is a risk of scar rupture (1:200), and a chance to develop endometritis or infection of the womb. Repeated C-sections can lead to placenta accrete, a

condition where the placenta grows deep into the C-section scar of your womb. If you have 135 Page

© 2020 ARIMGSAS

one more C-section, the next deliveries should always be by C-section and it is advisable not to have more than 3 C-sections.

• Further Management o Confirm management o Do all antenatal blood checks o Start on folic acid o Advice regarding down syndrome screening o Needs to go for shared antenatal care with ultrasound done at 18 and 32 weeks, sweet drink test at 26-28 weeks. During each visit, you will be monitored for any complications. And if any complications happen, you will be managed at the high-risk pregnancy clinic. o [KEYPOINT] I need to arrange for a specialist consultation at 26 weeks for discussion about the possible mode of delivery, and another at 36 weeks for a definite decision. o During delivery, you and the baby will be continuously monitored and the delivery should be done in a tertiary hospital, under specialist guidance. You can also have excellent pain relief options like an epidural at the time of delivery. You will be continuously watched for any possible rupture due to the previous CS you had. o Here are reading materials regarding VBAC to give you more insight about this. o Please observe to eat a healthy diet, and engage in regular exercise. Please avoid smoking, alcohol, or recreational drug use. o I will arrange a review with you once your blood tests are out o Are you happy with this plan?

o HOME VBAC: not usually advised. Only if it is supervised by a specialist

136 Page

© 2020 ARIMGSAS

Home Birth Advice

Next patient at your GP is Rachel, a 25-year-old primigravida who is 30 weeks pregnant. Until now, her pregnancy has been uneventful. Her 18 weeks ultrasound revealed a single baby, proper position of the placenta. Her OGTT was normal. She has come to see you today to discuss with you home birth options.

TASKS 1. Take a further history from the patient 2. Examination findings from the examiner 3. Advice regarding home birth

▪ History Ask the reason why she wishes to have a home delivery Home delivery is a good option, but there are certain concerns. Labors are very unpredictable and can go wrong at any time. In a hospital, interventions will be there. It doesn't always necessarily mean that she would have a C-section, as doctors wouldn’t do unnecessary procedures on her. "Know your midwife program" - only one midwife who will monitor her from the beginning Family birth suites - home-like environment attached to the hospital run by midwives

APPROACH ▪ RAPPORT ▪ History o How is your pregnancy going so far? o Any tummy pain, bleeding, water discharge from down below? o Is the baby kicking well or not? o Signs of preeclampsia: any headache, blurring of vision, any edema? o When was your last antenatal check? Are you reviewed regularly? • Do you know your blood group? Down syndrome screening is done? • Ultrasound at 18 weeks done? Results? • Folic acid taken o How's your diet? Do you regularly engage in physical exercise? o SAD o Past Medical History o Family History o Why do you prefer a home birth? [KEY POINT] her friend had a home birth 2 weeks back and she said it was a wonderful experience for her. And she prefers to have a home birth so that she'll be in a familiar environment and not surrounded by strangers, and she need not go anywhere once the contractions start. The doctors may also put her on unnecessary

interventions like a C-section. • I will discuss this in detail with you, but before that is it ok if I examine you?

• Physical Examination 137

o GA: pallor, edema (PICCLED) Page

© 2020 ARIMGSAS

o VS: BP o ABDOMEN • Fundal height; 30cm • Fetal heart rate: normal • Fetal lie: longitudinal • Fetal presentation: cephalic o OFFICE TESTS: UDS, BSL

• Counselling I can understand that you want a home delivery as you have said that it can be a more familiar environment for you where you could feel more comfortable, but home birth has got certain limitations which I will discuss with you now. As I can see, until now your pregnancy has been going very well but you still have 8-9 weeks to go before your delivery. Just in case you or your baby develop any complications like a sharp rise of blood pressure in you, any bleeding, if you have an early rupture of membranes or preterm labor, that is you go into labor before 36 weeks, then home birth is NOT advised. At any time if the baby becomes unwell or if the baby kicks become less, home delivery is not advisable. The complications are more likely to happen during your first pregnancy rather than the subsequent ones. Now coming to labor, all labors are unpredictable, and complications at the time of labor sometimes cannot be foreseen. The first thing we have to consider is whether your pelvis is roomy or not, and as you have no previous deliveries, we do not know how the pelvis is going to behave. There could be fetal distress at the time of labor (baby can be unwell at the time of labor), or there could be a cord prolapse when the membranes rupture, or at any stage, the labor can become obstructed. There could be excessive bleeding during and after delivery as well. So it is always safe to have a delivery at the hospital especially your first delivery, as if any complications arise, they will be tackled with utmost efficiency. At the hospital, all the necessary equipment are there to monitor you and your baby's well-being. At no stage will unnecessary interventions be done unless absolutely necessary. But by the end of the day, it is your choice and if you're quite sure that you want to have a home birth, we need to make a back-up plan in consultation with the specialist. For that, I need to know how far you are living away from the hospital, The transport facilities available for you, and if you have enough support. I can enrol you to the Know Your Midwife Program. The same midwife will take care of your pregnancy and also your delivery and after delivery as well. She will be knowing everything about you as well--your pregnancy, conditions, etc. but just in case you need to have a look at the hospital maternity ward, I can arrange for that as well. Another option is to give delivery at birth centres or family birth suites which is a home-like maternity care facility. A kind of half-way between the home and the hospital. And it is always attached to a hospital and is run by a team of midwives. The suite offers a relaxing environment but with full equipment. Your family members can also be there with you during the time of delivery. And just in case any complications happen, you will be taken to the hospital immediately. But the only problem is that they cannot give you epidurals during your labor, but what they can offer you is non-medical pain relief regimens.

I can refer you to a specialist who will be the best person to discuss this with you. And I can also give you reading materials regarding this. I can fix up another appointment with you, and if you wish you can bring your partner along as well. 138

Do you have any questions? Page

© 2020 ARIMGSAS

C-section Counselling

You are at your GP when Susan, 25 year old, at her 20th week of her first gestation presents to you with a request of C-section to be done as she does not want a vaginal delivery.

TASKS 1. Take a further history 2. Counsel the patient

APPROACH ▪ History o How is your pregnancy going so far? o Why do you prefer a C-section over a vaginal delivery? Scared of the pain associated with the vaginal delivery • Must focus on pain relief options. Explain that pain duration in c section is more o Have you started to feel the baby kicks? Any tummy pain, discharge, or bleeding, from down below? o Are you having regular antenatal checks? Do you know your blood group? Are blood tests done? o Down syndrome screening done? Is ultrasound done at 18 weeks? ==> no ultrasound so far. (address this in the management--I will arrange for an ultrasound now and give you a referral for that) o SAD o Past Medical History o Family history o Do you have enough support for this pregnancy?

• Counselling Vaginal birth is a natural method of delivery, whereas as you know, C-section is a surgery where we put a cut in the lower part of your tummy and your womb to deliver the baby, and the placenta, and its membranes under anesthesia. We usually go for a C-sections if there are definite indications. C section can be done in a planned or an elective way when there are certain indications like placenta previa, cephalopelvic disproportion if detected early in the pregnancy (due to abnormalities in the baby---when there is a big baby, or abnormalities in your pelvis--like if there is a narrow pelvis), any abnormal lies of the baby (transverse or oblique), previous2 CS or a vertical CS, multiple pregnancies An emergency CS can be done in the following situations: if the baby becomes unwell, or if there is arrested labor at any point in time. if the patient has got eclampsia, or if the cord prolapses.

C-sections carries certain complications to you and to your baby. Immediate complications can be a risk of anesthesia--breathing difficulties, bleeding problems, injury to the surrounding structures like the bowel or the bladder. These complications are rare when done by a trained obstetrician. You may develop clots in your legs after the surgery, or develop an infection in the womb 139

(endometritis) later. For complications in the baby, he can go in for respiratory distress, breathing Page

© 2020 ARIMGSAS

problems are more common in C-section babies compared to the babies delivered via vaginal delivery. Disadvantages you may have with a C-section include: there is a longer duration of pain that you'll have when compared with vaginal birth. With a vaginal birth, you have the pain at the time of delivery but for a C-section, the pain continues for the next few days until the healing of the wound takes place. C-section would entail a longer stay in the hospital (3-5 days), and there can be problems with future attempts of vaginal birth like scar ruptures. Here are some advantages of the vaginal birth over a C section: there are no complications like bleeding or risk of anesthesia, or infection from the womb. The pain duration is less. You will have a shorter hospital stay. Recovery is quick, and you can have any number of future vaginal deliveries. However, there are some risks with vaginal birth as well. These include a failure which can lead to a C-section, there could also be some damage to the pelvic floor muscles that can lead to future complications like incontinence. But it can very well be managed if you start doing pelvic floor muscle strengthening exercises after delivery. As I know that you are concerned with the pain associated with vaginal birth, there are excellent pain relief options that are available for you at the time of labor. It could be pharmacological or non-pharmacological. o Pharmacological options: • epidural, where anesthetic drugs will be introduced into the outer covering of your spine which can numb the nerves to your womb and also the muscles surrounding it. And it can be topped up anytime. • Second pain relief options are injections of pethidine • The third is by giving nitrous oxide and oxygen inhalation via a mask o Non-pharmacological methods • Using TENS (transcutaneous electrical nerve stimulation) where 2 electrodes are placed on either side of the spine and a small electric current will be passed which can inhibit the pain fibers. • Certain positions during labor can also reduce the pain • Deep breathing techniques can also be used as well • Hydrotherapy can also be done, which is giving birth while on water You are only at 20 weeks, and there is still a long way to go. If any complications will occur, we may go ahead with the C-section if indicated. But again, by the end of the day, it is your choice. If you still think that C-section is the best option for you, I'll arrange an appointment with the specialist, and the specialist can discuss this further with you. Now, I'll give you a referral for the ultrasound scan, reading materials regarding vaginal birth and c section, and also pain relief options that are available during your labor and delivery.

OTHER CASE: the reason they give you: incontinence following vaginal birth Vaginal birth is not alone is not a cause of incontinence. If you have other causes like chronic constipation, extra weight gain, and menopause, all these can contribute to incontinence as well. But vaginal birth can lead to weakness to the pelvic floor muscles, but if you start

strengthening exercises to the pelvic floor muscles, called the KEGEL exercises, 6 weeks after delivery, you can prevent this to a certain extent. There are post-natal classes you can enroll in, who will teach you the Kegel exercises. 140

Page

© 2020 ARIMGSAS

141 Page

© 2020 ARIMGSAS

Primary Amenorrhea

You are a GP, and 17-year-old Maya presents with a complaint of not starting her periods yet.

TASKS 1. Focused history 2. PE from examiner 3. Discuss diagnosis and differential diagnosis with patient 4. Investigations and Management

Primary amenorrhea: does not start periods by 15-16 years old in the presence of normal secondary sexual characteristics or by 13-14 years old with no secondary sexual characteristics

Causes: ▪ Hypothalamic causes o Eating disorders o Exercise-induced o Stress-induced o Chronic illnesses (severe liver or renal disorders, diabetes, severe depression) o Kalman’s syndrome (primary amenorrhea + anosmia) ▪ Pituitary causes o Hyperprolactinemia ▪ Ovarian causes o PCOS o Premature ovarian failure o Turner's syndrome (streak ovaries) o Chemotherapy or radiotherapy to the ovaries ▪ Uterine causes o Pregnancy o Mullerian agenesis (absent uterus + hypoplastic or absent vagina; with normal secondary sexual characteristics) o Androgen insensitivity syndrome (gonadically male, but phenotypically female; absent uterus; with normal secondary sexual characteristics) ▪ Vaginal causes o Imperforate hymen o Transverse vaginal septum ▪ Other hormonal causes o Hyper/hypothyroidism o Cushing's syndrome

Positive points in history: has breast development, has pubic and axillary hair growth, not sexually active Positive points in the PE/Investigations: Tanner stage 5

142 Page

© 2020 ARIMGSAS

APPROACH ▪ History o What are your concerns? o Have you had any spotting? o Any cyclical tummy pains? o Secondary sexual characteristics: • Do you have proper breast development? • Do you have pubic and axillary hair growth? o Rule out PCOS • Any recent weight gains? • Any excessive hair growth especially on your face? • Any acne? o Rule out POF • Any hot flushes, heavy sweating, mood changes? o Rule out thyroid problems • Any weather preferences? How are your bowel habits? o Rest of Differentials • Anorexia: Any eating disorders? Do you think that you are excessively overweight? ▪ Do you try to lose weight by crash dieting, or self-starvation? ▪ Do you exercise? How often? How many hours per day? • Hyperprolactinemia: any headache, blurring of vision, milky discharge from the breasts? • Diabetes: any frequent urination, frequent thirsts? • Kallman's syndrome: any problems with your smell? • Depression: how is your mood? How is your appetite, sleep? o Sexual history • Are you sexually active? Have you ever been sexually active? Have you taken the Gardasil vaccine? o Constitutional delayed puberty: • Do you have any sisters? When did they have their menarche? How about your mom? o Any stress at home? Any stress at school? o Any other medical or surgical condition such as liver or kidney problems? o Any medications that you are taking? (antipsychotics can lead to hyperprolactinemia which can, in turn, lead to primary amenorrhea) o Any family history of genetic abnormalities?

▪ Physical Exam o General appearance: BMI, dysmorphic features (features of Turner's syndrome: webbed neck, short stature, broad chest with widely spaced nipples, wide carrying angle; Androgen/PCOS: hirsutism, acne, virilising features) o Vital signs o Thyroid o CVS/Respiratory/CNS

o Abdomen 143

o With the consent of the patient, ask the examiner for the Tanner staging Page

© 2020 ARIMGSAS

• Axillary hair • Pubic hair • Breast development o Pelvic examination • Inspection of the vulva and vagina: external genitalia (imperforate hymen - usually presents as a bulge and bluish in color; atrophic vagina - premature ovarian failure, pubic hair) o Office tests: UPT

▪ Diagnosis and Differential Diagnosis o Draw the hypothalamo-pituitary-ovarian axis This is the normal centres of the brain, called the hypothalamus and the pituitary which secretes hormones that in turn will act on the ovary. The ovary will then secrete estrogen and progestogen, which acts on the uterus causing periods. A disruption at any part of this axis, can lead to an absence of periods or primary amenorrhea. This could be due to several causes. Sometimes, lifestyle factors can contribute to this, like eating disorders, when you have excessively low body weight, which interrupts many hormonal functions in your body thereby preventing ovulation. It could also be due to excessive stress or exercise which alters the functioning of your hypothalamus which controls the hormones that regulate your cycle. It can also be due to other chronic medical conditions like liver or kidney problems, diabetes, or severe depression. It can be due to hormonal imbalances as in polycystic ovarian syndrome where there are relatively high and sustained levels of hormones, rather than the fluctuating levels necessary for a normal cycle. And an overactive or underactive thyroid, and pituitary tumours leading to increased secretion of a hormone called prolactin, can also lead to amenorrhea. It could be due to premature ovarian failure, where the ovaries fail to respond to the hormones from the brain. It can be due to structural problems like a lack of reproductive organs such as the uterus, cervix and the vagina, in genetic conditions like Mullerian agenesis. It could be due to a simple outflow tract obstruction, like imperforate hymen, where the membrane blocks the outflow of blood from the uterus. It could also be due to genetic defects such as Turner's syndrome and Kallman's syndrome.

▪ Investigations and Management Basic investigations: FBE, UEC, LFT, RFT, TFT, serum prolactin Further investigations: o Normal pubertal development: pelvic ultrasound: look if the uterus is present or not • If the uterus is present, the first thing to look for is an obstruction in the outflow tract ▪ If the obstruction is present, it could either be an imperforate hymen or transverse vaginal septum ▪ If the obstruction is absent, do a further test called progesterone challenge test (this test is done to look if your body is producing adequate estrogen to prime the uterus for periods; give 5mg medroxyprogesterone acetate x 10 days. Then withdraw the progesterone, if the uterus is growing well and estrogen is present, the person will have a withdrawal bleed) • If the uterus is absent, do a karyotyping and look for androgen insensitivity syndrome

and Mullerian agenesis o Estimate LH and FSH • If low, it can be due to eating disorders, exercise, stress-induced, chronic illnesses • If high, it is the ovaries that are not responding 144 ▪

Order a karyotyping and look for Turner's syndrome Page

© 2020 ARIMGSAS

▪ Order estrogen and progestogen estimations to look for Premature ovarian failure

145 Page

© 2020 ARIMGSAS

Secondary amenorrhea - Exercise-induced

Maria presents to your GP with complaints about the absence of periods since the last 6 months.

TASKS 1. Further history 2. PE from examiner 3. Discuss management with the patient

Secondary amenorrhea: ballet, dancing, swimming --often associated Absence of periods for 3 months if she is having regular periods and 6 months if she is having irregular periods

Causes: ▪ Pregnancy ▪ PCOS ▪ Premature Ovarian failure ▪ Post-pill amenorrhea ▪ Asherman's syndrome ▪ Thyroid disorders ▪ Hyperprolactinemia ▪ Eating disorders ▪ Exercise-induced amenorrhea ▪ Stress-related amenorrhea

Positive points in the history: not planning for a pregnancy, partner uses condoms, no period for the last 6 months, the period has become irregular for a few months before it stopped, heavy bleed before it stopped, ballet dancer, does ballet dancing 3 hours per day, tries to stick on a healthy diet because she doesn't want to put on weight Positive points in the PE/Investigations: BMI: 18.5

APPROACH ▪ History o What are your concerns? o Do you get spots at the time of your normal periods? o How were your periods before it stopped? o How do you quantify your bleed before it stopped?

o Any pain at the time of periods? o Sexual history

• Are you sexually active? Are you in a stable relationship? Were you planning for 146 pregnancy?

• What contraception were you using? Page

© 2020 ARIMGSAS

• Have you ever been pregnant? • Have you noticed any weight gain? Any excessive hair growth or acne? o Any menopausal symptoms like hot flushes, heavy sweating, mood changes? o Any weather preferences? How are your bowel habits? o Any headache, blurring of vision, milky discharge from the nipples? o What is your occupation? • How many hours of ballet dancing do you do per day? o Are you much bothered by your weight and appearance? o Have you ever thought that you are overweight? o Do you try to lose weight by crash dieting or self-starvation? o Any other exercise that you do? o How is your diet? o How is your home situation? Any stress at home? Any stress at work? o SADMA history o Past history o Family history

▪ Physical Exam o General appearance: BMI, pallor, dehydration • PCOS: any excessive hair growth, acne • The elasticity of skin - low estrogen can decrease the elasticity o Vital signs o Thyroid o CVS/Respiratory/CNS o Abdomen: visible distention, mass, palpate for mass and tenderness o Pelvic examination • Inspection of the vulva and vagina: any discharge or bleeding, look for atrophic vagina • Speculum exam: cervix healthy or not, any discharge/bleed • Bimanual exam: CMT, uterine size, tenderness, adnexal mass and tenderness o Office tests: UPT, UDT, BSL

▪ Diagnosis and Management Most likely you have a condition called exercise-induced amenorrhea. Amenorrhea is when you have a loss of periods for around 6 months after established periods. Draw the HPO axis. The estrogen and the progesterone that is released by the ovary, under the influence of the hormones secreted by the hypothalamus and the pituitary is responsible for a normal menstrual cycle. These sex hormones could be affected by a range of factors like excessive exercise when certain exercise-related hormones like beta-endorphin and catecholamines are released. The high levels of these hormones suppress the areas in your brain which alters the secretion of estrogen and progesterone. The other contributing factors could be eating disorders and low levels of body fat and also the emotional stress that is associated. So, an imbalance between the exercise and increased nutritional demands along with stress can bring about amenorrhea, as it starts

interfering with the normal hormonal pathways. If this is not treated, this can lead to long-term complications, like the decrease in the fertility or decrease in the bone density and the bones become brittle and break easily, increased cholesterol

levels and also premature aging. 147 Page

© 2020 ARIMGSAS

But first, we need to rule out all other possible causes of amenorrhea. We have already done a pregnancy test, and it has come out to be negative. All the basic bloods need to be done like an FBE, UEC, LFT, RFT, TFT, vitamin D (25-OH vitamin D), serum prolactin, serum lipid profile, estimate all the hormones like GnRH, FSH, LH, estrogen, progesterone, and also a pelvic ultrasound. You will be referred to the specialist. Adopt lifestyle modifications: o Try to put around 2-3 kilos of weight. I will refer you to the dietician who will give you proper dietary advice. o I will put you on calcium and vitamin D supplementation. o Limit exercise to a maximum of 8 hours per week. If these measures are not working in 6 months’ time, then we can put you on combined oral contraceptives. I will give you reading materials regarding exercise-induced amenorrhea and I will arrange a review with you in 1 month.

148 Page

© 2020 ARIMGSAS

Recurrent Candidiasis

You are at your GP when 30-year-old Lisa presents to you with complaints of itching and burning sensation in her vagina, with discharge. She tells you that this is the 4th time that this has been happening for her during the past 3 months. The last time, her GP had done culture and it came out to be severe Mobilises, and she was treated with Nystatin cream for 5 days. She is pretty much worried about this and wants you to address her concern.

TASKS 1. Further history 2. PE from examiner 3. Discuss further Investigations and Management with the patient

Recurrent candidiasis: 4 episodes or more in 1 year Causes: ▪ Pregnancy ▪ Immune deficiency states ▪ Medications like steroids ▪ Antibiotics ▪ Oral contraceptive pills ▪ Diabetes ▪ Usage of vaginal douches or creams

Positive points in the history: 4 episodes in the last 3 months, has , on combined OCP Positive points in the PE/Investigations: erythematous vulva and vagina (brick red vagina - candidiasis)

APPROACH ▪ History o What are your concerns? o What symptoms are you having now? o Any discharge? What is the color of the discharge? Any blood stains? Is it smelly or not? (in candidiasis, it has no smell) Is it white, curdy or cheese-like? o Any ulcers, rash? o When was your last menstrual period? Is it regular? Any problems with periods? o Are you sexually active? Are you in a stable relationship? Do you have dyspareunia? What contraception do you use? Since how long have you been on OCP? What type of pill are you taking?

o Any symptoms in your partner? (it is not sexually transmitted, but still, the partner can also present with symptoms), any history of STIs? o Any pregnancies in the past? When was your last pap smear and what was the result? o Causes of recurrent candidiasis: 149

• Diabetes: any history of diabetes? Any frequent thirsts, frequent urination? Page

© 2020 ARIMGSAS

• Antibiotics: have you taken any repeated doses of antibiotics? • Any vaginal creams or douches that you have used? • Any other medical or surgical illness? • Any medications you are taking, especially steroids? • Any known allergies? o How are your bowel and bladder habits? Any burning or stinging while passing urine? o Do you use tight clothing or tight jeans, pantyhose? (can also predispose to recurrent candidiasis)

▪ Physical Exam o General appearance: BMI, pallor, lymph nodes, rash o Vital signs: temperature, BP o CVS/Respiratory, CNS o Abdomen: visible distention, mass, palpable mass and tenderness o Pelvic examination • Inspection of the vulva and vagina: discharge, color of the discharge, is it smelly, any swelling or erythema of the vulva or vagina • Speculum: cervix healthy or not, discharge or bleeding, erythema of the cervix • Per vaginal: DON'T do because you will be carrying infection inside the vagina o Office test: UDT, BSL

▪ Investigations o Basic bloods: FBE, UEC, LFT (you will give an antifungal, you want to know if the liver is okay), RFT, TFT, ESR/CRP o 2 vaginal swabs for microscopic culture and sensitivity for confirmation

▪ Diagnosis and Management What you are having is recurrent Candidiasis or commonly known as Thrush. It is a common yeast infection that affects many women and could be irritating and painful. Four or more episodes in 1 years’ time are classified as recurrent Candidiasis. It is caused by an overgrowth of a fungus called Candida albicans, which is normally present in your vagina. The other healthy bacteria in the vagina usually prevents the overgrowth of Candida. However, changes in your lifestyle and other health conditions like Diabetes, or conditions where your immunity goes low, pregnancy, use of medications like steroids, and use of combined oral contraceptives cause Candida to multiply rapidly leading to thrush.

*There are different protocols for the management of recurrent Candidiasis. What we will follow are RACGP guidelines. a. First, we induce symptom remission by using oral antifungal agents like fluconazole 50mg once daily, or itraconazole 100mg once daily for 2 weeks or even up to 6 months until your symptoms completely subside. We can also use clotrimazole 1% or nystatin cream intravaginally at night. (here we do not prefer that because she was already put on nystatin before and she is not responding)

b. Once symptom remission is achieved, we need to maintain the remission by taking fluconazole or itraconazole weekly for 6 months. You can also use clotrimazole 500mg pessary or nystatin intravaginally weekly for 6 months.

General measures: 150 Page

© 2020 ARIMGSAS

o Stop the combined oral contraceptives as the estrogen in the pills can favour the growth of Candida. You have other options for contraception like mini pills or progestogen only pills, or injectable like Depo-Provera, implants like Implanon, or IUCD like Mirena, or your partner can also use condoms if he is willing. o Even though it is not an STI, better to refrain from sexual intercourse until the symptoms subside. o Meanwhile, maintain good genital hygiene, wear cotton (not synthetic) undergarments, avoid wearing tight-fitting clothing like tight jeans and pantyhose, and if you go for swimming, remove the wet clothing immediately after swimming. I will give you reading materials regarding recurrent candidiasis and I will review you in 2 weeks' time.

151 Page

© 2020 ARIMGSAS

UTI in Pregnancy/Pyelonephritis

You are an HMO in the ED of a major hospital. Your next patient is an 11-week pregnant lady, married, presenting with nausea and vomiting.

TASKS 1. Further history 2. PE from examiner 3. Investigations 4. Diagnosis and Management

Positive points in the history: vomiting for the past 2 days, can take some food and fluids, no diarrhea or loose stools, has on and off fever, took Panadol, has bit of pain the lower tummy, has burning or stinging while passing urine, dark coloured urine and a little smelly, due for antenatal check in a few days' time, did a home PT to confirm pregnancy Positive points in the PE/Investigations: temp: 38.0, uterine size 11 weeks,

Differentials: ▪ Hyperemesis gravidarum ▪ AGE ▪ Acute abdomen ▪ UTI

APPROACH ▪ History o Is my patient hemodynamically stable? o What are you concerns? o Since how long have you been vomiting? o What is the color of the vomitus? Any blood stains? It is projectile or does it fall far from you rather than near to you? Any relation with eating or meals? Can you keep something down? (if she cannot keep something down, you need to admit her) o AGE: How are your bowel habits? Any diarrhea or loose stools? o UTI: • Any fever? ▪ Is it a continuous or on and off fever? Have you recorded the temperature? ▪ Any rash? ▪ Any medications taken? Is it working? • Any tummy pains?

▪ Any pain in your back as well? (rule out pyelonephritis) ▪ Any chills or rigors? • Any burning or stinging while passing urine? ▪ What is the color? Is it smelly or not? 152 o Pregnancy Page

© 2020 ARIMGSAS

• Have you had your first antenatal checks? • How did you come to know of your pregnancy? ▪ We need to confirm your pregnancy with office test UPT as well • Any early signs of pregnancy? • How long have you been off your contraception? • Are you in a stable relationship? • Any previous miscarriages? • Any history of STIs? (can predispose to ectopic) • When was your last pap smear, and what was the result? • Any bleeding or discharge from down below? • Have you taken your folic acid? o Other medical or surgical illness? o Any medications taken? Any known allergies? o SAD history

▪ Physical Exam o General appearance: pallor, dehydration, lymph nodes o Vital signs: BP with a postural drop, temperature o ENT: look for any focus of infection o CVS/Respiratory/CNS o Abdomen: visible distention, mass, palpate for mass, tenderness, suprapubic tenderness, renal angle tenderness (key point), auscultate for the bowel sounds o Pelvic exam • Inspection of the vulva and vagina: discharge, bleed, rash, vesicles • Speculum: the cervix is healthy, discharge, bleed • Per vaginal exam (because she has never gone for antenatal check): CMT, uterine size and tenderness, adnexal mass and tenderness o Office test: UDT: nitrites, leukocytes, RBCs, KETONES, BSL, UPT

▪ Diagnosis and Management What you are having is most probably a . Draw the urinary tract. This is the urinary tract, and I am suspecting a lower urinary tract infection which is an infection of the bladder or the urethra. Is this the first episode of urinary tract infection? This is a common infection during pregnancy due to the hormonal and mechanical changes leading to the risk of urinary stasis. Sometimes, sexual intercourse can also predispose you to UTI. The complications if this is left untreated is pyelonephritis or the infection can ascend high into the kidney. But with treatment, this has a good prognosis. The first thing is to do an investigation like urine microscopic culture and sensitivity. Then we need to put you on antibiotics such as Cephalexin 500mg 4x/day for 5-7 days, or Augmentin (co- amoxiclav) 2x/day for 5-7 days. Analgesics like Panadol will also be given to relieve you of the pain and fever. General measures:

o Drink plenty of oral fluids to flush the organism out of the system o Avoid taking baths but take showers o Toilet hygiene • Wipe from front to back after opening the bowels 153 •

Make sure that you empty the bladder completely each time when you go to the toilet Page

© 2020 ARIMGSAS

Just in case you continue with the fever, fever gets higher, you develop chills or rigor, if the vomiting becomes worse, or you develop back pain, please report back immediately. We need to repeat the urine microscopic culture and sensitivity 1 week after finishing the antibiotics. I will give you reading materials regarding UTI and I will review you in 1 week.

Pyelonephritis Do all blood tests like FBE, UEC, ESR/CRP, LFT, RFT, blood culture (depending on the temperature), urine microscopy culture and sensitivity. Requires admission. Give adequate IV fluids. Refer to a specialist. Renal ultrasound if recommended by the specialist. Put on IV antibiotics like Ceftriaxone 1g daily x 3 days or Cefotaxime 1g every 8 hours x 3 days. After 3 days, if she is responding well, then you can change her to oral antibiotics like cephalexin 500mg 4x/day or Augmentin 2x/day. Give analgesics like Panadol. If vomiting is a problem, you can put her on metoclopramide, or ondansetron. Once she improves, she may be discharged, and a urine MCS will be repeated 1 week after stopping the antibiotics.

154 Page

© 2020 ARIMGSAS

HRT Counselling

You are at your GP when 53-year-old Tracy presents to you. She has come to you to discuss about HRT.

TASKS 1. Focused history 2. Counsel regarding HRT

Positive points in the history: she is experiencing hot flushes after the menopause, menopause for 2 years

APPROACH ▪ History o Why are considering starting HRT? o Symptoms of menopause: • Vasomotor symptoms: Are you experiencing hot flushes, heavy sweating? • Psychological: Are you having mood changes, sleep disturbances, depression? • Atrophic vaginitis: Any vaginal dryness, itchiness, discharge? • Atrophic urethritis: Any burning or stinging while passing urine? • Somatic symptoms: any muscle aches and pains? Any bone pains? Any history of fractures? ▪ Is it interfering with your lifestyle? o At what age did you have your menopause? Any bleeding after menopause? o Contraindications for HRT: (Make sure you rule out all of these before you consider starting her on HRT) • Any increased clotting in your veins? (DVT) • Any undiagnosed vaginal bleed? • Any recent heart problems like angina or heart attacks? • Any suspected current or past history of breast cancer? (even if she says she had breast cancer 10 years ago, fully treated, NEVER start HRT in the patient) • Any history of ovarian or endometrial cancer? • Any active liver disease? (HRT is processed in the liver) • Any uncontrolled high blood pressure? • Any history of stroke? o When was your last pap smear? What was the result? When was your last mammogram? (make sure she does a mammogram before you start HRT) o Any medical or surgical conditions in the past?

▪ Counselling After menopause, the ovaries shut down completely so that very low levels of estrogen and no progesterone is found in the body. The lack of estrogen contributes to menopausal symptoms that you are having. HRT has both estrogen and progesterone in it, and this replaces these hormones in 155

your body. Why progesterone is added to estrogen is to prevent the thickening of the Page

© 2020 ARIMGSAS

endometrium or the inner lining of your womb which happens due to estrogenic influence. The benefits of HRT are that it can relieve all menopausal symptoms, and it is found that there is a possible decrease in the incidence of Alzheimer's and other forms of dementia, and it decreases the incidence of colorectal or bowel cancer. Key point: the risk associated with HRT is you can go in for thromboembolic disease or DVT or increased clotting tendency in your veins, stroke, breast cancer (especially if you use it for more than 5 years), endometrial and ovarian cancer, and also gallbladder disease. Before starting HRT, you need to do certain blood tests like FBE, UEC, LFT, BSL, lipid profile. (No need to estimate the hormones because she is already menopause) HRT will be started with your informed consent. We will start you on continuous combined HRT.

There are different methods for giving HRT. HRT can be given in 3 separate ways: o Estrogen alone: this is given if the patient had hysterectomy already o Cyclic/Sequential HRT: if the patient is menopausal for up to 1 year, or perimenopausal • Give estrogen continuously for 28 days, add progestogen in the last 14 days of the cycle • Once you stop this, the patient will have withdrawal bleeding • You prefer this type of HRT for these group of patients, because these patients still have some follicular activity, and along with relieving the menopausal symptoms, the bleeding becomes regular • Estrogen can be given in the form of tablets, pessaries, creams, patches • Progestogen can be given orally or as patches o Continuous combined HRT: if the patient is menopausal for more than 1 year • She will not get a bleed at all • If she bleeds, we consider that a post-menopausal bleed, and that needs to be investigated further • Can be given orally, patches, estrogen orally and put in Mirena Side effects of HRT: o Nausea, headache, breast tenderness, blotting, breakthrough bleeding o Usually settles in 3-6 months' time When can you consider stopping HRT? o Review her in 6 months' time once you put her on HRT, and then every year after that o Do a general health check, breast check, and make sure she does her pap smear and mammogram every 2 years o Stop HRT in 2 years' time, and better not to continue HRT beyond 5 years o NEVER start HRT beyond 60 years o NEVER stop HRT abruptly, it should be tapered for 3-6 months and then stopped

Another case: the patient has no indications for HRT, comes to you for HRT because her friends tell her it's good to be started on HRT Look for indications for HRT. Tell her what HRT is. Just in case she develops any indications for HRT, and at that time there are no contraindications,

then she can be considered for starting HRT after doing certain blood tests and examinations. HRT should NEVER be started without clear indications as there a lot of risks associated with it. Elaborate the risks of HRT. 156

Page

© 2020 ARIMGSAS

Another case: the patient has clear indications for HRT, but the patient had breast cancer 10 years back, fully treated The patient has a contraindication for HRT. Advise the patient lifestyle modifications: healthy balanced diet, no spicy foods, coffee should be limited, no smoking, better to quit alcohol as well Advise to wear light cotton clothing. Exercise like swimming (because the water can keep the body cool), and jogging Ask her to sleep in cool rooms. Put her on medications (alternatives to HRT): o First choice: Gabapentin - can bring down all the vasomotor and psychological symptoms o Pregabalin (Lyrica)- derivative of Gabapentin o SNRIs/SSRIs - venlafaxine/paroxetine • Along with the improvement of the psychological symptoms, there is a 60% reduction in the vasomotor symptoms as well

*Hormonal alternative to HRT (but also same contraindications): - a SERM which has estrogenic, progestogenic and weak androgenic activity o Considered in patients menopausal after 1 year and women with intact uterus o Carries a small risk for breast cancer and DVT o It is less effective compared to HRT that is why it is not widely used

157 Page

© 2020 ARIMGSAS

HRT Counselling II

48-year-old Jane is your next patient at your GP. She has come to you to discuss the options of starting HRT. She is on combined oral contraceptives for the past 5 years.

TASKS 1. Further relevant history 2. Discuss with her about the options that she has regarding HRT

**As long as the patient is on combined OCPs, usually they do not get menopausal symptoms, and if they do, it is during the dummy pill period. But if the patient on combined OCPs get perimenopausal symptoms, it tells you that the estrogen content in the pill is not sufficient to replace the loss of estrogen that she is having. Normally, the option here is to increase to high dose COC. BUT never increase the dose if the patient is perimenopausal. If she is and she presents with perimenopausal symptoms on combined pills, stop her on combined pills, estimate her hormones (FSH, LH, estrogen -- usually FSH is the main predictor of menopause; FSH and LH high, estrogen is low), then start her on HRT and advice on an alternative method of contraception like condoms. ------Another case: She is on the combined pill, no symptoms of menopause, she is coming to you for HRT because her friends tell her that you feel more feminine if you start on HRT. As long as she is on COC, she will get bleeding and you will not know if she is menopausal. Ask her to go on with the COC and stop the combined COC at 50 years old. If she has no periods after that, she is menopausal. If she is getting irregular periods, estimate her hormones (FSH, LH, estrogen) and then at that time, if she has clear indications with no contraindications, you start her on HRT. If she is menopausal after 50 years, always ask her to use alternative methods of contraception like condoms for 1 year after menopause. If she attains menopause before 50 years, she needs to use condoms for 2 years.

Positive points in history: 1st case: wants HRT because she has hot flushes and sweating, has mood changes 2nd case: no symptoms but wants HRT because her friends are on HRT

APPROACH ▪ History o What are your concerns? What do you think you need to be started on HRT? o Since how long have you been having these symptoms of hot flushes and sweating? o Look for other indications for HRT: • Psychological: Do you have any other symptoms like irritability, mood changes, sleep

disturbances? • Somatic: Do you have muscle aches and pains, bone pain, history of fractures? • Reproductive: Do you experience vaginal dryness, discharge, burning or itching? • Bladder: How is your waterworks? Any incontinence? 158

o Ask for contraindications of HRT: Page

© 2020 ARIMGSAS

• Do you have increased clotting in your vessels or veins? • Do you have any recent heart problems like heart attack, angina? • Any history of breast cancer, endometrial cancer, ovarian cancer? • Any undiagnosed vaginal bleeding? • Any liver disorders? • Any history of stroke? • Any uncontrolled high blood pressure? o Combined OCPs • Since how long have you been on the pills? • What type of pill are you using? • Any side effects of the pills that you are experiencing? • Before you were started on the pills, what contraceptive were you on? • During which part of the pill schedule do you get the symptoms, during hormonal or dummy pills? (more chance of symptoms on the dummy pills) o Period history • When was your LMP? • Are your periods usually regular? • Any bleeding in between periods? (contraindication for starting HRT) • When was your last pap smear done? What was the result? ▪ You are due for a pap smear now, I can do it with your consent • Have you done a mammogram? ▪ I will arrange for a mammogram now o Sexual history • Are you sexually active? Do you have a stable partner? • How many pregnancies have you had in the past? Any miscarriages? o How is your diet? How is your exercise? o SADMA • Do you smoke, drink alcohol, take recreational drugs? o Any past history of any medical or surgical illness?

▪ Diagnosis and Management HRT is hormone replacement therapy where we replace the hormones that become deficient as you go towards menopause, and also after menopause. The symptoms that you are having is due to a decrease in the secretion of estrogen from the ovaries, as the ovaries start to become less functional as you go towards menopause, and this estrogen is replaced by HRT. But remember that HRT never acts as a contraceptive (key point). Even though the combined pills contain estrogen, the estrogen in the pill does not replace the estrogen that is deficient from your body. At this age, we do not recommend you to be shifted to a high dose combined OCPs. What we can do however is to stop the combined OCPs that you are currently taking and start you on HRT instead as you have clear indications, and no contraindications. But I must also advise you that HRT carries multiple risks (key point). It can lead to thromboembolic disease or increased clotting in your veins, stroke, womb cancer or endometrial cancer, ovarian cancer, breast cancer, and also gallbladder disease. Only with your informed consent can we start HRT. The benefits of HRT are:

you can get rid of perimenopausal symptoms, decreases the incidence of bowel cancer, and there is a possible decrease in the incidence of Alzheimer's disease and other forms of dementia. After 1 week of stopping the combined pills, certain blood tests including hormones need to be done like FBE, UEC, LFT, BSL, lipid profile, FSH, LH, and estrogen levels. If you are going in for menopause, 159

the FSH and LH will be high, and the estrogen levels will be low. Page

© 2020 ARIMGSAS

After a complete physical examination and a pelvic examination, I will start you on HRT on a cyclical or sequential manner. Taking the menstrual cycle for 28 days, we will put you on continuous estrogen, and progestogens during the last 14 days, and after finishing the progestogens, you will expect to have withdrawal bleeding. You may experience a bit of nausea, abdominal bloating, headaches, and breast tenderness, and sometimes a breakthrough bleed. We may stop you on HRT once you become clear of the symptoms, and it is usually advisable to take HRT for only 2 years, and by any chance not more than 5 years. As I have told you, HRT cannot act as a method of contraception, so you need to use alternate methods of contraception like condoms (key point). I will give you reading materials regarding HRT for further insight and I will arrange a review with you regularly.

2nd case: History taking is the same, but no need to rule out contraindications for HRT because she has no symptoms or indications for HRT. No indications in the woman for HRT.

Counselling: As from the information that you have given me, there are no indications in you to be started on HRT. The indications for HRT are when you experience perimenopausal symptoms like mood changes, irritability, sleep disturbances, muscle aches and pains, history of bone fractures, vaginal dryness, burning or itching, and problems with your waterworks. HRT should always be started with caution, as there are lots of risks associated with HRT. The risks are… Because of these risks, HRT is never started if there are no clear indications for it. HRT will never act as a contraceptive. What is advisable this time is that you continue on your combined pills if you are comfortable with that, until 50 years of age. After then, you can stop your pills, because as long as you're on the pills, you will go on getting your periods, and we will not know whether you've hit menopause or not. Once you stop the pills by 50 years old and if you do not get your periods anymore, that could mean that you are already menopausal. However, if you get irregular bleeds after stopping the pill, and there are indications in you for HRT, we will estimate your hormones like FSH, LH, and estrogen. If FSH, LH is high and estrogen is low, that means that you are menopausal, and at that time if you have no contraindications for HRT like…, we can consider starting you on HRT with your informed consent. We will do a full physical examination and pelvic examination on you before we start you on HRT. Make sure that your pap smear and mammogram is up to date as well by doing it every 2 years. It is advisable to use alternate methods of contraception like condoms for 1 year after menopause if the menopause happens after 50 years of age. I will give you reading materials regarding HRT for further insight and I will arrange a review with you regularly.

160 Page

© 2020 ARIMGSAS

Thalassemia Minor

40-year-old Jane who was 28 weeks pregnant visits your GP to get the results of the blood tests that you had ordered for her during the last visit. Her OGTT was normal, but her FBE showed a low haemoglobin, and low MCV. Iron studies were found to be normal.

TASKS 1. Take a further history 2. Discuss your most probable diagnosis and further management with the patient

When the MCV is low, the size of the RBCs is decreased. The picture you get here is microcytic hypochromic anemia. There are two reasons for this during pregnancy that comes in the AMC are iron deficiency anemia and thalassemia. Thalassemia key questions: her descent, and her partner's descent

Positive points in the history: she is of Italian descent, her partner is also of Italian descent, the father seems to be anemic, but she is not sure,

APPROACH ▪ History o How has been your pregnancy going so far? o Are you experiencing any problems such as excessive vomiting, tummy pain, bleeding from down below, burning or stinging on passing urine, fatigue, headache, blurring of vision & swelling, fever/malaise? o Is the baby kicking well? o Symptoms of anemia: • Are you feeling tired, any chest pain, funny racing of the heart, are you feeling short of breath, any dizziness? • Before pregnancy, have you experienced any of these symptoms? o Pregnancy history • Did you do all your antenatal tests? How were the results? ▪ Routine first visit tests o FBE o Blood group and antibody screen o Rubella antibody status o Syphilis serology o Midstream urine

o Chlamydia o HIV o Hepatitis B and C serology o Varicella 161 o Cervical cytology/Pap smear Page

© 2020 ARIMGSAS

o Screening for Down syndrome (if >35 years old) ▪ Ultrasound at 18 to 20 weeks? o Any past history of medical illness? Any kidney or liver disorders? o Thalassemia questions: • Where is your country of origin? (key point) (Thalassemia is common in Mediterranean countries: Greek, Italian, Middle Eastern, Turkish; Asian, African) • What about your partner's country of origin? (key point) o Any family history of anemia? o Any history of miscarriage and bleeding disorders?

▪ Diagnosis and Management The investigation results show that the sweet drink test is normal which means that you do not have diabetes during pregnancy. Your full blood test results, however, show that you have low levels of hemoglobin, which is the red pigment or protein in your red blood cells that carry oxygen to different parts of your body, and the size of your red blood cells is also reduced. This sort of a picture can be seen in two conditions: first is iron deficiency anemia, but the iron studies that we have done for you has turned out to be normal; another condition is thalassemia. Thalassemia is a group of inherited blood disorders where there is defective production of hemoglobin. This happens when the genes are mutated, and they are permanently altered. This is a lifelong condition. What you are having is a minor version of this condition, or what we aptly call as Thalassemia minor. In this condition, you do not experience symptoms other than anemia. Usually, there is no risk for you during pregnancy, except for the anemia that could become prominent during your later pregnancy. It is more likely that the baby will be born healthy. We need to confirm this by doing certain blood tests called hemoglobin electrophoresis. It would be better to get your partner tested as well because we can better predict if the baby will be having Thalassemia as well. If you alone have Thalassemia minor, there is a 50 percent chance of the baby becoming normal, and 50 percent chance as well of having Thalassemia minor. However, if both of you have Thalassemia minor, there is a 50 percent chance of the baby having Thalassemia minor and 25% chance of having Thalassemia major, and a 25% chance of becoming normal. I will refer you to the hematologist for further assessment. We need to start you on folic acid 5mg/day, and it will be given throughout the pregnancy. (keypoints) I will also refer you to the high-risk pregnancy clinic. I will refer you for genetic counselling as well. If everything goes well for you and the baby, you can go in for normal vaginal delivery. I will give you reading materials regarding Thalassemia, and I will arrange a review with you when the results of the hemoglobin electrophoresis come in.

162 Page

© 2020 ARIMGSAS

Twin Pregnancy

32-year-old Mary attends your GP clinic. She is 20 weeks pregnant and has come to discuss the results of the ultrasound scan that she did 1 week ago. The ultrasound showed a twin pregnancy with each foetus having a separate amnion and chorion.

TASKS 1. Take a further relevant history 2. Explain the ultrasound results to her 3. Discuss your further management regarding her pregnancy

APPROACH ▪ History o Was this a planned pregnancy? Is this your first pregnancy? o Was this a natural conception, or did you go in for artificial methods of conception? o How is your pregnancy going so far? o Are you experiencing any problems such as tummy pain, bleeding from down below, burning or stinging on passing urine, fatigue, headache, blurring of vision & swelling, fever/malaise? o Have you started feeling the babies kicking? o During the early weeks of pregnancy, any exaggerated vomiting, any nausea? o Did you do all your antenatal tests? How were the results? ▪ Routine first visit tests o FBE o Blood group and antibody screen o Rubella antibody status o Syphilis serology o Midstream urine o Chlamydia o HIV o Hepatitis B and C serology o Varicella o Cervical cytology/Pap smear o Screening for Down syndrome (if >35 years old) ▪ Ultrasound at 18 to 20 weeks? o General well-being questions: o Are you taking your folic acid? When did you start taking it? (folic acid 0.5mg/day 3 months before and 3 months after)

o Are you eating a healthy balanced diet? Do you drink a lot of fluids? o Are you taking any vitamin or mineral supplementation? o Any change in appetite? Have you noticed weight gain/loss? o Do you have a regular exercise routine? 163 o Do you open your bowels regularly? Any problems with your bowels? Page

© 2020 ARIMGSAS

o Are your influenza and pertussis vaccinations up to date? o Any family history of multiple pregnancies? o SADMA o Do you smoke? ▪ How long have you been smoking? How many per day? o Do you drink alcohol? ▪ What kind? How often? How much per session? o Do you take any recreational drugs? ▪ What kind? How often? o Do you take any prescription or over the counter medications? ▪ What medication? For what reason? Who prescribed it? o Do you have any known allergies? ▪ Do you take any medications for this? o Any past history of any medical illness especially clotting problems (thromboembolic disease), diabetes, epilepsy, thyroid problems, or high blood pressure? o Do you have good support? o Have you done your pap smear? What were the results?

▪ Diagnosis and Management How are you feeling about your pregnancy? There are different types of twin pregnancies. The type that you are having is more favourable because each of the babies has a separate placenta as well as a separate bag of its own. These are independent twin pregnancies, and usually carries a better prognosis. These could likely be non- identical twins. I know that you are a bit concerned about this pregnancy but let me reassure you that many women who have twin pregnancies go through a normal pregnancy and will have healthy, normal babies. But we need to monitor you as well as the babies for certain complications. The complications we have to look out for you are first, anemia, you can feel a bit tired and exhausted as the pregnancy goes by; there is also a risk of high blood pressure and pre-eclamptic toxemia where there is a sharp rise in blood pressure with leakage of proteins into the urine; there is also a higher chance of developing diabetes during pregnancy; also as your tummy will be bigger towards the end of your pregnancy, you may feel a little short of breath and experience back pain; you may also have antepartum hemorrhage or bleeding during pregnancy, cord prolapse, and a greater risk of postpartum hemorrhage. The complications in the babies are low birth weight, intrauterine growth retardation of one baby, birth defects, and malpresentation. But we will monitor you and your babies for these complications. I will refer you to the high-risk pregnancy clinic for better management. You need to go for more frequent antenatal checks: every 2 weeks until 28 weeks and weekly until you deliver. During each visit, your weight will be recorded, and your BP will be monitored. A sweet drink test will be done at 28 weeks to check for any Gestational Diabetes, a repeat blood test to check for anemia, ultrasound screening every 2 weeks from 28 weeks, CTG to look at the wellbeing of the baby twice a week from 34 weeks, and a bug test will be done as well. Delivery will be planned at 38 weeks. There are two options for you: one is to do a C-section, and

two is a normal vaginal delivery. However, normal vaginal delivery can be done only if certain criteria are met: o The first baby should be in cephalic presentation o The second baby should be less than 500 grams compared to the first baby (if the second 164

baby is big, it will obstruct the passage of the first baby) Page

© 2020 ARIMGSAS

The delivery should be done in a tertiary hospital, under the guidance of a specialist. Continuous monitoring of the babies will be done via CTG during the delivery. How far are you living from the hospital? It is better to stay near to the hospital during the last weeks of pregnancy because there is a greater chance of you going into premature labor because of this twin pregnancy. You need to take a healthy balanced diet and take iron, calcium and folic acid supplements. Just in case you develop tummy pain, bleeding and excessive discharge from down below, blurring of vision, headache, excessive vomiting, regular painful contractions, please go to the ED immediately. It is always advisable to maintain kick charts for the baby as well so we can monitor the wellbeing of the baby.

165 Page

© 2020 ARIMGSAS

Transverse Lie

35-year-old Jenny is your next patient at your GP clinic. She is at 37 weeks pregnant and has come to you for antenatal checks. This is her fourth pregnancy.

TASKS 1. Take a further history 2. PE from examiner 3. Discuss further management with the patient

Positive points in history: none Positive points in the PE/Investigations: fundal height: 36cm, normal FHR, lie: can feel a hard ballotable mass on one iliac fossa and a soft mass towards the other iliac fossa (hard is the head, soft is the bum) - transverse lie, lower pole of the uterus is empty

APPROACH ▪ History o How is your pregnancy going so far? o Any tummy pain, bleeding or discharge from down below? o Is the baby kicking well? o Any headache, blurring of vision, any swelling? o Did you do all your antenatal tests? How were the results? ▪ Routine first visit tests o FBE o Blood group and antibody screen o Rubella antibody status o Syphilis serology o Midstream urine o Chlamydia o HIV o Hepatitis B and C serology o Varicella o Cervical cytology/Pap smear o Screening for Down syndrome (if >35 years old) ▪ Ultrasound at 18 to 20 weeks? o Is it a single baby? o Any birth defects that the baby is having? o What is the position of the placenta?

o Any other abnormalities detected in the ultrasound? ▪ Sweet drink test at 26/28 weeks? ▪ Repeat ultrasound at 32/34 weeks? ▪ Bug test at 36 weeks? 166 o General well-being questions: Page

© 2020 ARIMGSAS

o Are you taking your folic acid? When did you start taking it? (folic acid 0.5mg/day 3 months before and 3 months after) o Are you eating a healthy balanced diet? Do you drink a lot of fluids? o Are you taking any vitamin or mineral supplementation? o Any change in appetite? Have you noticed weight gain/loss? o Do you have a regular exercise routine? o Do you open your bowels regularly? Any problems with your bowels? o Are your influenza and pertussis vaccinations up to date? o How were your previous pregnancies and deliveries? Any complications in during pregnancy, delivery or after delivery? o When was your last pregnancy? (can predispose to anemia if no proper spacing) o SADMA o Do you smoke? ▪ How long have you been smoking? How many per day? o Do you drink alcohol? ▪ What kind? How often? How much per session? o Do you take any recreational drugs? ▪ What kind? How often? o Do you take any prescription or over the counter medications? ▪ What medication? For what reason? Who prescribed it? o Do you have any known allergies? ▪ Do you take any medications for this? o Any past history of any medical illness especially clotting problems (thromboembolic disease), diabetes, epilepsy, thyroid problems, or high blood pressure? o Do you have good support? o Have you done your pap smear? What were the results?

▪ Physical Exam o General appearance o Vital signs o CVS/Respiratory/CNS o Abdomen: fundal height, FHR, lie and presentation, is the lower pole of the uterus empty or not? Any uterine contractions or tenderness? o Pelvic: o Inspection of the vulva and vagina: any bleed, discharge, rash, vesicles o Speculum: cervical os open or closed, discharge or bleeding from the os o Don't go in for a per vaginal exam because you're thinking of transverse lie and/or placenta previa (Previa can predispose to transverse lie) o Office test: UDT, BSL

▪ Diagnosis and Management Most likely your baby is in a transverse lie. Normally, the baby lies in a longitudinal position which means that the baby is parallel to your spine. But in a transverse lie, the baby lies perpendicular to

your spine. *Draw a picture of a longitudinal and transverse lie. The concern here is, when you go into labor with the baby in a transverse lie, the labor might now progress, and you will end up having an obstructed labor, and there is a chance of cord prolapse 167

as well. The cord usually comes out after the delivery of the baby, but in cord prolapse, the cord Page

© 2020 ARIMGSAS

comes out first and the cord can get compressed between the baby's head and the birth canal leading to decreased oxygen and nutrient delivery to the baby. The baby then becomes distressed and unwell. The cause of this that I suspect in you is multiparity. As this is your fourth pregnancy, the uterus could be lax, and this allows the baby to adopt an abnormal position and lies. The other causes can be polyhydramnios or excessive fluid in the sac surrounding the baby, but that is not a probability here as the uterine size is less. Another cause is placenta previa when the uterus attaches itself to the lower pole of the uterus, but this could have been detected in the ultrasound that has been done earlier. Another cause could also be birth defects in the baby, but this has also been ruled off from the ultrasound as well. Fibroids or non-cancerous tumors in the uterus can also lead to a transverse lie in the baby, but this again could have been detected in the ultrasound. But we need to look out for these causes as well. I need to refer you to a tertiary hospital, where you will be admitted and seen by the specialist. An ultrasound will be done to rule out the other causes of a transverse lie, and a CTG will also be done to look for the baby's wellbeing. There are two options for you as far as delivery goes: the first option is an elective C-section by 37/38 weeks, and the second option is to do an external cephalic version. ECV will be done by the specialist after ruling out contraindications like fetal distress or unwell baby, placenta previa, a cephalopelvic disproportion, or a short cord or oligohydramnios. In ECV, the baby will be manipulated over the tummy by using the hands and finally, the head of the baby will be brought down so that the baby will have a longitudinal lie and a cephalic presentation or head down. Labor will then be induced by artificial rupture of the membranes. There are complications associated with ECV such as fetal distress, abruptio placentae or placenta separates from the wall of the uterus, rupture of membranes, and the most important complication is the cord can get twisted around the neck of the baby. It is usually done around 37/38 weeks. After 38 weeks, it is not done anymore. Do you have somebody to take you to the hospital now? I will give you reading materials regarding transverse lie.

168 Page

© 2020 ARIMGSAS

Permanent Sterilization

35-year-old Susan is your next patient at your GP. She has come to you to discuss a permanent method of contraception available to her.

TASKS 1. Counsel the patient regarding permanent contraception

Positive points in the history: on combined OCPs but has difficulty taking the pill

APPROACH ▪ History o What are your concerns? o How many children do you already have? o Are you sure of not having any children in the future? o Have you had a discussion with your partner regarding this? (but partner's consent is not necessary) o Any contraceptives that you are in now? o Why are you considering permanent sterilization? o Have you tried an IUCD, injectable or implants or other non-permanent forms of contraception? o Any past medical illness? o Any previous surgeries done to her uterus, tubes, or ovaries? o SADMA history o Are all your pregnancies normal vaginal deliveries?

▪ Counselling Female sterilization should be considered as a permanent method of contraception and should be assumed that his procedure cannot be reversed. You should be very sure of not wanting any children in the future. The efficacy of this procedure is greater than 99.5%, and the advantages are there is no need to use any other forms of contraception unless you are concerned about sexually transmitted infections, then you need condoms for that; it does not interfere with your menstruation or menopause; and your sexual desire will not be interfered with as well. There are two procedures that could be considered as a form of permanent sterilization: o The first is a tubal clipping or ligation. This is done by a keyhole surgery or laparoscopy. Under general anesthesia, two cuts will be made around your belly button, and a gas will be piped into the abdomen to lift up the wall. A laparoscope with a camera at one end will be passed through one of the cuts, and the sterilizing instruments will be passed through the

other cut. The tubes are then identified, and the first procedure done is clipping of both the tubes, or two parts of the tube will be tied or ligated and then the tube is cut into two. This prevents the sperm from reaching the egg and thereby prevents pregnancy. This is a day

procedure. The complications of this procedure could be risks of anesthesia, bleeding, 169 Page

© 2020 ARIMGSAS

infection, injury to the surrounding structures like the bowel and urinary tract, and you can also get tummy pain and shoulder pain (due to the gas irritating the diaphragm). o The second procedure is called the tubal occlusion technique or otherwise called Essure procedure. This is done by placing a tiny flexible device made of titanium into each of the tubes through hysteroscopy or an instrument introduced from down below. As this is a foreign body in the tube, scar tissue starts forming around it, until finally the tube will be occluded. As it takes quite some time for the scar tissue to form, you might need to use alternate methods of contraception for at least 3 months. An ultrasound will be done to make sure that the stent is in place and also to see if the tubes have been occluded or not. The disadvantages of permanent sterilization are: it does not protect against STIs, and even though it is technically possible to reverse the sterilization, it should be considered permanent. After the reversal, there is only a 50% chance of pregnancy. Tubal occlusion is not reversible at all. If you are happy to discuss any other method of contraception, I would be happy to help you with that.

Another case: 47-year-old patient asking for permanent sterilization.

History o Why are you considering permanent sterilization now? o What sort of contraception are you in now? o Are you still having your periods? When was your LMP? Is it coming regularly? (if going for menopause, has irregular bleeds) o Are you still sexually active? How many pregnancies and deliveries have you had before? o Are you experiencing any perimenopausal symptoms like hot flushed, sweating, mood changes? o Are you sure that you don't want any children in the future? o Have you discussed this with your partner?

Counselling Discuss permanent sterilization but discuss other points as well since she is already nearing her menopause. In 3 or 4 years, you could go in for menopause. If menopause happens, then there is no risk of pregnancy. In the meantime, you can use other methods of contraception like a copper IUCD, which could offer your contraception for the next 5 to 10 years, and by that time, you would have attained your menopause. Another option is to consider a vasectomy in your partner if he is willing. Vasectomy is a simpler procedure in comparison to the permanent sterilization that could be performed in you. I will give you reading materials regarding permanent sterilization for further insight.

170 Page

© 2020 ARIMGSAS

Bacterial Vaginosis/Trichomoniasis

You are at your GP clinic when 26-year-old Tracy presents to you with complaints of discharge from her vagina for the past few days. She is pretty concerned about this is the first time and she wants you to address her concerns.

TASKS 1. Further relevant history 2. PE from examiner 3. Discuss your investigations and management

Positive points in the history: whitish to greyish discharge, fishy smell, on oral pills, does not use condoms Positive points in the PE/investigations: abdomen is soft and non-tender, thin profuse whitish to greyish discharge, vulva and vagina are normal

Differential Diagnosis for discharge: ▪ Bacterial vaginosis ▪ Candidiasis ▪ Trichomoniasis ▪ Other STIs

APPROACH ▪ History o Since how long have you been having this discharge? o What is the consistency of the discharge? o How severe is the discharge? Do you need to use pads? o What is the color of the discharge? o Does it have any peculiar smell? (fishy smell: bacterial vaginosis and trichomoniasis) o Any blood stains? o Any itching, soreness, rash on down below? o Any fever, lumps and bumps around your groin, tummy pain? o Any problems with your waterworks? (dysuria is common in candidiasis) o Period history • When was your LMP? • Is it regular? • Any problems with your periods? o Sexual history

• Are you sexually active? Are you in a stable relationship? • Any history of STIs in you or your partner?

• Any pregnancies or miscarriages in the past? 171 • What form of contraception do you use?

• When was your last pap smear and what was the result? Page

© 2020 ARIMGSAS

o Any medical or surgical illness in the past? o Did you take any antibiotics recently? o SAD history

▪ Physical Exam o General appearance: BMI, pallor, lymph node enlargement o Vital signs: temperature o CVS/Respiratory/CNS o Abdomen: look for signs of PID: any visible distention, any mass, palpate for any mass and tenderness o Pelvic examination • Inspection of the vulva and vagina: discharge: color, consistency, odour; any erythema of the vulva or vagina, any swelling • Speculum: cervix healthy or not • Don’t do a per vaginal exam o Office test: UDT, BSL

▪ Diagnosis and Management What I am suspecting in you is a condition called bacterial vaginosis. It is caused by an imbalance of the bacteria normally present in your vagina. This happens when the normal helping bacteria are replaced by an overgrowth of other mixed bacteria. (normal bacteria are lactobacilli) It is one of the most common causes of abnormal vaginal discharge. The exact cause of why this occurs is unknown, but it could be sexually transmitted. It usually produces a watery, white or grey discharge, and has a strong unusual fishy smell. To confirm this, I need to take a high vaginal swab which is given for microscopy and gram stain, and it will show clue cells, which is a normal vaginal epithelial cell with bacteria attached all around. Another test is an amine whiff test where 10% potassium hydroxide is added, and it will give a pungent fishy smell. Another one is to estimate the pH of the vaginal fluid and usually, the pH will be greater than 4.5 if it is bacterial vaginosis. Even after treatment, in about half of the women, it can sometimes recur in the next 6-12 months and could be associated with other STIs as well. Management of this is by prescription of antibiotics, Metronidazole 400mg orally twice daily with food x 7 days OR Metronidazole gel intravaginally at night x 5 nights, OR Clindamycin 300mg orally twice daily x 7 days (this is also the preferred antibiotic if pregnant). Avoid vaginal douching because that can also alter the bacteria in your vagina. If you are as well concerned about sexually transmitted infections, I can also offer you an STI screen. Your partner does not require any treatment as of the moment but always practice safe sex. I will give you reading materials regarding bacterial vaginosis and I will review you in 5 days' time.

------Trichomoniasis

This is the most severe of infections. The discharge is a greenish, yellowish, frothy discharge. The patient has dyspareunia, dysuria and soreness but is NOT itchy. When you look at the vulva and the vagina, it shows swelling and erythema and the cervix might

also show evidence of infection. 172 Page

© 2020 ARIMGSAS

Investigations: high vaginal swab or saline prep for microscopy (you will see a typical flagellated protozoans) Treatment: Metronidazole 500mg twice daily orally x 7 days The partner also needs to be treated. Offer her STI screen as well. Notify the DHS.

173 Page

© 2020 ARIMGSAS

Reduced Foetal Movement

You are at your GP when a 25-year-old lady, Amanda presents to you. She is 38 weeks pregnant. It is her first pregnancy and she noticed that her baby did not move around as much as usual and she got frightened and decided to see you.

TASKS 1. Take a relevant history 2. PE from examiner 3. Discuss your Diagnosis and Management with the patient

Causes: ▪ Non-pathological o Position of the mom (if mom is sitting and standing, she may not feel the baby kicks well, but if she lies down especially in the left lateral position, she can feel it much better) o Busy lifestyle o Foetal sleep o Obesity (if the abdominal wall is thick, she cannot feel the baby kicks very well) o Placental position (normal position of the placenta is fundus towards posterior; if it is attached anteriorly, it can interfere with the foetal movement) ▪ Pathological o Oligohydramnios o IUGR o Congenital anomalies o Maternal infections o Anemia o Thyroid problems

Positive points in the history: since the morning the number of the kicks reduced but in the past one hour, she has not felt the baby kicking at all Positive points in the PE/Investigations: fundal height: 38cm, FHR: 150bpm, lie: longitudinal, presentation: cephalic, no palpated foetal movements, no uterine contractions

APPROACH ▪ History o When did you first start feeling the baby kicking? (Primigravida 18 weeks, Multigravida 16 weeks) o Since how long have you not been feeling your baby kicking?

o Is it like no movements at all or a reduced number of kicks? o Have you counted the kicks? Have you attempted counting the kicks in the supine or lying down position? o Is this the first time that this is happening to you? 174 o Differentials Page

© 2020 ARIMGSAS

• Are you working now? • Any thyroid problems before you became pregnant? Any weather preferences, change in bowel habits? • Have you done your blood tests? Was there any anemia detected? • Any recent infections? Any fever that you are running? o Pregnancy • Are you having any tummy pain, bleeding or discharge from down below? Any blurring of vision, headache, swelling? • Did you do all your antenatal tests? How were the results? ▪ Routine first visit tests o FBE o Blood group and antibody screen o Rubella antibody status o Syphilis serology o Midstream urine o Chlamydia o HIV o Hepatitis B and C serology o Varicella o Cervical cytology/Pap smear o Screening for Down syndrome (if >35 years old) ▪ Ultrasound at 18 to 20 weeks? o How is the baby? What is the position of the placenta? ▪ Sweet drink test at 28 weeks? ▪ Repeat ultrasound at 32 weeks? o Is the baby growing well? Is there adequate fluid in the bag surrounding the baby? (AFI) ▪ Bug test at 36 weeks? o General well-being questions: ▪ Are you taking your folic acid? When did you start taking it? (folic acid 0.5mg/day 3 months before and 3 months after) ▪ Are you eating a healthy balanced diet? Do you drink a lot of fluids? ▪ Are you taking any vitamin or mineral supplementation? ▪ Any change in appetite? Have you noticed weight gain/loss? ▪ Do you have a regular exercise routine? ▪ Do you open your bowels regularly? Any problems with your bowels? ▪ Are your influenza and pertussis vaccinations up to date? o Social history ▪ Do you have a stable partner? ▪ Who do you live with? ▪ Do you have good support? o SADMAC ▪ Do you smoke?

▪ How long have you been smoking? How many per day? ▪ Do you drink alcohol? ▪ What kind? How often? How much per session? ▪ Do you take any recreational drugs? 175 ▪

What kind? How often? Page

© 2020 ARIMGSAS

▪ Do you take any prescription or over the counter medications? ▪ What medication? For what reason? Who prescribed it? ▪ Do you have any known allergies? ▪ Do you take any medications for this? ▪ Do you take coffee regularly? ▪ How much per day? o Any past history of any medical illness especially clotting problems (thromboembolic disease), diabetes, epilepsy, thyroid problems, or high blood pressure? o Any previous abdominal and gynecological surgery in the past? o Any family history of any medical, surgical, or mental illness?

▪ Physical Exam o General appearance: pallor, oedema o Vital signs: temperature, BP o CVS/Respiratory/CNS o Abdomen: fundal height, FHR, lie, presentation; can you feel foetal movements? Are there uterine contractions? Is the head engaged or not? (patient is a primigravida, the head should get engaged at this time) o Pelvic examination ▪ Inspection of the vulva and vagina: bleed, discharge, rash, vesicles ▪ Speculum: is the os closed or open?

▪ Diagnosis and Management What I am suspecting is a reduced fetal movement. Baby kicks in a primigravid start at around 18 to 20 weeks, and it is a good indicator of the baby's health. The baby kicks cannot be felt well if the mom is having a busy lifestyle, and if you are sitting or standing, and it is best felt if you lie down on your left side. One of the most common causes of reduced fetal movement is fetal sleep or the baby is sleeping. Usually, the baby movements will be more towards the afternoon and evening, and it is usually interspersed with sleep cycles lasting for 20 to 40 minutes, or sometimes, although rarely, it can go greater than 90 minutes. There can also be pathological causes for this like if the fluid in the bag surrounding the baby is less or oligohydramnios, or if the baby is not growing well. But as far as your examination goes, everything is doing well with your baby, and the baby's heart rate is also good. Certain complications in your, such as recent infections, anemia, and thyroid problems, can affect the blood supply to the placenta and the baby, thereby decreasing the movements of the baby. But it is unlikely to present in you based on your history and examination. The most likely cause is that the baby is sleeping. Many pregnant women experience a single episode of reduced fetal movement during their pregnancy, and in more than 70% of these women, there are no complications. However, even though everything is normal at this stage, we need to confirm it further. I need to refer you to the hospital and in the hospital, a CTG will be done to monitor the baby's heart rate at least for 20 minutes, and an ultrasound will also be done to look for the baby's heart movements or cardiac activity, and also to rule out growth retardation in the baby, and oligohydramnios. You

will be discharged after a brief period of monitoring, but you need to continue maintaining a kick chart of your baby, and the baby should move 10 times in a 2-hour period. Do you have someone to take you to the hospital? I will give you reading materials regarding reduced fetal movements. 176

Page

© 2020 ARIMGSAS

Iron Deficiency Anemia

You are a GP, and your next patient is a 33-year-old lady, Helen. She is 30 weeks pregnant, and this is her fourth pregnancy. Her blood test results done 1 week earlier showed decreased iron levels.

TASKS 1. Further relevant history 2. PE from examiner 3. Discuss management to the patient

Positive points in the history: feels extremely tired and lethargic, was anemic in previous pregnancies, took iron supplements during pregnancy but stopped afterwards, children are 2, 3/12 and 5 years old (poor spacing), does not eat vegetables Positive points in the PE/Investigations: fundal height is 30cm, FHR normal, longitudinal cephalic

APPROACH ▪ History o Why did you have the blood test done? o Anemia symptoms: • Any tiredness, chest pain, palpitations, shortness of breath, oedema? • Did you have anemia before you got pregnant or anytime during your previous pregnancies? • Have you taken any iron supplements? • What are the ages of your children? • What is your usual diet? Does it include meat and green leafy vegetables? Are you on any special diet? • Do you have any decreased appetite? • Any diarrhea? Any nausea or vomiting? • Any exaggerated symptoms of pregnancy? • Any bleeding disorders, or are you bleeding from anywhere in the body like gums, black stools? • How were your periods before you became pregnant? Any excessive bleeding? o Pregnancy • Any bleeding or discharge from down below? Any tummy pains? (you don't ask for symptoms of preeclampsia yet because the symptoms occur at around 25/26 weeks) • Have you started feeling the baby kicking? • Did you do all your antenatal tests? How were the results? ▪

Routine first visit tests 177

o FBE Page

© 2020 ARIMGSAS

o Blood group and antibody screen o Rubella antibody status o Syphilis serology o Midstream urine o Chlamydia o HIV o Hepatitis B and C serology o Varicella o Cervical cytology/Pap smear o Screening for Down syndrome (if >35 years old) ▪ Ultrasound at 18 to 20 weeks? o How is the baby? • How were your previous pregnancies and deliveries? Any complications during those? • Any history of medical or surgical illness? • Any family history of anemia? • Any medications or allergies?

▪ Physical Exam o General appearance: pallor, skin for petechiae or bruising o Vital signs o CVS/Respiratory/CNS o Abdomen: fundal height, FHR, lie, presentation o Pelvic examination • Inspection of the vulva and vagina: bleed, discharge • Speculum: cervix for any bleed or discharge o Office test: UDT, BSL

▪ Diagnosis and Management What I am suspecting in you is an iron deficiency anemia. It is a common condition that happens in pregnancy because there is increased demand during pregnancy because you need to supply iron for yourself and your baby. It is a risky condition because the iron level is correlated with the oxygen supply, which is important for the baby's growth and development. But if it diagnosed and treated early, it does not usually cause a problem during pregnancy. We need to be concerned if it is severe, untreated, and if it lasts long. If it is untreated, it can result in maternal complications like lethargy, tiredness, exhaustion and dizziness, it can lead to heart failure and also postpartum haemorrhage or bleeding after delivery. In the baby, it can result in low birth weight and growth retardation. The causes of this condition are multiple. It could be due to the diet, or the pregnancies that are close together, and as you had a past history of anemia. I need to start you on elemental iron, 40- 80mg per day. The iron tablets should not be taken along with dairy products, but you can take orange juice along with it as vitamin C increases the absorption of iron. After you start on iron, your haemoglobin levels will go up in two weeks' time, but you need to continue taking the iron tablets still for at least 3 months after your iron comes back to normal to replenish the iron stores.

Once you start taking the iron tablets, you may experience a bit of abdominal discomfort, and constipation, your stools may turn black. You should take a lot of water to avoid constipation. Include green leafy vegetables, red meat, beans and lentils in your diet, and cut down on coffee,

tea, soy products as all these can decrease your iron absorption (key point). 178 Page

© 2020 ARIMGSAS

If no improvement is seen after taking the supplements, then I will refer you to the haematologist for further assessment and management. (If Hgb is less than 6, go in for blood transfusion; if less than 8 but greater than 6, IM iron injections; above 8, oral iron supplements) Anemia: if less than 110 prior to 20 weeks, and less than 105 after 20 weeks.

179 Page

© 2020 ARIMGSAS

Post-dated Pregnancy

You are at your GP, and your next patient is a 31-year-old lady, Mary. She is 41 weeks pregnant and this is her first pregnancy. Today she has come to your GP clinic because she is concerned about her pregnancy.

TASKS 1. Take a relevant history 2. PE from examiner 3. Discuss your management with the patient

APPROACH ▪ History o What are your concerns? o How is your pregnancy going so far? o Any tummy pain, bleeding or discharge from down below? o Any tightening of the abdomen? o Is your baby kicking well? o Are you experiencing any problems such as excessive vomiting, burning or stinging on passing urine, fatigue, headache, blurring of vision & swelling, fever/malaise? o Did you do all your antenatal tests? How were the results? ▪ Routine first visit tests o FBE o Blood group and antibody screen o Rubella antibody status o Syphilis serology o Midstream urine o Chlamydia o HIV o Hepatitis B and C serology o Varicella o Cervical cytology/Pap smear o Screening for Down syndrome (if >35 years old) ▪ Ultrasound at 18 to 20 weeks? o Is it a single baby? Any birth defects? What is the position of the placenta? ▪ Sweet drink test at 28 weeks? ▪ Repeat ultrasound at 32 weeks? o Is the baby growing well? Is the fluid in the bag surrounding the baby adequate?

▪ Bug test at 36 weeks? o General well-being questions: • Are you taking your folic acid? When did you start taking it? (folic acid 0.5mg/day 3 months before and 3 months after) 180

• Are you eating a healthy balanced diet? Do you drink a lot of fluids? Page

© 2020 ARIMGSAS

• Are you taking any vitamin or mineral supplementation? • Any change in appetite? Have you noticed weight gain/loss? • Do you have a regular exercise routine? • Do you open your bowels regularly? Any problems with your bowels? • Are your influenza and pertussis vaccinations up to date? o Social history • Do you have a stable partner? • Who do you live with? • Do you have good support? • How far do you live from the hospital? (because she can go in for labor anytime) o SADMAC • Do you smoke? ▪ How long have you been smoking? How many per day? • Do you drink alcohol? ▪ What kind? How often? How much per session? • Do you take any recreational drugs? ▪ What kind? How often? • Do you take any prescription or over the counter medications? ▪ What medication? For what reason? Who prescribed it? • Do you have any known allergies? ▪ Do you take any medications for this? • Do you take coffee regularly? ▪ How much per day? o Any past history of any medical illness especially clotting problems (thromboembolic disease), diabetes, epilepsy, thyroid problems, or high blood pressure? o Any previous abdominal and gynecological surgery in the past? o Any family history of any medical, surgical, or mental illness?

▪ Physical Exam o G/A: pallor, edema o V/S: BP o I would to quickly examine the Cardio, Respiratory then focus on my examination in the abdomen. o Abdomen: fundal height, fetal heart rate, fetal lie, presentation, is the head engaged? Are there any uterine contractions? o Pelvic examination: o Inspection of vulva and vagina: discharge/bleeding, rash/vesicles o Speculum examination: I would like to do a sterile speculum examination ▪ Cervix: bleeding and discharge, open or close cervical OS o Office test: Aside from physical examination findings, I would like to check for UA, BSL

▪ Diagnosis and Management Any pregnancy is defined as post-dated if it is more than 42 weeks. Normal pregnancy is 40 weeks

plus or minus 2 weeks. We can consider that you are still within the normal period. This is not an uncommon condition. It is more common to happen during the first pregnancy, and also if you have a previous post-term pregnancy. Risks for post-date pregnancy in you are obstructed labor as there is a chance of a big baby with 181

ossified skull and molding does not happen during delivery, and because of this, you can have Page

© 2020 ARIMGSAS

perineal tears. There is also increased chance of infections, emotional stress, increased C-section rates, and postpartum hemorrhage. In the baby, the baby can have fetal distress and have meconium aspiration. Because of distress, the baby can open his bowels within the womb, and this could be aspirated by the baby when the baby is born. The placenta can become insufficient leading to intrauterine growth retardation. I will refer you to the specialist, and a CTG will be done along with an ultrasound to look for any placental insufficiency. A Doppler study will also be done, and amniotic fluid will be assessed. You have two options now. First is an elective induction of labor. The cervix needs to be dilated by using mechanical devices by using Foley's catheter or by using medications like prostaglandin E2. Once the cervix is dilated, uterine contractions will be induced by rupturing the membranes, and if necessary, by an oxytocin drip. The second option is expectant management where you can wait for one more week. But here again, CTG needs to be done twice weekly along with ultrasound and Doppler studies weekly. Meanwhile, maintain a kick chart of the baby, and if anything goes wrong, a C-section needs to be done. The decision is yours, after consultation with the specialist. I will give you reading materials regarding post-dated pregnancy. Do you have any further concerns?

182 Page

© 2020 ARIMGSAS

Bipolar Disorder in Pregnancy

You are at your GP, when 28-year-old Laura presents to you. She is having bipolar disorder since the past 2 years and is taking Lithium. She is planning to start a family and would like your advice regarding that.

TASKS 1. Take a further relevant history 2. Counsel the patient accordingly

Positive points in the history:

APPROACH ▪ History o Ensure confidentiality o Bipolar Disorder • Since how long have you been having Bipolar disorder? • When was the last episode of your bipolar disorder (if she has not had an episode in 1 year, you can stop Lithium)? • What symptoms did you have at that time? • Did you have any ideas of self-harm during those episodes? • Do you go for regular appointments with your psychiatrist? • When was your last appointment with the psychiatrist? o Psychosocial history • How is your mood? • Do you have passing thoughts of harming yourself or others? • How is your sleep? (sleep deprivation can lead to bipolar disorder) • Are you eating well? o Medications • Since how long are you on your medications? • Any other medications that you are taking apart from Lithium? • Any side effects of Lithium that you are experiencing like tremor, dizziness? • Any weather preferences, hair loss? (side effects of Lithium: can induce hypothyroidism, hair loss) • How often are your Lithium levels monitored? • Any hospitalizations in the past? o Reproductive health • Period history ▪ When is your LMP?

▪ Is it regular? ▪ What is the duration of your cycle? ▪ What is the amount of your bleeding, mild, moderate, severe? ▪ Any problems during your periods? 183

• Sexual history Page

© 2020 ARIMGSAS

▪ Are you into a stable relationship? ▪ Do you have very good support from your partner? ▪ Any history of STIs? ▪ When was your last pap smear, and what was the result? ▪ Have you ever been pregnant before? Any miscarriages in the past? o Any previous history of any medical or surgical illness, mental illness? o SADMA history

▪ Counselling Many women who have bipolar disorder and on medications for bipolar disorder go for a normal and healthy pregnancy and deliver healthy babies. Bipolar disorder can sometimes worsen during pregnancy and that might require hospital admission. The bipolar medications can cause birth defects in the baby especially if it is taken during the first trimester. The baby can develop neural tube defects, heart defects, and once the baby is born, the baby can have developmental delays and behavioural problems. There is also a high chance of relapse of bipolar disorder within the first month of delivery. When are you planning for your pregnancy? Now, I need to refer you to the psychiatrist for review of your medications. There are three strategies that the psychiatrist can do, depending on the severity of the bipolar episodes. o First is a medication-free pregnancy, and also after delivery if you are asymptomatic. It is usually done if a patient has a few episodes of the disorder, long periods of mood stability (at least 1 year), low risk of self-harm, good support, and if the patient is able to identify early warning signs. The specialist will decide on a safe and supervised withdrawal of Lithium over the next 2-6 weeks. o The second strategy is to go for partial prophylaxis. The aim is to go for a medication-free first trimester (because Lithium is contraindicated in the first trimester) and restart Lithium after the first trimester. The baby will be monitored by ultrasound and fetal echocardiogram to check for any cardiac defects. Serum levels of Lithium will be monitored at regular intervals. o The third strategy is to give a mood stabilizer throughout the pregnancy. The specialist will consider the option of changing Lithium to another mood stabilizer carrying less risk like Lamotrigine, Olanzapine, or Quetiapine. If the bipolar episodes are not controlled with these medications, Lithium will be reintroduced after the first trimester. Prior to your pregnancy, we will do baseline antenatal blood tests like FBE, UEC, Blood group and Rh, coagulation profile, rubella serology, hepatitis serology, and TFTs. Serum levels of your medications will also be checked. I will also advise you to take folic acid, a high dose of 5mg/day 3 months before and for the first 3 months of your pregnancy if you will be taking Lamotrigine, otherwise, take the usual dose. Once you become pregnant, you will be monitored in the high-risk pregnancy clinic and will be seen by a multidisciplinary team composed of the obstetrician and the psychiatrist. During your pregnancy, a Down syndrome screening, ultrasound at 18 to 20 weeks, OGTT at 28 weeks (high chance of Diabetes with Olanzapine and Quetiapine), repeat ultrasound at 32 weeks will be done, and a bug test at 36 weeks will be done.

You will also be advised to go for more frequent antenatal checks, and as the pregnancy progresses, CTG monitoring of the baby will be done. And if you will be put on Lithium during pregnancy, the baby will be monitored for cardiac defects. Usually, the dose of Lithium has to be decreased by 38 weeks to reduce high levels of Lithium in 184

the baby. Page

© 2020 ARIMGSAS

Renal function tests and thyroid functions test will also be regularly checked. You may go in for normal vaginal delivery, but it should be in a tertiary hospital under the guidance of a specialist. Follow a healthy lifestyle, which is very important in pregnancy. Get adequate sleep, follow a healthy diet, get regular exercise, and drink plenty of water to prevent Lithium toxicity. After delivery, if you will not be breastfeeding, you will be put back on the full therapeutic dose of Lithium. However, if you intend to breastfeed, then Olanzapine will be given to you instead. Our goal during your pregnancy is to maintain maternal wellbeing, ensure your baby's safety, and also prepare you for the post-partum period. I will give you reading materials regarding bipolar disorder during pregnancy for further insight and will review you regularly. If you experience any tremors, dizziness, suicidal ideations, please report to the ED immediately.

185 Page

© 2020 ARIMGSAS

Urinary Incontinence

Tamara, 68 years old, presents to you at your GP clinic with complaints of inability to hold urine for the past 6 months. She is quite worried about this and wants you to help her.

TASKS 1. Take a further history 2. PE from examiner 3. Investigations with the examiner 4. Diagnosis and Management with the patient

Types of Incontinence in the AMC: ▪ Stress incontinence: happens due to weakness of the sphincter ▪ Urge incontinence: happens due to detrusor muscle irritability and small bladder capacity ▪ Mixed incontinence: a combination of urge and stress incontinence

Causes ▪ The rise in the intraabdominal pressure o Cough o Constipation ▪ Bad obstetric history ▪ Menopause ▪ Urinary tract infection ▪ Medications (cholinergic, tranquillizers, diuretics) ▪ Bladder irritants (smoking, alcohol, recreational drugs, excessive coffee)

Positive points in the history: leakage of urine for the past 6 months, leaks a large amount of urine before I go to the toilet, feels stressed when I go out because I feel that I might pass urine, leaks small amount urine during coughing, sneezing, gets up around 2-3 times at night to go to the loo, fears that she might wet her bed, had menopausal symptoms for 1 year then it went away, has bad obstetric history Positive points in the PE/Investigations: BP 110/80, Temp 36.7, PR 86 regular, RR 18, 99% sats; BMI is 30, can see leakage of urine on cough test (stress incontinence)

APPROACH ▪ History o Ensure confidentiality o Reassurance

o History of present illness • How long have you been experiencing the leakage of urine?

• When do you experience this? Is it when you cough, sneeze, or do exercise? 186 • How much of urine do you leak at that time?

• Any sensation of something protruding from down below? (prolapse) Page

© 2020 ARIMGSAS

o Urge incontinence • One you feel the urge to go to the loo, can you hold the urine back? • Does urine leak out before you reach the loo? • How much urine do you leak at that time? • At night, do you need to get up quite often to go to the loo? o Identify the cause • How is your waterworks? Any burning or stinging while passing urine? Do you have any fever? • How are your bowels? Do you have any constipation? • Any chronic cough that you are having? • Do you smoke, drink alcohol, take recreational drugs? How much coffee do you consume in a day? o Menopausal history • When did you have your menopause? • Any mood changes, heavy sweating, hot flushes after menopause? • Did you take any HRT? • When did you last have your pap smear? • When did you last have your mammogram? o Sexual history • Are you still sexually active? • Do you have good support? • How were your past pregnancies and deliveries? How many have you had? • Any history of prolonged labour, obstructed labour, big babies, or any instrumental delivery? • Any other surgeries that you had on down below? o Are you taking any prescription or over the counter medications? o Any allergies to any medications? o Any medical or surgical illness? o Do you have any difficulty moving around?

▪ Physical Exam o General appearance: BMI, PICCCLED o Vital signs: temperature o CVS, Respiratory, CNS o Abdomen: visible mass/distention, tenderness, palpate for mass tenderness o Pelvic examination • Inspection of the vulva and vagina: any bleed, visible discharge, prolapse, atrophic vagina • Speculum: the cervix is healthy or not, discharge or bleeding from the cervix, Cough test: after placing a pad I would like to ask the patient to cough; any leakage of urine from the urethra, any prolapse that you can see? • Per vaginal exam: uterine size and tenderness, adnexal mass and tenderness o Office test: UDT, BSL

▪ Investigations o Blood tests: FBE, UEC, ESR/CRP, LFT, urine MCS (check for asymptomatic bacteriuria)

o Refer to urogynecologist for Urodynamic studies (check for urge incontinence) 187 Page

© 2020 ARIMGSAS

• Has 4 parts: (all 4 parts need to be done in urge and mixed; only post void in stress incontinence) ▪ Urethrocystoscopy: look at the urethra and bladder to look for any polyps and other lesions that might irritate the bladder ▪ Uroflowmetry: measure the amount of urine that is passed and speed that it is passed ▪ Cystometry: look at bladder capacity, how full the bladder is when she feels the urge to void ▪ Post void residual volume: ask the patient to void and then view the bladder (a most important part of the urodynamic studies) • If greater than 50mL, it is severe

▪ Diagnosis and Management Most likely you are having a mixed incontinence. It is a combination of stress and urge incontinence. Stress incontinence is leakage of urine when there is a sharp rise in the tummy pressure, usually happens when you cough, sneeze, or when you strain during a bowel movement, and during exercise. Urge incontinence is a sudden strong urge to urinate due to an overactive bladder, where the bladder muscles contract at wrong times. The most important cause of incontinence is a pelvic muscle floor weakness. Pelvic floor muscles and ligaments support the bladder and the urethra along with the uterus and the vagina. Pelvic floor muscle weakness can happen due to several reasons: o One is gaining extra weight or overweight when the pelvic muscles start becoming weak; o Second a bad obstetric history, which causes stretching and damage to the pelvic floor muscles; o But the major contributing factor is menopause. The estrogen or the female hormone is necessary to maintain the stability and strength of the pelvic floor muscles. At the time of menopause, the ovaries will shut down, so that very low levels of estrogen are being formed. This estrogen deficiency contributes to pelvic muscle floor weakness. The first step of management is lifestyle modifications. Your BMI is a bit high, so you need to bring down your weight to a more optimum level. I will refer you to the dietician who can help you with a diet plan appropriate for your weight goals. Adopt a regular physical exercise as well. (if the patient has cough and constipation, taking excessive coffee, smoking, alcohol, fizzy drinks, address it here as well) Maintain a bladder diary and record in it the times that you are having incontinence so that we can correctly identify the type of incontinence and see whether the treatment is effective.

If it is stress incontinence: o Do pelvic muscle floor exercises (Kegel exercise)? I will refer you to a physiotherapist who can better advise you regarding that. You can do the exercise in a sitting, standing or lying down position. Contract your bottom muscles, for a count of 8, relax it for the same amount of time that you have contracted the

muscles, or count of 8 also, and do it 8 times at a go, three times a day. o If conservative measures fail (or if there is no improvement in 6 months' time), I can refer you to the surgeon who might do several options: • a sling surgery via keyhole surgery. A U-shaped mesh tape in the form of a sling will be 188

installed to support the bladder and the urethra. Page

© 2020 ARIMGSAS

• Urethra-bulking agents. Substances like collagen will be injected into the bladder neck to make it tighter and stronger. • Colpo suspension. We use stitches to suspend or lift up the neck of the bladder and urethra and attach those to the pubic bone.

If it is urge incontinence: o Along with the pelvic muscle floor exercises, you have to do bladder re-training exercise. When you feel the urge to go to the toilet, try to hold it back, initially for 5 minutes, then gradually increase the span of time before you go to the toilet. Once you reach the toilet and start passing the urine, try to hold back the urine at times, and then restart again. (Goal is to try to increase the urine volume capacity of the bladder, and decrease the irritation of the detrusor muscle) o Botox injections in the bladder wall can decrease the spasms of the bladder wall and thereby can improve the urge incontinence. o [surgery for urge incontinence is rarely done; it is called an augmentation cystoplasty: attach a loop of bowel to the bladder to increase bladder capacity, but it is a complicated surgery that is why it is not widely done]

Key point is to start the patient on medications (antimuscarinic agents) o Oxybutynin o Propantheline bromide Initially for 1 month and if there is a definite improvement, continue for 6 months.

If it is mixed incontinence, it is a mix of the management for stress and urges incontinence.

189 Page

© 2020 ARIMGSAS

OCP-induced Hypertension

You are a GP and 26 years old Susan comes to see you. She was started on combined pills 3 months ago by a colleague of yours. She's having a headache for the past 2 weeks.

TASKS 1. Further history 2. PE from examiner 3. Discuss management plan with the patient

Differentials: ▪ COC-induced hypertension ▪ PCOS ▪ Hyperthyroid

Positive points in the history: COC for the past 3 months, the character of the headache is undefined Positive points in the PE/Investigations: BMI is 26, BP 155/95

APPROACH ▪ History o Ask the pain scale. Offer painkiller. o Headache questions • Where exactly is the pain? • How long have you been having a headache? On/off or continuous? • Does it go anywhere else? • What sort of pain are you having? Throbbing (migraine), band-like constricting (tension) • Anything making it better or worse? When exposed to light (migraine) • Any food that you take that may trigger your headache -- chocolates, cheese, red wine? • Associated symptoms (Differentials) ▪ URTI/sinusitis: any fever, colds, facial pain? ▪ Referred pain: Any pain in your ears or around your teeth? ▪ Meningitis: Any nausea, vomiting, blurring of vision? Any rash? ▪ Trauma: Did you hit your head somewhere? ▪ CVS symptoms (a side effect of pills): Any dizziness that you're having, any chest pain, funny racing of the heart? o COC history

• What type of pill are you having? • What contraception was you on before this?

• Any particular reason why did you opt for these pills? 190 • Any other side effects of these pills? ▪ Breakthrough bleeding, weight gain, bloating, pain or swelling in your calf? Page

© 2020 ARIMGSAS

o Period history • LMP, regularity, pain, any bleeding in between your menses o Sexual history • Sexually active? Stable relationship? Pregnancies or miscarriages for you? STI? o When did you have your pap smear? What was the result? o Lifestyle • What sort of diet do you usually take? • Any regular exercise that you do? • Do you smoke, drink alcohol, take recreational drugs? • Any stress at home or at work? o Any other medical or surgical illness? Any history of high blood pressure? Any heart disorders? Any migraine? o Any other medications that you are taking other than the combined pills? Any allergies?

▪ Physical Exam o General appearance: BMI, PICCCLED, any hirsutism, acne (PCOS) o Vital signs: BP o Neck: any thyroid swelling o CVS: S1 S2 normally heard, any murmurs o CNS: fundoscopy to look for papilledema o Abdomen: renal or suprarenal mass, bruit o Pelvic examination o Office test: UDT, BSL

▪ Diagnosis and Management Most likely you are having a combined oral contraceptive pill-induced hypertension. This is also the reason why you are having headaches. However, with only one reading, we cannot say for sure that it is already hypertension. We need to check your blood pressure two more times to say that it is hypertension. Do you have a BP monitoring apparatus at home? If you do, please record your blood pressure for the next two days. If not, please visit the GP for blood pressure monitoring for 2 days. Meanwhile, you can take Panadol for the headache. If it is confirmed that you have hypertension on the 3 readings, you need to stop the combined OCPs. Then we will recheck your blood pressure after 2 days of stopping the combined OCPs. If your blood pressure is already low after stopping, we can confirm that it is just the OCPs that is causing high blood pressure. I would like to discuss with you other options for contraception. If your blood pressure remains high despite stopping the OCPs, then we need to look for other causes of high blood pressure. We need to organize investigations such as FBE, UEC, LFTs, TFTs, serum lipid profile, BSL, and also ECG. I would advise you to adopt lifestyle modifications as your BMI is quite high. Do relaxation exercises as well to decrease stress that might also contribute to elevated blood pressure. I will give you reading materials about hypertension. I will review you on the third day of monitoring your blood pressure. If you experience worsening of your symptoms, please report back.

191 Page

© 2020 ARIMGSAS

Fibroadenoma

A 25-year-old lady, married, comes to your GP with a history of a breast lump which she accidentally found out one week ago. She's pretty worried about this and she wants you to address her concern.

TASKS 1. Further history 2. PE from examiner 3. Discuss your management with the patient

Positive points in the history: lump is a size of a small cherry on the left breast, family history of breast cancer: mom has breast cancer, on combined OCPs Positive points in the PE/Investigations: lump is 1-2 cm, smooth round mobile swelling, soft to firm in consistency, borders well-defined and non-tender, no lymph node swelling

Differential Diagnoses ▪ Fibroadenoma ▪ Breast cysts (happens in perimenopausal women ▪ Fibro adenosis - lumpy breast +/- lump ▪ Breast malignancy

APPROACH ▪ History o History of present illness • How long have you noticed that lump? • How did you notice the lump? • Which breast is the lump in? • Is it a single or multiple lumps? • Is it increasing in size or not? • Any pain in the lump? • Is the lump moving freely? • Is the lump soft or hard to feel? • Any redness in the skin over the lump? • Any lumps in the opposite breast? • Any lumps and bumps in the armpits or neck area? • Is this the first time that you have felt a lump in your breast? (fibroadenoma can be recurring) • Nipples ▪ Any watery discharge, or blood-stained discharge?

▪ Any ulcerations or distortion of the nipple? o Differentials

• Fibro adenosis: any increased pain in your breast just before your periods? 192 • Breast malignancy: any weight loss?

• Breast metastasis: any back pain, headache, shortness of breath? (spine, brain, lungs) Page

© 2020 ARIMGSAS

• Fat necrosis: any history of trauma? o Period history • When is your LMP? Are your periods regular? Any problems with your periods? o Sexual history • Are you into a stable relationship? Are you sexually active? • Any pregnancies so far? • Do you use any contraception? What type? • Since how long have you been on the pills? • Any complications with the pills that you are experiencing? o Any family history of breast lumps? • At what age was she diagnosed with breast cancer? (screening will depend on the age) How is she now? • Any sisters that have been diagnosed with breast cancer? o When was your last pap smear? What was the result? o SADMA o Past Medical History

▪ Physical Exam o General appearance: BMI, PICCCLED, axillary lymph nodes on the right and left side, supraclavicular and cervical lymph nodes o Vital signs o Breast: I would like to examiner the left breast, comparing it with the right breast. • Any lump or mass that you can see or feel? ▪ Where is the mass seen or felt? ▪ What is the size, shape, mobility, borders, consistency? Is there tenderness? ▪ How is the skin on the surface of the lump? Any redness, dilated veins, tethering? ▪ Any changes around the breast that you can see? • Any discharge is seen from the nipple? Any nipple retraction, distortion or ulceration? • Any lumps that can be felt on the same breast or on the opposite breast? • Any axillary, cervical or supraclavicular lymph nodes? o Lungs o Liver: hepatomegaly o Abdomen o Spine: specific tenderness o Office tests: BSL, UDT

▪ Diagnosis and Management Most likely you have a condition called Fibroadenoma. It is a benign non-cancerous lump that arises from the breast tissue. It is a very condition that is seen in this age group. It is not a nasty growth, and you will not undergo a nasty change. But we have to confirm this by doing a triple test. o The first part is a clinical examination which I have done on you.

o The second part is imaging. For women below 35 years, we do an ultrasound, and for above 35 years, we do a mammogram. o The third part is by cytology. We do this by doing a fine needle aspiration biopsy. A fine needle is introduced into the lump, and cells from the lump are aspirated and observed 193

under the microscope for any nasty growth. Page

© 2020 ARIMGSAS

If this is confirmed to be a fibroadenoma, then there are two options for you: o The first option is that we can leave it behind, but with regular monitoring. To begin with, we will do an ultrasound every 6 months and then annually. If you see that the lump is increasing in size, if it is becoming fixed or hard, or you develop any other lumps in your breast or armpits, please report back. o The second option is to go for surgery. Surgery can be in the form of lumpectomy wherein only the lump is removed under local anesthesia, but the complications of lumpectomy are: there is a chance of recurrence and there could be some scarring and dimpling in the breast, and also a little bit of duct damage; or another method is cryocoagulation, where a wand- like device is introduced into the lump and gas is pumped in to freeze the lump.

Can I continue with the combined OCPs? Yes, if it is just a benign lump in the breast, she can continue with the OCPs. This is protective against other benign lumps as well.

Is FNAC confirmatory or diagnostic of a benign lump? The needle is directed at different points of the lump and is aspirated. It is around 90-95% confirmatory.

Since your mom has been diagnosed or breast cancer before 50 years of age, it is advisable to do a mammogram annually from 40 years of age. The same criteria hold if two first-degree relatives from the same side have been diagnosed with breast cancer at 50 years or older.

If you have any sisters, advise her to do the mammogram from 40 years as well.

I will give you reading materials about breast lumps and I will arrange a review with you once the investigation results come in.

Breast Cysts o age is usually 40-50 years old o Usually in perimenopausal women o Usually associated with mammary dysplasia and fibroadenosis of the breast o It decreases after menopause o Presents as a discrete mass, firm, relatively mobile, sometimes can be fluctuant as well o Management: Triple test • FNAC: can aspirate the whole cyst (diagnostic and therapeutic) ▪ If the fluid is blood-stained, and if after aspirating the lump does not subside, or if the cyst recurs 1 month after the aspiration, then you have to suspect a malignant lump.

Fibroadenosis (Mammary Dysplasia) o 30 - 40-year-old

o Presents as nodularity in the breast

o In between the nodularity, you get a discrete mass o There can be pre-menstrual discomfort, pain, and increased swelling of the breast which

settles after the periods 194 Page

© 2020 ARIMGSAS

o Can be associated with a nipple discharge (greenish gray in color, but rarely can be blood- stained; but if it is blood-stained, your first consideration is an intraductal papilloma) o Clinical exam + mammography or ultrasound = if no lump o Triple test = if with lump o Management: painkillers, breast supports, reduce coffee, alcohol intake, evening primrose oil but is not very effective o Self-breast awareness (breast examination): if in case she develops breast lumps, report back.

195 Page

© 2020 ARIMGSAS

OCP Counselling in a 14-year-old

You are working as a GP, and a 14-year-old student came in your clinic asking a prescription of oral pills. She is planning to become sexually active.

TASKS 1. Take a history from the patient 2. PE from examiner 3. Counsel her

Positive points in history: in a stable relationship, been with boyfriend for 6 months, boyfriend is 15 years old

APPROACH ▪ History o Hi, I'm Dr____, your GP today. o I read from the notes that you want me to prescribe you some pill and you want to be sexually active soon. o I know this might be a little sensitive, I want to assure you that everything we talk about here will be private and confidential unless it poses a risk to you or to others. o In this case, you are a 14-year-old, I would like to ask you a few things before I talk to you about the pills. Is that alright? • Do you still live with your parents or do you live independently? • When do you plan to be sexually active? • Are you in a stable relationship? • How old is your boyfriend? • What is your relationship with your partner prior to becoming your boyfriend? Is he your friend, classmate? o I would like to ask you a few things that are really important, just to make sure you will not have a problem if and when you start taking the pills. • Absolute contraindications ▪ Do you have any history of clotting? ▪ Do you have any history of stroke? ▪ Any history of heart disease? ▪ Any history of migraine? ▪ Any problems with your liver? Any discolouration of your skin? Any tummy pains? ▪ Any nasty growth in your breast, chest, or anywhere in your body?

• Relative contraindications? ▪ Do you smoke? ▪ Do you have any abnormal vaginal bleeding? ▪ How is your mood? Have you ever been diagnosed with depression? 196

▪ Do you have any high blood pressure? Page

© 2020 ARIMGSAS

▪ Do you have any blood sugar problems? ▪ Do you have high blood cholesterol? o Period history • When did you last have your period? • Is it regular? • Any problems with your periods? • Have you taken your Gardasil vaccine? (9-25 years old) o Any past history of any medical or surgical illness? o SADMA • Do you smoke? • Do you drink alcohol? • Do you take recreational drugs? • Any known allergies? Do you take any medications?

▪ Physical Exam o I need to examine you now, is it alright with you? I would like to talk to the examiner. o General appearance: pallor, LN, BMI o Vital signs: BP o CVS o Thyroid: palpate o Breast: any breast lump o Abdomen: mass, hepatomegaly o Leg: pain, swelling of the calf muscle o With the patient's consent, and in the presence of a chaperone, I would like to do a pelvic examination • Vulva and vagina; any discharge a bleeding • I would not proceed with a per vaginal exam as this patient has not been sexually active before

▪ Diagnosis and Management o Content of pills • Usually, the OCP contains a low dose of estrogen and moderate dose progestogens which are hormones that are usually found in your body that regulates your cycle. o Action • The main action of the pill is preventing ovulation or release of an egg from your ovary • To a lesser extent, it increases the cervical mucous thickness so that the sperm find it difficult to penetrate through and it changes the lining of the womb o Administration • There are 28 pills in a pack, out of which, 21 are hormonal and 7 are dummy or sugar pills • Start taking the pill from the first day of your next periods. • It offers immediate protection. • When you have finished the hormonal pill, and you reached the dummy pills, you have

your periods also called a withdrawal bleed • Take the pills on your hand. (available in the table during the exam, show it to the patient) 197

Can I start it tom, a week before my period? Page

© 2020 ARIMGSAS

• Yes, you can but you need to take alternative contraceptives, barrier methods during the first seven days.

o Side effects • Minor ▪ Nausea and vomiting, breast tenderness, breakthrough bleeding, spotting between periods, usually it will come down in 3-4 months' time. • Major ▪ DVT, MI, stroke. Look the OCP are low dose pills and the chance of you getting the major side effects is very low. o Advantages • Periods become regular, lighter and shorter • Decreased incidence of anemia • Decreased dysmenorrhea • Decreased incidence of cancer of the womb and the ovary • Decreased thyroid disorders • Decreased incidence of a benign breast lump, fibroadenoma, cysts o Disadvantages • Do not protect against sexually transmitted infections. If you are concerned about this, use barrier methods. o Precautions • If you develop any diarrhea and vomiting within 2 hours of taking the pill, take one more pill and if diarrhea and vomiting continue, use alternative contraception until it lasts. • Whenever you go to a pharmacist or doctor, mention that you are on OCP pills. Some medications can decrease the efficacy of the OCP (anticonvulsant, vitamin C, rifampicin, griseofulvin, antiretrovirals) o Missed Pills • When you miss a pill, there is an instruction on the cover on how you should proceed • Remember it in 24 hours, take the missed pill and just keep going with the rest, even if it means taking two pills on the same day • If she misses a pill for more than 24 hours, she has to take the recently missed pill even if it means taking 2 pills on the same day. When she keeps going with the rest if, within 7 days of missing the pill, she falls into the dummy pill period, skip the dummy pills and start taking the hormonal pills from the next packet. • If she had missed her pill (18-19-20) and 21st day she remembered, take the 20th pill and 21st pill, on 22nd day due for dummy pills, skip the dummy pills and start taking the hormonal pills from the next pack. • You have to use an alternate method of contraception after missing period. o Alright, can you tell me the important points about the pill that you should know? o Follow up • Arrange a review in 3 months to check BMI and BP then yearly review • I will teach you self-breast examination • Advice regarding pap smear, every 2 years

• Reading material regarding the oral pills • And if concerns feel free to contact me.

Consent: 198 ▪

When you are under 12 years old, no one is allowed to have sex with you. Page

© 2020 ARIMGSAS

▪ A person who had sex with a child under 12 has committed serious crimes, called rape and sexual penetration of a child, and can be charged, jailed, and placed on the sex offender register. ▪ When you are 12 to 15 years old, a person can legally have sex with you only if: o You agree to it and they were less than 2 years older than you o You agree to it and they honestly believed that you were 16 or older ▪ A person who had sex with a 12 to the 15-year-old outside of these limited situations has committed serious crimes, called rape and sexual penetration of a child, and can be charged, jailed, a placed on the sex offender register ▪ When you are 16 years or older, another person 16 or older can have sex with you if you both agree to it ▪ A person who had sex with someone who is 16 or older (but who did not agree) has committed a serious crime, called rape and can be charged and jailed ▪ While you are under 18, however, it is also a serious criminal offence for someone who is caring for you, supervising you or has authority over you. o Teacher, sports coach, youth worker, counsellor, foster carer, religious instruction, health professional, police officer, employer to have sex with you ▪ They can be charged, jailed, and placed on the sex offender register ▪ No member of your family is allowed to have sex with you ▪ A person who had sex with a child or young person with a member of their family has committed a serious crime called incest and possibly also rape and sexual penetration of a child, and can be charged, jailed and placed on the sex offender register

199 Page

© 2020 ARIMGSAS

Stage 1 Breast Cancer

You are at your GP and is about to see 58-year-old Susan who had a lump in her left breast. She was seen by another GP who had ordered a core biopsy and the biopsy result has shown invasive ductal cancer with progestogen and estrogen receptor positive. The lump was 1.5cm in size and she has come to collect the result of the cone biopsy.

TASKS 1. Take further history 2. Discuss the results with her 3. Discuss your further management with the patient

ER/PR positive: good prognosis ▪ Treatment: Tamoxifen or aromatase inhibitors x 5 years Her2neu positive: bad prognosis ▪ Treatment: Trastuzumab x 1 year

Management: ▪ Check lymph node status o Sentinel lymph node biopsy • Injects a dye or radioactive substance near the tumour. The lymph nodes which get stained with this dye or radioactive material are identified and removed and the pathologist will be looking if there are cancer cells. o Axillary lymph node dissection • Remove the axillary nodes as a whole ▪ Surgery o Breast conservation surgery o Partial mastectomy o Total mastectomy ▪ Radiotherapy ▪ Adjuvant therapy: chemotherapy +/- hormone therapy o Chemotherapy: (CMF) Cyclophosphamide, Methotrexate, 5-Fluorouracil

Positive points in history: ▪ Noticed the lump since the last 2 months ▪ It is a single lump ▪ It is in the left breast ▪ It is increasing in size ▪ Has watery discharge from the breast, not blood-stained

▪ Sister also was diagnosed with breast cancer ▪ Not having a good idea of what is happening to her

200 Page

© 2020 ARIMGSAS

APPROACH ▪ History o History of present complaint • Lump ▪ How long has the lump been there? ▪ Is it a single or multiple lumps? ▪ Where is the site of the lump? Is it towards the inner or the outer side? ▪ Is it increasing in size? ▪ Any pain in the site of the lump? ▪ Any skin changes that you have noticed over the lump like dimpling or puckering? ▪ Any ulcers or any discharge? ▪ Is the lump fixed or is it mobile? ▪ Is its soft lump or a hard lump? ▪ Any lumps in the opposite breast? • Nipple ▪ Have you observed any changes in the nipple like retraction or indrawing, distortion or ulceration? ▪ Any discharge from the nipple? (can get serous or bloody discharge in carcinoma of the breast) Is it blood-stained? • Lymph nodes ▪ Have you noticed any lumps or bumps in your armpit on the same side or opposite side? How about in the neck? ▪ Has the doctor noticed any lumps in your armpit in his previous examination? • Secondary sites ▪ Have you noticed any weight loss? ▪ Any back pain that you are having? (spinal metastasis) ▪ Any shortness of breath? (lung metastasis) ▪ Any yellowing of the skin? (liver metastasis) ▪ Any headaches? (brain metastasis) • Risk factors ▪ Age greater than 40 ▪ Caucasian race ▪ Early menarche: when did you have your first period? ▪ Combined OCPs: did you use combined OCPs before? ▪ Nulliparity: How many pregnancies have you had in the past? ▪ Late menopause: At what age have you had your menopause? ▪ HRT: Did you take HRT after your menopause? How many years did you take it? • When did you last have your mammogram and pap smear? What were the results? o SADMA • Do you smoke? • Do you drink alcohol? (can predispose to breast cancer) ▪ How much do you drink in one session? How often do you drink?

• Do you take recreational drugs? • Do you take any over the counter and prescription medications? •

Do you have any known allergies? 201

o Any past history of breast cancer, ovarian cancer? Page

© 2020 ARIMGSAS

o Any previously diagnosed medical illness? o Any family history of breast cancer?

▪ Diagnosis and Management Breaking bad news: Do you know why the biopsy has been done? Have you ever been told about your medical situation so far? I am sorry to tell you, unfortunately, I have not so good news. Do you need somebody to be with you while I discuss this with you? The sample of tissue that we have taken from the lump in your breast has shown that it is a nasty growth or cancer. (offer box of tissues, wait for her to settle down) I know that this is not what you wanted to hear, and I also wish the news were better. I can see that this is upsetting news for you. Are you ready to discuss a treatment plan now, or should I arrange another appointment with you? What I am suspecting in you is a stage 1 breast cancer. And if this is the case, it is treatable and survivable. The 5-year survival rate is around 100%. The survival rates have significantly improved over time due to better diagnostic tests and scans following earlier detection and improvement in treatment methods. I will make an immediate referral to the cancer specialist for further investigations to check if it has spread to other parts of your body. We need to do all the blood tests like FBE, UEC, BSL, LFTs, chest x-ray, and a CT scan of the chest.

Management: o Stage 1: • The surgery will either be: 1) a lumpectomy where the cancerous lump is removed with a border of normal breast tissue, and after this, you need to have a radiotherapy to the rest of the breast, and also to the lymph nodes of the armpits or, 2) the surgeon might decide to do a mastectomy where the whole breast is removed and they will plan for a breast reconstruction where a new breast will be made so that you will not be cosmetically affected. No further radiotherapy needs to be done. Meanwhile, the lymph nodes close to your breast will also be checked to see if they also have any cancer cells and removed if necessary. The other treatments that you have to take is a chemotherapy usually if the tumour size is greater than 1 cm or if is high-grade cancer. You also need hormone therapy as the cancer cells in your breast is showing sensitivity for the female hormones, estrogen and progestogen. You will be given Tamoxifen or aromatase inhibitors which blocks these receptors. You need to take them for 5 years. We will all be here to support you through this. It is usually a multidisciplinary team who will manage you. It is also advisable to have yearly mammography of your opposite breast, or to the same breast after breast reconstruction has been done. o Stage 2:

• The tumour will be between 2cm - 5cm in size • Surgery is usually a breast conservation surgery or partial mastectomy. We will check the lymph nodes at that time with a sentinel lymph node biopsy or an axillary lymph

node dissection. After that, we will do whole breast irradiation. Chemotherapy is a 202 Page

© 2020 ARIMGSAS

must in this case, given before or after surgery. If the tumour is big in size, the tumour can shrink if chemotherapy is given before surgery. Hormonal therapy can also be given depending on receptor status. A total mastectomy can also be done will sentinel lymph node biopsy or axillary lymph node dissection. Chemotherapy will be given. Breast reconstruction will be done. o Stage 3: • If the tumour size is greater than 5cm • Give chemotherapy in the beginning to shrink the tumour. If the tumour has Her2neu positivity, then Herceptin (Trastuzumab) will be given before the surgery. If the tumour has shrunk well, you can go in for a partial mastectomy. You can also go in for a total mastectomy with lymph node dissection. Radiotherapy will be given after surgery. If you're planning for a breast reconstruction surgery, however, radiotherapy should be given before that.

• Inflammatory breast cancer: falls under stage 3 cancer ▪ It is a rare form of invasive breast cancer, where the breast becomes red and inflamed as it affects the lymphatic vessels in the skin of the breast. They become blocked and the breast becomes swollen and red similar to an infection. The incidence is 1 -5% of the breast cancers, more common in the younger age group, and usually in obese women. It is a highly aggressive and rapidly progressive cancer, usually with Her2neu receptor positivity. ▪ When the breast is examined, the breast will be red, swollen, inflamed, warm and tender. The skin will show dimpling and there can be involvement of the axillary and supraclavicular lymph nodes. ▪ Differential: Mastitis: occurs in breastfeeding women ▪ Investigations: • Triple test: diagnostic mammogram +/- ultrasound, core biopsy to look for receptors ▪ Management: a multi-modal approach • First, you have to give chemotherapy to shrink the tumor. • Give Herceptin if Her2neu positive • Surgery, which is usually total mastectomy with axillary lymph node dissection • Radiotherapy • Breast reconstruction *if the patient is going in for total mastectomy, ALWAYS discuss for breast reconstruction. It carries high marks in the AMC.

203 Page

© 2020 ARIMGSAS

Stage 2 Breast Cancer

You are in your GP when Jane, 35 years old comes to see you. You had seen her 2 years ago for a lump in her left breast. The lump was 2.5cm in size, you didn't feel any lymph nodes at that time. You had ordered an ultrasound and a core biopsy of that lump, and the results had come out to be an invasive ductal cancer in the left breast, with estrogen and progestogen receptor positivity, and Her2neu negativity. She is with a stable partner and is planning for pregnancy in a year's time.

TASKS 1. Explain the biopsy results 2. Counsel the patient about the management 3. Answer her questions

APPROACH ▪ History • History of present complaint • Lump ▪ How long has the lump been there? ▪ Is it a single or multiple lump? ▪ Where is the site of the lump? Is it towards the inner or the outer side? ▪ Is it increasing in size? ▪ Any pain in the site of the lump? ▪ Any skin changes that you have noticed over the lump like dimpling or puckering? ▪ Any ulcers or any discharge? ▪ Is the lump fixed or is it mobile? ▪ Is its soft lump or a hard lump? ▪ Any lumps in the opposite breast? • Nipple ▪ Have you observed any changes in the nipple like retraction or indrawing, distortion or ulceration? ▪ Any discharge from the nipple? (can get serous or bloody discharge in carcinoma of the breast) Is it blood-stained? • Lymph nodes ▪ Have you noticed any lumps or bumps in your armpit on the same side or opposite side? How about in the neck? ▪ Has the doctor noticed any lumps in your armpit in his previous examination? • Secondary sites

▪ Have you noticed any weight loss? ▪ Any back pain that you are having? (spinal metastasis) ▪ Any shortness of breath? (lung metastasis) ▪ Any yellowing of the skin? (liver metastasis) 204 ▪ Any headaches? (brain metastasis) Page

© 2020 ARIMGSAS

• Risk factors ▪ Age greater than 40 ▪ Caucasian race ▪ Early menarche: when did you have your first period? ▪ Combined OCPs: did you use combined OCPs before? ▪ Nulliparity: How many pregnancies have you had in the past? ▪ Late menopause: At what age have you had your menopause? ▪ HRT: Did you take HRT after your menopause? How many years did you take it? • When did you last have your mammogram and pap smear? What were the results? • SADMA • Do you smoke? • Do you drink alcohol? (can predispose to breast cancer) ▪ How much do you drink in one session? How often do you drink? • Do you take recreational drugs? • Do you take any over the counter and prescription medications? • Do you have any known allergies? • Any past history of breast cancer, ovarian cancer? • Any previously diagnosed medical illness? • Any family history of breast cancer?

▪ Counselling Do you know why the biopsy has been done? Have you ever been told about your medical situation so far? I am sorry to tell you, unfortunately, I have not so good news. Do you need somebody to be with you while I discuss this with you? The sample of tissue that we have taken from the lump in your breast has shown that it is a nasty growth or cancer. (offer box of tissues, wait for her to settle down) I know that this is not what you wanted to hear, and I also wish the news were better. I can see that this is upsetting news for you. Are you ready to discuss a treatment plan now, or should I arrange another appointment with you? I will make an immediate referral to the cancer specialist for further investigations to check if it has spread to other parts of your body. We need to do all the blood tests like FBE, UEC, BSL, LFTs, chest x-ray, and a CT scan of the chest. • Stage 2 cancer • The tumour will be between 2cm - 5cm in size • Surgery is usually a breast conservation surgery or partial mastectomy. We will check the lymph nodes at that time with a sentinel lymph node biopsy or an axillary lymph node dissection. After that, we will do whole breast irradiation. Chemotherapy is a must, in this case, given before or after surgery. If the tumour is big in size, the tumour can shrink if chemotherapy is given before surgery. Hormonal therapy can also be given depending on receptor status.

A total mastectomy can also be done will sentinel lymph node biopsy or axillary lymph node dissection. Chemotherapy will be given. Breast reconstruction will be done.

▪ Questions 205 o

What is chemotherapy? Page

© 2020 ARIMGSAS

• Chemotherapy is certain anti-cancer medications that can destroy cancer cells. By surgery, we are just removing the cancer cells in that particular area, but we do not know if the cancer cells have gone somewhere else or not. Chemotherapy lowers the risk of breast cancer recurrence. It is usually a combination of 2 or 3 drugs, can be given intravenous or injections through your vein every 3-4 weeks, and given for 4-6 cycles. o What is Tamoxifen? • It is a medication that blocks the action of estrogen. It lowers the risk of breast cancer coming back after surgery or developing cancer in the other breast. o Any side effects of this? • The side effects include a fluid build-up, thereby having weight gain and leg cramps which can be improved with walking. It can also cause period irregularities or even amenorrhea, dryness and itching of the vagina, thinning of hair, feeling sick, tiredness, and eye problems like cataract. • Tamoxifen during pregnancy is highly contraindicated, and it must be stopped 2 months before she plans for pregnancy. o Can I go for a pregnancy? • This is advisable to wait for at least 2 years after the treatment, which is when the risk of cancer is highest. During this time, you can use non-hormonal contraceptive methods (condoms, non-copper-containing IUCD) • The planned pregnancy should be discussed with the oncologist, and Tamoxifen needs to be stopped at least 2 months before • Women with a good prognosis (stage 1 cancer, small lump, low grade, ER/PR positive), need not wait for 2 years to become pregnant. It is the oncologist who is the best person who can advise her regarding the proper time to have a pregnancy o What is the chance for recurrence in the same breast after a lumpectomy? • The chance of recurrence in 10 years is 3 - 15%. But it also depends on other factors like hormone receptor status, lymph node involvement, and if the tumor margin has cancer cells or not.

206 Page

© 2020 ARIMGSAS

Cyclical and Non-cyclical Mastalgia

Cyclical Mastalgia ▪ Age group: 30 - 50 years old ▪ Always has a relation to the periods. Starts usually 3 -5 days before the periods, sometimes even 2 weeks before ▪ The breast will be more swollen and lumpier and will come down after period starts ▪ Severity can vary from month to month ▪ The pain usually affects both breasts ▪ Usually worse in the upper and outer part of the breast ▪ May radiate towards the inner part of the arm ▪ Breast tissue is most sensitive to the normal hormonal changes that occur each month

APPROACH ▪ History o When does the pain start? o Does it come down when the period starts? o Where is the site of pain? Is it on one breast or both breasts? o Does it go anywhere else? o Does it interfere with activities of daily living?

▪ Diagnosis and Management o Reassure the patient • Not due to any hormone disease or any other breast conditions • Not a symptom of cancer • Not a serious condition • It can settle by itself in around 3-6 months' time. But it can recur o Medications • Panadol or NSAIDS (Neurofen) on the days that she has the pain • Apply topical NSAIDs (Diclofenac gel) • Evening primrose oil is not much effective and has to be taken for at least 4 months to decide if it is helpful or not • Always make sure to review her meds ▪ If on COC or HRT, consider changing it • May cause cyclical mastalgia • If high dose COC, give low dose COC ▪ Antidepressants may cause cyclical mastalgia • Medications that block the hormones ▪ Danazol

▪ Tamoxifen ▪ Usually not considered for cyclical mastalgia due to unwanted side effects

UNLESS you have severe pain during most of the months and if other 207

treatments fail Page

© 2020 ARIMGSAS

o Referral to a specialist may be considered o General measures • Good breast support: well-fitting undergarments • Lifestyle changes can also help like a low-fat, increased fiber diet, decrease in coffee and alcohol, and regular exercises. ------Non-Cyclical Mastalgia ▪ Can present with breast pain all the time ▪ Comes and goes randomly ▪ Not related to periods ▪ It is usually one breast that is involved, or it can be localized to one area of the breast, rarely both ▪ Can be due to several other causes: o Pain can be due to the breast tissue itself, and the cause is unknown o Costochondritis o Mastitis o Shingles o Pregnancy o Fluid-filled cysts/breast cysts o Breast cancer, but an uncommon cause ▪ Management: Try to find out the cause, and address the cause if any. Give analgesics for the pain like Panadol, topic NSAIDs. Advise lifestyle changes, good breast support, evening primrose oil.

208 Page

© 2020 ARIMGSAS

Lymphedema

You are at your GP, when 58-year-old Lisa presents to you with a swelling of her right arm. She had breast cancer in her right breast diagnosed to be stage 2 cancer. Following which, she had a total mastectomy with axillary lymph node dissection, which was done 3 months ago.

TASKS 1. Take a further history 2. Discuss your management with the patient

Positive points in history: ▪ Swelling for the past one month ▪ Building up gradually ▪ No pain, tingling, numbness ▪ Feels it to be a bit tight to move the arm ▪ Finding it difficult to dress, cook, and garden because the arm feels quite heavy

APPROACH ▪ History o Swelling • Since how long have you had this swelling in your arm? • Is the onset gradual or sudden? • Is it progressing? • Is there pain in your arm? • Have you noticed any redness or skin sores? • Any tingling or numbness in your arm? • Are you having trouble bending your finger, wrist, elbow or shoulder? • Do you think that it is affecting your daily activities? • Is there swelling in any other part of your body? o Breast cancer • I'm sorry to hear that you had breast cancer. I read from the notes that you had surgery before for that. Currently, are you on any treatment for that? • Has radiotherapy been done on you? • Any immediate problems that you had after surgery? • Any other surgeries done? • Did you have a swelling before the surgery? • Any other health conditions that you are having like diabetes, high blood pressure,

heart or kidney disorders?

▪ Diagnosis and Management Most likely what you are having now is lymphedema of your right arm. In your body, we have got 209

a system called the lymphatic system, which has a network of tiny vessels and small bean-shaped Page

© 2020 ARIMGSAS

structures called lymph nodes. They carry lymph throughout the body. Lymph is a clear colorless fluid having a few blood cells. This system maintains fluid of our body by filtering and draining the waste products from each body region, and it also helps to fight infections. Often during a breast cancer treatment, some of the lymph nodes under the arm also need to be removed. These lymph nodes usually drain the lymphatic vessels from the upper arm, the underarm area, and the majority of the breast. Removal of these lymph nodes disrupts the flow of lymph which leads to swelling or lymphedema of the arm. This can happen any time after surgery or even the following radiation to the lymph nodes. This is a condition which cannot be cured but can be well-managed and thereby can improve your quality of life. I will refer you to a lymphedema clinic which has a doctor and a qualified lymphedema therapist. The measures that you need to do are: o Exercise of the arm, which can improve the lymph drainage. Specific exercises will be advised by the therapist. o Compression: Application of compression sleeves or elastic bandages, which prevent pooling or build-up of fluid, and also helps to move the fluid. o Elevation: keep the arm raised above the level of the heart whenever possible, which lets the gravity drain the fluid. o Manual lymphatic drainage or massage therapy: will be given by the lymphedema therapist. This helps to move the fluid out of the swollen area. I will advise you to adopt lifestyle modifications as well as a healthy diet, regular exercise, as controlling the bodyweight is really important. There are certain precautions that you need to take as the arm is at more risk of infection: o Avoid taking injections or blood tests and blood pressure measurements on the affected arm o Avoid carrying weight, avoid cuts and wounds o Practice good care of the skin and fingernails o Keep the skin clean and well-moisturized o Use gloves when you are doing housework or other activities like gardening o Make sure that you do not expose the arm to extreme hot and cold temperatures o Do not place heating pads or ice packs o Avoid getting sunburns on the arm as well Any time that you are having any redness, warmth, pain, if you start running a temperature, or you think that the swelling is getting worse, report immediately. I will give you reading materials regarding lymphedema and review you regularly.

210 Page

© 2020 ARIMGSAS

Speculum Exam

Prime the patient: ▪ I've been asked to do a speculum examination on you, is that okay with you? ▪ I need to first inspect your genitalia to look for any bleed, discharge, rash or vesicles ▪ After that, I'll be putting this instrument very gently ▪ You can have a look. It has two lips which can open like this. ▪ What I'll be doing is, I will be introducing this, holding the handle 45 degrees into your vagina ▪ Once it goes into the vagina, I will slowly rotate it so that it becomes a right angle to your vagina ▪ Then I will be gently opening this ▪ When the correct opening is obtained, I'll be pushing it like this ▪ With proper lighting, I will be looking at the neck of your womb or cervix ▪ I'll be looking for any changes in the cervix ▪ Once it is finished, I'll be pulling it out

▪ Go behind the curtain, and please expose yourself from the waist down ▪ Everything will be done in a gentle way, but anytime you feel mild discomfort, please tell me ▪ Please lie down on the bed and cover yourself with the towel ▪ Put your legs with the feet close to your bum, and your knees to the sides ▪ All this time, we have a female chaperone, but if you are not comfortable, tell me.

▪ Put on your gloves ▪ Separate the labia with your first and second finger in an inverted V ▪ Hold the speculum, put your thumb in between ▪ Slowly introduce the speculum in the vagina ▪ Then when you have the right size of the opening, push the lock with your thumb to lock it in place ▪ When you are finished, slowly pull it out

211 Page

© 2020 ARIMGSAS

Contraceptive advice

You are working in a general practice. A 24-year-old woman has come to see you for advice as to the most appropriate pill she should go on for contraception for the next two to three years. She knows that various types of pills are available and wants to know how to decide which is the most appropriate pill for her.

TASKS ▪ Take a further relevant and focused history. ▪ Ask the examiner about findings you wish to elicit on general and gynaecological examination. ▪ Advise the patient of the appropriateness of oral contraceptive pill (OCP) therapy, which pill should be given, and how it should be administered.

APPROACH ▪ History o Hi Jane, I'm Dr. ______your GP today. I understand from the notes that you are here today for advice regarding pill use, is that correct? Is it alright if I ask you a few questions so we'd know which pill would be most appropriate for you? o Exclude absolute contraindications to OCP use: • Deep vein thrombosis: (ADD COST VMPF) ▪ Do you take any over the counter or prescription medications? Have you taken contraceptive pills before? ▪ What is your occupation? ▪ Have you had any surgeries in the past 3 months, or any trauma? ▪ Do you smoke, drink alcohol or take recreational drugs? ▪ Any recent travel, especially greater than 12 hours? Did you walk around during the flight? ▪ Have you noticed any prominent veins in your legs? Any calf pain? Any previous DVT? ▪ Are you sexually active? Do you have a stable partner? Do you use contraception? Any previous pregnancy before? Any chance that you might be pregnant right now? ▪ Any family history of DVT or any coagulation disorders? ▪ Any recent fractures? • Breast cancer: ▪ Any weight loss, loss of appetite, lumps and bumps around the body? ▪ Have you noticed any lumps in your breast? Any breast tenderness? • Active liver disease or previous cholestatic jaundice: ▪ Have you noticed any yellowing of your skin? Any abnormal striae in your

abdomen? Any palmar erythema, finger clubbing? • Unexplained vaginal bleeding: do you have any bleeding from down below? • Focal migraines: any history of migraine?

o Exclude relative contraindications to OCP use: 212 Page

© 2020 ARIMGSAS

• Hypertension and diabetes: Do you experience frequent thirsts? Do you have to urinate more than usual? • Very irregular cycles or : ▪ Period history: When was your last menstrual period? Are your cycles regular? What is the average duration of your period? Is your bleeding mild, moderate and severe? Any pain or clots during periods? Any bleeding in between periods? When did you last do your pap smear and what was the result? • Cigarette smoking: Do you smoke, drink alcohol or use recreational drugs? o Any previous history of heart disorders, liver disorders, stroke, high blood pressure, diabetes, breast malignancy or severe depression?

▪ Physical Examination o General appearance: • What is the BMI of my patient? • Any pallor, icterus, cyanosis, lymph node enlargement, edema, poor skin turgor, dry mucous membranes, delayed capillary refill time? Any abnormal hair growth? o Vital signs: What is the BP, PR, RR, Temp and Sats of my patient? o Systemic examination: • CVS: How are the heart sounds? Is the rhythm regular? Any murmurs? • R/S: Is air entry equal? Any adventitious breath sounds? • CNS: How is the motor and sensory exam of the upper and lower limbs? (stroke symptoms) • Abdomen: Is there any visible distention or mass of the abdomen? Is there any hepatosplenomegaly, any mass or tenderness? • Musculoskeletal: Any edema or tenderness of the lower leg? • Breast: Any palpable breast lumps, tenderness or visible distortion or dimpling? • Pelvic exam: ▪ Inspection of the vulva and vagina: Are there any visible lesions in the vulva and vagina? Any discharge or bleeding? Any rash or vesicles? ▪ Speculum exam: is the cervix healthy? Is there any bleeding or discharge from the cervix? ▪ Per vaginal exam: CMT, uterine size and tenderness, adnexal mass and tenderness • Office tests: I'd like to do a urine dipstick test and a blood sugar level.

▪ Management Okay, it seems like it would be possible for you to start on combined OCPs. I would recommend you to take a low dose estrogen pill. This has a low breakthrough bleeding and low failure rate. And because your cycles are irregular, it would be better to choose a triphasic preparation as this has less post-pill amenorrhea or cessation of periods after taking the pill. Combined OCPs contains 2 hormones, estrogen and progesterone which is normally present in your body which regulates your periods. What it does is it inhibits ovulation, the release of an egg from the ovary. To a lesser extent, it increases the thickness of your cervical secretions so the sperm will find it

difficult to get through. And just in case fertilization happens, it changes the lining of your womb so that implantation does not happen, because only after implantation does a full bloom pregnancy happen. (TAKE PILL PACK) In a pill pack, there are 28 pills, 21 are hormonal pills, 7 are sugar-coated or 213

dummy pills. Starting taking the hormonal pill from the 1st day of your next period, 1 pill a day, at Page

© 2020 ARIMGSAS

the same time every day. Continue the hormonal pills for 21 days and then on starting the sugar pills, you get your periods. However, if you want to start taking the pill right away without waiting for your next period, you may, but use alternate methods like condoms for 7 days. Contraceptive efficacy is satisfactory after seven hormone tablets have been taken. While taking the pill, you may experience side effects such as nausea and vomiting, abdominal bloating and breast tenderness. Breakthrough bleeding or bleeding in between periods will usually settle in 3-4 months. Your breasts will just feel sore in the first 1-2 cycles. Major side effects such as DVT, stroke and MI could happen but are rare with low dose pills, such as what you will be taking. Advantages of the pill include periods become more regular, lighter and shorter. There is less dysmenorrhea. There is a decreased incidence of benign breast lumps and pelvic inflammatory diseases, decreased incidence of endometrial and ovarian cancer, and thyroid disorders. However, you must remember that OCPs do not protect against sexually transmitted infections, so you must use condoms along with it just in case you're concerned about STIs. If you experience diarrhea and vomiting within 2 hours of taking the pill, take a pill again and keep going with the rest. You need to use condoms as long as the diarrhea and vomiting lasts. If you go to a doctor or pharmacist, make sure you tell them that you're on pills as there are medications that decrease the efficacy of pills like vitamin C, some antibiotics or antifungals, and anti- epileptics. If you miss a pill for more than 24 hours, take the recently missed pill and just keep going with the rest even if it means taking 2 pills on the same day. If you keep going with the rest, and the dummy pill period falls within 7 days of missing the pill, skip taking the dummy pills and start the hormonal pills from the next pack. This will mean that you will miss having your periods. Alternate methods of contraception like condoms should be used for 7 days after missing the pill. I will need to follow-up with you in about three months after starting on the pill so I could check your blood pressure and to check whether the pills are working well for you or it needs to be changed because of any problems. I will give you some reading materials about combined OCPs for further insight.

KEY ISSUES ▪ Ability to take an adequate history to exclude absolute contraindications to the OCP and facts that would influence the pill chosen, and its dose. ▪ Ability to advise a patient as to how to take the pill, the timing of its effectiveness and the likely problems during its use.

CRITICAL ERRORS ▪ Failure to exclude absolute contraindications to OCP use.

214 Page

© 2020 ARIMGSAS

Rape of a 20-year old Woman

You are a Hospital Medical Officer (HMO) in the Emergency Department of a metropolitan general hospital. Your patient is a 20-year old university student who is brought to the Emergency Department of the hospital because she was raped by a man that she met at a disco and who offered her a lift home. The rape occurred six hours ago after he had stopped the car in an undeveloped area. She has decided not to involve the police as the person concerned is known to her family. She has had no previous operations or illness and no pregnancies

TASKS 1. Take any further relevant history you require 2. Ask the examiner about the appropriate findings likely to be evident on initial general and gynecologic examination 3. Advise the patient of the investigations required and the management you would propose.

APPROACH ▪ I am very sorry to hear that you have been raped. Let me assure you that there is a lot of support for you and you are not alone in this moment of crisis. Do you want me to call someone to be with your right now before we proceed with this consultation? ▪ I want to help you with your condition. However, I need to ask you some questions revolving what happened to you and some personal questions to assess for possible risks. I assure you everything will be kept confidential between us unless it poses harm to you or to others. Is that alright? ▪ HISTORY o Were you hurt anywhere else? o PERIOD HISTORY • When was your last period? • Do you get it every month? How many days do you usually bleed? o PARTNER • Are you sexually active? In a stable relationship? Any history of STI? • I'm sorry to ask about this, but do you know if the man suffered from any STIs? ▪ Did you see any discharge or lesions over his private part? ▪ Did he use a condom during the incident? o PILL • Are you on any forms of contraception? When were you last on the pill? o PREGNANT • Have you ever been pregnant before? o PAP • When did you have your last pap smear? o Do you have any history of bleeding disorders, hypertension, migraine, vaginal bleeding, or

breast cancer? Do you have any allergies? Do you take any medications? o Does anyone else know about this? Are you planning to take legal action about this? • I respect your decision but I would like to get samples from you during my

examination later and keep these in the hospital just in case you will change your 215 Page

© 2020 ARIMGSAS

mind later. We will have a strict chain of security and continuity in handling all of these specimens if the results of these are to be admissible in court o I know it can be very distressing for you to be checked on your private parts. However, I need to perform the examination on you to see for further evidence of injuries. With your consent, I would perform a full examination of your private parts, involving me to take a look outside, look using a speculum, and get swabs from your private parts to get possible DNA samples, and to check for signs of possible sexually transmitted infections, and also feeling for any soreness or masses in your private parts. Will that be okay? ▪ PE o GA o VS o Check for any other signs of physical abuse in other parts of the body: bruising, rashes, lacerations o PELVIC examination WITH CONSENT and with a chaperone (in the case, the patient initially refused to do this! That's why I included a statement prior to the PE to explain about why a pelvic & speculum exam is necessary) • Inspection: lacerations, bleeding, bruises, discharge • Speculum: cervix bleeding or not? Any lacerations? Discharge? Bleeding from anywhere else? • Per vaginal: CMT, uterine size, mobility, masses, tenderness? Adnexal tenderness? o CVS, RESPI, CNS, ABDOMEN ▪ PE FROM BOOK o GA: no evidence of bruising or trauma o Vulva: looks normal--not bruised and not bleeding o Speculum exam and per vaginal has not yet been done. The candidate should now discuss these with the patient and advise on the management plan.

▪ INVESTIGATIONS AND MANAGEMENT o I can imagine how this can be a very difficult situation for you, again we are here to give whatever support you'll need. As for now, I can suggest a management plan for you, which includes a medical and psychological plan to support you during these times. o Due to the nature of what happened to you, I'm really concerned about the possible sexually transmitted infections which you could have acquired from your experience. Because of this, and with your consent, I would like to take blood samples from you to test for HIV, syphilis, hepatitis B&C, HSV, and take urine samples to test for chlamydia for PCR. I will also give you an antibiotic (Azithromycin 1g SD) now to cover for possible STIs. o I will refer you to our rape crisis team or our medical social worker to give you more support. I will also refer you to a psychologist where you can undergo a talk therapy to help you cope up with this situation. o As you are already in the late phase of your cycle, I'm also concerned for a possible pregnancy that might occur because of what happened to you. Because of this, I would like to give you medications for emergency contraception. These medications contain the hormones that we normally have in our body, but just in doses enough to prevent a

pregnancy. We have two options to do it. First, I can give you the Postinor pill (Levonorgestrel), you can take 2 pills (0.75mg) 12 hours apart, or 1 pill (1.5mg) given up to 5 days from today, but this is best taken within 72 hours since the incident. This has 85%

efficacy if taken properly. 216 Page

© 2020 ARIMGSAS

o Another option is the Yuzpe method (50mcg estrogen + 250mcg progesterone), where you will take 2 tablets now and then 2 tablets after 12 hours. This has an 75% efficacy if taken properly. But then again, there are still chances of becoming pregnant despite these interventions. o Because of that, I will arrange a review with you in 3 weeks to check whether you have conceived and also to review tests results and decide if these need to be repeated. The screening tests that we'll do for syphilis and HIV will need to be repeated in 3 months' time to check if you have been infected or not. o "will I still become pregnant?" • Yes, you can still become pregnant as what happened to you will not affect a subsequent pregnancy if ever you decide to become pregnant o Again, we are here for you and we will give you all the support you need to recover from this crisis in your life.

▪ SIDE NOTES o If you'll choose to refer the patient to the rape crisis centre immediately, a summary of what needs to be done with her (as what is indicated above) should be given to her prior to the referral.

KEY ISSUES ▪ Ability to assess a patient who has recently been raped ▪ Ability to arrange the appropriate follow-up, investigations and care

CRITICAL ERRORS ▪ Failure to consider the need for post-rape contraceptive methods and management ▪ Failure to refer to the appropriate clinic or to discuss taking appropriate swab to exclude sexually transmissible infections (STI) and taking specimen for DNA analysis ▪ Failure to consider the use of prophylactic antibiotics to prevent pelvic inflammation with an STI

217 Page

© 2020 ARIMGSAS

Pre-pregnancy advise for DVT

You are working in a general practice. Your next patient is a 28-year-old woman, who had one pregnancy 18 months ago, which was complicated by deep vein thrombosis and a postpartum pulmonary embolus. She has come to see you for pre-pregnancy counselling as she wishes to conceive again. At the time of a previous assessment twelve months ago, she had ceased warfarin. When assessed six months ago, there were no sequelae or symptoms and she had no signs of chronic venous insufficiency in the legs. There are no abnormalities on physical examination and she is not overweight.

TASKS ▪ Take any further relevant history you require from the patient. ▪ Advise the patient on the management she will require before and during the next pregnancy.

APPROACH ▪ History o Hi Jane, I'm Dr. _____, your GP today. I read from the notes that you are planning another pregnancy. That's wonderful. However, I understand that you have some questions about it as you had some problems in your previous pregnancy, is that right? o Is it alright if I just ask you a few questions about your condition so we can appropriately plan for your next pregnancy? o Questions about previous pregnancy: • Can you tell me more about what happened in your previous pregnancy? • What was the mode of delivery? Was your baby born at term? Any problems during delivery? Was your baby alright after birth? • Did you take folic acid during your last pregnancy? • Did you do a rubella serology during your last pregnancy? What was the result? o Questions about the previous DVT: • When did the DVT and the pulmonary embolus happen? How was it treated? Are you still on any medications for that right now? Have you had any clotting episodes other than that? o Risk factors for DVT: (ADD COST VMPF) • Do you take any over the counter or prescription medications? • What is your occupation? • Do you take any contraceptive pills? • Have you had any surgeries in the past 3 months or any trauma especially on the legs or pelvis? • Do you smoke, drink alcohol or take recreational drugs? • Any recent travel, especially greater than 12 hours? Did you walk around during the flight?

• Have you noticed any prominent veins in your legs? Any calf pain? Any edema? • Have you noticed any lumps and bumps around your body? Have you experienced any weight loss or loss of appetite? • Are you sexually active? Do you have a stable partner? Are you on any contraception? 218

Any chance that you might be pregnant right now? Page

© 2020 ARIMGSAS

• Any family history of DVT or any coagulation disorders? • Any recent fractures? o Do you experience any shortness of breath?

▪ Management Look Jane, since you had a previous history of clotting during your first pregnancy, there is an increased risk of about 20 % of developing another one in your next pregnancy. During pregnancy itself, there is increased tendency for clotting because of the hormonal and physiological changes happening in the body. If you would decide to carry on with a pregnancy, you would be referred to the high-risk pregnancy clinic and be managed in consultation with a haematologist and an obstetrician. But before you become pregnant, we would need to screen for your clotting propensity by doing some investigations such as anticardiolipin antibody, lupus anticoagulant, protein S, protein C, anti- thrombin 2, and factor V Leiden, all of which are markers to screen for both inherited and acquired clotting diseases. At about 14 weeks of your pregnancy, we will start you on a low molecular weight heparin such as enoxaparin, which is a medication to prevent your blood from clotting. You will take these 12 hourly as subcutaneous injections throughout your whole pregnancy and about 4-6 weeks after you deliver. It is also advised that you use compression stockings by day throughout the pregnancy, and avoid prolonged immobilization. You should also start taking folic acid 0.5 mg orally, once a day, 3 months before pregnancy and up to the first 12 weeks of pregnancy. Labor will be induced and delivery will be done in a controlled manner at 38-39 weeks of gestation. During the planned date, we will withhold your morning dose of heparin. After delivery, warfarin would be given for 6 weeks and we will monitor INR every day and dose will be adjusted accordingly. I will give you reading materials about DVT for further insight.

KEY ISSUES ▪ Ability to recognise that she is at increased risk of recurrent thrombosis in her next pregnancy and requires at least low dose heparin during the puerperium if not for most of the antenatal period as well. ▪ Recognition of relative risks and indications for heparin and warfarin therapy. ▪ Recognition of significant risks of warfarin during pregnancy.

CRITICAL ERROR ▪ Failure to screen for an inherited or acquired coagulation disorder. ▪ Failure to advise anticoagulant therapy at least for 4-6 weeks postpartum in the next pregnancy. ▪ Advising warfarin therapy throughout pregnancy.

219 Page

© 2020 ARIMGSAS

Pre-pregnancy advise to a 24-year-old woman with Type 1 Diabetes Mellitus

You are working in the primary care facility of a teaching hospital. Your patient is a woman aged 24 years (para 0, gravida 0), a known diabetic for 15 years and well controlled on insulin. She has come to see you for counselling and advice about possible future pregnancies.

TASKS 1. Take any further relevant history you require 2. Advise the patient of the information she needs to be given for pre-pregnancy counselling

APPROACH ▪ I understand that you are here to seek advice regarding future pregnancies. Are you planning to be pregnant anytime soon? Let me ask you some questions to identify some factors in your life ▪ And I can see from your notes that you have been diabetic for 15 years, could you tell me more about it? o ASSESS DM SEVERITY & CONTROL (must be symptom-free, with good control for at least 3 months) • When was your last check-up for your diabetes? Do you regularly have check-ups? • How is your blood sugar control? DO you regularly monitor your sugar levels? What are your recent blood test results? • Have you ever had any hypoglycemics episodes or low blood sugar level episodes? ▪ If yes--were you admitted in the hospital? Any complications (hypoglycemics coma) • Do you feel thirsty all the time, go to the toilet to urinate more often? • Any concerns about your waterworks? Have you seen a kidney specialist? • Any blurring of vision? Have you been referred to an eye specialist? • Any infections from down below? o 5P's: I will be asking you some sensitive and personal questions to identify some factors which can affect your pregnancy, and rest assured everything will be kept confidential. Will that be ok? • Is this your first time to go for a prenatal check-up? • Any concerns with your periods? Do you get it every month? How many days do you usually bleed? How heavy is your bleeding? Any pain during your periods? • Are you in a stable relationship? Any history of STIs? Do you practice safe sex? • Are you on the pill or other forms of contraception? ▪ When were you last on the pill? • Have you ever been pregnant before? ▪ If yes, ask for details (when, how many times, complications, etc) • When was your last pap smear? ▪ PATIENT ADVISE

o It's good that you are planning to become pregnant and that you came here for advice 220

regarding your pregnancy. However, you should be aware that having diabetes in pregnancy Page

© 2020 ARIMGSAS

proves to have a high risk for complications both for you and for your baby. The state of being pregnant itself has anti-insulin effects which tend to further worsen your diabetic state. But don't worry too much, as you did the right thing to have a consult here first before becoming pregnant. We can do our best to minimize the possibilities of all of these complications from happening as long as proper blood sugar control prior to and during pregnancy is achieved. o From your history, it can be seen that you seem to have a fair control over your blood sugar levels. An initial test that I can do for you to confirm this is the HBA1c blood test together with your regular blood sugar level checks in your diary to give an assessment of your blood sugar control. We need to keep your sugar levels within 5-7mmol/L to reduce the risk of complications for you and your baby o But to give you further assessment, I will refer you to a diabetic specialist who will check your general state and check for complications of diabetes. He might do investigations such as renal function tests, 24-hour urinary protein checks. I will also refer you to an eye specialist to assess for your vision, as diabetes can often affect your vision as part of the complications of the disease. So, with this, it is very important that you do not get pregnant until you have reached optimal control of your blood sugar levels. o Providing all of your tests are normal, then we can now plan for your pregnancy. I will then start you with folic acid which you will take from the time pregnancy is attempted until at least 12 weeks of your pregnancy. o We can already do your basic tests for pregnancy now as part of your general assessment. We can do blood group testing, indirect Coombs test, full blood examination, urine microscopy and culture, and also STI screening tests which include hepatitis screening, VDRL, rubella serology and HIV testing with your consent. We can also arrange for your pap smear now as your last pap was done 3 years ago. o Once you become pregnant, you will be referred to the high-risk pregnancy clinic where you will be managed by a multidisciplinary team consisting of a diabetic specialist and an obstetrician. • Your insulin requirement will increase to keep your sugar levels controlled and keep the fetal malformation rate to a minimum, and to keep the size of your baby (avoid macrosomia) to an acceptable level. • Your iron and folic acid tablets will be continued during your pregnancy • You will have a planned delivery at around 38-40 weeks, either by induction or by C- section. Earlier delivery might be necessary if problems might occur during your pregnancy o Despite all of the adequate care during pregnancy, you must still be aware of the complications related to diabetes in pregnancy. Sorry for the medical terms I have to say to you, but you need to be aware of these. Your pregnancy can be complicated by preeclampsia or a sharp rise in BP during pregnancy, polyhydramnios or excessive water inside your womb which can predispose to early labor or early rupture of these bag of water, and a big baby necessitating early labour or C-sections. There is also an increased risk of unexplained fetal death in late pregnancy and possible breathing difficulties in the baby after delivery.

o I can imagine that this must be very distressing for you, but I am here to give you all the information you need before you proceed with your pregnancy. I am here to support whatever decision you make, and if you do decide to proceed with pregnancy, we will look after you and do the best we can to minimize these complications. 221 o

Here are reading materials for you to give you more insight about your condition. Page

© 2020 ARIMGSAS

o I will arrange a review again with you once I receive your results.

KEY ISSUES ▪ The ability of the candidate to counsel a diabetic woman, prior to pregnancy, about the care she will require and the methods of keeping the complication rate to herself and her fetus to a minimum

CRITICAL ERRORS ▪ Failure to ensure the patient is aware that her diabetic control prior to pregnancy should be good, to ensure the risk of fetal abnormality is kept as low as possible ▪ Failure to do pre-pregnancy blood tests (hemoglobin estimation, blood group, rubella antibodies and tests as above) and failure to recommend pre-pregnancy and early pregnancy folic acid therapy (routine pre-pregnancy counselling)

222 Page

© 2020 ARIMGSAS

Anencephaly

You are an HMO working in a primary care clinic attached to a teaching hospital. Your next patient is a 25-year-old primigravida who has just had an ultrasound performed at 18 weeks of gestation, which has revealed an anencephalic fetus. A maternal serum screening (MSS) was done at 16 weeks and this had shown elevated levels of alpha fetoprotein.

TASKS 1. Take any further relevant history 2. Advise the patient, in lay terms, of the relevance of the diagnosis and the subsequent management you would propose in this pregnancy 3. Advise the patient of the care you would recommend in a subsequent pregnancy

You will not be expected to request examination findings, nor to arrange any further investigations.

APPROACH ▪ History o I understand that you are here because of the results of the ultrasound done to you and the results of your blood tests. Do not worry, I will try to help you understand what your results mean. But before that, I would like to ask you a few questions, is that alright with you? o Obstetric history • How is your pregnancy so far? • Any issues in your pregnancy? Any tummy pain? Any bleeding or discharge from down below? • Did you take your folic acid for this pregnancy? • Were you able to do your blood tests, blood group and indirect Coomb's test? Do you remember the results? o Were you diagnosed with any medical and surgical illness especially asthma? o Do you smoke, drink alcohol or take any recreational drugs? o Are you taking any medications? Do you have any allergies? o Do you have a family history of neural tube defects or brain and spinal disorders in pregnancy?

▪ Diagnosis and Management o Jane, before I discuss with you the results of your ultrasound and serum screening test, do you want me to call someone to be with you? I am afraid that I have not so good news for you. o The results of your tests show that your baby has a condition we call anencephaly. I'm sorry for using medical terms. Do you know anything about this?

In the womb during the development of the baby, a structure called the neural tube goes on to develop into the baby's brain and spinal cord. When the neural tube does not fuse together properly, a problem with the development of the spine or brain occurs. Fetal anencephaly is a developmental defect of the brain which occurs somewhere between 223

five and eight weeks of gestation. Page

© 2020 ARIMGSAS

When the neural tube does not close at the head, a condition called anencephaly results, such as the case in your baby. This is when the top part of the brain, skull and scalp are partially or missing. I am very sorry to say, that in this condition, the baby usually dies at, or soon after birth. o At this point, I would like to inquire on your preference regarding your pregnancy. Do you wish to terminate the pregnancy forthwith, or continue with the pregnancy until labor occurs? o Because you wish to terminate your pregnancy now, I would like to discuss your options for you. Termination of the pregnancy could be performed using medications called prostaglandins or by the surgical procedure of dilatation and evacuation. Dilatation and evacuation have the advantage of being performed under general anaesthesia or you will be asleep, with the procedure being over when you wake up. However, it also carries the risk of weakening of your cervix (cervical incompetence) in your subsequent pregnancies, when the procedure is done after 16 weeks of pregnancy. Prostaglandin termination, on the other hand, may take several hours, or even days, and results in uterine contractions similar to those experienced in labor, followed by vaginal delivery of the fetus. But there will also be a possible need for curettage to remove any retained placental fragments. After termination of the pregnancy, the fetus will be examined to see if there is any other abnormality present. o The risk of recurrence of a neural tube defect such as anencephaly in the subsequent pregnancy is somewhere between 2% to 5%. But to reduce this risk, you must take a higher dose of folic acid, 5mg per day, a month before you are planning to get pregnant and continue until about 12 weeks of your pregnancy. Also, a serum alpha-fetoprotein assessment could be done at 16 weeks of gestation along with an ultrasound examination to detect such problems in your subsequent pregnancy.

KEY ISSUES Ability to advise the mother: ▪ empathically of the fact that her baby has a lethal abnormality ▪ of the appropriate options regarding her further care in this pregnancy ▪ of the methods available to reduce the recurrence risk of this abnormality

CRITICAL ERRORS Failure to: ▪ recognise and advise the patient that this is a lethal abnormality to the baby ▪ determine the preferences of the mother in respect to termination of pregnancy ▪ counsel the patient appropriately concerning management in a subsequent pregnancy

224 Page

© 2020 ARIMGSAS

Primary amenorrhoea

You are working in a general practice. Your next patient is an 18-year-old woman who is concerned because she has never had a menstrual period.

TASKS 1. Further relevant history 2. PE findings from examiner 3. Investigations and provisional diagnosis 4. Counsel the patient

APPROACH ▪ I understand that you are a bit concerned now because you haven't had your menstrual period. Let me just ask you a few questions for me to find out what is causing the problem. Is that alright with you? ▪ Assess patient's pubertal status: o Have you had a growth spurt already? o Have you noticed changes in your breast already? o Have you noticed any hair growth in your armpit and private areas? o How do you compare in height with your peers in school? o Have you had any history of tummy pain or pain down below? ▪ Differential diagnoses questions o Constitutional delay in puberty: Any family history of early or delayed menarche? Any of your family members experience the same problem? • When did your mom have her first menstrual period? • Do you have a sister? How old is she? When did she have her first menstrual period? Did she already have changes in her breast development? o Pregnancy: Are you sexually active? Do you have a stable partner? Do you use any form of contraception? o Do you smoke, drink alcohol, or use recreational drugs? o Any history of medical and surgical illnesses? Any history of irradiation to the pelvic area? o Thyroid disease: any weather preferences? Have you experienced any sweating, feel your heart racing, diarrhea or constipation, tremors? o CNS tumour: any significant headaches or vision changes? o PCOS/CAH/Cushing: have you noticed any excess hair growth anywhere in your body? Any problems with acne? ▪ Physical Exam o General appearance • What is the BMI of the patient? o Vital signs o Systemic exam • Assess for tanner stage of breasts, axillary and pubic hair • Valval inspection - is the hymen intact? Is it imperforate? 225

• Vaginal inspection - any lesions, deformities? Page

© 2020 ARIMGSAS

• DO NOT DO A PER VAGINAL EXAM. PATIENT IS VIRGINAL. ▪ Investigations o Ultrasound examination (abdominopelvic ultrasound NOT VAGINAL because patient is virginal) to check the development of uterus and vagina o Hormone tests: FHS, LH, prolactin, oestradiol levels o Chromosome analysis IF uterus is NOT normal and hormone levels are deranged to check for Turner syndrome ▪ Management From history and examination, most likely the reason for the absence of your menses is just a physiological delay or a normal delay in the start of your menses. I can say this because I have not found anything abnormal in your examination. However, I would arrange for some investigations to rule out any other causes of the absence of your menses. Usually, the first period occurs two years after the first breast development but can be delayed for 3 years or longer as normal variation in menarche. I will review you again once the results of the investigations come in, and again in 12 months’ time if your period has still not occurred by that time, and we will re-evaluate and repeat the measurement of your female hormone levels. Do you have any questions at this point?

KEY ISSUES ▪ Ability to define the most likely cause of primary amenorrhoea (delayed menarche). ▪ Ability to arrange the appropriate investigations.

CRITICAL ERROR ▪ Inadequate history to evaluate current pubertal status. ▪ Performance of pelvic vaginal examination as she is virginal. Requesting that pelvic examination should be done would be a significant and potentially failing error. ▪ Failure to order abdominal ultrasound. Pelvic (vaginal) ultrasound is also inappropriate ▪ Failure to order hormonal analyses of FSH, prolactin and oestradiol

226 Page

© 2020 ARIMGSAS

Investigation for male factor infertility

A married couple (husband 25, wife 23 years) have been trying to conceive for the last 12 months. Examination of both the husband and the wife is normal. Investigations arranged by you, from a general practice setting, have shown she is ovulating each month, and has patent Fallopian tubes.

The husband's recent semen analysis is not normal. His result is as follows

SEMEN ANALYSIS Collected after three days of abstinence Examined 30 minutes after collection by masturbation, normal values in brackets

VOLUME 6ml (2-6ml)

Count 2 million/ml (greater than 20 million/ml)

Motility 20% (greater than 40%)

Velocity 20 microns/second (Greater than 30 microns/second)

Abnormal morphology 95% (Less than 80%)

Antisperm antibodies Nil (Nil)

The husband has come to see you today for the result of the semen specimen. His wife is aware of her results. She was unable to come today.

When you examined him previously, you found no abnormality on general or genital examination. Both testes were normal in size (20ml estimated value), felt normal in consistency, there was no indication of a varicocele or hydrocele.

TASKS 1. Take any further relevant focused history from the husband in regard to the results obtained 2. Advise the husband regarding the couple's fertility problem.

227

Page

© 2020 ARIMGSAS

IDIOPATHIC/UNEXPLAINED: 40-50% of cases Testicular: Varicocele: 35-40% of infertile males Meds: cytotoxic agents, GnRH agonists, anabolic steroids, nitrofurantoin, sulfasalazine, spironolactone, alpha-blockers Drugs; Alcohol, tobacco, cocaine, marijuana Exposure: radiation, heavy metals

APPROACH ▪ I understand that you are here to discuss your test results with me. But before we go through that, I need to ask you some more questions to assess for possible factors that might affect your fertility. Will that be okay? ▪ I might also ask you some personal and sensitive questions in this consultation, but rest assured everything will be kept between us unless it poses harm to you or others. Is that okay? ▪ HISTORY o SEXUAL HISTORY • How long have you tried for a baby? • I'm sorry to ask you this but I need to know, have you or your wife have any kids from previous relationships? • How often do you engage in sexual intercourse? Do you live with your wife? • Do you or your wife experience any difficulty during intercourse? • Does she have any pain on penetration? • DO you have any difficulties in maintaining an erection? DO you think you can ejaculate completely? • Do you or your wife have a history of STIs? o SAD • Do you smoke, drink alcohol, or engage in recreational drugs like marijuana or hashish? o Do you take any medications? Steroid use? o WORK AND EXPOSURE • What is your occupation? Have you been ever exposed to certain chemicals? • Any stresses or concerns at work? o INJURIES/CONDITION TO THE TESTES • Do you have any history of hernias, undescended testes, or surgeries to your lower tummy and genital area? Have you ever had any injuries to your genital area? • Did you have any other medical illnesses, suffered from frequent URTIs (Kartagener's syndrome and Cystic fibrosis), especially mumps? o PSYCHOGENIC • How is your mood lately? ▪ ADVICE o I have the results of your semen analysis that was done for you. This contains information about the number, shape, and function of your sperm which may affect your fertility. In your case, the sperm count and motility are low, and the number of abnormal shapes is

high. Please don't worry too much about this. As sometimes, certain conditions can affect the functioning of your testis like stress, febrile illnesses, certain medications. Usually, these

are temporary and on repeating the tests, the sperm count usually reaches the normal level. 228 Because of this, we need to repeat another sperm analysis test in about three months' time.

If an abnormal test again is found, we will repeat the test again for the third time in another Page

© 2020 ARIMGSAS

3 months. I will give you written instructions regarding how to obtain a sample. You will need to follow them carefully as they can affect the outcome of the test. o And if these show the same findings as to the first and a second one, then clearly there is a problem which is almost certainly a major factor that contributes to your infertility. Even if you see that there are still sperm present, it still does not reach the optimum number needed and activity that sperm must have in order to successfully fertilize the egg cell. o However, if the semen analysis results improve spontaneously with time, the possibility of achieving a pregnancy is increased. o We will also perform other investigations to identify possible causes of your low sperm count. This will include blood tests which include serum FSH, LH, testosterone, and prolactin levels. We will also do an ultrasound of both your testes to check for its size and to rule out any masses or conditions that may obstruct the flow of sperm. o But in most cases, unfortunately, the exact cause of an abnormal sperm analysis is still unknown. And in terms of treatments or medications, there is no documented evidence for use of any treatment in improving the semen specimen. o Now, in the case we'll not have good results with these investigations, it seems that there is a definite place for the use of in-vitro fertilization (IVF) with intracytoplasmic sperm injection, or what we call an IVF with ICSI. Sorry for the medical term but let me explain this procedure to you. This involves getting a sperm sample from you and getting the egg cell of your wife and injecting the sperm to the egg cell in the laboratory for fertilization to take place. This procedure is done by a fertility specialist and has a pregnancy rate of about 20- 40% per cycle. Just doing IVF without ICSI has poor results compared if it is done with ICSI. Are you open to considering this procedure? o Another consideration is the use of donor sperm and performing artificial insemination, again sorry for the use of the medical terms. The pregnancy rate is about 20% per cycle of insemination, and this use is cheaper and more straightforward than other methods of treatment. However, the baby would not contain any of your genetic material if we do this method. o I'm afraid that if we use your sperm for intrauterine insemination it would have a very poor success rate (1-2%) as your sperm o Again, please don't stress yourself too much about this, as we will still do a repeat sperm analysis test and we have pending results for the investigations that we plan to do to determine the causes of your low sperm count. I will arrange another review with you once results become available and it is best if you bring your wife along. For now, I would like to give you reading materials to give you more insight into your condition and about IVF with ICSI. Rest assured, I will do my best to look after you and support you for whatever decisions you make with regard to your plans of having a child.

KEY ISSUES ▪ Need for appropriate history from husband ▪ Knowledge of appropriate tests to assess him, and of the possibility of improvement with time ▪ Need for empathetic counselling

▪ Ability to understand that a definitive cause is unlikely to be found

229 Page

© 2020 ARIMGSAS

CRITICAL ERRORS ▪ Failure to advise that at least a second semen specimen (3 months after the first) must be examined ▪ Failure to recognise that persisting severe abnormality of the semen specimen as currently obtained will result in a very low pregnancy rate ▪ Failure to understand that ICSI (within IVF) is the best method of achieving pregnancy using his genetic material

230 Page

© 2020 ARIMGSAS

Anemia in a 28 year old pregnant woman

This 28-year-old pregnant woman, who is attending a general practice in which you work, has just been found to have a hemoglobin level of 80 g/L when tested at 26 weeks of gestation.

TASKS 1. Take any further relevant history you require 2. Ask the examiner about relevant findings likely to be evident on general and obstetric examination 3. Advise the patient of the tests required to define the most likely diagnosis and the subsequent management you would advise

APPROACH ▪ I have read from your notes that you have a hemoglobin level of 80g/L. But if it's okay, I would like to ask you some questions first to identify some risks which may have caused this. Is that alright with you? ▪ QUICK HISTORY o SEVERITY of ANEMIA: Do you feel tired more than the usual? Any dizziness, racing of the heart, or shortness of breath or chest pain? Are you on any iron supplements? o DIET: Can you tell me about your diet? What do you usually eat? Are you on a special diet (Celiac disease can predispose to IDA) o PREGNANCY: How is your pregnancy so far? Have you had regular antenatal check-ups? How are your blood tests and ultrasound? Have undergone down screening test? Do you have any bleeding episodes? Is the baby kicking well? Are you taking folic acids or iron supplements during this pregnancy? • When was your last pregnancy? How many pregnancies have you had? Have you had any complications during your pregnancy? Bleeding episodes before, during, and after your delivery? Are you aware if you have low hemoglobin during your previous pregnancy? Did you take any medications (iron supplements) during your previous pregnancy? o PERIODS • How are your periods before this pregnancy? Are they heavy? o R/O Other causes of anemia • Do you have bleeding or bruising from other parts of your body? • Have you travelled recently? (R/O malaria) • Any lumps or bumps anywhere in your body? Weight loss? o Are you aware of your blood group? o Any history of medical or surgical conditions? o Do you have any family history of blood or bleeding disorders? Do you have any Mediterranean heritage in your family? o Do you have support for this pregnancy? ▪ PHYSICAL EXAMINATION o GA: pallor? Lymphadenopathy? Dehydration? o VS: BP with the postural drop? O2 sats? HR with rhythm? 231

o CVS, RESPI, CNS Page

© 2020 ARIMGSAS

o Focused abdominal exam: • Fundic height? • Fetal heart sounds? • Fetal lie • Fetal presentation • Uterine tenderness? o PELVIC: • Inspection: any bleeding or discharge from the vulva or vagina? Any rashes or vesicles present? • Speculum: is the cervical os open/closed? Any discharge or bleeding coming from the cervical os and the vagina? • Per vaginal: ▪ What is the size and position of the uterus? Is it mobile? Any palpable masses or tenderness? ▪ Any tenderness and palpable masses over the adnexa? ▪ OFFICE TESTS: UDS, BSL ▪ PHYSICAL EXAM: Apart from looking pale, the general examination is normal. The uterus is enlarged to about 4cm above the umbilicus, and measures 26cm above the pubic symphysis. ▪ DIAGNOSIS AND MANAGEMENT o As you can see from your blood test results, your hemoglobin levels in your blood are decreased, and we define this condition as anemia. Hemoglobin is the pigment in your red blood cells which is responsible in distributing oxygen to the rest of your body to optimize bodily functions. Because of this decrease, your body tends to lack oxygen needed to fully function that's why you're feeling tired, dizzy, and look pale. And based on your history and examination, it seems that the most likely cause of this anemia is an iron deficiency anemia. It is a common condition in pregnancy. It is often asymptomatic and only detected on screening tests o Iron is a substance that is needed in the development of red blood cells, which carry hemoglobin and oxygen in the body. There is a higher demand for iron in pregnancy, and most of the time we get the iron through our diet. And from your history it seems that you may have not been taking iron-rich foods nor supplements, which may have caused the gap between your pregnancy's demands with your iron supply, thereby causing you to have this condition. o There are some risks because of this anemia, as this can predispose you to develop infections, to have excessive blood loss during pregnancy, and can also affect your heart. Your baby might have reduced oxygen supply which can lead to growth restriction inside the womb, distress, and in severe cases, stillbirth (death). o But don't stress yourself too much about this. It is good that we caught it early on so that we can avoid these complications so that you'll have a normal course of pregnancy. o Again, iron deficiency anemia is a probable diagnosis, and in order to confirm it, we need to do certain investigations, which include FBE, iron studies (serum iron and ferritin). And if these show abnormal results which lead to findings suggestive of other causes of anemia, I will refer you to a hematologist who might suggest further investigations to determine the

cause of your anemia (hemoglobin electrophoresis to r/o thalassemia) o I will now start you with iron tablets (ferro-gradumet/Fefol), one tablet twice a day. Possible side effects of this drug include constipation, tummy pain, nausea, and dark stools. There is no need for a blood transfusion (or IV iron therapy) at this time as you don't have severe 232

symptoms or severely decreased hemoglobin levels. Page

© 2020 ARIMGSAS

o I also advise you to eat more iron-rich foods such as iron-fortified cereals, legumes, nuts, and nut butter, seeds, whole-grain bread, green leafy vegetables. Also, eat a lot of vitamin C rich foods like citrus fruits to increase the absorption of iron. o If ever you'll have bleeding, dizziness, weakness, chest pain, or difficulty of breathing, please report to the hospital immediately. o I will arrange a review with you after two weeks to check your hemoglobin levels again along with a reticulocyte count. If it does not improve, I will refer you to a specialist to advise concerning your diagnosis and appropriate treatment. o Providing that your anemia can be treated satisfactorily, there should be little effect on your pregnancy and your baby. o I can give reading materials for you to give you more insight to your problem.

Side notes: ▪ Parenteral iron indications: if close to delivery and if cannot tolerate oral iron and Hgb <7g/L ▪ Anemia: Hgb <110g/L in 1st trimester and <100g/L in late second or third trimester. Iron requirements increased to 1300mg/day.

KEY ISSUES ▪ Ability to evaluate appropriately a patient who has become anemic during pregnancy ▪ Ability to commence treatment and arrange appropriate follow-up in such a patient

CRITICAL ERRORS ▪ Failure to make a provisional diagnosis of probable iron-deficiency anemia due to the demands of successive pregnancies ▪ Failure to administer oral iron therapy ▪ Recommending blood transfusion at this time

233

Page

© 2020 ARIMGSAS

234 Page

© 2020 ARIMGSAS

Breech in labour

You are working in the Emergency Department of a general hospital. This patient is a 25-year-old woman in her second pregnancy, at 38 weeks of gestation and is in early labour. Vaginal examination unexpectedly reveals a breech presentation: the legs of the fetus are apparently both extended. The cervical dilatation is 4 cm. The previous pregnancy resulted in a normal cephalic vaginal delivery of a 4 kg baby at 41 weeks of gestation. The current pregnancy has been uneventful to date and the fundal height is 38 cm above the pubic symphysis at the time of admission in labour at 38 weeks.

TASKS 1. Advise the patient of the possibilities regarding subsequent management and the pros and cons of these

You may take any further relevant history you require but do this briefly as the essential features have been provided above.

APPROACH ▪ History o Hi Jane, I am Dr. ___. From my examination, your baby is now in breech presentation, with both legs extended or what we call a frank breech. Have you known this from your previous antenatal tests? o Let's discuss your options for your delivery, but before that, is it alright if I confirm with you some details from the notes and a few questions? I read from the notes that your current pregnancy has so far been uneventful. That's great. Did you do all your antenatal tests? Were all your tests normal? o I understand also that your previous pregnancy was a 4kg baby delivered at 41 weeks, via normal cephalic vaginal delivery, is that correct? Did you have any problems with your delivery? Were forceps used to deliver the baby? Did you have any problems during your pregnancy? o Do you have any past history or family history of diabetes? ▪ Management o So, as I said, your baby’s position is breech. Normally, the baby’s head is down and the bottom is up. In your case, the baby’s butt/bottom is presenting down. Since it is a frank breech, it would be possible for us to try a vaginal delivery or a caesarean delivery. What would be your preference? o Your baby's type of breech presentation is actually a favorable one, and since you're keen to avoid a Caesarean delivery, we could attempt to do a vaginal delivery. However, there are certain risks to this such as fetal distress because of cord prolapse, hip or shoulder

dislocation, fracture of humerus, femur or clavicle, and asphyxia. If breech presentation were diagnosed prior to labor, ultrasound would have been of value to check your baby's size, type of breech presentation, and to check whether the fetal neck is extended. However, as you are already in labor, an ultrasound examination will probably be difficult to 235

arrange urgently. What we would do is, as soon as your membranes rupture, we would do a Page

© 2020 ARIMGSAS

vaginal examination to exclude cord prolapse and confirm the type of breech presentation. We would also do continuous cardiotocograph monitoring so that we could pick up any abnormalities during labor and delivery. However, if there will be slow progress or inadequate progress of labor, or there is fetal distress or any significant CTG abnormality occurs, we would need to do an immediate caesarean section. But do not be stressed, a successful outcome of labor can be anticipated with your condition.

KEY ISSUES ▪ The ability of the candidate to advice and counsel a patient of the current options in regard to breech delivery by vaginal or Caesarean delivery.

CRITICAL ERROR ▪ Failure to advise of the appropriate risks of vaginal breech delivery ▪ Recommending that external cephalic version should be attempted despite the fact she is in labour. ▪ Indicating to the patient that vaginal breech delivery is absolutely contraindicated despite her desires.

236 Page

© 2020 ARIMGSAS

Vaginal bleeding in a 23-year-old woman

You are working in a hospital emergency department. Your next patient is a 23-year old nulliparous woman who has been trying to conceive, and she believes she is pregnant. She has developed vaginal bleeding after eight weeks of amenorrhea.

TASKS 1. Take any further relevant history you require 2. Ask the examiner about the findings you would look for on general and gynaecological examination and the results of any lead you would expect to be available at the time you are seeing the patient 3. Advise the patient of the probable diagnosis and subsequent management you would institute, including any further investigations you would arrange.

APPROACH ▪ I'm sorry to hear that you currently have bleeding. But before we proceed further with this consultation, may I ask if you have any pain right now? Can you point to me where exactly is the pain? From a scale of 1-10, 10 being the most painful, how bad is your pain right now? Do you have any allergies? I will talk to my examiner first, and get back to you, is that alright? ▪ I would like to offer some pain killers for my patient. And I would like to know what is her BP and is there a postural drop, pulse and its rhythm, respiratory rate and oxygen saturation, and temperature? o If not stable, transfer to resuscitation room, insert IV lines and draw blood ▪ Before we proceed further, I am going to ask you some sensitive and personal questions to help me assess your condition. Rest assured everything will remain confidential between us unless it poses harm to you or to others. Is that alright with you? ▪ I understand from your notes that you have vaginal bleeding and had been trying to get pregnant. Can you tell me more about it? o ASK ABOUT VAGINAL BLEEDING (PERIOD HISTORY) • When did it start? What were you doing before this started? Is it continuous (always there) or is it on and off? Is it getting worse (has it increased in amount since it started)? What is its color? Is it smelly or not? How many pads did you use up until now? Did you pass any clots? Did you feel any dizziness? Has this happened before? Any tiredness or dizziness? Any fever or discharge from down below? Any rash or skin lesions noted? • Do you know your blood group? • Do you have any pain associated with this bleeding? If yes, ask details about the pain, especially if the pain preceded the bleeding. • Do you have any bleeding or bruising from anywhere else in your body?

• PERIOD DETAILS ▪ When was your last period? Do you get it every month? How long do you usually bleed? Do you usually have a light, moderate, or heavy flow? Do you have any pain or dizziness with your menses? 237

▪ How does this bleeding episode compare with your usual menses? Page

© 2020 ARIMGSAS

o ASK ABOUT PREGNANCY DETAILS (PREGNANCY & PARTNER & CONTRACEPTIVE HISTORY) • I understand that you are sexually active and has been trying to get pregnant, is that correct? Are you on a stable relationship? • Prior to this occurrence, were you on any forms of contraception? ▪ When were you last on the pill? ▪ Is there any chance that you are pregnant now? • Have you had a pregnancy test done? When was it done and what was the result? • Have you ever been pregnant before? (If yes, ask for details) • Do you have any nausea, vomiting, or breast tenderness? (early pregnancy symptoms) • In case you're pregnant now, will you have support for this pregnancy? • Any history of sexually transmitted infections? o PAP SMEAR • When did you have your last pap smear, and what was the result? (must be within 2 yrs) ▪ If not within 2 yrs, I will arrange another review with you to arrange for your pap smear. o R/O DDX (triggers for the bleeding) -- **FOR COMPLETION, BUT THE CASE SUGGESTS THAT THIS WILL ONLY TAKE YOUR TIME. JUST FOCUS ON THE 5Ps • How is your waterworks? Any burning or stinging sensation? • Are there any changes in your bowel movement? • Do you have any cough or colds, or headache? • Any recent falls or injuries? o SADMA**FOR COMPLETION, BUT THE CASE SUGGESTS THAT THIS WILL ONLY TAKE YOUR TIME. JUST FOCUS ON THE 5Ps • Do you smoke? Drink alcohol? Engage in recreational drugs? ▪ If positive: "It's best to stop smoking/drinking alcohol/engage in recreational drugs now, as this will be harmful to you, and especially to the baby in case you're pregnant.: • Any intake of prescription or over the counter medications? • Any allergies? o Do you have previous illnesses or surgeries? o Do you have any family history of bleeding disorders, thyroid disorders, or womb conditions such as myoma, polyps, etc.? ▪ PHYSICAL EXAMINATION o General appearance: what is the BMI? Any pallor, cyanosis, lymphadenopathies, edema, dehydration? o VS - already asked in the beginning o Run through ENT, CVS, RESPI, CNS exam o ABDOMEN: any distention or visible masses? Any palpable masses or tenderness? o PELVIC: • Inspection: any bleeding or discharge from the vulva or vagina? Any rashes or

vesicles present? • Speculum: is the cervical os open/closed? Any discharge or bleeding coming from the cervical os and the vagina?

• Per vaginal: is there cervical motion tenderness? Is the cervix firm/soft? 238 Page

© 2020 ARIMGSAS

▪ What is the size and position of the uterus? Is it mobile? Any palpable masses or tenderness? ▪ Any tenderness and palpable masses over the adnexa? ▪ I would like to end my examination by performing a urine pregnancy test, Urine dipstick, blood sugar level. ▪ PE EXAMINATION FINDINGS: o PULSE: 80/min, regular o BP 120/80, not distressed o Pelvic exam: cervix closed, firm, no blood in the vagina o Uterus retroverted, enlarged to the size of an 8th-week pregnancy o Adnexa: no mass or tenderness o Pregnancy test positive previously confirmed on spot urine testing now o RH negative patient ▪ MANAGEMENT o Hello Mary, based on my history and examination findings, it seems that most likely you are having a threatened miscarriage. Do you have any idea what it is? Let me explain this to you. Your pregnancy test result came out positive. o (ILLUSTRATE) Normally, the baby inside your womb is attached to you through a cord, which we call as the placenta. Sometimes, because of the attachment of the placenta to your womb, some bleeding can happen. It is quite common, especially during the early period of pregnancy. In the majority of cases, this bleeding is quite harmless. It will stop on its own within a few days, and your pregnancy may continue without any problems. o However, because of your bleeding, we may have to admit you in the hospital to monitor you and your baby and do investigations to further confirm your pregnancy and to check for other reasons why you are having this bleeding. I will refer you to an obstetrician as well. The tests include basic blood tests such as an FBE, UEC, Blood group, (urine mcs if positive urine findings). We need to do an ultrasound to fully confirm your pregnancy, to check the sac surrounding the baby, the liquor volume or the amount of water that the baby needs to thrive, and to check for the presence of the baby's heart sounds. We need all of these to be done, and if everything comes out normal--meaning that you have no infections and that the baby is thriving well inside your womb, the OB might advise you to just take some rest, and we confirm the condition of you having a threatened miscarriage with a good prospect of continuing your pregnancy. If this is truly a threatened miscarriage, you will only be subjected to bed rest and to avoid stressors in your life. Unfortunately, there is no specific medication effective in improving the pregnancy outcome. Again, providing the ultrasound examination is normal, the chance of a successful pregnancy is between 90-95%. If this bleeding continues, we will do serial ultrasound checks to see the condition of the baby. o As you have an O- blood test result, we will also check your blood for an indirect Coombs test. It checks some factors in the blood that if present, might affect the development of your baby during your next pregnancies. We will arrange medications for you (anti-D) in case you turn out positive for it. o Do you have any questions?

o Rest assured, we will do our best to manage your condition the best way we can. Here are some reading materials for you to give you more insight into your condition.

239 Page

© 2020 ARIMGSAS

KEY ISSUES ▪ Ability to define the diagnoses needing to be considered in the presence of eight weeks of amenorrhea ▪ Ability to appropriately investigate a woman with these symptoms

CRITICAL ERRORS ▪ Failure to confirm pregnancy by pregnancy testing ▪ Failure to arrange ultrasound to check site and visibility of pregnancy ▪ Failure to consider the use of anti-D in view of Rhesus negative state

240 Page

© 2020 ARIMGSAS

Cessation of periods in OCP use

Your patient is a 30-year-old woman who is taking the oral contraceptive pill (OCP). She has come to see you in a general practice because she did not have a period following the last two courses of pills.

TASKS: 1. Take a further focused history. 2. Ask the examiner about the findings you wish to elicit on general and gynaecological examination. 3. Advise the patient of the diagnosis and subsequent management (including any investigations you would arrange).

APPROACH ▪ History o I understand from the notes that you are here because you have not been getting your periods after taking your pills. Can you tell me more about it? When did your periods stop? Was it a gradual reduction in your periods? Did you have similar episodes before you took contraceptive pills? o Period history: When was your last menstrual period? Before this, were your periods regular? How long were your cycles? A usual number of days of periods? Is your bleeding mild/moderate or severe? Any pain or clots during periods? o How long have you been on the pill? Are you taking your pills regularly? Do you think you might have missed your pill anytime? Are you experiencing side effects with the pill? Are you taking any other medications aside from the contraceptive pill? o Sexual history: are you sexually active? Do you have a stable partner? Any problems with sexual activity? Any chance that you might be pregnant right now? When was your last pap smear and what was the result? o Rule out causes of secondary amenorrhea: ▪ Pregnancy: Do you experience any nausea, vomiting, breast enlargement or nipple discomfort? ▪ PCOS: any recent weight gain, acne, abnormal hair growth? ▪ Hypothyroidism: any weather preferences, puffy face, edema? ▪ Eating disorder/exercise induced: do you ever feel guilty about your weight? How often do you exercise? ▪ Hyperprolactinemia: any breast discharge, headache, nausea and vomiting? ▪ Asherman syndrome: any previous gynecological procedures or curettage? ▪ Stress: do you experience any excessive stress at home or at work? ▪ Post-Pill Amenorrhea o Have you ever been diagnosed of any medical or surgical illness? o Do you smoke, drink alcohol, take recreational drugs? ▪ Physical Exam o General appearance: look for visible hirsutism, acne, puffy face or edema, BMI o Vital signs o Vision: visual fields, fundoscopy, visual acuity 241

o Neck: thyroid enlargement Page

© 2020 ARIMGSAS

o Breast examination: nipple discharge o Abdomen: any visible distention? Palpable mass or tenderness? o Pelvic exam: ▪ Inspection: discharge, atrophic vagina ▪ Speculum: discharge, bleeding from the cervix ▪ Bimanual exam: size, position and mobility of the uterus, adnexal masses, CMT o Office test: urine dipstick, BSL, pregnancy test

▪ Diagnosis and Management From my history and examination, most likely the cessation of your periods is due to endometrial secondary to the combined pill. Do you have any idea what this is? The tissue lining your womb is called endometrium. Each menstrual cycle, part of it grows and becomes filled with blood and then is shed as a period. However in your case, there is thinning of the lining of the womb. This thinning is due to the progestogen component of the combined pill that you are taking. However, although unlikely, it is prudent for us to do a pregnancy test to exclude the possibility of pregnancy. But do not worry, this pill-induced endometrial atrophy does not cause any serious problems or affect your fertility, and your periods will come back once you discontinue the pill. However if you are really concerned about this, we can change your current pill to a higher estrogen containing pill such as Microgynon 50 or a triphasic pill such as Triguilar which can increase the menstrual loss, or we can cease the combined pill altogether and you can use some other method of contraception such as condoms. I will give you reading materials regarding post-pill amenorrhea for further insight and will review you regularly.

KEY ISSUES ▪ Ability to diagnose the cause of amenorrhoea when on the OCP. ▪ Ability to counsel the patient appropriately.

CRITICAL ERRORS ▪ Failure to perform a pregnancy test (/J-HCG) to exclude the unlikely possibility of a pregnancy occurring whilst taking the OCP.

242 Page

© 2020 ARIMGSAS

A positive culture for GBS

Your next patient is a 26-year-old woman who is now at 37 weeks of gestation in her first pregnancy. You have been looking after her pregnancy in a shared care arrangement in a general practice setting. All have been normal, and at 36 weeks you ordered a vaginal and rectal swab for Group B streptococcal (GBS) testing. This test has shown GBS organisms were detected in the lower vagina. She has returned to receive the results and any implications if the test is positive.

TASKS 1. Advise the patient of the results of the GBS test 2. Advise her about the subsequent management you would advise

There is no need for you to take further history or to request any examination findings or investigation results from the examiner.

APPROACH ▪ I understand that you are here to discuss the results of your Group B streptococcal test. ▪ Before I proceed further, let me explain to you why we usually do this test. ▪ (ILLUSTRATE) Group B streptococcus, or GBS, is a bug that is part of the normal vaginal flora in healthy women and is found in 10-15% of pregnant women at your stage of the pregnancy. It doesn't cause any harm to the mother, however, if present in the mother, it can possibly harm the baby when the baby is delivered vaginally. Around 40-50% of babies are colonized by this bug, but only 1% of these babies develop a severe life-threatening infection. And worse, by the time baby presents with these signs of infection, it may be too late to treat it effectively. ▪ In essence, the important principle of doing this test is to prevent your baby from developing an overwhelming infection. Do you understand so far? ▪ As for your results, your vaginal swab shows the presence of GBS. But do not stress yourself about this. It is good that we have picked it up at this stage and we will do our best to prevent your baby from developing an infection. ▪ Do you have any allergies to any medications? ▪ Having found that you are GBS positive, we will give you IV antibiotics during your labor (penicillin 3g initially as a loading dose, then 1.5, or erythromycin if allergic to penicillin) which is started at least 4 hours before your delivery or when your bag of water ruptures even before the onset of labor. The antibiotics we'll give to you will cross the placenta and protect your baby before passing through your vagina--where the GBS is present--during delivery. This is the best means that we can prevent your baby from developing an infection ▪ However, you have to be aware that there are some risk factors which can increase the risk of the baby developing the infection. That is when the mother develops preterm labor, when the bag of water ruptures even prior to onset of the labor, and if the mother develops fever. Antibiotics are

given immediately to the mother in these cases, but it puts 0.5% of babies at significant risk to develop the infection. ▪ After the delivery, your baby will be assessed by the paediatrician. If he is seen to be completely

healthy, no further antibiotics will be given to him. 243 Page

© 2020 ARIMGSAS

▪ If you develop any fever, abdominal pain, difficulty and pain in urination, please come back to me so that I can review you again. ▪ I will arrange another review for you next week, as your labor can start anytime soon. ▪ I can give you reading materials to give you more insight into your condition. ▪ Do you have any questions? ▪ Thank you and see you again during your next review.

KEY POINTS ▪ Defining the management plan ▪ Counselling the patient as to why antibiotic treatment in labor is recommended

CRITICAL ERRORS ▪ Failure to advise the patient of the significance of GBS organisms to the mother and her baby. ▪ Failure to advise antibiotic treatment of the pregnant woman if the membranes rupture, or when labor commences, to protect the fetus from the risk of severe infection

244 Page

© 2020 ARIMGSAS

Vaginal bleeding

Your patient is a 25-year-old married nulliparous woman who presents to you in general practice with vaginal bleeding after eight weeks of amenorrhoea. Her cycles are often irregular with the periods occurring at intervals of 4-8 weeks.

TASKS: ▪ Take a further focused history. ▪ Ask the examiner about the findings you wish to elicit on general and gynaecological/obstetric examination. ▪ Advise the patient of the probable diagnosis and subsequent management, including any investigations you would arrange.

APPROACH ▪ History o I understand from the notes that you are because of vaginal bleeding. Is it alright if I ask you a few questions? o When did the bleeding start? Is this the first episode of the bleed? What is the colour of the bleed? Does it have any offensive smell? Are there clots? How heavy is the bleed? How many pads do you soak in a day? Is it fully soaked? Is it a continuous bleed? Is it associated with pain? Is it associated with sexual activity? o Period history: when was your last menstrual period? Are your cycles regular? What is the usual length of your cycles? What is the usual duration of your periods? Any pain or clots during your periods? o Sexual history: are you sexually active? Do you have a stable partner? Do you use any contraception? Any chance that you might be pregnant right now? When was your last pap smear and what was the result? o Symptoms of pregnancy: Do you experience any nausea, vomiting, breast enlargement or nipple discomfort? o Any previous medical or surgical illness especially bleeding disorders? o Any family history of any gynecological disorders? o Do you smoke, drink alcohol, take recreational drugs? o Do you take any prescription or over the counter medications? Any allergies?

▪ Physical Exam o General appearance: pallor, edema, lymph node enlargement, BMI o Vital signs o Systemic examination

• CNS/CVS/Respiratory • Abdomen: any visible distention? Any palpable mass or tenderness? • Pelvic examination ▪ Inspection of the vulva and vagina 245 ▪ Speculum: is the cervix closed or open? Is there discharge or bleeding? Page

© 2020 ARIMGSAS

▪ Per vaginal exam: what is the size and position of the uterus? Any tenderness? Any mass or tenderness in the adnexa? o Office tests: Pregnancy test ▪ Diagnosis, Investigations and Management From the history and examination, the cause of your vaginal bleeding is still unclear. However, the most important thing that I want to confirm if whether you are pregnant or not. To confirm this, I would first like to do a beta HCG to see if you are pregnant. If beta HCG is negative, your bleeding might just be a late period, and we will just wait and observe the pattern of your periods. If your periods remain irregular, then we can order some hormonal tests such as FSH, LH, prolactin and thyroid function tests to look for other causes of your irregular periods, and plan subsequent treatment accordingly. However, if beta HCG is positive, we will do an estimation of the beta HCG level. If it is greater than 1000 U/L, then we will do an ultrasound to check the site and normality of the pregnancy, and the gestation and due date. We will also investigate the cause of the vaginal bleeding about the pregnancy, and manage accordingly. I will review you once the results of the tests are available.

KEY ISSUES ▪ Ability to evaluate a patient with bleeding after amenorrhoea. ▪ Ability to confirm or exclude pregnancy as a cause.

CRITICAL ERRORS ▪ Failure to consider non-pregnancy as well as pregnancy causes. ▪ Failure to arrange ultrasound if pregnant and β -hCG is greater than 1000 U/L.

246 Page

© 2020 ARIMGSAS

Eclampsia in a 22 year old primigravida

This 22-year-old primigravida has been seeing you in general practice clinic for her shared antenatal care since early in her pregnancy. She is now at 38 weeks of gestation.

The pregnancy has been progressing normally until now. Whilst in the waiting room along with her mother waiting to see you for her routine antenatal visit, she has had a grand mal fit. She had brought a urine specimen with her to the appointment

TASKS 1. Take any further relevant history from the mother of the patient who is in the waiting room 2. Ask the examiner about the specific findings you would look for on general and obstetric examination and any office test results which should be available to you 3. Advise the mother of the patient, in lay terms, of the diagnosis and the subsequent management you would advise for her daughter.

APPROACH ▪ I understand that your daughter had a fit while waiting here, but before I proceed further, I would like to talk to my examiner first if it's okay with you. ▪ To examiner: I would like to know what is her BP and is there a postural drop, pulse and rhythm, respiratory rate, oxygen saturation, and temperature of my patient? Can I ask for the fetal heart sound rate and rhythm? o If not stable: I would like to transfer my patient to the treatment room, insert large-bore iv lines, and draw some blood for investigations: FBE, UEC, LFTs, RFTs. I would like to check her urine for a urine dipstick to check for proteinuria. o If stable, go with the history ▪ HISTORY o I understand that you are very distressed about your daughter's condition as she had a fit a while ago. Could you tell me more about how this happened? • How long did the fit last? • Can you describe how she looked like when she had the fit? • What happened after the episode? • Further history related to preeclampsia/eclampsia: ▪ Did she complain about recent headaches, visual disturbances, swelling lately? • If positive, ask further • Headaches: Pain Qs • Visual disturbance: Describe what is the visual disturbance, always there or on and off? • Swelling: where is the swelling? one leg or both legs? Swelling up to

where? Any pain? When is it most severe (worsens as the day goes by)? • Rule out other causes of seizures: ▪ Did she have any recent cough, colds, problems with her water works, loose 247

bowel motions? Any nausea or vomiting? Any fever or rashes? Page

© 2020 ARIMGSAS

• Does she have any history of hypertension, kidney disease, or other medical problems in the past? • Thank you. I would like to talk to my examiner ▪ PHYSICAL EXAMINATION o GA: is she alert, drowsy, rousable? Any signs of pallor, icterus, cyanosis, lymphadenopathy present? • Edema: is there generalized edema? If not, where is it localized? o VS: BP, HR with rhythm, RR with saturation, Temperature o CNS: level of consciousness, motor power, tone, reflexes, sensory examination? Any unilateral localizing signs? o Abdominal exam: • Inspection: distention, visible masses? Tenderness? • Fundic height? • Fetal heart sounds? • Fetal lie and presentation? • Head engaged? o PELVIC EXAM • Inspection: any bleeding discharge, rashes vesicles • Speculum: bleeding, discharge, cervical os open or not • Per vaginal: uterine size, cervix soft/firm, cervix dilated? o I would like to end my examination by doing a urine dipstick to check for proteinuria, blood sugar level ▪ DIAGNOSIS AND MANAGEMENT o From history and physical examination, it seems that most likely your daughter developed ECLAMPSIA. Have you heard about it? It is a condition which occurs late in pregnancy and generally in women having their first baby. It is a condition where there is a sharp rise in your daughter's blood pressure with massive swelling and leakage of proteins through urine, which prompt the body to respond by having a fit or in medical terms, a seizure. This leakage of proteins was manifested by the swelling she had for the past few days, and as well as her headaches. Unfortunately, the exact cause is unknown but anything that decreases the blood supply to the placenta can cause the placenta to secrete certain chemicals which could damage the lining of the blood vessels of all major organs. Providing it is well controlled, no long-term harm usually occurs to either the mother or the baby, although it is potentially very dangerous to both of them. o Because of this, I will refer her to the obstetrician and she needs to be transferred via an ambulance and be admitted to the hospital immediately, and the delivery of the baby arranged as soon as her blood pressure and any further fits are controlled. She needs an urgent admission as these seizure episodes compromise the blood supply of the baby. o As we are waiting for the ambulance, I will start an anticonvulsant for her which we will give through her veins (IV diazepam) to prevent another seizure episode. We will also arrange basic blood investigations such as a renal function test, liver function test, FBE, coagulation profile, blood group and cross matching. o In the hospital, she will be seen by the obstetrician and she will receive IV medications to

prevent further fits and to control her blood pressure (4 grams of MgSO4 initially, over 10- 15minutes, and then 1g per hour as continuous infusion, IV hydralazine 5mg or diazoxide). We will continue to monitor her vital signs and her urine output and urine proteins, also check the condition of the baby through a monitor called cardiotocogram (CTG). She will be 248

placed in a dark, quiet room fit to manage another seizure episode in case it happens again. Page

© 2020 ARIMGSAS

o The method of the delivery of her baby will depend on her over-all assessment by the obstetrician. This will depend on her BP control, the condition of the baby, and the findings on cervical examination. If favorable for a vaginal delivery, her labor will be induced, and she will be given an epidural anesthetic for pain relief and to also aid in her blood pressure control with continued monitoring. However, if anything would show abnormalities, especially with the condition of the baby, the OB might decide to perform a cesarean section. o I can imagine how this can be very distressing for you. Do you want me to call someone who can assist you now? o We will do what we can to give the best care for your daughter and her baby. Thank you.

KEY ISSUES ▪ Knowledge of the causes of fitting in pregnancy ▪ Ability to manage a patient who has had an eclamptic fit in late pregnancy and is not in the hospital

CRITICAL ERRORS ▪ Failure to diagnose eclampsia and recognise the risk of this to mother and baby ▪ Failure to sedate, and failure to transfer her immediately to the hospital ▪ Failure to outline the three principles of management in the hospital -- sedation, lower blood pressure, and delivery of the baby

249

Page

© 2020 ARIMGSAS

Abnormal GTT

This patient is a 34-year-old obese primigravida whom you are managing in a country general practice. She has had a screening glucose tolerance test performed at 28 weeks of gestation. This revealed a fasting blood glucose of 7.5 mmol/L and a two hour level of 9.5 mmol/L (Normal levels — fasting < 5.5 mmol/L; two hours < 8.0 mmol/L). Progress of her pregnancy has until now been normal. No other investigations have been done apart from routine screening tests at the first antenatal visit which was all normal.

TASKS: ▪ Take any further relevant history you require. This should be limited to 1-2 minutes only. ▪ Ask the examiner for the findings you would expect on general and obstetric examination. ▪ Advise the patient of the diagnosis you have made. ▪ Advise the patient of the management you would give in the remainder of the pregnancy.

APPROACH ▪ History o I understand from the notes that you have the results of your sweet drink test with you. I am happy to discuss your results with you, but is it alright if I ask you a few questions before we get to that? o How many weeks are you pregnant right now? How is your pregnancy going so far? Any issues in your pregnancy? Any tummy pain? Any bleeding or discharge from down below? Is the baby kicking well? Is this your first pregnancy? o Diabetes questions: Any symptoms of frequent urination? Do you feel more thirsty nowadays? Any recurrent skin or vaginal infections? Any numbness or tingling sensation in your extremities? Have you been tested for diabetes previously? o Pre-eclampsia questions: any headaches, blurring of vision, edema or swelling? o Routine antenatal history: Any blood test and blood group done? Down syndrome screening? Folic acid? Ultrasound at 18 weeks? Any complications in the position of the placenta? o How is your diet? Activity and exercise? o Any smoking, alcohol? o Do you have good support? o Any other medical and surgical illness? Any medications? o Any family history of diabetes?

▪ Physical Exam o General appearance: BMI

o Vital signs: BP o Systemic examination

• CNS: signs of peripheral neuropathy 250 • Eyes: fundoscopy for cotton wool spots

• CVS/Respiratory Page

© 2020 ARIMGSAS

• Abdomen: fundic height, FHR, lie, presentation • Pelvic exam: inspection, speculum

▪ Diagnosis and Management The results of your sweet drink test show that the value is a bit high, which means that you have a condition called gestational diabetes. During pregnancy, the placenta secretes certain hormones like human placental lactogen, beta HCG, and cortisol, all of which has got anti-insulin properties. And insulin is the hormone that keeps your blood sugar level under control. Usually, during pregnancy, your production of insulin is heightened up, however in your case, it isn't the case. If your blood sugar doesn’t get controlled, complications can happen in your or the baby. Complications for you include polyhydramnios. This is a condition where the fluid in the bag surrounding the baby becomes high. Due to this, you could also go into preterm labor or premature rupture of membranes, or membranes rupture before labor pains set in. There is also an increased chance of induction or C-section. Another complication is pre-eclampsia, which is a condition where there is a sharp rise in blood pressure with leakage of proteins in the urine. Next is placental abruption, which is a condition where the placenta separates from the womb resulting in bleeding. In your baby, complications include macrosomia or a big baby. To detect this, we will do an ultrasound at 32-34 weeks. He can also have birth defects like a neural tube or nervous system defects, heart defects, and vertebral defects. And if the blood sugar level gets uncontrolled for a long time, the baby can even go for intrauterine growth retardation. Therefore, weekly CTGs will be performed until delivery or twice weekly if you will be on insulin or if macrosomia or polyhydramnios is detected. There can be complications after birth as well, such as low blood sugar levels in the baby. He can also develop respiratory distress syndrome or difficulty in breathing, and a high incidence of jaundice or yellowing of the skin. It is still possible to avoid these complications as long as we keep your blood sugar level under control. I will refer you to the high pregnancy clinic where you will be seen by the obstetrician, the diabetic physician, the dietician and the diabetic educator. The first thing to do is a strict diet control for 2 weeks. You will be given a diet chart by the dietician and you need to monitor your blood sugar level 3-4 times a day, especially about two hours after a meal and record that in a diary. Your aim is to maintain the blood sugar level between 4-6 mmol/litre before meals. If the diet control is not working, the diabetic physician will decide to start you on insulin. Again, you have to monitor your blood sugar level 3-4 times/day. You will monitor your blood sugar using a glucometer. This and the proper administration of insulin will be taught to you by the diabetic educator. You will be monitored for diabetic control by doing an HbA1c. You may also be seen by the ophthalmologist and the nephrologist if necessary. You need to have weekly antenatal checks from 30 weeks, ultrasound at 32 weeks and then 4 weekly, CTG weekly from 32 weeks. You can go in for normal vaginal delivery if everything goes on well with her antenatal checks, but it should be a term at the latest. During delivery, your blood sugars will be checked intermittently and insulin injections will be given as needed. The baby will be

monitored by continuous CTG. After delivery, there is a high chance that your blood sugar level will come back to normal because the hormones present during pregnancy will now be gone. You can already stop insulin injections when that happens. Your baby, however, will be checked by the 251

pediatrician and monitored for low blood sugar levels. Page

© 2020 ARIMGSAS

Once you have gestational diabetes, there is a 30-60% chance, that you could develop diabetes later in life. We will repeat an OGTT, 6 weeks after delivery, and then a blood sugar level 3 yearly. You must also control any weight gain in the future.

KEY ISSUES ▪ Ability to recognise that the blood sugar results are diagnostic of gestational diabetes. ▪ Ability to appropriately assess the control of diabetes during the remainder of the pregnancy, and to appropriately manage the patient, in consultation with a physician and obstetrician. ▪ Ability to recognise the need for insulin if the blood glucose levels are not reduced satisfactorily with diet alone. ▪ Ability to recognise the increased risks to the fetus, and the need for close monitoring.

CRITICAL ERRORS ▪ Failure to diagnose gestational diabetes. ▪ Failure to advise diabetic diet and testing of blood sugar levels 3-4 times daily. ▪ Failure to arrange for a consultation with a diabetic physician and obstetrician.

252 Page

© 2020 ARIMGSAS

Abdominal pain and vaginal bleeding

Your patient is a 39-year-old woman who has been married for 12 months and suffered a spontaneous abortion at eight weeks of gestation six months ago. This was her only previous pregnancy.

An ambulance has brought her to the hospital today because of severe lower abdominal pain and heavy vaginal bleeding for the last 12 hours. Her last period was nine weeks ago. You are seeing her in the Emergency Department at the local hospital.

TASKS 1. Take any further relevant history you require (it should not take you more than 3-4 minutes to do this) 2. Ask the examiner for the appropriate findings you would look for on general and gynecological examination, and then ask for any investigation results you feel are necessary to enable you to make a diagnosis 3. Advise the patient, in lay terms, of the diagnosis and the subsequent management required.

APPROACH ▪ I'm sorry to hear that you have been suffering from tummy pain and vaginal bleeding. Are you comfortable enough to go through this consultation or do you want me to do something about the pain first? From the scale of 1-10, 10 being the most painful, how bad is your pain? Do you have any allergies? ▪ To examiner: I would like to offer painkillers to my patient (morphine) and I would like to know what is her BP and is there a postural drop, pulse and rhythm, respiratory rate, oxygen saturation, and temperature of my patient? o If not stable: I would like to transfer my patient to the resuscitation room, insert large-bore iv lines, and draw some blood for investigations: FBE, UEC, blood group with cross-matching, indirect Coombs test. o If stable, go with the history ▪ HISTORY o Which came first? Abdominal pain or bleeding? • ABDOMINAL PAIN Q ▪ Can you point to me where exactly is the pain? When did it start, what were you doing before it started? was it sudden or gradual? Is it always there or does it come and go? Is it cramping, stabbing? Does the pain go anywhere else? Is there anything that makes the pain better or worse? • Any recent injuries before the pain started? • Do you feel any stinging or burning sensation when you pass urine? • Any fever, cough, or colds?

• VAGINAL BLEEDING ▪ When did it start? What were you doing before this started? Is it continuous (always there) or is it on and off? Is it getting worse (has it increased in amount since it started)? What is its color? Is it smelly or not? How many pads did you 253

use up until now? Did you pass any clots or tissues? Did you feel any dizziness? Page

© 2020 ARIMGSAS

Has this happened before? Any tiredness or dizziness? Any fever or discharge from down below? Any rash or skin lesions noted? ▪ Do you know your blood group? ▪ Do you have any bleeding or bruising from anywhere else in your body? • PERIOD DETAILS ▪ When was your last period? Do you get it every month? How long do you usually bleed? Do you usually have a light, moderate, or heavy flow? Do you have any pain or dizziness with your menses? ▪ How does this bleeding episode compare with your usual menses? • ASK ABOUT PREGNANCY DETAILS (PREGNANCY & PARTNER & CONTRACEPTIVE HISTORY) • I understand that you are sexually active and has been trying to get pregnant, is that correct? • I'm sorry to hear that you just had a miscarriage six months ago. I would just like to ask if there were any procedures done to you or any medications given to you during that time? (ask if she underwent D&C or just took meds, etc) • And have you been actively trying to get pregnant since then? • Do you have any nausea, vomiting, or breast tenderness? (early pregnancy symptoms) • In case you're pregnant now, will you have support for this pregnancy? • Any history of sexually transmitted infections? • PAP SMEAR • When did you have your last pap smear, and what was the result? (must be within 2 yrs) ▪ If not within 2 yrs, I will arrange another review with you to arrange for your pap smear. o SADMA**FOR COMPLETION, BUT THE CASE SUGGESTS THAT THIS WILL ONLY TAKE YOUR TIME. JUST FOCUS ON THE 5Ps • Do you smoke? Drink alcohol? Engage in recreational drugs? ▪ If positive: "It's best to stop smoking/drinking alcohol/engage in recreational drugs now, as this will be harmful to you, and especially to the baby in case you're pregnant." • Any intake of prescription or over the counter medications? • Any allergies? o Do you have previous illnesses or surgeries? o Do you have any family history of bleeding disorders, thyroid disorders, or womb conditions such as myoma, polyps, etc. ? ▪ PHYSICAL EXAMINATION o GA: is she alert, drowsy, rousable? Any signs of pallor, icterus, cyanosis, lymphadenopathy, edema present? • Dehydration: what is the CRT? Is there oral mucosa dry or moist? How is the skin turgor? o VS: BP, HR with rhythm, RR with saturation, Temperature

o CVS, RESPI, CNS: level of consciousness? o ABDOMEN: • Any distention or visible masses? Any guarding or rigidity? Any palpable masses or tenderness? 254 o

PELVIC: Page

© 2020 ARIMGSAS

• Inspection: any bleeding or discharge from the vulva or vagina? Any rashes or vesicles present? • Speculum: is the cervical os open/closed? Any discharge or bleeding or products of conception coming from the cervical os and the vagina? • Per vaginal: is there cervical motion tenderness? Is the cervix firm/soft? ▪ What is the size and position of the uterus? Is it mobile? Any palpable masses or tenderness? ▪ Is the cervix dilated? By how much is it dilated? ▪ Any tenderness and palpable masses over the adnexa? o I would like to end my examination by performing a blood sugar level test. • DO NOT REQUEST FOR A URINE PREGNANCY TEST OR AN ULTRASOUND IF THERE ARE PRODUCTS OF CONCEPTION PRESENT ON THE PELVIC EXAM. YOU WILL BE MARKED DOWN! ▪ PE FINDINGS FROM THE CASE o GA: clammy, pale, obviously distressed and in pain o CVS: pulse 90/min, BP 80/50 mm hg o Abdomen: Lax, non-tender, no mass or viscus palpable o Speculum: cervix open, products of conception in cervical os (if the candidate does not ask if the cervix is open or closed, do not give this information but comment on the uterine size alone) o Pelvic: uterus enlarged to the size of the eighth week of pregnancy, anteverted, and mobile. Cervical os is open and easily admits the tip of one finger. No adnexal masses or tenderness o Investigations: • Blood group: O Rh-negative, indirect coombs test negative • Hb 112 g/L ▪ DIAGNOSIS AND MANAGEMENT o From my history and examination, I am sorry to say that you are having a miscarriage. Most miscarriages occur without any obvious reasons, and this loss was probably because of a genetic abnormality of the fetus itself especially during miscarriages occurring in the first 14 weeks. Unfortunately, these cases become more common as women get older. But let me reassure you that this is not your fault. You did not do anything wrong so please do not feel guilty about this. o Since you are actively bleeding and your vitals still unstable as of now, I will have to admit you. I have informed the OB registrar and sent all the blood for investigations. Your bleeding is caused by retained tissues of the fetus inside your womb, causing you to bleed more as long as it stays there. Thus, they will probably do a procedure called curettage as soon as possible to control your bleeding. This essentially involves inserting a small instrument we call curette (ILLUSTRATE) inside your womb to scrape out the remaining tissues that cause you to bleed. This will be done under anesthesia and is basically a pain-free procedure. o As we are waiting, fluids have been started in your IV line, and we will also give you medicines for your womb to contract to control your bleeding (IV ergometrine or a similar oxytocic agent). o We will also give you an injection called anti-D immunoglobulin as we have seen your blood

to be O negative to prevent future bleeding to you and to your child during your next pregnancy. o If your blood pressure remains low despite these interventions, it is possible that we give

you more IV fluids or a possible blood transfusion, depending on specialist advice. 255 Page

© 2020 ARIMGSAS

o You will still get pregnant, but it is advisable to wait for at least one normal period to occur before trying to get pregnant again. o The likelihood of having a miscarriage in your next pregnancy is probably about 25-30%. But do not stress too much about this, there is still a good chance that you will become pregnant. Thus, in your next pregnancy, we will do a regular review for you as your advanced age also increases the risk of having fetal abnormalities for your baby. We will do an ultrasound as early as seven weeks as well as blood tests for screening and confirmation of congenital disorders to provide reassurance that all is normal. o I can imagine how this must be very hard for you. But let me reassure you that we will do what we can to provide you with the best care possible. Do you want me to call anyone for you? Do you have enough support?

KEY POINTS ▪ Ability to define the likely cause of the symptoms and the low blood pressure found ▪ Ability to recognize that no investigations are required in this patient other than the urgent determination of blood group, as the diagnosis can be made on clinical grounds ▪ Ability to define the subsequent management plan

CRITICAL ERRORS ▪ Inappropriate investigation requested such as pregnancy test after results of physical examination are known, ultrasound examination, or coagulation screen ▪ Failure to recognize the need to remove POC from the cervix ▪ Failure to check the blood group to see if the anti-D antibody was indicated

256 Page

© 2020 ARIMGSAS

Fundus greater than dates

Your patient is a 26-year-old primigravida. She has been attending the general practice where you are working and seeing the doctors there in a shared care arrangement with a specialist in a major city 30 km away. She is not due to see the specialist again for a further six weeks. All appeared to be normal up to and including her last visit at 26 weeks of gestation when the symphysis-fundal height was 28 cm. Today, four weeks later at 30 weeks of gestation, the symphysis fundal height is 40 cm, and a weight gain of 6 kg has occurred during the four-week time interval.

TASKS: 1. Take any further relevant history you require. 2. Ask the examiner about the relevant findings on examination and the results of specific previous investigations which you believe would have been performed. 3. Advise the patient of the diagnosis and subsequent management.

APPROACH ▪ History o I read from the notes that you are now at your 30th week of pregnancy. How is your pregnancy so far? o Is the baby kicking well? Did you do your antenatal checks, blood tests and blood group and Rh typing? Have you done your pap smear? What was the result? Have you done your down syndrome screening? Did you take your folic acid? How was your ultrasound at 18 weeks? Was it a single pregnancy? Were there any uterine fibroids seen? Was correct dating confirmed on ultrasound? What was the result of your sweet drink test at 28 weeks? o I also read from the notes that you gained a substantial amount of weight for 4 weeks back, and your fundal height seems to be much larger than your age of gestation. What was your initial weight prior to pregnancy? Do you know your weight right now? o Diabetes questions: Any symptoms of frequent urination? Do you feel more thirsty nowadays? Any recurrent skin or vaginal infections? Any numbness or tingling sensation in your extremities? Have you been tested for diabetes previously? o Pre-eclampsia questions: any headaches, blurring of vision, edema or swelling, proteins or glucose in the urine? o Any past history of medical or surgical illness? o Do you have any family history of diabetes? o Any smoking, alcohol or recreational drugs? Any medications taken? o Do you have a good support? ▪ Physical Exam o General appearance: pallor, edema, lymph node enlargement, BMI

o Vital signs o Systemic examination

• CNS/CVS/Respiratory 257 • Abdomen: fundic height, FHR, lie, presentation, uterine tenderness, fluid thrill

• Pelvic examination Page

© 2020 ARIMGSAS

▪ Inspection of the vulva and vagina ▪ Speculum: is the cervix closed or open? Is there discharge or bleeding? ▪ Investigations o Blood group and Rh typing? Indirect Coombs test o Urine test: protein and glucose o Sweet drink test at 28 weeks o FBE: hemoglobin o Ultrasound at 18 weeks: single baby?

▪ Diagnosis and Management From the history and examination, most likely the cause why your uterus is larger than it should be is polyhydramnios or an excessive amount of amniotic fluid. There are other causes as to why your uterus may be larger than it should be such as wrong dating, multiple pregnancy, macrosomic or big baby, or uterine fibroids. However, these are unlikely because your ultrasound at 18 weeks has confirmed your dates, a singleton pregnancy, there is no evidence of diabetes, and there were no fibroids detected on your ultrasound. However, I will need to refer you back to the specialist now or within the next few days, and we need to confirm the diagnosis by doing an ultrasound which would also look for the possible cause of the excessive amount of amniotic fluid, such as fetal malformation of the central nervous system, gastrointestinal system, abdominal wall, or a benign tumor (chorioangioma) of the placenta. Other causes are diabetes and fetal infection with cytomegalovirus and toxoplasmosis. That is why we would also need to do a glucose tolerance test, a confirmatory test, to exclude diabetes since the glucose challenge test that you did before is only a screening test. Polyhydramnios can cause problems in late pregnancy and labour including malpresentation, premature rupture of the membranes, premature labour, and placental abruption following membrane rupture. As you are already at 30 weeks of gestation, it would probably beneficial for you to take extra bed rest to prevent premature delivery. However, if at any time you feel that you might be in premature labor, you should immediately go to the ED. Depending also on the assessment of the specialist, she may decide to give you prophylactic steroid therapy to reduce the likelihood of respiratory distress in the baby in the event that delivery will occur before 34 weeks of gestation. I will give you reading materials about polyhydramnios for further insight.

KEY ISSUES ▪ Knowledge of the causes of excessive uterine enlargement in pregnancy. ▪ Diagnosis of polyhydramnios. ▪ Determining the cause of polyhydramnios in this patient.

CRITICAL ERRORS ▪ Failure to consider and confirm that the problem is most likely due to polyhydramnios. ▪ Failure to consider the possible causes of polyhydramnios and failure to arrange the appropriate investigations or failure to refer to a specialist for these within the next few days.

258 Page

© 2020 ARIMGSAS

Severe postpartum hemorrhage

This 25-year-old primigravida had a normal vaginal delivery by the midwife 20 minutes ago in a country District Hospital which you are a Hospital Medical Officer (HMO), and currently on call for the Obstetric Unit. The pregnancy had been perfectly normal. The labor was of 14 hours duration. Only one dose of analgesia had been required. The estimated blood loss at delivery was only about 250ml. However a further 1500ml of bright blood has been passed in the last 15 minutes. The midwife has just phoned you to advise you of these facts, and to ask you to come and help with the patient's care.

TASKS 1. Ask the midwife the appropriate questions to define the probable cause of the haemorrhage and to assist you to define what care is now required. You should not take more than four minutes to do this task. 2. Advise the midwife of what she should do between now and when you will arrive in the delivery suite. You are currently at your flat which is ten minutes from the hospital 3. Advise the examiner of the most probably cause of the haemorrhage, and what you will do when you arrive in the delivery suite.

APPROACH ▪ DDx of post-partum haemorrhage: o Uterine Atony - most common o Genital tract lacerations: vagina/cervix/uterine rupture o Retained placental fragments o Coagulation disorders ▪ QUESTIONS TO MIDWIFE o HEMODYNAMIC STABILITY--VITAL SIGNS: What is her current BP and is there a postural drop, pulse and rhythm, respiratory rate, oxygen saturation, and temperature? • If UNSTABLE: please transfer the patient to the resuscitation room, secure IV lines, and take blood for blood grouping and cross-matching and coagulation studies. Please start IV fluids: normal saline, Hartmann solution or Haemaccel whichever is available. • Please give her an oxygen mask and start high flow oxygen at 10/L o Is there any shortness of breath? How is her level of consciousness? o What mode of delivery was done? Was instrumental delivery performed? o Was it a single or a multiple pregnancy? How much did the baby weigh? And how is the baby? o Can you describe the bleeding for me? What is the color (bright or dark red)? Is it continuously pouring out? Are there clots? How much blood was lost so far? o Was episiotomy done? Any visible genital/perineal tears from outside? o Is the uterus lax or contracted? What is its position (central or not)? Is there abdominal

tenderness? o Have you checked the placenta? Is it complete? When was it delivered? Any possibilities that there are retained products inside the uterus? o Is the patient bleeding from anywhere else? 259

o What has been done so far? Was ergometrine or oxytocin given to the patient? Page

© 2020 ARIMGSAS

o Does she have any history of bleeding disorders or previous uterine surgeries? o Do you know her latest hemoglobin count? o Is she on a urinary catheter? If not, please insert a catheter to drain her bladder. ▪ ADVISE TO MIDWIFE o I will be on my way there but as of now please do the following interventions • Please put a firm force over the uterine fundus and massage it • Continue the IV fluids that we started. Transfuse it on a fast drip-rate. • Give IV or Intramuscular ergometrine at 0.25mg, or intramuscular oxytocin immediately. ▪ Alternative: prostaglandin per rectum • Follow-up blood test results as soon as possible • Observe any clotting on the blood that is passed o COMMENTARY TO THE EXAMINER • The cause of the bleeding can be uterine atony, retained placental fragments, genital lacerations from the vagina, cervix, or uterus, or a coagulation disorder. But most likely, the cause is uterine atony because of the prolonged labor of 14 hours, lack of administration of oxytocics and the abdominal findings of having a lax uterus. Although the other causes cannot be fully excluded now, the initial treatment should be targeted to address uterine atony. • As soon as I arrive, I will do the following unless the bleeding has substantially decreased: ▪ I will insert an intravenous drip if it has not been completed by the midwife, and commence/continue crystalloid fluids until blood is available ▪ Inspect the placenta to ensure that it is complete and normal ▪ Do a speculum examination to check for any vaginal or cervical lacerations that may require suturing ▪ Regularly monitor the vital signs ▪ Give oxygen by mask ▪ Give blood as soon as available and cross-matched ▪ If the bleeding continues: • I will add 20-50units/litre of Oxytocin (Syntocinon) to the intravenous fluids being given. • I will refer the patient to the OB specialist, as it is likely that the patient requires an examination under anesthesia (EUA) to ensure no retained products of conception are present, and that the uterus is intact, and that there is no uterine inversion. This examination should not be performed until a blood transfusion is running and she is hemodynamically stable. ▪ It is only after a EUA has been performed that the use of intramyometrial prostaglandins such as PGF2a and major surgery such as an internal iliac ligation, or even a hysterectomy should be considered.

KEY ISSUES ▪ Knowledge of the causes of primary postpartum haemorrhage

▪ Ability to recognize the most likely cause of a primary postpartum haemorrhage ▪ Ability to advise the midwife of the initial management required

▪ Ability to understand the steps required if the initial management is not successful in controlling 260 the haemorrhage

Page

© 2020 ARIMGSAS

CRITICAL ERRORS ▪ Failure to define that uterine atony is likely to be present on the information obtained from the midwife ▪ Failure to ask for information regarding the vital signs ▪ Failure to advise the midwife to initiate initial actions in detail. The performance of coagulation studies is not mandatory but other actions are.

261 Page

© 2020 ARIMGSAS

Sterilization in intellectually disabled child

You are working in a general practice. Mrs Davis is a widow and has been a patient of yours for approximately twelve years. She has a 14-year-old daughter Evelyn, who has a significant intellectual disability. Evelyn has also been a patient under your care since the family moved to the suburb 12 years ago. Evelyn also suffers from epilepsy, has an ataxic gait and significant behavioural problems. While able to dress and feed herself Evelyn requires significant assistance with washing and is not capable of any form of independent living. Mrs Davis, now 54 years old, is concerned that Evelyn's behavioural problems will be exacerbated with the onset of menstruation. Mrs Davis is also extremely anxious as to her own ability to care for Evelyn during her menstrual cycle and has the clear view that Evelyn would be intellectually incapable of understanding the physiological changes to her body in addition to being physically incapable of meeting her own hygiene needs. As Evelyn has been cared for solely by her mother, and requires high levels of supportive care, Mrs Davis has decided that she will approach you to organise for Evelyn to undergo a hysterectomy and oophorectomy. Today Mrs Davis has made an appointment to talk to you about this operative procedure for Evelyn.

TASKS: Respond to the mother's questions and provide information to her about the consent required prior to the operative procedures she is seeking for her daughter.

APPROACH ▪ Good morning Mrs Davis. I understand that you have some concerns about Evelyn. Can you please tell me more about it? ▪ How is your daily situation with Evelyn? What are your concerns about this? ▪ Has Evelyn started her periods already? Does she already exhibit any other signs of puberty such as enlargement of the breasts, armpit and pubic hair? How did she react to these changes in her body? ▪ I understand your concerns Mrs Davis, and I can only imagine how hard it must be for you to manage all these by yourself. I understand that you want Evelyn to undergo permanent sterilization for reasons you have stated, however, the decision or consent to undergo sterilization needs to be determined through legislation or the case law. Even if you are the mother and sole guardian of Evelyn, you do not have the legal authority to give consent to the sterilization of Evelyn because this procedure is outside the scope of a parent's legal authority to validly consent to medical treatment of your child. Even in this circumstance that your child has an intellectual disability and incapable of giving valid consent, a procedure such as sterilisation cannot be carried out lawfully without the authority of the Family Court, or similar legal body such as a Guardianship Board. I understand and acknowledge that Evelyn has some intellectual disability, however, this does not deprive her of the legal and ethical right to be treated medically as would any other

patient, so a procedure would not be carried out unless it was medically indicated for the best interests of the patient, and this sterilization procedure might predispose Evelyn to some form of harm or risks.

▪ Question from the mother: 262 Page

© 2020 ARIMGSAS

'Why can't you have her admitted for the surgery? When Evelyn needed her appendix out you admitted her to hospital.' Yes, at that time, I admitted her to the hospital because an appendectomy can be a medical emergency, and that decision was made for the best interest of the patient.

KEY ISSUES ▪ Recognise that medical procedures like sterilisation, are different in nature from those procedures for which a parent has the legal authority to consent on behalf of their child. ▪ Identify that parents do not have the legal authority to consent to the sterilisation of their child. ▪ Identify that the doctor does not have the legal authority to consent to sterilisation of the child. ▪ Recognise the need for a valid consent before the procedure can be undertaken. ▪ Recognise that only the Australian Family Court or in some states the Guardianship Board has the jurisdiction to authorise the carrying out of the procedure in the particular circumstances. ▪ Articulate the 'harm' that potentially flows from the sterilisation of a minor and the recognition of the issues associated with discrimination in relation to individuals with disabilities.

CRITICAL ERRORS ▪ To agree Evelyn's parent can give legally valid consent for Evelyn to undergo such surgery. ▪ To agree that the candidate, as the treating medical practitioner, can give legally valid consent to the procedure. ▪ To fail to recognise that even though Evelyn has an intellectual disability, valid consent is necessary.

263 Page

© 2020 ARIMGSAS

Blood transfusion consent

A 33-year-old woman, Miriam, has just come to see you in the Emergency Department of a major urban hospital with severe antepartum hemorrhage. She is seven months old (30 weeks) pregnant, and prior to this time, has been fit and well. This is her second pregnancy--her first baby is alive and well.

On examination, she is conscious and able to speak. Her appearance is pale and sweaty, she is tachycardic and her blood pressure is 80/45mmHg. The hemorrhage is continuing. An emergency ultrasound suggests central placenta previa.

You start to take appropriate measures, including insertion of an intravenous cannula, and taking blood for grouping and cross-matching. You have begun transfusion with Hartmann balanced electrolyte solution. You explain to her that she will need an emergency blood transfusion as part of her treatment, and that this will start as soon as possible while preparations are made for an emergency Cesarean section.

She says that she is a Jehovah's Witness and will not accept a blood transfusion. Her husband is also a Jehovah's Witness. He is overseas at the moment and cannot be contacted.

TASKS 1. Ascertain fully the patient's views about her treatment by blood transfusion 2. Explain the risks and benefits of the suggested treatment for both Miriam and the baby 3. After six minutes, answer the examiner's questions.

APPROACH ▪ Good morning/afternoon, Miriam. It seems that you have a condition that we call an antepartum hemorrhage or severe bleeding during pregnancy because of the very low attachment of the placenta to your womb which we call as placenta previa which we have seen on your ultrasound examination. Sorry for using a medical term, but let me explain this to you. ▪ (ILLUSTRATE) Inside the womb, the baby is attached to the mother through an organ we call as the placenta, which provides nutrition and blood to the baby. Normally this is attached way above your cervix and vaginal canal. However, in your case, it seems that the placenta is lying in the way of your baby. Because of its placement the placenta is prone to be pushed by the baby constantly or is forced to contract by the womb muscles causing bleeding episodes. This condition presents as a common cause of bleeding in pregnancy, and it usually presents with a causeless, painless, recurrent bleeding. Sometimes it can present with severe life-threatening bleeding, unfortunately as in your case, which can adversely affect you and your baby. ▪ I can imagine how this can be very distressing for you. Would you like me to call anyone to be with you right now, or is it okay to for me to continue explaining what we can do to address your

condition? ▪ Currently we can see that your BP is low and your pulse is fast, with you being sweaty and having cold hands and feet, and shortness of breath. These are signs of what we call as haemorrhagic shock due to blood loss. This means that your organs may not be receiving the adequate oxygen 264

they need to optimally function because of the blood loss, as blood carries oxygen to our organs Page

© 2020 ARIMGSAS

and to your baby. With your symptoms, it seems that most likely you lost at least 30% of your blood volume and still you are actively bleeding. This is an indication for a blood transfusion to increase the chances of survival both for you and for your baby. o NO DOCTOR, I DON’T WANT TO BE TRANSFUSED! ▪ It is your right to refuse the treatment. However, it is important for me to explain the possible outcomes, the advantages and disadvantages of this procedure so that you can make an informed decision. We can discuss your treatment options in private, as I understand you are a Jehovah's Witness. It that okay with you? ▪ Right now, the best chances we have to save your baby is by doing an emergency caesarean section--that is, we operate on your tummy and womb to deliver your baby as soon as possible. Your bleeding will stop as soon as we empty your womb and remove your placenta. However, the operation itself is associated with blood loss (500ml) and may worsen your condition if blood transfusion is not given BEFORE and AFTER this procedure. Do you understand so far? ▪ We can maximize giving you fluids and use synthetic blood substitutes (Haemaccel) which can reduce the shock. However, if the bleeding continues and once 40% blood loss has occurred, the biggest problem we'll have is oxygen deprivation (Hypoxia) for both you and your baby, as these blood substitutes cannot carry oxygen to your organs, thereby shutting down your organs which can lead to death. And the only way we can address this is to have a blood transfusion as only blood has the capacity to carry oxygen throughout your body. o NO DOCTOR, I STILL DON'T WANT TO BE TRANSFUSED ▪ Can you please tell me if you fully understand all the possible consequences which may arise without the blood transfusion? The baby may survive if the operation is done immediately, but the chances of you dying are markedly increased without transfusion and without proper preparation. Do you have any questions for me so far? ▪ I respect and understand your decision. We will do our best to save you and your baby's life without the blood transfusion. ▪ TO THE EXAMINER o Summarize the legal and ethical issues in this situation • The case raises the issues of legally valid consent. It is a process for ensuring that patients can have control over what happens to them, and ensuring that they are competent to decide and have sufficient information to understand the implications of their decisions • LEGAL ISSUES ▪ Every competent patient has the legal right for autonomy -- or to accept or refuse treatment. She has the right as an adult of sound mind to self-determine her treatment. ▪ Even if the patient's rights can be restricted by potential damage to a third person--as in this case her baby, in Australia, the fetus/unborn baby has "no rights" justifying treatment being forced on the mother against her wishes. ▪ Australian law does not protect the unborn child from maternal decisions regarding care. ▪ Human Tissue Act (1982) allows children to be transfused without parental consent providing that without a transfusion the child was likely to die, and

providing two medical practitioners concurred in that opinion before administration of the blood transfusion. However, this does not apply as the child is still unborn

• ETHICAL ISSUES 265 Page

© 2020 ARIMGSAS

▪ Miriam's right to be an autonomous decision-maker is not actually restricted by any impact her decision will have on her baby, although it would be restricted if the decision was one involving blood transfusion to her baby following delivery if this was necessary to preserve life. o What will you do--how will you manage this situation? • Respect the patient's decision • Aggressive IV fluids and synthetic blood alternatives to be given to prepare the patient for an emergency caesarean section

KEY POINTS ▪ Ability to deal with strong religious views in a respectful manner ▪ Ability to recognize the priorities in this emergency situation and respond appropriately ▪ Recognizing that urgent Caesarean section is required as the bleeding is continuing and causing persisting shock and urgent operation is the only way of controlling persisting bleeding, despite the risks which operation would entail

CRITICAL ERRORS ▪ Not finding out Miriam's definitive wishes about treatment ▪ Indicating they would transfuse Miriam without obtaining a valid consent (either from Miriam or from her husband if Miriam was no longer conscious and capable of providing any opinion, for example, after Miriam had lost consciousness)

266 Page

© 2020 ARIMGSAS

Meconium staining

Your patient is a 25-year-old primigravida who is in early labour at 41 weeks of gestation. She is in the local district hospital where you are attending as a general practitioner. The hospital has good facilities but a consultant obstetrician is not available. Pelvic examination 30 minutes ago showed the cervix was 3 cm dilated, well effaced, and well applied to the presenting part. The cephalic presentation was position left occipito transverse (LOT), at zero station, with no caput or moulding evident. The membranes were still intact and allowed to remain so. Spontaneous rupture of the membranes then occurred and revealed profuse, thick meconium-stained liquor. The pregnancy had been uneventful to date, and blood pressure and urine testing have been normal in labour. The fetal heart rate, as defined using auscultation, has been between 130 and 140/min.

TASKS: ▪ Take any further relevant history you require. ▪ Ask the examiner about relevant findings likely to be evident on general and obstetric examination. ▪ Advise the patient of the diagnosis and subsequent management during and after delivery.

APPROACH ▪ History o I understand from the notes that you are already in your 41 weeks of pregnancy. When is your due date? o How was your pregnancy course? Any problems during your pregnancy? Did you have a CTG done? What was the result? Do you know your blood group? o Do you feel your baby kicking? Are the kicks consistent? No decrease in the fetal movements? o Are your contractions coming regularly? How often do they come? How long does one contraction last? o Have you noticed any discharge or bleeding from down below? o When did your water break? Was it green in color?

▪ Physical Exam o General appearance: pallor, edema, lymph node enlargement, BMI o Vital signs: BP, temp o Systemic examination • CNS/CVS/Respiratory • Abdomen: fundic height, FHR, lie, presentation, station, uterine tenderness, contractions

• Pelvic examination ▪ Inspection of the vulva and vagina ▪ Speculum: discharge? Meconium present? ▪ Per vaginal exam: cm dilation of the cervix? Percent effacement? Evidence of 267

caput? Evidence of cord prolapse? Page

© 2020 ARIMGSAS

▪ Diagnosis and Management It seems that the greenish material that we see is what we call meconium. Meconium is the baby's first stool. Meconium staining of the liquor is common in post-term labour. Sometimes it means the baby is having problems, such as fetal distress due to lack of oxygen, but in most instances, the baby is perfectly healthy. We will also do a cardiotocography to assess the fetal condition. A cardiotocograph is a safe, non-invasive method commonly used during pregnancy and labor. We will place 2 sensors in your abdomen to record your baby's movements, heart rate and your uterine contractions. If we find something abnormal in the CTG, then further investigations will be done such as fetal scalp pH or lactate level to find out the significance of this abnormality in the CTG, and if necessary, a caesarean section or vaginal manipulative delivery will be done. We would also do a pelvic examination to assess the progress of labour and to make sure that the cord has not prolapsed, which can be another cause of meconium staining of the liquor. If we don’t find any abnormality in the CTG or abnormality in the pelvic examination, then we would just let you progress with labor and monitor you accordingly until vaginal delivery is ultimately possible. At the time of vaginal delivery, adequate aspiration of the mouth, pharynx, and nasal cavity will be done to prevent any inhalation of meconium into the respiratory tract of the baby. Visualisation of the vocal cords, immediately after birth, with the aspiration of meconium in any adjacent region will also be done. During delivery, a pediatrician will also be there to ensure that resuscitation will be adequate and to commence any further therapy which may be required in the event that your baby inhales any meconium.

KEY ISSUES ▪ Understanding the relevance of the meconium staining of the liquor. ▪ Understanding the management required.

CRITICAL ERRORS ▪ Failure to do a vaginal examination to check cervical dilatation and exclude cord prolapse. ▪ Failure to adequately monitor the baby throughout the rest of the labour using continuous CTG assessment. ▪ Failure to aspirate the mouth and pharynx adequately, at delivery, to reduce the risk of meconium aspiration.

268

Page

© 2020 ARIMGSAS

Urinary incontinence in a 50-year-old woman

Your next patient is a 50-year-old woman with three children aged 29, 25, and 22 years. She comes to see you in a general practice setting because of a problem of urinary incontinence necessitating her to wear a pad all the time

TASKS 1. Take any further relevant history you require 2. Ask the examiner about relevant findings likely to be evident on general and gynecological examination 3. Advise the patient of the diagnosis and subsequent management including any investigations you would advise

Causes of incontinence (DIAPPEERSS) D – Delirium I – infection of UT A - Atrophic urethritis P – Pharmacological (diuretics) P – Psychological (acute distress) E – Endocrine (hypercalcemia) E – Environmental (unfamiliar surrounding) R – Restricted mobility S – Stool impaction S – Sphincter damage or weakness

Risk factors: UTI Obesity Smoking Caffeine Constipation Chronic cough Multiparity Menopause

APPROACH

▪ HISTORY 269 Page

© 2020 ARIMGSAS

o I'm sorry to hear you are having urinary incontinence as this can be very distressing. But don't worry I am here to help you and look after you today. Can you tell me more about your incontinence? o ASK ABOUT URINARY SYMPTOMS (identify the type of incontinence) • How much urine usually flows? Is it a small or large amount during episodes? How many pads have you used in a day? • Do you tend to rush to the toilet or else you'll have incontinence? (URGE) • Do you feel urine leaking when you cough, sneeze, laugh or strain? (STRESS) • Do you have constant dribbling? (OVERFLOW) • Any feeling of a mass bulging down below? • Do you have any burning or stinging sensation while you urinate? Do you go to the toilet more than the usual? Any frothy or smelly urine? Any changes in the color of your urine? • Any discharge or rashes from down below? • Any fever? Abdominal pain? (r/o UTI/infections) • Any weight loss? Do you feel thirsty all the time (r/o DM) o ASK ABOUT RISK FACTORS • I am going to ask you some private and sensitive questions to identify some factors which could have led to your condition. Is that okay with you? • Pregnancy history: I can see from your notes that you had three children. So, you had three pregnancies, is that correct? Did you have difficult labor for all pregnancies? Do you remember the weight of your babies? What type of delivery did you have, any instruments used? Are you aware if you had an episiotomy/repair of lacerations? • Periods: when was your last period? ▪ If menopause: at what age did you have your menopause? Any mood swings, hot flushes? ▪ Are you on any hormone replacement therapy? • Partner: are you sexually active? Are you in a stable relationship? Do you have any pain during intercourse? Bleeding during intercourse? Any history from STIs? • When was your last Pap smear? Did you already have your mammography? • Do you have any cough? • Any changes in your bowel motion? Do you have any hard stools? o SADMA • Do you smoke, drink alcohol, or engage in recreational drugs? • Do you drink coffee (caffeine as a risk factor), how many cups do you usually take in a day? • Do you take any medications? Any allergies? • How is your situation at home? • What's your occupation? Any stresses at work? o PMHx: Have you been diagnosed with any illnesses? Did you have any surgeries, especially done down there? o Family History ▪ PHYSICAL EXAMINATION

o GA: BMI**, dehydration** any edema, lymphadenopathy? o VITALS: Temperature, BP o Chest/Heart: r/o infections o Abdomen: any distention, visible masses? Tenderness? Is there renal angle tenderness? 270 o

PELVIC Page

© 2020 ARIMGSAS

• INSPECTION: any bleeding, discharge, rashes, or vesicles? Is any prolapse visible (third degree already visible) ▪ Vagina: any thin, dry, atrophic vagina? • SPECULUM: is the cervix healthy or not, any discharge or bleeding that you can see? ▪ Can you cough or bear down/STRAIN? ==> CHECK IF THERE IS ANY PROLAPSE (CYSTOCOELE IN THIS CASE) • Is incontinence present? ▪ DEGREES OF PROLAPSE • First degree: remains inside the vagina (coming down, but does not reach the introitus) • Second degree: cervix comes at or near the introitus*** • Third degree: most of the uterus and cervix lie outside the vagina • PER VAGINAL ▪ UTERINE SIZE, TENDERNESS ▪ ADNEXA: MASS, TENDERNESS ▪ OFFICE TESTS: BSL, URINE DIPSTICK ▪ DIAGNOSIS AND MANAGEMENT o From your history and examination, it seems that most likely you have a condition called stress incontinence. Have you heard about it? Let me explain this to you. (ILLUSTRATE) Because you are already in menopause, you have decreased levels of estrogen and this estrogen is needed to maintain a good tone of your lower pelvic muscles. Moreover, with your history of having several vaginal deliveries and with your increased weight, your pelvic muscles tend to become lax and loose thus causing the anterior vaginal wall (the part of the vaginal wall with the urethra/urine passage) to go out of your vagina, causing involuntary loss of urine whenever there is increased pressure inside your abdomen--that is when you tend to cough and strain. However, it is also likely that you have an "urge" incontinence, that is incontinence associated with a sudden need to void. Due to the mixed nature of your incontinence, we need to do further investigations to confirm your diagnosis, and to see which component appears to be worse. o I can imagine how this can be very distressing for you, but let me reassure you that this is a manageable condition. o I will refer you to a specialist who will do a urodynamic assessment to confirm your diagnosis. When the results of this assessment are available, decisions can be made on non- operative care. We will also do basic blood tests (FBE, UEC, CRP, ESR, RFTs), and urine microscopy and culture to identify other possible causes of your incontinence. o For now, this is what we can do. As your symptoms present more as stress incontinence, you can do pelvic floor exercises and I will refer you to a physiotherapist to teach you. You can do it in a sitting, standing, or a lying down position. Contract your bottom muscles for a count of 8, relax it for the same amount of time you have contracted your muscles, and do it 8 times at a go, 3 times a day. Eventually, you can slowly increase the time you can do this o You can also use a pessary to correct the prolapse you have. You have to maintain bladder diary. o It is very important that you lose weight to help reduce the pressure you have to your pelvic

muscles. I can refer you to a dietitian to assist you in your diet. And you should engage in exercise, for 30 minutes at least once a day. You should also stop smoking, as the latter will certainly reduce your coughing episodes that will reduce your abdominal pressure thus

minimizing your symptoms. 271 Page

© 2020 ARIMGSAS

o In case we see any instability of the muscle of your bladder (illustrate--detrusor instability), the specialist may start some medications that can control your symptoms (propantheline, imipramine, or oxybutynin). Bladder retraining will also be done so that it can accept a larger amount of urine before it will empty out. o Hormone replacement therapy can also be commenced to reduce the laxity of your pelvic floor muscles, and also reduce your menopause symptoms. o However, if these interventions will not work out for you, the specialist might consider surgery where he will elevate, support and buttress the bladder neck to reduce your symptoms. o Do you understand so far? Do you have any questions?

KEY ISSUES ▪ Knowledge of the causes and types of urinary incontinence which occur in middle-aged women ▪ Knowledge of the investigations required to assess such incontinence and the general principles of the management programmes available ▪ Ability to recognize the probability that this is not just stress incontinence, that urodynamic assessment is advisable, and treatment will depend on results obtained

CRITICAL ERRORS ▪ Failure to arrange urine microscopy and culture to exclude urinary infection

272 Page

© 2020 ARIMGSAS

Fundus less than dates

Your next patient is a 28-year-old primigravida who works as a nurse in a renal transplant unit. You have been looking after her pregnancy since the first trimester. You are seeing her in a general practice setting in a shared care arrangement with the local obstetric hospital. All appeared to be normal up to and including her last visit at 30 weeks of gestation, when the symphysis-fundal height was 28 cm. Today, four weeks later, the symphysis-fundal height is 29cm and there appears to be a reduced amount of liquor present.

TASKS: ▪ Take any further relevant history you require. ▪ Ask the examiner about relevant findings likely to be evident on general and obstetric examination and available investigation results. ▪ Advise the patient of the diagnosis and subsequent management including any further investigations you would arrange.

APPROACH ▪ History o I read from the notes that you are currently in your 31st week of pregnancy now, and your fundal height seems to be smaller than expected and there is a reduced amount of liquor in your tummy. o How is the baby? Is the baby kicking well? Are the fetal movement same as it was before? o Possible causes of the small fundus: • Are your dates certain? Did you do your ultrasound at 18 weeks? What was the result? • Were you ever diagnosed with hypertension or renal disease? Ever been diagnosed with lupus or arthritis or any thrombotic disorder in the past? • Pre-eclampsia questions: any headaches, blurring of vision, edema or swelling, proteins or glucose in the urine? • Do you have pets at home? Did you have any contact with dogs or cats? o Routine antenatal history: Any blood test and blood group done? Down syndrome screening? Folic acid? Any complications in the position of the placenta? Sweet drink test at 28 weeks? o Do you smoke, drink alcohol or take recreational drugs? o Do you have enough support at home? Who do you live with? ▪ Physical Exam o General appearance: pallor, edema, lymph node enlargement, BMI o Vital signs

o Systemic examination • CNS/CVS/Respiratory • Abdomen: fundic height, FHR, lie, presentation, is the head engaged, uterine tenderness 273

• Pelvic examination Page

© 2020 ARIMGSAS

▪ Inspection of the vulva and vagina ▪ Speculum: is the cervix closed or open? Is there discharge or bleeding? o Office test: UDT for proteins ▪ Investigations o CMV antibody testing o Toxoplasma antibody testing ▪ Management From history and physical examination, most likely your baby is having an intrauterine growth restriction. This is when the baby is not able to grow properly inside the womb. This is why your fundal height is less than expected and you have a reduced amount of liquor. The reason for this and its severity need to be assessed by some investigations such as ultrasound examination to confirm the size of the baby, to look for the amniotic fluid volume, and to see whether there is any obvious congenital abnormality which might explain the IUGR. Ultrasound will probably need to be repeated every 2-3 weeks. A Doppler study could also be done to assess the blood flow in the umbilical cord which supplies the needs of the baby. A cardiotocographic evaluation will be done as well twice a week from now until the time of delivery to assess and monitor the condition of the baby. There are many causes why IUGR could happen. It could be due to karyotypic abnormalities or problems in the genetic makeup of the baby, kidney disease, pre-eclampsia or abnormal, abrupt elevation of the patient's blood pressure with leakage of proteins into the urine, or congenital infections due to CMV or toxoplasmosis or placental dysfunction. That is why, in addition to the ultrasound, Doppler and CTG, we would also do serum urea, uric acid, and creatinine to look for evidence of renal compromise, lupus anticoagulant and anticardiolipin antibodies, antibodies for toxoplasmosis, and amniocentesis to assess the karyotype of the baby and to rule out toxoplasmosis in the specimen as well. Your baby will most probably be delivered via caesarean section and will be done prior to your due date. The timing of this will depend on the ultrasound evaluation, the CTG record and the amount of growth which occurs subsequently. But do not be too stressed, as long as the baby does not become deprived of oxygen and becomes acidotic and does not have a congenital malformation or a congenital infection, the long-term prognosis for the baby is satisfactory.

KEY ISSUES ▪ Understanding the causes of oligohydramnios. ▪ Management of a growth-restricted fetus in late pregnancy.

CRITICAL ERRORS ▪ Failure to recognise that the clinical picture is of intrauterine growth restriction. ▪ Failure to arrange appropriate assessment (or referral to a specialist). At the least ultrasound and serial CTG must be done.

274 Page

© 2020 ARIMGSAS

Nausea and vomiting in the first trimester

Your next patient is a 38-year-old woman who has come to the general practice because of severe nausea and vomiting for the last two weeks in this, her first pregnancy. She claims that she has been unable to keep foods or fluids down. Her last menstrual period was eight weeks previously, and pelvic examination by your colleague in the general practice two weeks ago showed the uterine size was appropriate for gestation and a pregnancy test was positive. She has had no previous operations or illnesses

TASKS 1. Take any further relevant history you require 2. Ask the examiner about relevant findings evident on general and obstetric examination which would assist you in making a diagnosis 3. Advise the patient of the likely diagnosis 4. Advise the patient of the care you would advise for her, including any investigations you would arrange

APPROACH o HEMODYNAMIC STABILITY--VITAL SIGNS: What is her current BP and is there a postural drop, pulse and rhythm, respiratory rate, oxygen saturation, and temperature? o If UNSTABLE: please transfer the patient to the treatment room, secure IV lines, and take blood for FBE, UEC, ESR, CRP, blood grouping and cross-matching. I would also like to do urine dipstick to check for ketones. Please start IV fluids: normal saline, Hartmann solution or Haemaccel whichever is available. o Please give her an oxygen mask and start high flow oxygen at 10/L (if RR / o2 sat unstable) o HISTORY o Congratulations on your pregnancy. I have read from your notes that you are currently at 8 weeks of pregnancy, and you have been suffering from vomiting for about two weeks now. I know this can be very distressing for you, but we'll do our best to manage you. Could you tell me more about your vomiting? • Is it getting worse? Does it usually come in the morning or at a particular time during the day? Could you describe to me the manner in how you vomit--is it projectile, do you retch, etc? What does your vomit usually consist of? What's its color? Does it have any blood? Did you eat anything out of the usual before you had these symptoms? • Any changes in your bowel motion? Do you still pass gas? (r/o bowel obstruction or gastroenteritis) • Assess dehydration ▪ How is your appetite? Are you still eating or drinking? Did you have any fever, diarrhea, or dizziness? How is your waterworks? Any burning or stinging

sensation? Do you go to the toilet more/less than the usual? Any change in the color of the urine? Any loin pain? o QUESTIONS ABOUT CURRENT PREGNANCY • Is this a planned pregnancy? How were you able to confirm your pregnancy? 275

• Are you in a stable relationship? Do you have support for this? Page

© 2020 ARIMGSAS

• R/O causes of vomiting ▪ Do you have any family history of twins? ▪ Is this a natural or an assisted pregnancy? ▪ Any bleeding or tummy cramps? • Did you have your initial blood tests requested during your previous visits to your GP? Were you advised regarding screening and confirmatory tests for diseases in the baby? o Any history of STIs? o When was your last pap smear? o Do you know your blood group? o Do you smoke, drink alcohol, engage in recreational drugs? o Do you take other medications? Any allergies? o Do you have any history of any medical or surgical conditions? o PHYSICAL EXAMINATION o GA: dehydration--skin turgor, CRT, tongue & oral mucosa moist? Lymphadenopathies, pallor? o VS: BP with the postural drop? Temperature? HR? o CVS, CNS, Respiratory system o Abdomen: distended or any masses? Tenderness? Rigidity/guarding? Any bowel sounds? o Pelvic exam not needed o Office tests: Urine dipstick and BSL o Check for ketones o PE findings from the case: o She looks unwell and drawn. Her tongue dry and firm. Tissue turgor of the skin is diminished. o Pulse: 110/min o BP: 120/80 o Temperature 36.8C o Abdominal examination, uterus not palpable o No loin tenderness o Pelvic examination not repeated o DIAGNOSIS AND MANAGEMENT o From history and examination, it seems that most likely you have a condition we call as Hyperemesis gravidarum. Have you heard about it? It is a condition common in early pregnancy manifested by excessive nausea and vomiting. Its cause is multifactorial; however, it is usually implicated that the excessive vomiting and nausea is due to increased hormone levels especially the b-HCG hormone which supports your pregnancy. o This condition usually goes away on its own, especially by 14 weeks of gestation as your body becomes used to these new hormones, and the level of b-HCG goes down. o However, it was seen from your examination that you are severely dehydrated, supported by findings of ketones in your urine. Because of this, I will have to refer you to a hospital so that you can be admitted for intravenous rehydration and monitoring. I will call an ambulance to transfer you to the hospital.

o I will also refer you to a specialist who will see you and might do further investigations. Although hyperemesis gravidarum can be a complication of a normal pregnancy, it also occurs with increased frequency in association with other conditions such as multiple pregnancies, a urinary tract infection, or even a condition we call as a hydatidiform mole-- 276

where there is an abnormal growth of placenta mimicking pregnancy. To rule out these Page

© 2020 ARIMGSAS

conditions, you will undergo blood tests such as an FBE, UEC, serum b hcg, liver function tests, and also urine microscopy and culture, and a transvaginal ultrasound. You will be given fluids through IV, and anti-vomiting/nausea meds (metoclopramide - maloxon/stemetil), vitamin B supplementation (pyridoxine) to address your symptoms. You and your baby will continually be monitored throughout your admission. o Another thing that I would like to address is that you are currently 38 years old now, and this puts you at a very high risk of having a baby with Down syndrome or other genetic abnormalities. • SHORTCUT: Because of this, I will arrange genetic counselling for you to undergo screening tests for down syndrome which consists of a blood test done as early as 9- 13 weeks looking for factors in blood which denote possible presence of a baby with down syndrome, combined with an ultrasound examination done at 11-13 weeks. We can also do a screening test in your second trimester which will also involve a blood test looking for 4 factors in the blood (AFP, BHCG, ESTRIOL, INHIBIN) which will screen for the condition. We can also do confirmatory tests: the chorionic villous sampling test done during 11-14 weeks, which involves getting samples from your placenta and to send it for genetic testing. We can also do amniocentesis, which can be done at 15- 18 weeks, which involves getting a sample of your bag of water and we send it for genetic analysis for down syndrome. All of these confirmatory tests have certain risks for miscarriage with 1:100 for CVS, and 1:200 for Amniocentesis. • FULL EXPLANATION: We have two screening tests, we usually do blood tests looking for factors namely the bhcg and pappa as early as 9-13 weeks, together with an ultrasound of the baby's back of the neck at about 11--13 weeks of your pregnancy. The bhcg is increased, and pappa is decreased in a baby suspected of having downs. The detection rate of this combined test is 87% Another test that we do is the Noninvasive Prenatal Test (NIPT) which is done at 10 weeks. We get a blood sample from you and we send it for genetic testing. The detection rate is 99% but it is not covered by Medicare and usually costs around 600- 700aud. For the second trimester, 15-17wks. we can also do what we call a quadruple and a triple screen. We test factors in your blood, namely the bhcg, inhibin, estriol and AFP. Both bhcg and inhibin are increased while the other two are decreased in a quad screen. We test the bHCG, estriol, and AFP in the triple screen. Detection rates are 81% and 71% respectively. Once we get positive for downs in the screening tests, we do confirmatory testing for downs Chorionic Villous sampling that we do during 11-14 wks. of pregnancy where we insert a needle guided by ultrasound from down there and get a sample from the placenta which we send for genetic testing. However miscarriage rates are 1:100. We can also do an amniocentesis, which is done at around 15-18 weeks. We pass a needle guided by an ultrasound to your womb, to get a sample of your bag of water and we send it for genetic analysis. Risk of miscarriage for this is

1:200.

277 Page

© 2020 ARIMGSAS

KEY ISSUES ▪ Ability to investigate and treat a woman with hyperemesis gravidarum ▪ Recognition of the need for genetic counselling in the view of advanced maternal age

CRITICAL ERRORS ▪ Failure to recognise the need for hospitalization ▪ Failure to do ultrasound and urine examination to check pregnancy, diagnose twins, molar pregnancy, urinary infection, and the presence of urinary ketones

278 Page

© 2020 ARIMGSAS